You are on page 1of 648

PREFACE

This book provides the mathematical knowledge that is needed to pursue a career as a
Merchant Navy Engineering Officer.

The UK system of certification utilises Higher Education programmes to meet the


academic knowledge required for the operational and management level Certificates
of Competency.

This book covers the fundamental topics of mathematics (algebra, trigonometry


and calculus) in detail and also gives an introduction to higher-level topics (Laplace
Transforms and Fourier Series). It will be an invaluable aid to all students of technical-
based courses in which mathematics and analytical methods play an important part.

Basic principles are introduced from an elementary stage to facilitate self-study.

Each chapter includes fully worked examples interwoven into the text, as well as test
examples set at the end of each chapter. This will ensure that readers can monitor the
progress they are making in this study of mathematics.

Each chapter concludes with typical examination questions.

Non-programmable calculators may be used and a comprehensive set of formulae are


provided.

The author has gone beyond the normal practice of merely supplying bare answers to
the test examples and examination questions by providing fully worked step-by-step
solutions leading to the final answers. It is this sequential learning method that will
enable readers to understand the topics and apply them to other analytical situations
in all disciplines of engineering, including marine engineering.

This new edition is a major update in the subject and learning methods, and brings the
material for study completely up to date.
INDICES AND
1
LOGARITHMS
Indices
A prime number can only be divided by two factors, 1 and itself.

Any number can be written as a multiplication of its prime factors.

Example
200 = 2 × 2 × 2 × 5 × 5

These factors can be written as 23 × 52 , where 2 and 5 are called bases and the numbers
3 and 2 are called indices. Whole number indices are called powers.

When an integer is written without an index, the power is 1. Example: 6 = 61 .

The reciprocal of a number means 1 divided by the number and has the power −1.
1
So the reciprocal of 5 is 5−1 and has a value of , or 0.2.
5
If the base number is a fraction its reciprocal is simply the fraction turned upside down
(or inverted).

Example
 −1  −1
5 3 3 5
= and, naturally, =
3 5 5 3

A quotient means the division of two quantities so, as in the above example, the
3
quotient of 3 and 5 is = 0. 6.
5
A product means the multiplication of two (or more) quantities.
2 • Mathematics

Behaviour of Indices
Important: The following rules apply only to terms having the same base.

Consider quantities with the base ‘a’.

Example

(a) a3 × a2 = (a × a × a) × (a × a) = a5
So a3 × a2 = a(3+2) = a5
a×a×a×a×a×a
(b) a6 ÷ a2 = = a4
a×a
So a6 ÷ a2 = a(6−2) = a4
 3
(c) a2 = a2 × a2 × a2 = a(2+2+2) = a6
 3
So a2 = a(2×3) = a6

These three statements are examples of the three laws of indices:

• am × an = a(m+n)
• am ÷ an = a(m−n)
• (am )n = amn

These laws can be used to find other identities.

Example

(d) ab ÷ ab = a(b−b) = a0
but ab ÷ ab = 1, as any value divided by the same value must be equal to 1
So a0 = 1

So: any value (numerical or algebraic) to the power zero equals 1.

(e) ab × a−b = a(b+( b)) = a0 = 1


1
Divide both sides by ab gives a−b =
ab
So: a negative power means 1 divided by the positive power (and vice versa)

(f ) A root of a value (square, cube, . . . ) can also be written as a power.



Let a = ap .
√ 2
Since a = a1 it follows that (ap )2 = a1 . The third law means that (ap )2 = a2p .
1
So a2p = a1 so 2p = 1 ⇒ p =
√ 2
Therefore a = a1/2
Indices and Logarithms • 3


Similarly 3 a = a1/3 etc.
If the power is a fraction it means the following – the denominator is the type of root
and the numerator is the power.
b   √ b
a c = c ab or c
a

(g) The square root of a number has an index of 12 , so the square root of 16 can be

written as 16 or 161/2 .
√ √
As 4 × 4 = 16 then 16 = 4, but (−4) × (−4) = 16 so 16 = −4.
Therefore every square root has two possible values, one positive, the other

negative, so 16 = ±4.
This is the case for any even root.

Example

(h) Find the value of 53 × 55 ÷ 54


 
53 × 55 ÷ 54 = 53 × 55 ÷ 54 = 5(3+5) ÷ 54 = 58 ÷ 54 = 5(8−4) = 54 = 625
or 53 × 55 ÷ 54 = 5(3+5−4) = 54 = 625
 2
(i) Find the value of 53 ÷ 54
 3 2
5 ÷ 54 = 5(3×2) ÷ 54 = 56 ÷ 54 = 5(6−4) = 52 = 25
 2
or 53 ÷ 54 = 5(3×2−4) = 52 = 25
 3 3
5
(j) Find the value of
55 × 56
 3 3
5 5(3×3) 59 1 1
= = = 5(9−11) = 5−2 = 2 =
55 × 56 5(5+6) 511 5 25
(k) Evaluate 642/3
 √ 2 √ √
64 /3 = 64 = 42 = 16 or 64 /3 = 642 = 4096 = 16
2 3 2 3 3

Obviously the first way is much easier!

These rules can be combined to give some useful tips.


 2
6
Consider the following:
2
 2  2
6 6 62 36
Now = 32 = 9, but = 2 = =9
2 2 2 4
4 • Mathematics

This means that ‘the power of a quotient gives the same result as the quotient of the
powers’.

Since a power can also represent the root of a number the same rule applies.
36
Consider the following:
4

36 √ 36 36 6
Now = 9 = ±3, but = √ = ± = ±3
4 4 4 2
A root of a number that does not have an exact value is called a surd. For example,
√ √
2, 5.

Applying powers to the product of terms gives the same type of result.
√ √ √ √ √ √
So 12 = 4 × 3 = 4 × 3 = ± 2 × 3 = ±2 3
8
Consider the following ratio: √
12
√ √ 8 8 4
Since 12 = ±2 3, it follows that √ = √ = ±√
12 ±2 3 3
Although this is numerically correct, it is not considered to be written correctly as roots
are never left as terms on the denominator. The problem is removed by multiplying
numerator and denominator by an appropriate root.
√ √ √
4 4 3 4 3 4 3
Therefore ± √ =± √ × √ = ± √ 2 = ±
3 3 3 3
3
√ √
3 3 2 3 2
Similarly √ =√ ×√ =
2 2 2 2

Questions

Attempt the following questions, giving the answer in index form:

1. 44 × 42 2. 23 × 22 × 2−4 3. 66 ÷ 62
 5 2 23 × 24 23
4. 3 5. 6.  
22 21 × 2−3
 3
33 × 9 (16 × 32)3 37
7. 8. 9.  
(27 × 3) 4
(8 × 16) 5 3−4 × 33
 −2 2  2 −3
24 × 2−3 52 × 53 × 5−4 2 × 2
10. 11. 12.  5
2 53 × 50 × 5−2 4 −1
Indices and Logarithms • 5

Answers

1. 46 2. 21 3. 64 4. 310 5. 25 6. 25
7. 3−1 8. 2−8 9. 38 10. 20 11. 50 12. 2−20

Logarithms
If N is a number such that N = ax then x is the logarithm of N to the base a.

This is written as x = loga N

Example: 1000 = 103 . Therefore log10 1000 = 3.

Questions

In each of the following find the value of x:

1. logx 9 = 2 2. logx 81 = 4
3. log2 16 = x 4. log5 125 = x
5. log3 x = 2 6. log4 x = 3
7. log10 x = 2 8. log7 x = 0
9. logx 8 = 3 10. logx 27 = 3

Answers

1. 3 2. 3 3. 4 4. 3 5. 9
6. 64 7. 100 8. 1 9. 2 10. 3

The usual base of a logarithm is 10, where the abbreviation used is log. This is called a
common logarithm.

Another base is the number 2. 718 · · · (given the symbol ‘e’): this is called the natural
(Naperian) logarithm and is abbreviated as ln.
6 • Mathematics

Rules for the use of logarithms

1. The logarithm of two numbers multiplied together is given by the sum of the
individual logarithms:
log(xy) = log(x) + log(y)

2. The logarithm of the division of two numbers is given by subtracting the individual
logarithms:
log(x ÷ y) = log(x) − log(y)

3. The logarithm of a number raised to a power is given by the logarithm of the number
multiplied by the power:
log(x y ) = y · log(x)

Relation between logarithms of different bases

loga (N)
logb (N) =
loga (b)

Example

Find the logarithm of 8 to base 2 using common logs.

log (8) 0. 9030


log2 (8) = = =3
log (2) 0. 3010

This is obviously correct as 23 = 8.

Questions
Without using a calculator, find the value of:

1. log 8 + log 12. 5 2. log 30 − log 3


3. log2 28 − log2 7 4. log5 12. 5 + log5 10
5. 2 log 2 + log 75 − log 3 6. log 25 ÷ log 5
7. log 8 ÷ log 2 8. log 27 ÷ log 3
9. 3 log 2 + log 200 − log 16 10. log 30 − log 3
Indices and Logarithms • 7

11. log2 16 − log2 8 12. log3 2. 7 + log3 10


13. log 4 + 2 log 5 14. log 32 ÷ log 2
15. log 7 + 2 log 5 − log 1. 75 16. log 18 + log 5 − log 9
17. log6 42 − log6 7 18. log 3 + log 15 − log 4. 5
19. log5 40 − log5 1. 6 20. log8 72 − log8 1. 125

Answers

1. 2 2. 1 3. 2 4. 3 5. 2
6. 2 7. 3 8. 3 9. 2 10. 1
11. 1 12. 3 13. 2 14. 5 15. 2
16. 1 17. 1 18. 1 19. 2 20. 2

Test Examples 1
Questions 1 to 5 to be answered without the use of a calculator:
103/2 × 104 × 103/4
1. Find the value of
103 × 101/4 × 102
42 × 4 /3 × 4−2
2
2. Evaluate
41/2 × 41/6
8−2
3. Find the value of −5
8
 2/5 5/3 √
3 × 3×3
4. Evaluate
3−4/3 × 33
 2
2 4 √
5. Find the value of 1 + + 200
3 81
For help with the next questions refer to Appendix 1.

6. The ratio of the volumes of two spheres is equal to the ratio of the cubes of their
diameters; the volume of one sphere is 24.25 cm3 , find the volume of another
sphere whose diameter is twice as much.
7. A pump can empty a tank in 12 h, another pump can empty the same tank in 4 h
and a third can empty this tank in 9 h. If all three pumps are set to work together
on emptying the tank, how long would it take to empty it?
8 • Mathematics

8. The strength of a beam varies directly as its breadth, directly as the square of its
depth and inversely as its length. A beam is 5 m long, 40 mm broad and 100 mm
deep, find the breadth of another beam of similar material, 3 m long and 80 mm
deep, to have equal strength.
9. A piece of mild steel 50 mm long between gauge points and 80 mm2 cross-
sectional area, was tested in a tensile testing machine and broke when the
gauge length was 62.5 mm and cross-sectional area 48 mm2 . Find the percentage
elongation in length and the percentage reduction in cross-sectional area.
10. In a certain three cylinder engine, the power developed in No. 1 cylinder is 15%
more than in No. 3, and 5% less power is developed in No. 2 than in No. 3. What per-
centage of the total engine power is developed in each cylinder? Give the answers
correct to one decimal place.
11. A brass casting is composed of 71% copper, 1% tin, 3% lead and the remainder
zinc. Find the mass required of each constituent to make 500 kg of this alloy.
12. The heights of an indicator diagram measured at regular intervals along its length
are as follows: 27, 39, 47, 51, 48, 32, 20, 11, 8 and 5 mm respectively. Find the mean
height of the diagram in mm.
13. 200 tonnes of oil were bought in one port $600/tonne and 600 tonnes of oil at
another port at $700/tonne. What is the average cost of the oil per tonne?
ALGEBRA
2
Algebra is a convenient system of notation in which letters or symbols are used to
represent quantities, instead of words.

Example

Suppose a box contained 25 bolts, 34 nuts and 47 washers. To the contents 15 bolts and
18 washers were added but 14 nuts were removed.

This could be set down as follows:

25b + 34n + 47w


add 15b − 14n + 18w
result 40b + 20n + 65w

Example

The area of a rectangle is found by its length multiplied by its width.

If the area is A, the length L and the width W, then A = L × W.

If the length and width are subtracted then the answer is L − W, if they are added L + W
L
and if they are divided L ÷ W or .
W
Note: the ‘×’ symbol is often ignored so L × W becomes LW (or L · W)

When a quantity, y, is multiplied by a number, for example 3, the answer is written 3y. If
a quantity appears by itself, for example k, it means one times k.

If a combination of letters and numbers are multiplied together the order does not mat-
ter so d4e is the same as ed4 and so forth. However, it is usual to write the number first
and the letters alphabetical afterwards, i.e. 4de.
10 • Mathematics

An expression which consists of two terms is called a binomial expression, and one
consisting of three terms is called a trinomial expression.

Simplification of Algebraic Expressions


Note: Different symbols can be combined by multiplication or division, but cannot be
combined by addition or subtraction.

Addition of terms

Example

x + 3x = 4x but c + d stays as c + d.

Example

Add 2a + 7b + 3c and 6c + 5a − 2b

2a + 7b + 3c
Add 5a − 2b + 6c
7a + 5b + 9c

Example

Add 3x − 5y + 6z and 4x + 2y − 8z

3x − 5y + 6z
Add 4x + 2y − 8z
7x − 3y − 2z

Subtraction of terms

Example

From 8x subtract 5x
8x − 5x = 3x
Algebra • 11

From 8x subtract −5x

8x − (−5x) = 8x + 5x = 13x

From −8x subtract 5x

−8x − 5x = −13x

From −8x subtract −5x

−8x − (−5x) = −8x + 5x = −3x

Example

Subtract 6c − 3b + 4a − 7d from 5b + 3d + 2a − 9c

4a − 3b + 6c − 7d subtract
2a + 5b − 9c + 3d
2a − 8b + 15c − 10d

However, 2abc + 5bac = 7abc as each term has the same set of letters although the
order differs.

m2 n and mn2 are different as the first term is m × m × n and the second is m × n × n.

Questions
Simplify, where possible:

1. p+q 2. a + 3a 3. d + 7d − 3d

4. s + 5s − 3t 5. 6y − (−5y) 6. c×d

7. 2x × 5x 8. 6s × 2t 9. q × 7q3

10. 6xy ÷ 3 11. 9x 2 y ÷ 3x 12. j − (−3k)

13. 16q − 8qt 14. 2a2 b − 6a2 15. nm + n + 2m + 2

16. 6 + 2d − cd − 3c 17. m2 n − mn2


12 • Mathematics

Answers

1. p+q 2. 4a 3. 5d 4. 6s − 3t

5. 11y 6. cd 7. 10x 2 8. 12st

9. 7q4 10. 2xy 11. 3xy 12. j + 3k

13. 8q(2 − t) 14. 2a2 (b − 3) 15. (n + 2)(m + 1) 16. (3 + d)(2 − c)

17. mn(m − n)

An algebraic expression surrounded by a bracket with a number in front means all the
terms inside the bracket are to be multiplied by the number outside the bracket.

Example

3 (4s + 5t) = 3 × 4s + 3 × 5t = 12s + 15t

One very important rule is necessary when removing brackets.

If there is a negative sign in front of the bracket, ALL the signs inside the bracket
change when the bracket is removed.

Example
SIGN
24x − 3(6y + 4x) = 24x − 18y − 12x = 12x − 18y CHANGE

Just as brackets can be removed from an expression, they can equally well be inserted.

Method

• Find out the common terms that can be divided out from the expression. Set this
outside the brackets and place inside the brackets whatever is left when the factor is
removed.
• It is often useful to put 1 in front of a term that appears by itself. Example, x = 1x,
a2 b = 1a2 b . . .

Example

18xy + 12y = 6y(3x + 2) as 6y divides exactly into 18xy and 12y leaving 3x and 2
respectively.
Algebra • 13

Questions
Multiply out the brackets and collect like terms:

1. 4x(a + b + 2c) 2. 3m(x − 3y + 2z) 3. −2a (p − q − r + s)


     
4. x x2 + x + 2 5. −3m m2 − 2m + 4 6. −2a 3a2 + 4a − 2
     
7. u2 2u3 + 5u2 − 4u − 3 8. 3d2 d2 + 4d + 1 9. −4m2 m2 − 2m − 3

10. a (a − m) − m (a + m) 11. m (m + 2u) − u (2m − u) 12. b (2b − 3a) + x (4b − x)


   
13. 2c (c + 2d) − 3d (c − 2d) 14. x2 (3x − 1)− 2x x2 − x + 2 15. 2a a2 + 3a + 3a2 (a − 4)
 
16. 5mn (m − 3n)−2mn (m + 4n) 17. 3u u2 − v2 + 2uv (u − 2v)

18. 3a2 (2a − b) − ab(a + 2b) + b2 (2a − 3b)

19. 4x2 (x − 3y) + 2xy (6x + 3y) − 3y2 (2x − 3y)

20. 4x2 (x − 3y) − 2xy (6x + 3y) + 3y2 (2x − 3y)

Answers
1. 4ax + 4bx + 8cx 2. 3mx − 9my + 6mz 3. −2ap + 2aq + 2ar − 2as

4. x3 + x2 + 2x 5. −3m3 + 6m2 − 12m 6. −6a3 − 8a2 + 4a

7. 2u5 + 5u4 − 4u3 − 3u2 8. 3d4 + 12d3 + 3d2 9. −4m4 + 8m3 + 12m2

10. a2 − 2am − m2 11. m 2 + u2 12. 2b2 − 3ab + 4bx − x2

13. 2c2 + cd + 6d2 14. x3 + x2 − 4x 15. 5a3 − 6a2

16. 3m2 n − 23mn2 17. 3u3 + 2u2 v − 7uv 2 18. 6a3 − 4a2 b − 3b3

19. 4x3 + 9y3 20. 4x3 − 24x2 y − 9y3

Questions

Factorise the following:

1. ab + ac 2. 5x + 25 3. b2 − bc

4. 4b2 − 2bc 5. 6ab − 12a 6. 8a2 b − 32ab

7. 5a3 b2 c − 10a2 b2 c − 15abc2 8. 6ab2 c + 9abc + 15a2 bc3


14 • Mathematics

Answers

1. a(b + c) 2. 5(x + 5) 3. b(b − c)

4. 2b(2b − c) 5. 6a(b − 2) 6. 8ab(a − 4)


   
7. 5abc a2 b − 2ab − 3c 8. 3abc 2b + 3 + 5ac2

When terms are divided, division can only take place when the term on the denominator
(bottom line) is part of every term on the numerator (top line).

Example

Divide 9a3 b2 + 12a2 b3 − 18a5 b2 by 3a2 b2

Solution

Each term has to be divided by 3a2 b2 so:

9a3 b2 + 12a2 b3 − 18a5 b2 9a3 b2 12a2 b3 18a5 b2


= 2 2+ − = 3a + 4b − 6a3
3a2 b2 3a b 3a2 b2 3a2 b2

Questions
Divide

1. 4a7 + 2a5 − 4a3 by −2a2 2. 3b3 m − 12b2 m + 6bm by 3bm

3. 4u3 v − 8u2 v2 + 16uv3 by 4uv 4. 6m4 n − 2m3 n2 − 8m2 n3 + 4mn4 by −2mn

5. −3x6 − 9x5 + 6x4 − 3x3 by −3x2 6. −8a4 u2 + 4a3 u3 − 12a2 u4 by 4a2 u

7. 6p4 q3 − 9p3 q2 − 3p2 q3 + 6p3 q4 by −3pq2 8. −8h3 k3 − 6h2 k2 + 2h2 k4 − 4h4 k2 by −2h2 k2

3x5 y2 − 9x4 y3 + 3x3 y4 − 6x2 y5 4a3 m2 u2 + 6am3 u2 − 2a3 m2 u − 4am2 u3


9. 10.
−3x2 y2 2am2 u

Answers

1. −2a5 − a3 + 2a 2. b2 − 4b + 2

3. u2 − 2uv + 4v 2 4. −3m3 + m2 n + 4mn2 − 2n3

5. x 4 + 3x 3 − 2x 2 + x 6. −2a2 u + au2 − 3u
Algebra • 15

7. −2p3 q + 3p2 + pq − 2p2q2 8. 4hk + 3 − k2 + 2h2

9. −x 3 + 3x 2 y − xy2 + 2y3 10. 2a2 u + 3mu − a2 − 2u2

Highest Common Factor (HCF) and


Lowest Common Multiple (LCM)
Example

Find the HCF of 8ab2 c3 , 6a2 b2 and 10ab2 c2

8ab2 c3 = 2 × 2 × 2 × a × b × b × c × c × c
6a2 b2 = 2 × 3 × a × a × b × b
10ab2 c2 = 2 × 5 × a × b × b × c × c × c

The biggest collection of factors that is contained in all three expressions is 2×a×b ×b.

So the HCF is 2ab2 .

Questions
Find the HCF of:

1. amxy and bmdx 2. 9jhk and 6ghk

3. mn2 and m2 n 4. 8d2 e and 6def

5. 10km2 n3 and 15k3 m2 n 6. 24a2 b4 and 16a3 b3

7. uvw, vwx and wxy 8. 8ab3 , 2a2 b2 and 6a3 b

9. 3c4 d3 , 9c2 d5 and 7c3 d4 10. 12ax 3 y2 , 4a3 y4 and 8a2 xy3

11. 15abm2 n, 6cmp2 b and 12b2 dmp

12. 4f 3 gh3 , 8g2 h4 k3 and fg3 h2 k

13. 18a2 bm2 n3 , 24a3 b2 m2 and 12a2 mn2

14. 16uv 2m3 n2 , 32u3 mn3 and 24v 3 m2 n2

15. 16a4 b3 x 3 , 24b2 m3 x 4 y and 20a2 b3 nx 3


16 • Mathematics

Answers

1. mx 2. 3hk 3. mn 4. 2de 5. 5km2 n

6. 8a2 b3 7. w 8. 2ab 9. c2 d 3 10. 4ay2

11. 3bm 12. gh2 13. 6a2 m 14. 8mn2 15. 4b2 x 3

Example

Find the LCM of 4a2 bc, 6ac and 8bc2

4a2 bc = 2 × 2 × a × a × b × c
6ac = 2 × 3 × a × c
8bc2 = 2 × 2 × 2 × b × c × c

The smallest collection of factors that contains all the factors of the three expressions is:

2×2×2×3×a×a×b×c×c

So the LCM is 24a2 bc2 .

Questions
Find the LCM of:

1. mnu and muv 2. 3abm and 2am2

3. ab, bc and cd 4. 4x 2 y and 6xy3

5. 9u2 mn and 6vm2 n 6. 3a2 b, abc and 6b2 c

7. m2 n2 , 5m and 10n 8. 3a2 b5 , 6a3 c2 and 12b3 c

9. 3f 2 g, 6g2 h and 4h2 k 10. 8pqr3 , 6p2 qs and 12q3 rs2

11. a2 bm3 n, ab2 m and abn2

12. 2cdu2 , 3d2 uv and 6c2 uv 2

13. 9c2 d3 , 3bcd2 e and 6bde

14. 6am2 x, 4anx 2 and 8am2 nx 2

15. 3a2 uv 4 , a4 mu2 v 3 and 4a3 m2 v 3


Algebra • 17

Answers
1. mnuv 2. 6abm2 3. abcd 4. 12x2 y3 5. 18m2 nu2 v

6. 6a2 b2 c 7. 10m2 n2 8. 12a3 b5 c2 9. 12f 2 g2 h2 k 10. 24p2 q3 r3 s2

11. a2 b2 m3 n2 12. 6c2 d2 u2 v2 13. 18bc2 d3 e 14. 24am2 nx2 15. 12a4 m2 u2 v4

Algebraic Fractions
Example
bd ?
What is the value of ? if = 2 2
2ac 4a bc
4a2 bc2 is obtained by multiplying 2ac by some quantity, call it Q

So the numerator bd must also be multiplied by Q to give ?


4a2 bc2
It follows that Q = = 2abc.
2ac
So ? = bd × 2abc = 2ab2 cd.

To add or subtract algebraic fractions, bring them all to a common denominator by


finding the LCM of the denominators of the given fractions, then add or subtract, like
terms being combined.

Example
3x + 1 x + 2 2x − 3
Simplify − −
2x 3x 6x
The LCM of the denominators is 6x.

For the first fraction, the denominator 2x has to be multiplied by 3 to make the LCM of
6x, and so the numerator also has to be multiplied by 3 to give 3(3x + 1).

For the second fraction, the denominator 3x has to be multiplied by 2 to make the LCM
of 6x, and so the numerator also has to be multiplied by 2 to give 2(x + 2).

For the third fraction, the denominator is already 6x, so the numerator stays as (2x − 3).
3x + 1 x + 2 2x − 3 3 (3x + 1) 2 (x + 2) (2x − 3)
So − − = − −
2x 3x 6x 6x 6x 6x
3 (3x + 1) − 2 (x + 2) − (2x − 3) 9x + 3 − 2x − 4 − 2x + 3 5x + 2
= = =
6x 6x 6x
18 • Mathematics

3x + 1 x + 2 2x − 3
Note: the given expression − − contains no brackets. The reason for
2x 3x 6x
this is that the division lines showing the fraction act as a double purpose. First they indicate
that the numerator is divided by the denominator but second, they show that everything
above (or below) the line is one complete term.
3x + 1 (3x + 1)
So could be written as
2x 2x

Example
1 3
Simplify +
3x 5y
The LCM of 3x and 5y is 15xy:
1 3 1 × 5y + 3 × 3x 5y + 9x
So + = =
3x 5y 15xy 15xy

Example
x y
Simplify +
yz xz2
The LCM of yz and xz2 is xyz2
x y x × xz + y × y x 2 z + y2
So + 2= =
yz xz xyz2 xyz2

Example
2
Simplify +5
n

2 2 2 n 2 5n
+5= +5×1= +5× = + =n
n n n n n n

Example
p
Simplify +3
qr

p p p qr p 3qr p + 3qr
+3= +3×1= +3× = + =
qr qr qr qr qr qr qr
Algebra • 19

Example
a+b b+c
Simplify +
ab bc

a + b b + c (a + b) c + (b + c) a (ac + bc + ba + ca) 2ac + ab + bc


+ = = =
ab bc abc abc abc

Questions

Find the expression for the ? in the following:

5a ? a 5ab 2ab a
1. = 2. = 3. =
3b 12bc bc ? ? 7bc
? 3d 2a + b ? a + 2b ?
4. = 5. = 6. =
a4 b2 c a3 b2 a 3ab 5ab 15a2 b

Simplify as far as possible:

3 2 2 3 ab
7. + 8. − 9. +3
4a 3a ab bc c
3 2 1 2 3 3+a 1
10. − 11. + − 12. −
ab bc a b ab 2 a
a + b a + 2b 2a + b 1 2 c a 1
13. + + 14. + +3 15. − −
2 3 4 ab bc ab bc abc

Answers

1. 20ac 2. 5b2 c 3. 14b2 c 4. 3acd

17 2c − 3a
5. 3b(2a + b) 6. 3a(a + 2b) 7. 8.
12a abc
ab + 3c 3c − 2a b + 2a − 3 a2 + 3x − 2
9. 10. 11. 12.
c abc ab 2a
16a + 17b c + 2a + 3abc c2 − a2 − 1
13. 14. 15.
12 abc abc
20 • Mathematics

Expansions and Factors


Expansion of (x + 3y)(x + 2y) means that each term in the first bracket has to be
multiplied by each term in the second.

So far only the removal of brackets has taken place when only one term exists outside
the bracket, for example: 3(5x + 2).

However, it is very commonplace to have two (or more) terms outside the bracket, for
example: (x + 3y)(x + 2y).

This is called a binomial expression because there are two terms in each bracket.

It is best to follow the same procedure each time two sets of brackets have to be
expanded.

Method

First times first, first times last, last times first, last times last.

Example

Expand (x + 3y)(x + 2y)


3
2
(x + 3y)(x + 2y) = x 2 + 2xy + 3yx + 6y2
1 2 3 4
1 4
or
x 2 + 5xy + 6y2

Questions

Expand the following and collect like terms:

1. (x + 1)(x + 3) 2. (x + 2)(x + 3) 3. (2x + 1)(x − 1)

4. (2x + 2)(2x − 1) 5. (x − 4)(x − 1) 6. (3x − 7)(4x − 3)

7. (x + 6)(5x − 6) 8. (x − 3)(4x + 2) 9. (4x − 1)(x + 3)

10. (3x − 6)(3x + 5) 11. (4x + y)(x − 2y) 12. (2x + 5y)(2x − 5y)
Algebra • 21

Answers

1. x 2 + 4x + 3 2. x 2 + 5x + 6 3. 2x 2 − x − 1

4. 4x 2 + 2x − 2 5. x 2 − 5x + 4 6. 12x 2 − 37x + 21

7. 5x 2 + 24x − 36 8. 4x 2 − 10x − 6 9. 4x 2 + 11x − 3

10. 9x 2 − 3x − 30 11. 4x 2 − 7xy − 2y2 12. 4x 2 − 25y2

With practice it will become unnecessary to write down all the working, and the answer
can be written down at once.

Example, in the expansion (x − 2y) (x − 3y) = x2 − 3xy − 2xy + 6y2 the terms.

−3xy and −2xy may be combined mentally and the answer x 2 − 5xy + 6y2 can be
written down at once.

Polynomial Division
A polynomial is an expression of the form f (x) = a + bx + cx 2 + dx 3 + · · · .

The method of dividing polynomials is best shown by looking at worked examples.

Example

Divide 3x 2 + x − 4 by x − 1

3x 2 + x − 4 is called the dividend and x − 1 the divisor. The usual layout is shown below
with both the dividend and divisor shown in descending order of powers.

3x + 4

 
x − 1 3x 2 + x − 4
3x 2 − 3x
4x − 4
4x − 4
0
22 • Mathematics

3x 2 + x − 4
So = 3x + 4
x−1

Reasoning

→ Dividing the first term of the dividend by the first term of the divisor.
3x 2
→ So gives 3x, which is put above the dividend as shown.
x
→ The divisor is then multiplied by 3x, i.e., 3x(x − 1) = 3x 2 − 3x, which is then placed
under the dividend as shown.
→ Subtracting gives 4x and the −4 is ‘dropped down’ to give 4x − 4.
→ The process is then repeated, i.e., the first term of the divisor, x, is divided into +4x,
giving +4, which is placed above the dividend as shown.
→ Then +4(x − 1) = +4x − 34 which is placed under the 4x − 4.
→ The remainder on subtraction is 0, which completes the division.
 
→ Therefore 3x 2 + x − 4 ÷ (x − 1) = (3x + 4)

A check can be made by multiplying x − 1 by 3x + 4.

Example

Divide x 2 + 5x + 4 by x + 4

x+1

 
x + 4 x 2 + 5x + 4
x 2 + 4x
x+4
x+4
0

x 2 + 5x + 4
So =x+1
x+4
Algebra • 23

Example

Divide 6x 2 − 11x − 12 by 3x + 2

2x − 5

 
3x + 2 6x 2 − 11x − 12
6x 2 + 4x
−15x − 12
−15x − 10
−2

6x 2 − 11x − 12 2
So = 2x − 5 −
3x + 2 3x + 2

Questions

Divide

1. a2 + 6a + 8 by a+2 2. b2 − 5b + 6 by b−3

3. c2 + 2c − 8 by c−2 4. d2 − 2d − 15 by d+3

5. e2 + 2e − 9 by e+4 6. 3f 2 + 5f − 2 by f +2

7. 2g2 − 13g + 17 by g−5 8. 3h2 + 14h + 12 by h+3

9. j2 + 3jk + 2k2 by j+k 10. 3m2 + 7mn − 6n2 by 3m − 2n

11. 10p2 − 2pq − 10q2 by 5p − 6q 12. 9r2 − 27rs + 24s2 by 3r − 5s

Answers

1. a+4 2. b−2 3. c+4

2 10
4. d−5 5. e−2− 6. 3f − 6 +
e+4 f +2
2 3
7. 2g − 3 + 8. 3h + 5 − 9. j + 2k
g−5 h+3
2q2 4s2
10. m + 3n 11. 2p + 2q + 12. 3r − 4s +
5p − 6q 3r − 5s
24 • Mathematics

Factorisation
Since 3 × 7 = 21, 3 and 7 are said to be the factors of 21.

Similarly, (x − 3) (x + 4) = x 2 + x − 12, so (x − 3) and (x + 4) are said to be the factors of


x 2 + x − 12.

An algebraic expression will not always have factors, for example x 2 + x + 1 does not
factorise.

Example

Factorise x 2 + 5x + 6

The idea is to write x 2 + 5x + 6 = ( )( )

The first term in the expression is x 2 which can only be x multiplied by x, so each bracket
begins with x, i.e., (x )(x )

The next term to consider is +6.

As this is a positive value it means that both the numbers are positive or both the num-
bers are negative. The sign in front of the ‘x’ term determines which is the correct sign.
In this case, both numbers are positive. So (x+ )(x+ )

The two value are those that multiply to give 6 but add to give 5, i.e., 2 and 3.

The answer is therefore (x + 2) (x + 3)

Example

Factorise x 2 − 11x + 10

Again the first term in each bracket is x. The last number is +10 so the sign of the ‘x’
term means that the factorisation is of the form (x− )(x− ).

The required numbers multiply to give 10 and add to give 11, i.e., 1 and 10.

So x 2 − 11x + 10 = (x − 1) (x − 10)

Example

Factorise x 2 − 4x − 12

The first term in each bracket is x.


Algebra • 25

As the last number is negative, it can only be the multiple of one positive and one
negative number, giving (x− )(x+ ).

The required numbers multiply to give −12 but add to give −4, i.e., +2 and −6

So x 2 − 4x − 12 = (x − 6) (x + 2)

Example

Multiply out the brackets: (x + y) (x − y)

(x + y) (x − y) = x 2 − xy + xy − y 2 = x 2 − y2

This means that x 2 − y2 factorises as (x + y) (x − y)

This is known as the difference of two squares

That is, the factors are (the square root of the first term plus the square root of the second)
and (the square root of the first term minus the square root of the second)

Example

Factorise 4a2 − 9b2


√ √
4a2 = 2a and 9b2 = 3b, so 4a2 − 9b2 = (2a + 3b) (2a − 3b)

Notice that
• When the number term is positive, the two numbers in the brackets are of the same
sign, and their total is the x coefficient;
• When the sign of the x term is negative, the two numbers are of opposite signs, so
that the coefficient of x is their difference.

Questions

Factorise the following where possible. If there are no factors say so.

1. x 2 + 4x + 3 2. x 2 + 3x + 2 3. x 2 + 13x + 12

4. x 2 − 3x + 2 5. x 2 − 7x + 12 6. x2 − x − 6

7. x 2 + 3x − 10 8. x 2 + 2x − 3 9. x2 + x + 1

10. x 2 + 5x − 14 11. x 2 + 3x − 2 12. x 2 + 7x + 12


26 • Mathematics

13. x 2 + x − 12 14. x 2 − 10x + 21 15. x 2 + 5x − 6

16. x 2 − 9x + 20 17. x 2 + 6x + 9 18. x 2 − 8x + 12

19. x 2 − 25 20. x 2 + 25

Answers

1. (x + 1)(x + 3) 2. (x + 2)(x + 1) 3. (x + 12)(x + 1)

4. (x − 2)(x − 1) 5. (x − 3)(x − 4) 6. (x − 3)(x + 2)

7. (x + 5)(x − 2) 8. (x − 2)(x − 1) 9. Cannot factorise

10. (x − 2)(x + 7) 11. Cannot factorise 12. (x + 3)(x + 4)

13. (x + 4)(x − 3) 14. (x − 3) (x − 7) 15. (x + 6)(x − 1)

16. (x − 5)(x − 4) 17. (x + 3)2 18. (x − 2) (x − 6)

19. (x + 5)(x − 5) 20. Cannot factorise

Example
 
Expand x 2 − 3x − 4 (2x − 5)

 2 
x − 3x − 4 (2x − 5) = 2x 3 − 6x 2 − 8x − 5x 2 + 15x + 20 = 2x 3 − 11x 2 + 7x + 20

Example

Simplify 2 (2x − y)2 − (3x + 2y) (x − 2y) − 4y 2

2 (2x − y)2 − (3x + 2y) (x − 2y) − 4y 2


= 2 (2x − y) (2x − y) − (3x + 2y) (x − 2y) − 4y 2
   
= 2 4x 2 − 4xy + y2 − 3x 2 − 4xy − 4y2 − 4y2
= 8x 2 − 8xy + 2y2 − 3x 2 + 4xy + 4y2 − 4y2
= 5x 2 − 4xy + 2y2
Algebra • 27

Questions
Simplify

1. (x + 3) (x − 4) + (x − 1) (x + 5) 2. (x + 6)(x − 4) − (x − 2)(x + 1)

3. (2x + 3) (x + 1) − 2x (x − 1) 4. 3x (2x − 3) − (3x − 1) (2x + 1)

5. (3x − 1)2 + (2x − 1)2 6. (2x + y)2 − (x − 2y)2

7. (3x + 1) (x + 3) − 3 (x + 1) (x − 4) 8. 5xy + (3x − 2y) (3x + 2y) −


(2x + 3y) (3x − 2y)

9. (x − 1) (x + 1) − (x + 2) (x − 2) + 10. 2 (2x + y) (x − 2y) −


(x + 5) (x − 5) (x + y) (x − y) − (2x − y)2

Answers

1. 2x 2 + 3x − 17 2. 3x − 22 3. 7x + 3 4. 1 − 10x

5. 13x 2 − 10x + 2 6. 3x 2 + 8xy − 3y2 7. 19x+ 15 8. 3x 2 + 2y2

9. x 2 − 22 10. −x 2 − 2xy − 4y2

Evaluation
Evaluation is the process of substituting the numerical values of the algebraic symbols
and working out the value of the whole expression.

The usual rules of arithmetic apply (BIDMAS is one way of remembering the order of
operations). Quantities in Brackets should be solved first, followed by Indices (powers
and roots). Next Division and Multiplication must be calculated before Addition and
Subtraction.

Example

Evaluate 3xy + x 2 − 4y when x = 2 and y = 3

3xy + x 2 − 4y = 3 × 2 × 3 + 22 − 4 × 3 = 18 + 4 − 12 = 10
28 • Mathematics

Example

Evaluate (a + b)2 − (c + d) + x 3 − y if a = 3, b = 4, c = 5, d = 6, x = −2, y = −3

(a + b)2 − (c + d) + x 3 − y = (3 + 4)2 − (5 + 6) − (−2)3 − (−3)


= 72 − 11 + (−8) − (−3)
= 49 − 11 − 8 + 3
= 33

This is much more relevant when the equations used are those that may occur in other
units.

Questions

w × v2
1. E= Find E if w = 120, v = 33 and g = 32.
2g

2Li
2. p= Find p if L = 230, i = 362 and t = 12. 6.
t

πd2
3. A= Find A if d = 71.
4

4. S = 4πR2 Find S if R = 18.

4
5. V = πR3 Find V if π = 3. 142 and R = 19.
3

l
6. T = 2π Find T if π = 3. 142, l = 21. 7 and g = 9. 81.
g

3V
7. R= 3
Find R if π = 3. 142 and V = 1046.
π

36VR
8. N= Find N if V = 85, R = 19, T = 14. 6 and P = 11. 53.
100TP

16T
9. d= 3
Find d if π = 3. 142, T = 136 and f = 52. 8.
πf

10. A = 2πr + πr2 Find A if π = 3. 142 and r = 5. 3.


Algebra • 29

Answers

1. 4083.75 2. 13215.9 3. 3595.2

4. 4071.50 5. 28730.9 6. 9.35

7. 10.0 8. 1.86 9. 2.36

10. 121.56

Questions
Force × Distance
1. Power =
Time
Evaluate the power when a force of 3760N raises an object a distance of 4.73 m in
35 s.
2. The potential difference, V volts, available at battery terminals is given by
V = E − i. R.
Evaluate V when E = 5. 62, i = 0. 70 and R = 4. 30.
m 2 
3. Given force F = v − u2 , find F when m = 18. 3, v = 12. 7 and u = 8. 24.
2
nE
4. The current, i amperes, flowing in a number of cells is given by i = .
R + nr
Evaluate the current when n = 56, E = 2. 20, R = 2. 80 and r = 0. 50.

L
5. The time, t seconds, of oscillation for a simple pendulum is given by t = 2π .
g
Determine the time when L = 54. 32 and g = 9. 81.
L. i2
6. Energy, E joules, is given by the formula E = .
2
Evaluate the energy when L = 5. 5 and i = 1. 2.
V
7. The current, I amperes, in an a.c. circuit is given by i =  .
R + X2
2

Evaluate the current when V = 250, R = 11. 0 and X = 16. 2.



8. The area, A, of any triangle is given by A = s (s − a) (s − b) (s − c) where
a+b+c
s= .
2
Evaluate the area when a = 3. 60 cm, b = 4. 00 cm and c = 5. 20 cm.
9. Given that a =0. 290, b = 14. 86, c = 0. 042, d = 31. 8 and e = 0. 650, evaluate v
a. b d
given that v = − .
c e
n
b
10. Calculate the value of X if X = T 1 + when T = b = n = 3/4.
100
30 • Mathematics

 
2
1
11. Calculate Z if Z = R2 + ωL −
ωC
when R = 24, L = 0. 30, ω = 352 and c = 6. 5 × 10−5 .
T
12. Calculate a where a =  
ku − Lv 2
2

and T = 17. 42, k = 0. 073, u = 5. 46, L = 0. 0890 and v = 3. 21.

Answers

1. Power = 508.1 W
2. V = 2. 61 v
3. F = 854. 5
4. I = 4 A
5. t = 14. 89 s
6. E = 3. 96 J
7. I = 12. 77 A
8. A = 7. 184 cm2
9. v = 7. 327
10. X = 0. 7542
11. Z = 66. 4
12. a = ±15. 52

Test Examples 2
1. (i) Add 3x + 4y − 5z and −2x − 5y + 4z
(ii) Add 2a2 b − ab + 3ab2 and 5ab2 − a2 b + ab
(iii) Subtract 2x + 5y − 3z from 5x − 4z + 3y
(iv) Subtract −4c − 5b − a from 3a + 6c − 2b

2. Simplify the following expressions by collecting like terms:


(i) 5x − 3z − 4x − 2y + 4y + 2z − y
(ii) 2. 5a + c − 1. 2a + 2. 5b − 3c + b
(iii) b2 − 3ab2 + 2a2 b − 4a2 − 2b2 + 5a2 − 2ab2
Algebra • 31

3. Simplify the following:


a × a3 × a5
(i)
a2 × a4
(ii) b × b−3 × b−2
5

(iii) c × c1/2 × c−1/3
 
(iv) d 4 e2
 3 √
(v) f 2 × f 1/2 × f −5
3

4. (i) Multiply (x + 2y) by (2x + y)


(ii) Multiply (2x + y) by (3x − 2y)
(iii) Multiply (3x − 4y) by (2x − 3y)

5. Work out the following:


(i) (a + b)2
(ii) (a − b)2
(iii) (a + b)3
(iv) (a − b)3
 
6. (i) Divide 8a2 − 8ab − 6b2 by (2a − 3b)
 
(ii) Divide 9x 3 − 9x 2 y − 10xy2 + 8y3 by (3x − 4y)
 
(iii) Divide x 3 − y3 by (x − y)

7. Simplify the following expressions by removing brackets and


collecting terms:
(i) (a + b) + (c − d) − (a − b) − (c + d) + (a − b)
(ii) 2 {a − 3 (a + 2) + 4 (2a − 1) + 5}
(iii) 2x − [2x − {2x − (2x − 2) − 2} − 2] − 2

8. Factorise the following expressions:


(i) 3b2 − 6b + 9
(ii) pv + pvx
(iii) ax 3 − 2bx 2 + 3cx
(iv) 12a3 b3 c3 − 8a2 b2 c2 + 4abc

9. Factorise the following expressions:


(i) D2 − d2
(ii) 1 − a2
32 • Mathematics

(iii) 4x 2 y2 − 9z2
(iv) T14 − T24

10. Factorise the following expressions:


(i) a2 + 8a + 16
(ii) d2 − 10d + 25
(iii) 9v 2 + 12v + 4
(iv) 4x 2 − 12xy + 9y2

11. Simplify the following expressions:


x x 2x
(i) + −
2 4 3
2a − 1 2a + 3 3a − 8
(ii) − −
b 2b 3b
4 18 3
(iii) + −
x + 2 x 2 − 2x − 8 x − 4

12. Find the value of: 2x 3 − x 2 y + xy2 − 3y3 when x = 2 and y = −2.
SIMPLE EQUATIONS
3
An equation is an expression consisting of two ‘sides’, one being equal in value to the
other. A simple equation contains one hidden value of the first order (e.g. x, and not x 2 ,
or x 3 , etc.) which is usually referred to as the unknown and to solve the equation means
to find the value of the unknown.

Since one side of the equation is always equal to the other side, it follows that if, in the
process of solving the equation, a change is made to one side of the equation the same
change must be made to the other side in order to maintain equality.

The most important thing to remember is that the equality of the equation must be
maintained.

This means if the values on the left-hand side of the equals sign are doubled then the
values on the right-hand side must also be doubled.

Any mathematical operation can be used, so long as the ‘balance’ of the equation is
maintained.

So equality will be maintained if

(i) the same quantity is added to both sides,


(ii) the same quantity is subtracted from both sides,
(iii) every term on both sides is multiplied by the same value,
(iv) every term on both sides is divided by the same value,
(v) both sides are raised to the same power,
(vi) the same root is taken of both sides.

Methods of approach

(i) eliminate fractions by multiplying all the terms by the LCM of the denominators,
(ii) remove brackets, following the rules of Algebra,
34 • Mathematics

(iii) place all the terms that involve the unknown on one side of the equation, and all
other terms on the other side of the equation,
(iv) collect and summarise terms on each side,
(v) find the value of the unknown.

This is a rough guide only.

Example

Solve the equation 6x = 24


6x 24
Dividing both sides of the equation by 6 gives =
6 6
Cancelling gives x = 4.

Every equation should be checked by substituting the answer into the original equation.

In this case, 6 × 4 = 24, so the answer is correct.

Example
2x
Solve the equation = 12
3
2x
Multiplying both sides by 3 gives × 3 = 12 × 3
3
Cancelling gives 2x = 36
2x 36
Dividing both sides by 2 gives =
2 2
Cancelling gives x = 18.
2 × 18 36
Check: = = 12. Therefore answer is correct.
3 3
Example

Solve the equation x − 8 = 3

Adding 8 to both sides gives x − 8 + 8 = 3 + 8

So, x = 11

Check: 11 − 8 = 3, so correct.

Example

Solve the equation x + 5 = 9


Simple Equations • 35

Subtracting 5 from both sides gives x + 5 − 5 = 9 − 5

So, x = 4

Check: 4 + 5 = 9, so correct.

Example

Solve the equation 4x + 10 = 18

Subtracting 10 from both sides gives 4x = 8

Dividing both sides by 4 gives x = 2

Check: 4 × 2 + 10 = 8 + 10 = 18, so correct.

Example

Solve the equation 5x + 6 = 3x + 12

In questions like this, all the terms involving the unknown quantity (in this case, x) must
be grouped on one side of the equals sign with everything else on the other side.

When this is done, changing from one side of an equals sign to the other must be
accompanied by a change of sign.

5x + 6 = 3x + 12
5x − 3x = 12 − 6
6
2x = 6 so x = =3
2
Check: (5 × 3) + 6 = 21, (3 × 3) + 12 = 21, so correct.

Example

Solve the equation 6 − 3x = 4x − 20

In order to keep the x terms positive, move the x’s to the right-hand side of the equals
sign, everything else to the left-hand side.

6 − 3x = 4x − 22
6 + 22 = 4x + 3x
28 = 7x
28
x= =4
7
Check: 6 − 3 × 4 = 6 − 12 = −6, 4 × 4 − 22 = 16 − 22 = −6, so correct.
36 • Mathematics

Example

Solve the equation 5 (x − 2) = 25

Removing the bracket gives 5x − 10 = 25

Rearranging gives 5x = 25 + 10
5x = 35
35
x= =7
5

Check: 5 (7 − 2) = 5 × 5 = 25, so correct.

Alternatively, divide both sides by 5, then add 2 to both sides.

Example

Solve the equation 3 (2x − 6) + 2 (x + 3) = 4 (x − 5)

Removing the brackets gives 6x − 18 + 2x + 6 = 4x − 20

Collecting like terms gives 6x + 2x − 4x = −20 + 18 − 6

Simplifying gives 4x = −8
−8
So x = = −2
4
Check: 3 (−4 − 6) + 2 (−2 + 3) = 3 × −10 + 2 × 1 = −30 + 2 = −28, 4 (−2 − 5)
= 4 × −7 = −28, so correct.

Example
3x + 2 5 (x − 1) 5x 5 − x
Solve the equation − = −
2 6 4 2
LCM of denominators is 12, so multiply throughout by 12 to give

       
3x + 2 5 (x − 1) 5x 5−x
12 − 12 = 12 − 12
2 6 4 2
6 (3x + 2) − 2 (5 (x − 1)) = 3 (5x) − 6 (5 − x)
(18x + 12) − (10x − 10) = 15x − 6 (5 − x)
18x + 12 − 10x + 10 = 15x − 30 + 6x
8x + 22 = 21x − 30
Simple Equations • 37

22 + 30 = 21x − 8x
52 = 13x
52
x=
13
=4

Example
9x 6
Solve the equation 8 − − = 2 (4 − 3x)
2 x
LCM of denominators is 2x, so multiplying every term by 2x gives
   
9x 6
2x (8) − 2x − 2x = 2x [2 (4 − 3x)]
2 x
16x − 9x 2 − 12 = 4x (4 − 3x)
16x − 9x 2 − 12 = 16x − 12x 2
12x 2 − 9x 2 = 12
3x 2 = 12
x2 = 4

x= 4
x = ±2

Note: the answer can be +2 or −2 as (+2)2 = 4 and (−2)2 = 4.

So the square root of a number always has two values, one positive, the other negative;
this is written using the symbol ±.

There is never just one way to solve an equation. Consider the following:

3x 2 + 10x + 3 x 2 − 4x + 3
=
x+3 x−1

The numerators look as though they might factorise and, if this is the case, the denom-
inators may cancel with one of the factors. This is worth trying as the result is a much
simpler equation to solve.

3x 2 + 10x + 3 x 2 − 4x + 3
=
x+3 x−1
38 • Mathematics

(x + 3) (3x + 1) (x − 1) (x − 3)
=
x+3 x−1
3x + 1 = x − 3
3x − x = −3 − 1
2x = −4
x = −2

Questions

Solve the following equations:

1. 2x + 5 = 7 2. 8 − 3x = 2

2
3. x−1= 3 4. 2x − 1 = 5x + 11
3

5. 7 − 4x = 2x − 3 6. 2. 6x − 1. 3 = 0. 9x + 0. 4

7. 2x + 6 − 5x = 0 8. 3x − 2 − 5x = 2x − 4

9. 20x − 3 + 3x = 11x + 5 − 8 10. 2 (x − 1) = 4

11. 16 = 4 (x + 2) 12. 5 (x − 2) − 3 (2x + 5) + 15 = 0

13. 2x = 4 (x − 3) 14. 6 (2 − 3x) − 42 = −2 (x − 1)

15. 2 (3x − 5) − 5 = 0 16. 4 (3x + 1) = 7 (x + 4) − 2 (x + 5)

17. 10 + 3 (x − 7) = 16 − (x + 2) 18. 8 + 4 (x − 1) − 5 (x − 3) = 2 (5 − 2x)

Answers

1. x = 1 2. x = 2 3. x = 6 4. x = −4 5. x = 1 23 6. x = 1

7. x = 2 8. x = 1
2 9. x = 0 10. x = 3 11. x = 2 12. x = −10

13. x = 6 14. x = −2 15. x = 2 12 16. x = 2 17. x = 6 14 18. x = −3


Simple Equations • 39

Further Examples on Simple Equations


Example
3 4
Solve the equation =
x 5
The lowest common multiple (LCM) of the denominators is 5x, so both sides of the
equation are multiplied by 5x.
3 4
× 5x = × 5x
x 5
15 = 4x
15 3
x= =3
4 4
3 4 12 4
Check: =3× = = , so correct.
15/4 15 15 5
Example
2x 3 1 3x
Solve the equation + +5= −
5 4 20 2
The LCM of the denominators is 20 so multiplying every term by 20 gives
2x 3 1 3x
× 20 + × 20 + 5 × 20 = × 20 − × 20
5 4 20 2
8x + 15 + 100 = 1 − 30x
8x + 30x = 1 − 15 − 100
38x = −114
−114
= −3x=
38
2 × −3 3 −6 3 11 1 3 × −3 1 −9 1 9 11
Check: + +5 = + +5 = 4 , − = − = + =4 ,
5 4 5 4 20 20 2 20 2 20 2 20
so correct.

Example
3 4
Solve the equation =
x − 2 3x + 4
Cross multiplying gives 3 (3x + 4) = 4 (x − 2)

Removing the brackets 9x + 12 = 4x − 8


40 • Mathematics

So 9x − 4x = −8 − 12

Giving 5x = −20
−20
So x = = −4
5
3 3 1 4 4 4 1
Check: = =− , = = = − , so correct.
−4 − 2 −6 2 3 × −4 + 4 −12 + 4 −8 2
Example

Solve the equation 2 x = 8

2 x 8
Dividing both sides by 2 gives =
2 2

So x = 4
√ 2
Squaring both sides gives x = 42

So x = 16

Check: 2 16 = 2 × 4 = 8, so correct.

Example

x+3
Solve the equation √ =2
x

To remove the fraction both sides are multiplied by x

x+3 √ √
So √ × x =2× x
x
√ √
Giving x + 3 = 2 x
√ √ √
Rearranging gives 3 = 2 x − x = x

Squaring both sides gives x = 9



9+3 3+3 6
Check: √ = = = 2, so correct.
9 3 3
Example
2 16
Solve the equation =
9 3x 2
Multiply both sides by the LCM which is 9x 2
2 16
So × 9x 2 = 2 × 9x 2
9 3x
Simple Equations • 41

So 2x 2 = 16 × 3 = 48

Dividing by 2 gives x 2 = 24

So x = 24 = ±4. 889 to 3 decimal places
16 16 16 2
Check: = = = , so correct.
3 × (4. 889)2 3 × 24 48 9

Summary of the Important Points


1. The equality of an equation must be maintained at every step.
2. Fractions are best removed. This is done by multiplying both sides by the lowest
common multiple of the denominators.
3. Brackets should be removed at the earliest possible opportunity.
4. The unknown terms should be grouped on one side of the equals sign and every-
thing else on the other side.
5. When the square root of the unknown is involved it is best to isolate that term on
one side of the equals sign, and then to square both sides.
6. When the square of the unknown is involved it is best to isolate that term on one
side of the equals sign, and then to take the square root of both sides.
• When taking the square root there are always two answers, one positive and the
other negative.

7. All solutions should be checked.


8. Do not try to take more than one step at a time.

Questions
Solve the following equations:

1 3 2 5
1. x+3=4 2. 2+ x =1+ x+
5 4 3 6

1 1 1 1 2
3. (2x − 1) + 3 = 4. (2x − 3) + (x − 4) + =0
4 2 5 6 15
42 • Mathematics

1 1 x x
5. (3x − 6) − (5x + 4) 6. − =2
5 4 3 5
1
+ (2x − 9) = −3
5
x x x 2 3
7. 1− =3+ − 8. =
3 3 6 x 8

1 1 7 x+3 x−3
9. + = 10. = +2
3x 4x 24 4 5

3x 6 − x 2x 3 x 7 5−y
11. = + − 12. + =
20 12 15 2 5 20 4

x−2 3 2 3
13. = 14. =
2x − 3 2x + 1 x − 3 2x + 1

x x+6 x+3 √
15. − = 16. 3 x=9
4 5 2
 
√ 3
17. 2 x=5 18. 4= +3
x
√ 
3 x x
19. √ = −6 20. 10 = 5 −1
1− x 2
 
x2 x+2 1
21. 16 = 22. =
9 x−2 2

6 2x 11 8
23. = 24. =5+ 2
x 3 2 x

Answers

1. x = 5 2. x = −2 3. x = −4 12 4. x = 2 5. x = 12 6. x = 15

7. x = −4 8. x = 5 13 9. x = 2 10. x = 13 11. x = −10 12. x = 2

13. x = 3 14. x = −11 15. x = −6 16. x = 9 17. x = 6 14 18. x = 3

19. x = 4 20. x = 10 21. x = ±12 22. x = −3 13 23. x = ±3 24. x = ±4


Simple Equations • 43

Practical Problems Involving Simple


Equations
Example

A copper wire has a length, L, of 1.5 km, a resistance, R, of 5Ω and a resistivity, ρ, of


17. 2 × 10−6 Ω mm
ρL
Find the cross-sectional area, A, of the wire, given that R = .
A
Solution
ρL
Since R = then
A
       
17. 2 × 10−6 Ωmm × 1. 5 × 103 m 17. 2 × 10−6 Ωmm × 1. 5 × 106 mm
5Ω = =
A A
From these units A is measured in mm2 .

So
   
5A = 17. 2 × 10−6 × 1. 5 × 106 giving
   
17. 2 × 10−6 × 1. 5 × 106 25. 8 × 100 25. 8
A= = = = 5. 16 mm2 .
5 5 5
 
Remember 100 = 1

The cross-sectional area A is 5.16 mm2 .

Example

The temperature coefficient of resistance, α, may be calculated from the formula

Rt = R0 (1 + αt) where t is the time in seconds, R0 , is the initial resistance and Rt is the
resistance after t seconds.

Find α given that Rt = 0. 928, Ro = 0. 8 and t = 40.

Solution

Since Rt = R0 (1 + αt) then 0. 928 = 0. 8 (1 + α (40))

So 0. 928 = 0. 8 + (0. 8) α (40)


0. 928 − 0. 8 = 32α
44 • Mathematics

0. 128 = 32α
0. 128
α= = 0. 004
32
The temperature coefficient of resistance is 0.004.

Example
1
The distance s metres travelled in t seconds is given by the formula s = ut + at2 , where
2
u is the initial velocity in m/s and a is the acceleration in m/s2 . Find the acceleration of
the body if it travels 168 m in 6 s, with an initial velocity of 10 m/s.

Solution
1
s = ut + at2 where s = 168, u = 10 and t = 6.
2
1 36
So 168 = (10) (6) + a (6)2 → 168 = 60 + a → 168 = 60 + 18a
2 2
108
18a = 168 − 60 → 18a = 108 → a = =6
18
So the acceleration is 6 m/s2 .

Example

When three springs are joined in series the total stiffness, KT , is given by
1 1 1 1
= + + where K1 is the stiffness of the first spring etc.
KT K1 K2 K3
Find the total stiffness if K1 = 2 kN/m, K2 = 3 kN/m and K3 = 5 kN/m.

Solution
1 1 1 1 1 1 1 1
= + + so = + +
KT K1 K2 K3 KT 2 3 5
The LCM of all the denominators is (KT ) (2) (3) (5) = 30KT so multiplying by the LCM gives
KT = 1. 43 kN/m (to 3 significant figures).

Questions
ρL
1. A formula used for calculating resistance of a cable is R = .
A
Given that R = 1. 25, L = 2500 and A = 2 × 10−4 find the values of ρ.
2. Force F newtons is given by F = ma, where m is the mass in kg and a is the accel-
eration in m/s2 . Find the acceleration when a force of 4 kN is applied to a mass of
500 kg.
Simple Equations • 45

3. PV = Mrt is the characteristic gas equation. Find the value of m when P = 100 × 103 ,
V = 3. 00, R = 288 and T = 300.
4. When three resistors are connected in parallel the total resistance RT is given by
1 1 1 1
= + + where R1 is the resistance of the first resistor etc.
RT R1 R2 R3
(a) Find the total resistance when R1 = 3Ω, R2 = 6Ω and R3 = 18Ω.
(b) Find the value of R3 when RT = 3Ω, R1 = 5Ω and R2 = 10Ω.
V
5. Ohm’s law may be represented by I = , where I is the current in amperes, V is the
R
voltage and R is the resistance in ohms. A soldering iron takes a current of 0.30 A
from a 240 V supply. Find the resistance of the soldering iron.

Answers

1. 10−7 2. 8 m/s2 3. 3.472 4. (a) 1.8 Ω, (b) 30 Ω 5. 800 Ω

Further Problems
Example

The extension, x metres, of an aluminium tie bar of length L metres and cross-sectional
area A m2 when carrying a load of F newtons is given by the modulus of elasticity
FL
E= .
Ax
Find the extension of the tie bar (in mm) if E = 70 × 109 N/m2 , F = 20 × 106 N, A = 0. 1
m2 and L = 1. 4 m.

Solution
 
FL 20 × 106 N × (1. 4m)
E= , so 70 × 109N/m2 = ,
Ax (0. 1) (x)

(the unit of x is therefore the metre)


 
70 × 109 × 0. 1 × (x) = 20 × 106 × (1. 4)
 
20 × 106 × (1. 4) 28 × 106
x= = = 4 × 10−3
70 × 109 × 0. 1 7 × 109

The extension of the tie bar is 4 × 10−3 m = 4 mm.


46 • Mathematics

Example
Power in a d.c. circuit is given by P = V 2 /R where V is the supply voltage and R is the
circuit resistance. Find the supply voltage if the circuit resistance is 1.25 Ω and the power
measured is 320 W.

Solution
V2 V2
Since P = then 320 =
R 1. 25

(320) (1. 25) = V 2


i. e. V 2 = 400

V = 400 = ±20 volts

Supply voltage = ±20 volts.

Example

A formula relating initial and final pressures P1 and P2 , volumes V1 and V2 and tem-
P1 · V1 P2 · V2
peratures T1 and T2 of an ideal gas is = . Find the value of P3 given
T1 T2
P1 = 100 × 10 , V1 = 1. 0, V2 = 0. 266, T1 = 423 and T2 = 293.
3

Solution
 
P1 · V1 P2 · V2 100 × 103 · (1. 0) P2 · (0. 266)
Since = then =
T1 T2 423 293
Rearranging gives

(100 × 103) · (1. 0) × (293) = P2 · (0. 266) × (423)


 
100 × 103 · (1. 0) × (293)
P2 =
(0. 266) × (423)
29300 × 103
P2 = = 260. 4 × 103 = 2. 604 × 105
112. . 518

To 2 significant figures P2 = 2. 6 × 105 or 260000.

Example  
D f +p
The stress f in a material of a thick cylinder can be obtained from = .
d f −p
Calculate the stress given that D = 21. 5, d = 10. 75 and p = 1800.
Simple Equations • 47

Solution
   
D f +p 21. 5 f + 1800
Since = then =
d f −p 10. 75 f − 1800
 
f + 1800
i.e. 2 = .
f − 1800

Squaring both sides gives:


f + 1800
4=
f − 1800
4 (f − 1800) = f + 1800
4f − 7200 = f + 1800
4f − f = 1800 + 7200
3f = 9000
f = 3000

So stress f = 3000.

Questions

1. A rectangle has a length of 20 cm and a width b cm.


When its width is reduced by 4 cm its area becomes 160 cm2 .
Find the original width and area of the rectangle.
2. Given R2 = R1 (1 + αt), find α given R1 = 5. 0, R2 = 6. 03 and t = 51. 5.
3. If v 2 = u2 + 2as, find u given that v = 24, a = −40 and s = 4. 05.
4. The relationship between the temperature on a Fahrenheit scale and a Celsius scale
is F = 95 C + 32.
Express 113◦F in degrees Celsius.

w
5. If T = 2π , find the value of S given that w = 1. 219, g = 9. 81 and t = 0. 3132.
Sg
6. An alloy contains 60% by weight of copper, the remainder being zinc. How much
copper must be mixed with 50 kg of alloy to give an alloy containing 75% copper?
7. A rectangular lab has a length equal to one and a half times its width and a perimeter
of 40 m. Find its length and width.

Answers

1. 12 cm, 240 cm2 2. 0.004 3. 30 4. 45◦ C 5. 50 6. 30 kg 7. 12 m, 8 m


48 • Mathematics

Problems Involving Simple Equations


From the facts given in the problem and what is required to be found, the first task is to
make up a straightforward equation.

Make a sketch of the problem if possible and show all the given data on the sketch,
indicating the unknown quantity to be found. If a sketch is not practical, it may help to
write down the given quantities.

It helps to give the unknown quantity a symbol that is representative of the quantity:
for example, ‘let t be the time taken, in seconds’, ‘let m be the mass in kg’.

Example

A port (left) tank and a starboard (right) tank, each of 200 tonne capacity, are each half
full with oil. Find what mass of oil must be pumped out of the port tank and into the
starboard tank so that there will be four times the mass of oil in one tank as in the other.

Port Starboard

Capacity M tonne
of each pumped in
tank 200
tonnes
M tonne taken out 100 + M tonne
Half full finally in tank
100 tonnes
100 – M tonne left

Initial mass in each tank = 100 tonne

Let M be the mass of oil pumped over, then

Final mass of oil in port tank = 100 − M tonne

Final mass in starboard tank = 100 + M tonne


Simple Equations • 49

Mass in starboard tank is four times that of mass in port tank, so

100 + M = 4 (100 − M)
100 + M = 400 − 4M
M + 4M = 400 − 100
5M = 300
M = 60

Therefore, mass to be transferred = 60 tonne.

This will leave 40 tonne in the port tank and have the starboard tank holding 160 tonne
(four times as much).

Example

An electric train leaves station A bound for station B at the same instant that a diesel
train leaves B for A on parallel lines along the same route. The speed of the electric train
is 80 km/h, that of the diesel train 64 km/h and the distance between stations is 72 km.
Find the distance from A where the trains will pass each other.
Passing point
Electric train C Diesel train
80 km/h 64 km/h
A B
x km 72 – x km

72 km

Let x = distance of passing place from A, then (72 − x) = distance of passing place
from B.

As the trains leave at the same instant, the time taken to reach the passing point must
be the same for each train.
⎫ ⎧
⎬ ⎨
Time for electric train Time for diesel train
=
to travel x km ⎭ ⎩ to travel (72 − x) km
Distance
Substituting, Time = for each train
Speed
distance travelled by electric train distance travelled by diesel train
=
speed of electric train speed of diesel train
x 72 − x
=
80 64
LCM = 320, multiplying both sides by 320
50 • Mathematics

4x = 5 (72 − x)
4x = 360 − 5x
4x + 5x = 360
9x = 360
x = 40

Therefore the trains pass each other 40 km from A.

Example

A ship travels upriver against the current for a distance of 25 nautical miles and then
downriver with the current over the same distance, taking 3.5 h for the double journey.
If the normal speed of the ship in still water is 15 knots, find the speed of the current.
Let x = speed of current (knot)

speed of ship against current = (15 − x) knot


then, speed of ship running with cuirrent = (15 + x) knot

Now time to go upriver + time to go downriver = total time


distance upriver distance downriver
So = = 3. 5 h
speed upriver speed downriver
25 25
Therefore + = 3. 5
(15 − x) (15 + x)
Multiply both sides by the LCM of the denominators, that is, by (15 − x)(15 + x).
25
(15 − x) (15 + x)
(15 − x)
25
+ (15 − x) (15 + x) = 3. 5 [(15 − x) (15 + x)]
(15 + x)
  ←−
25 (15 + x) + 25 (15 − x) = 3. 5 152 − x 2 ←− {difference of two squares}
 
375 + 25x + 375 − 25x = 3. 5 152 − x 2
750 = 787. 5 − 3. 5x 2
3. 5x 2 = 787. 5 − 750
3. 5x 2 = 37. 5

37. 5 37. 5
This gives x =
2 and so x = = ±3. 273
3. 5 3. 5
Therefore speed of current is 3.273 knots.
Simple Equations • 51

Transposition of Formulae
A formula is a group of algebraic symbols which expresses a rule for finding the value of
one of the symbols when values are substituted for the others.

To transpose means to change the order of the symbols to produce a re-arranged


equation giving the expression for one of the other symbols in terms of the rest.

The rules for the transposition of formulae are the same as those used for the solution
of simple equations.

Basically, the equality of the formula must be maintained.

In order to transpose a formula it is necessary to use inverse operations.

An inverse operation is one where, when it is applied immediately after some mathe-
matical operation, the expression remains unchanged.

Example

if the mathematical operation is ‘multiply by 6’ the inverse operation is ‘divide by 6’


6X
because (X × 6) ÷ 6 = = X.
6
So the inverse of ‘multiply’ is ‘divide’ and naturally, the inverse of ‘divide’ is ‘multiply’.

The following table gives some of the operations and their inverses:

Operation Inverse
Addition Subtraction (and vice versa)
Multiplication Division (and vice versa)
Square Square root (and vice versa)
Cube Cube root etc. (and vice versa)
Invert (turn upside down) Invert
Make negative Make negative

at2
Consider the formula S = ut + and what is more, the values of u, t and a are known.
2
It is required that this formula is transposed so ‘a’ is the new subject.
52 • Mathematics

Imagine the steps taken on a calculator to work out the value of s, starting with the
value of a (the letter that is to be the new subject).

Once the steps have been established all that is needed is the opposite operations to
be used working in reverse. The diagram should explain.

Start with a Multiply by t 2 Divide by 2 Add ut Result S

Result a Divide by t 2 Multiply by 2 Subtract ut Start with S

All that is needed is for the instructions to be carried out

• Start with S S
• Subtract ut S − ut
• Multiply by 2 2(S − ut)
2 (S − ut)
• Divide by t2
t2
2 (S − ut)
• Result a=
t2
It must be pointed out there is a slight problem with algebraic expressions: namely,
there is often more than one way of writing the same formula. It looks different but
would give the same answer when numbers were substituted for the symbols.
2 (S − ut)
Consider the answer found above: a =
t2
2 (S − ut) 2S − 2ut
Now = (multiply out the bracket)
t2 t2
2S − 2ut 2S 2ut
= 2 − 2 (split into two fractions with a common denominator)
t2 t t
2S 2u
− (cancel top and bottom of second fraction by ‘t’)
t2 t
2 (S − ut) 2S 2u
So either a = 2
or a = 2 − , and BOTH are equally correct.
t t t
Here is the problem: suppose this question was set in one of the exercises to be done
2S 2u
and, furthermore, you had arrived at the answer a = 2 − . However, on checking
t t
2 (S − ut)
the answers, the solution was given as a = . Unless you can show the two
t2
expressions mean the same, the natural conclusion is that you have made a mistake. If
in any doubt, ask for clarification.

Alternatively, the following sections give examples of the different types of formula that
may occur and how they can be transposed.
Simple Equations • 53

Type 1: Symbols connected by plus or minus signs

Example 1

Transpose c = d + e + f to make e the subject of the formula.

The aim is to end up with e by itself on the left-hand side of the equals sign.

Changing the equation around so that e is on the LHS gives

d+e+f =c

Subtracting d from both sides gives

d+e+f −d = c−d so e+f =c−d

Subtracting f from both sides gives

e+f −f = c−d−f so e= c−d−f

As with simple equations, if a term changes from one side of the equals sign to the other
either by addition or subtraction, there is an appropriate sign change. So directly

d+e+f =c gives e = c−d−f

Example 2

If 2c + 3d = f − g + 4h, make g the subject.

In this case the subject is negative, 2c + 3d = f − g + 4h

A negative term is always going to cause confusion so it is best to make the term
positive, and the easiest way is to move the term to the other side of the equals sign.

Therefore 2c + 3d + g = f + 4h

It is now a simple task to remove 2c and 3d from the left-hand side by subtraction, giving

g = f + 4h − 2c − 3d

Type 2: Formulae involving multiplications

Example 3

Transpose a = ω · r to make ω the subject.


54 • Mathematics

a ω·r a
Dividing both sides by r gives = so ω =
r r r
Example 4

Suppose (a + b)2 = 10cd

Express this formula with d as the subject.

In reality this is no different from the previous example: two terms d and 10c are mul-
tiplied together to make a third term (a + b)2 , so dividing both sideds by 10c gives
(a + b)2
= d.
10c
(a + b)2
Therefore d = .
10c

Type 3: Formulae involving fractions (or divisions)

Example 5
V
If Z = , rearrange to make V the subject.
R
Multiplying both sides by R gives V = Z · R

Example 6
V
If Z = , rearrange to make R the subject.
R
Multiply both sides by R, as in Example 5, so V = Z · R then divide both sides by Z.
V
Therefore R =
Z
Example 7
αV
Transpose the formula Z = to make (a) R the subject; (b) V the subject
R
The only difference between this and the previous examples is the introduction of a
multiplying constant, α.

(a) Multiplying both sides by R gives RZ = αV


αV
Dividing by Z gives R = .
Z
(b) From part (a): αV = RZ
Simple Equations • 55

RZ
Dividing by α gives V = .
α
Example 8
at2
If s = ut + transpose the equation to make a the subject
2
at2
ut + =s
2
at2
Moving ut gives: = s − ut
2
 2
at
Multiplying by 2 gives: 2 = 2 (s − ut) therefore at2 = 2 (s − ut)
2
at2 2 (s − ut) 2 (s − ut)
And dividing by t2 gives: = therefore a =
t2 t2 t2

Questions
Make the symbol shown in brackets the subject of the formula shown.

Express each answer in its simplest form.

1. a+b =c−d (d) 2. x + 3y = t (y)

3. c = 2πr (r) 4. y = mx + c (x)

E
5. i = PRT (T) 6. i= (R)
R
a 9
7. S= (r) 8. F = C + 32 (C)
1−r 5

Answers
t−x
1. d = c−a−b 2. y=
3
c y−c
3. 4. x=
2π m
i E
5. T= 6. R=
PR i
S−a a 5
7. R= or R = 1 − 8. C = (F − 32)
S S 9
56 • Mathematics

Type 4: Formulae with the new subject in a bracket

Example 9

Transpose the equation R = R0 (1 + αt) to make α the subject.

It is probably best to isolate the bracket on the LHS.


R0 (1 + αt) R
R0 (1 + αt) = R so =
R0 R0

R R
Therefore (1 + αt) = → 1 + αt =
R0 R0
R
1 + αt =
R0
R R − R0
αt = −1 → αt =
R0 R0
αt R − R0 R − R0
= → α=
t R0 t R0 t
R−S
So α =
St
Example 10
(u + v)
Transpose s = · t to make v the subject
2
(u + v)
·t = s
2
(u + v)
Multiplying by 2 gives: 2 . t = 2s → (u + v) . t = 2s
2
(u + v) 2s 2s
Dividing by t gives: .t = → u+v =
t t t
2s 2s
Subtracting u gives: u+v−u = −u → v = −u
t t
2s
So v = −u
t

Type 5: Formulae with powers and roots

Example 11
mv 2
Transpose k = to make v the subject.
2
Simple Equations • 57



mv 2 mv 2
=k → 2 = 2k → mv 2 = 2k
2 2
mv 2 2k 2k
mv 2 = 2k → = → v2 =
m m m
   
2k   2k 2k
v2 = → v2 = → v=
m m m
 
2k
So v =
m

Example 12

Pythagoras Theorem states:

‘In a right-angled triangle, the square on the hypotenuse is equal to the sum of the squares
on the other two sides.’

This can be expressed algebraically as

c
b

B C
a

AB2 = AC2 + BC2 or c2 = a2 + b2

Transpose Pythagoras Theorem to make b the subject

a2 + b2 = c2 → a2 + b2 − a2 = c2 − a2 → b2 = c2 − a2
       
b2 = c2 − a2 → b2 = c2 − a2 → b= c2 − a2 So b = c2 − a2

Example 13

If a simple pendulum consisting of a mass, m kg, attached to a mass-less string of length,


L metres,is allowed to swing freely the periodic time, T seconds, is given by the formula
L
T = 2π , where g is the acceleration due to gravity. Transpose the formula to make L
g
the subject:

L
 2π 
L g T L T
2π =T → = → =
g 2π 2π g 2π
58 • Mathematics

  2  2 

L T L T L T 2
= → = → =
g 2π g 2π g 2π
 2  2  2
L T L T T
= → g× =g× → L=g
g 2π g 2π 2π
 2
T gt2
So L = g or L =
2π 4π 2

Questions
Make the symbol shown in brackets the subject of the formula shown.

Express each answer in its simplest form.

b (c + d)
1. a= (c)
d
5L2
2. y3 = (P)
PN
1 1 1
3. = + (R2 )
R R1 R2
π 3
4. V= (D − d3 ) (d)
6
a2 b2
5. + =1 (x)
x 2 y2
r2 . θ
6. A= (θ )
2
πR2 θ
7. A= (R)
360

8. Z = R2 + (2πfL)2 (f )

Answers

d ad
1. c= (a − b) or c = d −
b b

5L2
2. P=
y3 N

R · R1
3. R2 =
R1 − R
Simple Equations • 59


6V
4. d= 3
D3 −
π

2A
6. θ=
r2
 
360A
7. R=
πθ

Z 2 − R2
8. f=
2πL

Type 6: Formulae where factorisation is needed

Example 14
2b3 x + 3b3 y
Transpose the formula F = to make b the subject
t
 3 
2b3 x + 3b3 y 2b x + 3b3 y
=F → · t = Ft → 2b3 x + 3b3 y = Ft
t t
2b3 x + 3b3 y = Ft → b3 (2x + 3y) = Ft
b3 (2x + 3y) Ft
b3 (2x + 3y) = Ft → = Ft → b3 =
(2x + 3y) (2x + 3y)
 
Ft √
3 3 Ft Ft
b3 = → b = 3 → b= 3
(2x + 3y) (2x + 3y) (2x + 3y)

Ft
So b = 3
(2x + 3y)

Example 15
a−1
If t = √ , make m the subject of the formula
mx + 2m

a−1 a−1 √
√ =t → √ × mx + 2m
mx + 2m mx + 2m
√ √
=t× mx + 2m → a − 1 = t mx + 2m

√ t mx + 2m a − 1 √ a−1
t mx + 2m = a − 1 → = → mx + 2m =
t t y
60 • Mathematics

√  2  2
2 a−1 a−1 (a − 1)2
mx + 2m = → mx + 2m = → m(x + 2) =
t t t2

(a − 1)2
So m =
t2 (x + 2)

Type 7: Formulae where the new subject appears in more than one term

Example 16
y
If x = , make y the subject
1+y
The only way to transpose for a subject that appears more than once in the formula is
to rearrange the terms so that all the terms that contain the repetition are on one side
of the equals sign and that all the terms that do not contain the repeated subject are on
the other side of the equals sign. The repetition is then factorised out, so it appears to
be written only once. Then proceed as usual.
y
=x → y = x(1 + y) → y = x + xy
1+y
y = x + xy → y − xy = x → 1y − xy = x → y (1 − x) = x
x
So y =
(1 − x)
Example 17
1 4K − 2b
If = , make K the subject
x 3c + 5K

1 4K − 2b
= → 1(3c + 5K) = (4K − 2b)x → 3c + 5K = 4Kx − 2b
x 3c + 5K
→ 3c + 2b = 4Kx − 5K → 3c + 2b = K(4x − 5)
3c + 2b
So K =
(4x − 5)
Example 18

Transpose the following expression to give the value of i in terms of P and t.

Find the value of i when P = 50 and t = 4


100
P=
(1 + i)t
Simple Equations • 61

Therefore P (1 + i)t = 100

100
(1 + i)t =
P

t 100
1+i= so
P
  1
t 100 100 /t
i= − 1 or i = −1
P P

100 √
Inserting the numerical values gives i = 4 − 1 = 4 2 − 1 = 0. 1892
50
Example 19

An adiabatic process is one where heat does not enter or leave a system and is typified
by equations of the type p1 · V1n = p2 · V2n

Transpose the expression to give V1 in terms of the other quantities.

Find the value of V1 when p1 = 1. 035, p2 = 4. 14, V2 = 0. 5 and n = 1. 3

p1 · V1n = p2 · V2n
p2 · V2n
V1n =
p
 1
p2 · V2n
so V1 = n
p1

p2 
V1 = n × n V2n
p1 
p2
V1 = V2 × n
p1
 1
p2 n
or V1 = V2 ×
p1
Substituting the values gives

  1
4. 14 1. 3
V1 = 0. 5 × = 0. 5 × 40.769 = 1. 452
1. 035
correct to 3 decimal places.

Questions

Transpose each equation so that the symbol indicated is the subject, and express the
answers in their simplest form.
62 • Mathematics

1. A = π · r2 (r)

4π · r3
2. V= (r)
3

r2 + R2
3. S = 4πR (r)
2

uv
4. f= (u) (the subject appears more than once)
u+v

5. d = 2 h (2r − h) (r)
 
πh 3R2 + h2
6. V= (R)
6

iR
7. v= (i)
E − iR

 2gh
8. q = A
  2 (A)
 A
−1
α

Answers

A
1. r=
π

3V
2. r=

    
S 2 2S2 S2
3. r= 2 − R2 ⇒ r= − R2 ⇒ r= − R2
4πR 16π 2 R2 8π 2 R2

fv
4. u=
f −v

d2 + 4h2
5. r=
8h

6V − πh3
6. R=
3πh
Simple Equations • 63

vE
7. i=
R (1 + v)


8. A= 
q − 2ghα 2
2

More practise

Questions

Make the symbol(s) indicated the subject of the formulae, and express the answers in
their simplest form.

N−M P + 3Q x
1. R= (D) 2. T = a + bN2 (N) 3. = (Q)
D Q − 3P y

m 2 P−p
4. v2 = u2 − 2as (s,u) 5. E= v − u2 (u) 6. e= (p)
2g PT − pt
    
W v2 2 3h
7. T −W = (W) 8. V = π r2 h + r (h) 9. v= gd 1 + (h)
gx 3 d
  
P h2 + k2 t−k
10. d= 3
(Q) 11. T = 2π (k) 12. e= (t)
Q−P 2gh k (1 + kt)

m2 − 2p2 2 (S − an) 
13. n= (p) 14. d= (a) 15. C = 2 2hr − h2 (r)
2m2 + p2 n(n − L)
 √ 
nsh s2 Mn2 4L2 + b2 r2
16. C= R2 − (R) 17. x= (b) 18. P= ω 2 M2 − ()
6 4 K 2π L
   
D f +p V ir
19. = (p) 20. n= 1− (V) 21. Z= r2 + (2π fL)2 (f)
d f −p ZpΦ V
 
1 1 R2 2V
22. V = i r2 + (C) 23. ω= − (R) 24. i=  r (r)
(ωC)2 LC 4L2 R+ L
n

Answers

T −a P(y + 3x)
1. M = N − RD 2. N=± 3. Q=
b x − 3y

u2 − v 2 √ 2Eg P (1 − eT)
4. s= , u = ± v2 + 2as 5. u = ± v2 − 6. p=
2a m 1 − et
64 • Mathematics

Tgx V 2 v2 − gd
7. W= 8. h= − r 9. h=
gx + v2 π r2 3 3g
 
P T 2 gh k 1 + e2
10. Q = 3 +P 11. k=± − h2 12. t=
d 2π 2 1 − e2 k2

1 − 2n2 s d (n − L) c2 + 4h2
13. p = ±m 14. a= − 15. r=
2 + n2 h 2 8h
 
36c2 s2 K 2 x2 1√ 2 2 2 2
16. R= + 17. b= − 4L2 18. ω= 4π L P + r
n2 s2 h2 4 M2 n4 M
 
f D2 − d 2 Z 2 − r2
19. p= 20. V = ir + nZpΦ 21. f=
D2 + d 2 4π 2 L2
 
i 1 4L − 4ω2 L2 C 2 2Vn nR
22. C= 23. R= 24. r= −
ω V 2 − i2 r 2 C iL L

As it is an essential skill, more practice questions are provided.

Questions
 

 4G
 K+
 3
1. v =
ρ
Make G the subject of the formula.

2. P = f (b − nd) t

Make f the subject of the formula.

6M
3. f =
bd2
Make d the subject of the formula.

WL
4. E =
Ax
Make x the subject of the formula.

Rt θ − θO
5. =
R θ − θd
Make θ the subject of the formula.
 a
6. p+ (V − b) = RT
v2
Make p the subject of the formula.
Simple Equations • 65

1 m 1
7. = +
Y μ YG
Make YG the subject of the equation.

πPr 4 t
8. η =
8vL
Make v the subject of the formula.

μ CZ 4 λ5/2 N
9. =
ρ A
Make λ the subject of the formula.
 
(M + 3m) L
10. T = 4π
3 (M + 2m) g
Make m the subject.

y+k
11. p =
y k
+
p1 p2
Make y the subject.

πh  2 
12. Make D2 the subject of the equation L = 2
D2 − D21 .
4d

Answers

3 2  P
1. G= v ρ −K 2. f=
4 (b − nd) t

6M WL
3. d= 4. x=
bf EA

Rt · θd − R · θO RT a
5. θ= 6. p= − 2
Rt − R V −b v

μY π Pr4 t
7. YG = 8. v=
μ − mY 8ηL
 2  
Aμ M 16π 2 L − 3gT 2
9. λ=
5
10. m=  
ρCZ 4 N 6 gT 2 − 8π 2 L
 
kp1 (p2 − p1 ) 4Ld2
11. y= 12. D2 = − D21
p2 (p − p1 ) πh
66 • Mathematics

3x − 1
A technique where algebraic fractions, for example,   , are ‘transposed’ is
(x + 2) x 2 + 4
that of Partial Fractions.

To demonstrate the concept consider the question: What is the result of


3 5
+ ?
(x + 2) (x − 4)
3 5 3 (x − 4) + 5 (x + 2) 3x − 12 + 5x + 10 8x − 2
Now + = = 2 = 2
(x + 2) (x − 4) (x + 2) (x − 4) x − 4x + 2x − 8 x − 2x − 8
3 5 8x − 2
Therefore + = 2
(x + 2) (x − 4) x − 2x − 8
8x − 2 3 5
and obviously = +
x 2 − 2x − 8 (x + 2) (x − 4)
8x − 2 3 5
The procedure of ‘breaking up’ into + is called resolving into
x 2 − 2x − 8 (x + 2) (x − 4)
partial fractions.

Partial Fractions
Certain conditions must be in place before an algebraic expression can be resolved into
partial fractions.

• The denominator must factorise, as in the above example x 2 − 2x − 8 = (x − 2) (x − 4).


It makes perfect sense that, if the denominator does not factorise, then it cannot be
resolved into partial fractions.
• The degree of the numerator must be at least one less than the degree of the numer-
ator. This means, the highest power of the variable in the numerator has to be at
least one less than the highest power in the denominator. In the above example, the
numerator is of degree 1 (the ‘8x’ term) and the degree of the denominator is 2 ( the
‘x 2 ’ term).
◦ If this is not the case then the numerator must first be divided by the denominator
using the technique of polynomial division. The remainder of this division can then
be resolved into partial fractions.

Partial fractions can be described as being one of three main types.


Simple Equations • 67

1. Linear denominators

Similar to the example above, where the denominator can be described as being the
product of several terms of degree 1.

In general, the denominator is of the form (x − a) (x − b) · · · (x − k), and the numerator is


a function of lesser degree.
2x − 1
For example,
(x − 1) (x + 2) (x + 4)
This would be resolved as follows:
2x − 1 A B C
= + +
(x − 1) (x + 2) (x + 4) (x − 1) (x + 2) (x + 4)
When resolving, the numerator of each individual fraction has to be one degree lower
than the numerator. In this case, all the denominators are of degree 1, so the numerators
have to be of degree 0 (a constant).

So
2x − 1 A B C
= + +
(x − 1) (x + 2) (x + 4) (x − 1) (x + 2) (x + 4)
A (x + 2) (x + 4) + B (x − 1) (x + 4) + C (x − 1) (x + 2)
=
(x − 1) (x + 2) (x + 4)
Therefore 2x − 1 = A (x + 2) (x + 4) + B (x − 1) (x + 4) + C (x − 1) (x + 2)

To obtain the basic equation it is quicker to use the following procedure:

• The common denominator is denominator of the left-hand side


◦ In this case (x − 1) (x + 2) (x + 4)

• All the individual fractions on the right-hand side must have this denominator in
common
◦ First term – has (x − 1), missing (x + 2)(x + 4). Hence A. (x + 2)(x + 4)

◦ Second term – has (x + 2), missing (x − 1)(x + 4). Hence B. (x − 1)(x + 4)


◦ Third term – has (x + 4), missing (x − 1)(x + 2). Hence C. (x − 1)(x + 2)
• Therefore 2x − 1 = A (x + 2) (x + 4) + B (x − 1) (x + 4) + C (x − 1) (x + 2).

This equation is true for all values of x, so a suitable choice of this value to substitute
into the equation will be crucial.
68 • Mathematics

Method: Choose a value of x that removes a bracket from the system.

2x − 1 = A (x + 2) (x + 4) + B (x − 1) (x + 4) + C (x − 1) (x + 2)

1
Let x = 1: 2 (1) − 1 = A (1 + 2) (1 + 4) ⇒ 1 = 15A ⇒ A=
15
5
Let x = −1: 2 (−2) − 1 = B (−2 − 1) (−2 + 4) ⇒ −5 = −6B ⇒ B=
6
9
Let x = −4: 2 (−4) − 1 = C (−4 − 1) (−4 + 2) ⇒ −9 = 10C ⇒ C=−
10
Therefore
  
2x − 1 1 15 56 −9 10
= + +
(x − 1) (x + 2) (x + 4) (x − 1) (x + 2) (x + 4)
1 5 9
= + −
15 (x − 1) 6 (x + 2) 10 (x + 4)

2. Quadratic denominator

 
One (or more) of the terms in the denominator will be of the form ax 2 + bx + c , where
the value of b could be 0. The numerator, being one degree less, would therefore be of
the form (Ax + B).
x 2 + 2x − 2
For example,  
(x + 1) x 2 + 1

x 2 + 2x − 2 A Bx + C
So  = + 2 
(x + 1) x + 1
2 (x + 1) x +1
 
This gives x 2 + 2x − 2 = A x 2 + 1 + (Bx + C)(x + 1)
 3
let x = −1: (−1)2 + 2 (−1) − 2 = A (−1)2 + 1 ⇒ −3 = A (2) ⇒ A = −
2
The next choice of x is 0, since this will remove B from the equation
3 1
let x = 0: −2 = A (1) + C (1) ⇒ −2 = A + C ⇒ −2 = − + C ⇒ C = −
2 2
There are now no other values of x that remove any of the unknowns from the equation.
Therefore, either any value of x can be taken, which will result in an equation involving
A, B and C. A and C can then be substituted, giving the value of B.

Or, the coefficients of powers of x can be equated.


Simple Equations • 69

3 5
Coefficient of x 2 : 1 = A+B ⇒ 1 = − +B ⇒ B =
2 2
Therefore
  
x 2 + 2x − 2 −3 2 5 2x − 1 2
 = +
(x + 1) x 2 + 1 (x + 1) (x 2 + 1)
3 5x − 1
=− +  2 
(x + 1) 2 x + 1

3. Repeated linear factors

The denominator will contain a linear term raised to a power, (x + a)n


x2 + 2
For example,
(x − 1)3
This is resolved into partial fractions whose denominators are the linear term raised to
the power 1 up to the linear term raised to the power n. In the case of the example,
n = 3.

So
x2 + 2 A B C
= + +
(x − 1)3 (x − 1) (x − 1)2 (x − 1)3

This gives x 2 + 2 = A (x − 1)2 + B (x − 1) + C

Let x = 1: (1)2 + 2 = C ⇒ C = 3
Coefficient of x 2 : 1 = A ⇒ A = 1

Choose another value of x:

Let x = 0 : 2 = A (−1)2 + B (−1) + C ⇒ 2 = A − B + C ⇒ B = A + C − 2 = 1 + 3 − 2

So B = 2

Therefore
x2 + 2 1 2 3
= + +
(x − 1) 3 (x − 1) (x − 1) 2
(x − 1)3
Examples

Find partial fractions for the following:


x+3 4x 2 − 7x − 5 3x − 4
(i) (ii)   (iii)
(x − 3)(x − 4) x 2 − 2x − 5 (x − 2) (x + 1) (x − 2)2
70 • Mathematics

Solutions
x+3 A B
(i) = + ⇒ x + 3 = A (x − 4) + B (x − 3)
(x − 3) (x − 4) (x − 3) (x − 4)
Let x = 4 ⇒ 7 = 1B ⇒ B = 7
Let x = 3 ⇒ 6 = −1A ⇒ A = −6
Therefore
x+3 −6 7
= +
(x − 3) (x − 4) (x − 3) (x − 4)

4x 2 − 7x − 5 (Ax + B) C
(ii)   = 2 +
x − 2x − 5 (x − 2)
2 x − 2x − 5 (x − 2)
 2 
⇒ 4x − 7x − 5 = (Ax + B) (x − 2) + C x − 2x − 5
2

3
Let x = 2 ⇒ 16 − 14 − 5 = −5C ⇒ −5C = −3 ⇒ C =
5
Let x = 0 ⇒ −5 = −2B − 5C ⇒ −5 = −2B − 3 ⇒ B = 1
17
Coefficient of x 2 ⇒ 4 = A + C ⇒ A =
5
Therefore
 
17 3
x + 1
4x 2 − 7x − 5 5 (17x + 5) 3
  = 2 + 5 =  +
x 2 − 2x − 5 (x − 2) x − 2x − 5 (x − 2) 5 x 2 − 2x − 5 5 (x − 2)

3x − 4 A B C
(iii) = + +
(x + 1) (x − 2)2 (x + 1) (x − 2) (x − 2)2
⇒ 3x − 4 = A (x − 2)2 + B (x + 1) (x − 2) + C (x + 1)
2
Let x = 2 ⇒ 2 = 3C ⇒ C =
3
−7
Let x = −1 ⇒ −7 = 9A ⇒ A =
9
7
Coefficient of x 2 ⇒ 0 = A + B ⇒ B =
9
Therefore
3x − 4 −7 7 2
= + +
(x + 1) (x − 2)2 9 (x + 1) 9 (x − 2) 3 (x − 2)2

Questions
Find partial fractions for the following:
x+7 8x 2 − 19x − 24 5x 2 − 13x + 5
(i) (ii)   (iii)
(x − 2)(x − 5) x 2 − 2x − 5 (x − 2) (x − 2)3
Simple Equations • 71

Answers
x+7 A B
(i) = + ⇒ x + 7 = A (x − 5) + B (x − 2)
(x − 2) (x − 5) (x − 2) (x − 5)
x = 5 ⇒ 12 = 3B ⇒ B = 4
x = 2 ⇒ 9 = −3A ⇒ A = −3
Therefore
x+7 −3 4
= +
(x − 2) (x − 5) (x − 2) (x − 5)
8x 2 − 19x − 24 (Ax + B) C
(ii)   = 2 +
x − 2x − 5 (x − 2)
2 x − 2x − 5 (x − 2)
 
⇒ 8x − 19x − 24 = (Ax + B) (x − 2) + C x 2 − 2x − 5
2

x = 2 ⇒ 32 − 38 − 24 = −5C ⇒ C = 6
x = 0 ⇒ −24 = −2B − 5C ⇒ −24 = −2B − 30 ⇒ B = −3
Coefficient of x 2 ⇒ 8 = A + C ⇒ A = 2
Therefore
8x 2 − 19x − 24 (2x − 3) 6
  = 2 +
x − 2x − 5 (x − 2)
2 x − 2x − 5 (x − 2)
5x 2 − 13x + 5 A B C
(iii) = + +
(x − 2) 3 (x − 2) (x − 2) 2
(x − 2)3
⇒ 5x 2 − 13x + 5 = A (x − 2) + B (x − 2) + C
2

x = 2 ⇒ 20 − 26 + 5 = C ⇒ C = −1
Coefficient of x 2 ⇒ 5 = A ⇒ A = 5
x = 0 ⇒ 5 = 4A − 2B + C ⇒ 5 = 20 − 2B − 1 ⇒ B = 7
Therefore
5x 2 − 13x + 5 7 7 1
= + −
(x − 2) 3 (x − 2) (x − 2) 2
(x − 2)3

Logarithmic or Indicial Equations


Some equations result in the unknown value being a power, or index.

Example
 n
T1 p1
Given that = find the value of n when T1 = 570, T2 = 250, p1 = 19 and
T2 p2
p2 = 1.16.
72 • Mathematics

The method in cases like this is to take logarithms of both sides.


   n 
T1 p1
So log = log
T2 p2
The third law of logarithms states:

The logarithm of a number raised to a power is given by the logarithm of the


number multiplied by the power: log(x y ) = y · log(x).
   
T1 p1
Therefore log = n × log
T2 p2
 
T1
log
T log (T1 ) − log (T2 )
and so n =  2 = , using the second law of logarithms
p1 log (P1 ) − log (P2 )
log
p2
log (570) − log (250)
Therefore n = = 0. 2948 correct to 4 decimal places
log (19) − log (1. 16)
Note: either base 10 or natural logarithms may be used.

Example
  n−1
T1 p1 n
Given that =
T2 p2
find the value of n when T1 = 710, T2 = 260, p1 = 30 and p2 = 0. 95

⎡ ⎤
  n−1     n−1
710 30 n 710 30 n
= ⇒ log = log ⎣ ⎦
260 0. 95 260 0. 95

    
710 n−1 30
So log = × log
260 n 0. 95
 
710
  log
n−1 260
Therefore =   = 0. 29097
n 30
log
0. 95
n−1
but = 0. 29097 ⇒ n − 1 = 0. 29097n ⇒ n − 0. 29097n = 1
n
1
Therefore 0. 70903n = 1 ⇒ n = = 1. 410 correct to 3 decimal places.
0. 70903
Simple Equations • 73

Test Examples 3
1. Find the value of x which satisfies the following equation:

8 + 5x − 7 = 3x + 9

2. Find the value of a in the equation:

2(a + 3) + 3(2a − 4) = 4(11 − 3a)

3. Solve the equation:

3[3 − {x + 2(1 − x)} − 4x] = 2[x − 3(2 + x) − 4]

4. Find x for the following equation:


2x x 5 4x 1
+ − = +
3 4 6 5 3
5. Find a in the following:
 
1 − 2a2 1 a 1 5
− + = −
6a 4 3 6 2a
2 3 4
6. If + = 2 , find x.
x−3 x+2 x −x−6
7. A ship travelling at 17.5 knot leaves one port bound for another 4.5 h after a
another ship, whose speed is 16 knots, leaves the same port set on the same course.
After how many hours and at what distance from port will the fast ship overtake the
slower?
8. A rectangular plate is to be cut so that its length is four times the breadth and
having an area of 1 m2 . Find the length and breadth.
R2 (1 + αθ2 )
9. Transpose = to find an expression for θ2 .
R1 (1 + αθ1 )
Calculate the value of θ2 when: R1 = 200, R2 = 240, θ1 = 15 and α = 0. 0042.

10. The diameter in mm of coupling bolts should not be less than that given by the
formula: 
D3
d=
3. 5 × n × R
where D = diameter of shafting in mm; n = number of bolts per coupling; and R =
pitch circle radius in mm.
Express R in terms of the other quantities and find the pitch circle radius when
d = 82. 5 mm, D = 381 mm and n = 8 bolts per coupling.
74 • Mathematics

11. Given
  n−1
T1 p1 n
=
T2 p2
find the value of n when T1 = 797, T2 = 301, p1 = 34. 4 and p2 = 1. 05
12. Find the values of x in each of the following:
(a) 40.59x = 56. 36
(b) x 1.95 = 12. 4x 0.53
(c) e5x = 46. 382.6

(d) 3 x = e2.5
SIMULTANEOUS
4
EQUATIONS
It has been seen that only one equation is needed to find the value of a single
unknown quantity. However, if one equation has two unknown quantities, for example,
3x + y = 12, there is no longer a unique answer. In this example, x could be 4 with y
equalling 0 or x could equal 0 with y equalling 12.

If two different equations involving the same two unknowns are available then a single
solution can be found. Similarly, if there are 3 unknowns there must be three different
equations etc.

When such sets of equations have to be solved for the unknowns, they are known as
simultaneous equations.

There are several methods available but the method shown below is that of elimination.

Two Unknowns Where the Coefficients


Are Whole Numbers
Example

Solve the following equations for x and y : 3x + 4y = 5 and 12 + 2x − 5y = 0:

• The two equations should be written down with similar unknown terms under each
other. It is usual to put the unknown terms on the LHS of the equation with the
76 • Mathematics

constant term on the RHS. So

3x + 4y = 5
2x − 5y = −12

• The two equations are numbered. This makes reference to the equations much easier.
So

3x + 4y = 5 (1)
2x − 5y = −12 (2)

• The equations have to be multiplied in such a way that the coefficients of one of the
unknowns become equal. The choice of either x or y is arbitrary.
• If, in this example, the x coefficients are to be made equal then equation (1) has to
be multiplied by the x coefficient in equation (2) (i.e. 2) and equation (2) has to be
multiplied by the x coefficient in equation (1) (i.e. 3). This gives

6x + 8y = 10 (3)
6x − 15y = −36 (4)

The new equations are renumbered.


• By adding or subtracting these new equations one unknown can be eliminated. If
the sign of the common coefficients is the same (i.e. both are positive or both are
negative) then one equation should be subtracted from the other. If the signs of the
coefficients are different then the equations should be added.
• As the x coefficients are both positive the equations should be subtracted. So
equation (4) is subtracted from equation (3) giving:

6x + 8y = 10
6x − 15y = −36

0 + 23y = 46

• This new equation has only one unknown and can therefore be solved.

23y = 46 gives y = 2

• This value should be substituted into one of the original equations

So, in (1) 3x + 8 = 5, so 3x = −3 giving x = −1

• The solution to the simultaneous equation is x = −1, y = 2.


Simultaneous Equations • 77

Example

Solve

8x − 3y = 39 (1)
7x + 5y = −4 (2)

To eliminate the y’s, equation (1) is multiplied by 5 and equation (2) by 3.

This gives

40x − 15y = 195 (3)


21x + 15y = −12 (4)

When equation (3) is added to equation (4) (added because the y coefficients have
different signs) the result is
61x = 183

183
So x = =3
61
Substituting this value into equation (1) gives

24 − 3y = 39
−3y = 15
15
y=
−3
= −5

Check: In equation (2), 7 × 3 + 5 × (−5) = 21 − 25 = −4. Therefore correct.

Example

Solve

4x − 18 = 3y
1 + x + 2y = 0

Rearranging gives

4x − 3y = 18 (1)
x + 2y = −1 (2)
78 • Mathematics

The simplest way to make the coefficients the same is to multiply equation (2) by 4.

So

4x − 3y = 18 (3)
4x + 8y = −4 (4)

Subtracting (3) from (4) gives

8y − (−3y) = −4 − 18
11y = −22
y = −2

Substituting into equation (3) gives

4x − 3 × (−2) = 18
4x + 6 = 18

So

4x = 18 − 6 = 12
12
x= =3
4

Check: In equation (2), 3 + 2 × (−2) = 3 − 4 = −1. Therefore correct.

Questions

1. a+b = 7 2. 2x + 5y = 7
a−b = 3 x + 3y = 4
3. 3s + 2t = 12 4. 3x − 2y = 13
4s − t = 5 2x + 5y = −4
5. 5m − 3n = 11 6. 8a − 3b = 51
3m + n = 8 3a + 4b = 14
7. 5x = 2y 8. 1 − 3d = 5c
3x + 7y = 41 2d + c + 4 = 0
Simultaneous Equations • 79

Answers

1. a = 5, b = 2 2. x = 1, y = 1 3. s = 2, t = 3 4. x = 4, y = −2

1 1
5. m=2 , n= 6. a = 6, b = −1 7. x = 2, y = 5 8. c = 2, d = −3
2 2

Two Unknowns Where the Coefficients


Are Fractions
Example

Solve
a b
− =0 (1)
6 8
a b
+ =6 (2)
3 4

It is best to get rid of the fractions before the solution of the simultaneous equations is
started. Multiply each equation by the LCM of its denominators.

So, multiply (1) by 24 and (2) by 12, giving

4a − 3b = 0 (3)
4a + 3b = 72 (4)

Adding equations (3) and (4) will eliminate b, giving 8a = 72, so a = 9.

Substituting into equation (3) gives

(4 × 9) − 3b = 0
3b = 36
so b = 12

9 12 3 3
Checking by substituting into equation (2) gives − = − = 0.
6 8 2 2
So correct.
80 • Mathematics

Two Unknowns Where the Coefficients


Are Decimals
Example

Solve

1. 2x − 1. 8y = −21 (1)
2. 5x + 0. 6y = 65 (2)

It is best to get rid of the decimals before the solution of the simultaneous equations is
started. Multiply each equation by the appropriate power of 10, that is, 1 decimal place
multiply by 10, 2 decimal places multiply by 100 etc.

So multiplying equations (1) and (2) by 10 gives

12x − 18y = −210 (3)


25x + 6y = 650 (4)

To eliminate y, multiply equation (4) by 3 giving

12x − 18y = −210 (5)


75x + 18y = 1950 (6)

Adding (5) and (6) gives

87x = 1740
1740
x=
87
x = 20

Substituting in (4) gives

(25 × 20) + 6y = 650


500 + 6y = 650
Simultaneous Equations • 81

So

6y = 650 − 500
6y = 150
150
y=
6
y = 25

Checking by substituting into equation (1) gives (1. 2 × 20) − (1. 8 × 25) = 24 − 45
= −21.

So correct.

It is important to realise that numbers must not be rounded at any stage. The full result of
each multiplication and division must be used if accuracy errors are to be avoided.

Two Unknowns Involving Reciprocals


Example

Solve
3 6
+ =4
a b
9 12
− = −3
a b

1 1
The following substitutions should be made: let = X and = Y.
a b
This gives

3X + 6Y = 4 (1)
9X − 12Y = −3 (2)

These can now be solved as normal.

Equation (1) multiplied by 2 gives

6X + 12Y = 8 (3)
9X − 12Y = −3 (4)
82 • Mathematics

Adding (3) and (4) gives

15X = 5
5
X=
15
1
X=
3

Substituting into (4) gives


 
1
9× − 12Y = −3
3
3 − 12Y = −3
−12Y = −6
−6 1
So Y= =
−12 2

1 1 1 1 1 1
But = X and = Y so = a and = b giving a = = 3 and b = = 2.
a b X Y 1/3 1/2
3 6
Checking by substituting into the original first equation gives + = 1 + 3 = 4.
3 2
So correct.

Example

Solve

x−1 y+2 2
+ = (1)
3 5 15
1−x 5+y 5
+ = (2)
6 2 6

Before the equations can be solved simultaneously the fractions have to be removed
and the equations rearranged.

Equation (1) multiplied by 15 and equation (2) multiplied by 6 gives

5 (x − 1)) + 3 (y + 2) = 2 (3)
(1 − x) + 3 (5 + y) = 5 (4)
Simultaneous Equations • 83

Expanding the brackets gives

5x − 5 + 3y + 6 = 2 (5)
1 − x + 15 + 3y = 5 (6)

Rearranging gives

5x + 3y = 1 (7)
−x + 3y = −11 (8)

Equation (7) subtract equation (8) gives

6x = 12 so x = 2

Substituting into equation (7) gives

(5 × 2) + 3y = 1
10 + 3y = 1
3y = −9
y = −3

Substituting in equation (8) gives (−2) + (3 × −3) = −2 + (−9) = −11, so correct.

Questions

In Questions 1–7 solve the following pairs of simultaneous equations and check the
results:

3 2 4 3
1. + = 14 2. − = 18
x y a b
5 3 2 5
− = −2 + = −4
x y a b

1 3 5 3
3. + =5 4. + = 1. 1
2p 5q x y
5 1 35 3 7
− = − = −1. 1
p 2q 2 x y
84 • Mathematics

c+1 d+2 3r + 2 2s − 1 11
5. − +1=0 6. − =
4 3 5 4 5
1 − c 3 − d 13 3 + 2r 5 − s 15
+ + =0 + =
5 4 20 4 3 4

5 20 4 16
7. = =
x + y 27 2x − y 33

3 4 5 xy + 1
8. If 5x − = 1 and x + = find the value of
y y 2 y

Answers

1 1 1 1 1 1
1. x= , y= 2. a= , b=− 3. p= , q= 4. x = 10, y = 5
2 4 3 2 4 5
1 3
5. c = 3, d = 4 6. r = 3, s = 7. x = 5, y = 1 8. 1
2 4

Practical Problems: Simultaneous


Equations in Two Unknowns
Example

The law connecting friction F and load L for an experiment is of the form F = aL + b,
where a and b are constants.

When F = 5. 6, L = 8. 0 and when F = 4. 4, L = 2. 0.

Find the values of a and b and the value of F when L = 6. 5.

Substituting the values into the law gives

5. 6 = 8. 0a + b (1)
4. 4 = 2. 0a + b (2)

Subtracting equation (2) from equation (1) gives


1. 2
1. 2 = 6a so a = = 0. 2
6
Simultaneous Equations • 85

Substituting this value into equation (1) gives

5. 6 = (8. 0 × 0. 2) + b
5. 6 = 1. 6 + b
b=4

Checking these values in equation (2) gives, (2. 0 × 0. 2) + 4 = 0. 4 + 4 = 4. 4 so correct.

When L = 6. 5, F = (0. 2 × 6. 5) + 4 = 1. 3 + 4 = 5. 3.

Example

When Kirchhoff’s laws are applied to the electrical circuit shown, the currents i1 and i2
are connected by the equations:

27 = 1. 5i1 + 8 (i1 − i2 ) (1)


−26 = 2i2 − 8 (i1 − i2 ) (2)

i1 i2

27 (i1 – i2) 26

1.5 Ω 8Ω 2Ω

Solve the equations for the currents i1 and i2 .

Removing the brackets from equation (1) gives

27 = 1. 5i1 + 8i1 − 8i2

Rearranging gives

9. 5i1 − 8i2 = 27 (3)

Removing the brackets from equation (2) gives

−26 = 2i2 − 8i1 + 8i2

Rearranging gives

−8i1 + 10i2 = −26 (4)


86 • Mathematics

Multiplying equation (3) by 10 gives

95i1 − 80i2 = 270 (5)

Multiplying equation (4) by 8 gives

−64i1 + 80i2 = −208 (6)

Adding equations (5) and (6) gives

31i1 = 62
62
i1 = =2
31

Substituting in equation (5) gives

190 − 80i2 = 270


−80i2 = 80
So i2 = −1

Checking in equation (4) gives (−8 × 2) + (10 × −1) = (−16) + (−10) = −26

So correct.

Example

The distance, s metres from a fixed point, of a vehicle travelling in a straight line with
constant acceleration, a m/s2 , is given by s = ut + 12 at2 , where u is the initial velocity in
m/s and t is the time in seconds. Determine the initial velocity and acceleration given
that s = 42 m when t = 2 s and s = 144 m when t = 4 s.

Find also the distance travelled after 3 s.


1 1
Substituting s = 42 and t = 2 into s = ut + at2 gives 42 = 2u + a (2)2
2 2
So 42 = 2u + 2a (1)

1 1
Substituting s = 144 and t = 4 into s = ut + at2 gives 144 = 4u + a (4)2
2 2
So 144 = 4u + 8a (2)

Multiplying equation (1) by 2 gives

84 = 4u + 4a (3)
Simultaneous Equations • 87

Subtracting equation (3) from equation (2) gives

60 = 4a
60
a=
4
a = 15

Substituting into equation (1) gives

42 = 2u + (2 × 15)
42 = 2u + 30

So

2u = 12
12
u= =6
2

Checking in equation (2) gives (4 × 6) + (8 × 15) = 24 + 120 = 144

So correct.

The distance travelled after 3 seconds is found by substituting t = 3, u = 6 and a = 15


1
into s = ut + at2 .
2
1 15 × 9
So s = (6 × 3) + (15) (3)2 = 18 + = 18 + 67. 5 = 85. 5 m.
2 2

Questions

1. In a system of pulleys, the effort P required to raise a load W is given by P = aW + b,


where a and b are constants. If W = 40 when P = 12 and W = 90 when P = 22, find
the values of a and b.
2. Applying Kirchhoff’s laws to an electrical circuit produces the following equations:

5 = 0. 2i1 + 2 (i1 − i2 )
12 = 3i2 + 0. 4i2 − 2 (i1 − i2 )

Determine the values of i1 and i2 · i1 and i2 .


3. Velocity v is given by the formula v = u + at. If v = 20 when t = 2 and v = 40 when
t = 7 find the values of u and a.
Find also the value of v when t = 3. 5.
88 • Mathematics

4. The molar heat capacity of a solid compound is given by the equation c = a + bt.
When c = 52, T = 100 and when c = 172, T = 400. Find the values of a and b.
b
5. In an engineering process two variables p and q are related by q = ap + , where a
p
and b are constants. Evaluate a and b if q = 13 when p = 2 and q = 22 when p = 5.
6. Equations connecting parallel resistances in a circuit are

4 6 9
+ + =6
R1 R2 R3
15 11 2 1
+ + =8
R1 R2 R3 12

If R2 = R3 , find the values of R1 , R2 and R3 .


7. x grams of cartridge brass (70% copper, 30% zinc, by mass) and y grams of naval
brass (62% copper, 37% zinc, 1% tin, by mass) when fused together give a new alloy
containing 342.6 grams of copper and 165.6 grams of zinc. Find x and y.

Answers
1
1. a= , b=4 2. i1 = 6. 47, i2 = 4. 62 3. u = 12, a = 4, v = 26 4. a = 12, b = 0. 40
5

5. a = 4, b = 10 6. R1 = 4, R2 = R3 = 3 7. X = 330, Y = 180

A More Complicated Situation


Given the simultaneous equations

1. 5x × 2y = 18
4x × 1. 5y = 54

find the values of x and y.

As seen earlier in the text, a method to deal with unknown values that are powers is to use
logarithms.
Simultaneous Equations • 89

Taking log10 of both sides of the equations gives:

log (1. 5x × 2y ) = log (18)

log (4x × 1. 5y ) = log (54) which gives

log (1. 5x ) + log (2y ) = log (18)

log (4x ) + log (1. 5y ) = log (54) therefore

x × log (1. 5) + y × log (2) = log (18) and

x × log (4) + y × log (1. 5) = log (54)

Inserting the log values gives

0. 1761x + 0. 301y = 1. 2253 (i)


0. 6021x + 0. 1761y = 1. 7324 (ii)

Dividing (i) by 0.1761, and (ii) by 0.6021 gives

x + 1. 709y = 7. 127 (iii)


x + 0. 2925y = 2. 877 (iv)

Subtracting (iv) from (iii) gives

1. 4165y = 4. 25
y=3

Substituting y = 3 into (iii) gives

x + 1. 709 × 3 = 7. 127
x = 7. 127 − 5. 127
x=2

So x = 2 and y = 3.

There are other methods of approach to the solution of simultaneous linear equations.

A second method is described below:

• Transpose one of the equations to write one unknown in terms of the other quantities.
• Substitute this expression in place of the variable in the second equation, producing
a single equation with one unknown value.
90 • Mathematics

• Solve this equation.


• Calculate the value of the second unknown.

Example

2x + 5y = 34
4x − 3y = 16

Transpose the first equation to write x in terms of the other quantities

2x + 5y = 34
2x = 34 − 5y
x = 17 − 2. 5y (i)

Substitute this expression into the second equation

4x − 3y = 16
4 (17 − 2. 5y) − 3y = 16
68 − 10y − 3y = 16
68 − 13y = 16
68 − 16 = 13y
52 = 13y
y=4

Substitute y = 4 into (i) gives

x = 17 − 2. 5 × 4
x = 17 − 10
x=7

The values are x = 7 and y = 4.

A third method is:

• Transpose both equations to find expressions for the same unknown.


• Equate these expressions.
• Solve for the unknown.
• Substitute this value into one of the original equations andcalculate the second
unknown.
Simultaneous Equations • 91

Using the same example:

2x + 5y = 34 (i)
4x − 3y = 16

means that

x = 17 − 2. 5y
x = 4 + 0. 75y

and so

17 − 2. 5y = 4 + 0. 75y
17 − 4 = 0. 75y + 2. 5y
13 = 3. 25y
13
y=
3. 25
y=4

Substituting into (i) gives

2x + 5 × 4 = 34
2x = 34 − 20
2x = 14
x=7

No single method is better or worse than another. Use the method that is most
‘understandable’.

If you find algebra to be really confusing there is always Cramer’s Rule which works for
two unknowns and is totally arithmetical.

Using the same example

2x + 5y = 34
4x − 3y = 16

write down just the numbers.


92 • Mathematics

So the equations give


2 5 34
4 −3 16

Using the first two columns: calculate (top left times bottom right) minus (top right
times bottom left).

So (2 × −3) − (5 × 4) = −6 − 20 = −26.

Replace the first column with the third column and perform the same calculation:

34 5
gives (34 × −3) − (5 × 16) = −102 − 80 = −182
16 −3

−182
Divide this answer by the first answer, that is, = 7. This is the value of the first
−26
variable, x.

Replace the second column of the original block by the third column and repeat the
process to find the value of the second variable, y.

2 34 −104
gives (2 × 16) − (34 × 4) = 32 − 136 = −104. =4=y
4 16 −26

So the solution is x = 7, y = 4.

Three unknowns

Two unknowns need two equations so three unknowns need three equations.

Example

Find the values for x, y and z which satisfy the equations:

3x + 2y − z = 4 (i)
2x + y + z = 7 (ii)
x−y+z =2 (iii)

Eliminate one variable at a time, in this case, z seems to be the easiest.


Simultaneous Equations • 93

Add (i) and (ii)

3x + 2y − z = 4 (i)
2x + y + z = 7 (ii)
So 5x + 3y = 11 (iv)

Subtract (iii) from (ii)

2x + y + z = 7 (ii)
x−y+z = 2 (iii)
x + 2y = 5 (v)

Multiply (v) by 5 and subtract (iv)

5x + 10y = 25
5x + 3y = 11 (iv)
7y = 14
y=2

Substitute y = 2 into (v)

x+4=5
x=1

Substitute x = 1 and y = 2 into (iii)

1−2+z= 2
z =2+2−1
z=3

Therefore x = 1, y = 2 and z = 3.

Test Examples 4
 
1. When 21 2 times one number are added to 31 2 times another the result is 19; and
 
when 31 2 times the first number is subtracted from 21 2 times the second, the
result is 3. Find the numbers.
94 • Mathematics

2. Find the value of x and y in the simultaneous equations:


2x 3y 3 x y 13
− = and − =
3 5 4 2 4 16
3. Find the values of a and b which satisfy the equations:

a (1 + 2b) = 3 and a (1 − 3b) = 0. 5

4. A man and his wife are 72 and 68 years old respectively, and have one grandson and
one grand-daughter. The man’s age is equal to the sum of four times the grandson’s
age and three times the grand-daughter’s. The woman age is equal to the sum of
three times the grandson’s age and four times the grand-daughter’s.
Find the ages of the grandchildren.
5. The difference between two numbers is 2 and the difference between their squares
is 6. Find the numbers.
6. The linear law of a simple lifting machine is given by F = a + bm where m = mass
lifted, F = effort applied, a and b being constants.
In a certain machine it was found that when m = 30 kg, F = 35 N and when
m = 70 kg, F = 55 N. Find the constants a and b, write the law for the lifting machine
and find the effort required to lift a mass of 60 kg.
7. Two ships, A and B, leave one port bound for another on the same course. B leaves
one hour later than A and overtakes in 8 h. If the speeds of each ship had been 4
knots slower, B would have overtaken A 2 h earlier. Find the original speeds of the
ships.
8. Given the simultaneous equations:
x 6 2x 9
+ =4 and − =1
2−y x 2−y x
find the values of
x 1
, , x and y.
2−y x
9. Find the values of x and y in the simultaneous equations:

2x = 4y and 4x−1 = 2y+1

10. Given the simultaneous equations

1. 259x+1 × 1. 175y−1 = 2. 323


3. 162x × 1. 778y = 25. 12

find the values of x and y.


Simultaneous Equations • 95

11. Given the following relationship:


d2 x 2 y2
2
= 2= 2
x y D
express x in terms of d and D, and find the values of x and y when D = 75 and
d = 25.
12. Find the values for a, b and c in the simultaneous equations:

3a + 6b − 2c = 7. 25
2a + 3b + 4c = 26
4a − 2b + c = 10. 25
5
QUADRATIC
EQUATIONS
A quadratic equation is one where the highest power of the unknown is 2.

For example, x 2 − 3x − 6 = 0 is a quadratic equation.

There are 3 main methods of solving of this type of equation:

1. by Factorisation (not always possible),


2. by ‘Completing the square’,
3. by Using the ‘quadratic formula’.

The solutions of the quadratic equation are called the roots of the equation.

Solution by Factorisation
When (x + 3) and (x − 2) are multiplied together the result is x 2 + x − 6. The reverse
process, called factorisation, means that x 2 + x − 6 = (x + 3) (x − 2).

For example, the solution of x 2 + x − 6 = 0 is the same as the solution of


(x + 3) (x − 2) = 0

This means that either (x + 3) = 0, in which case x = −3, or (x − 2) = 0, in which case


x = 2.

Example

Solve x 2 − 4x − 12 = 0 by factorisation.
Quadratic Equations • 97

The first term in each bracket is x.

−12 is the product of, either,

−12 and +1, +12 and −1, −6 and +2, +6 and −2, −4 and +3 or +4 and −3.

The only pair that gives −4 as the x coefficient is −6 and +2.

So x 2 − 4x − 12 = (x − 6) (x + 2)

Either x−6= 0 i.e. x = 6, or


x+2=0 i.e. x = −2

So the roots of x 2 − 4x − 12 = 0 are x = 6 and x = −2.

Example

Solve 3x 2 − 11x − 4 = 0 by factorisation.

The factors of 3x 2 are 3x and x, so the brackets are (3x ) and (x ).

The factors of −4 are −4 and +1, +4 and −1 and +2 and −2.

The x coefficient is given by the outside product added to the inside product which, in
this case, is −11.
#
As a diagram, this means that (3x ?) and (x= ?) when added together give −11x.

The only possible option is (3x + 1) (x − 4) giving either 3x = −1 or x = 4.


1
The roots of 3x 2 − 11x − 4 = 0 are therefore x = − and x = 4.
3
Example

By factorising, find the roots of x 2 − 8x + 16 = 0.

x 2 − 8x + 16 = 0 → (x − 4) (x − 4) = 0 i. e. (x − 4)2 = 0

The LHS is called a ‘perfect square’ and so x = 4 is the only root of x 2 − 8x + 16 = 0.

Example

By factorising, find the roots of 9x 2 − 49 = 0.

The LHS is called ‘the difference of two squares’ as 9x 2 − 49 = (3x)2 − 72 .


7 7
9x 2 − 49 factorises to give (3x + 7) (3x − 7) = 0 and so the roots are − and + .
3 3
98 • Mathematics

Given the roots

If the two roots are α and β then (x − α) (x − β) = 0.

Example
1
The roots of a quadratic equation are and −2. Determine the equation.
3
 
1 1
If α = and β = −2 then x − (x − (−2)) = 0
3 3
So
 
1
x− (x + 2) = 0
3
1 2
x 2 + 2x − x − = 0
3 3
5 2
x2 + x − = 0
3 3

Multiplying by 3 eliminates the fractions so 3x 2 + 5x − 2 = 0.

Questions

Solve the following questions by factorisation (if possible):

1. x 2 + 6x + 8 = 0 2. x 2 + 10x + 16 = 0 3. x 2 + 12x + 27 = 0

4. x 2 + 8x + 15 = 0 5. x 2 + 7x + 10 = 0 6. x 2 + 3x − 18 = 0

7. x 2 + 3x − 10 = 0 8. x 2 − 11x + 28 = 0 9. x 2 + 4x + 3 = 0

10. x 2 − 6x + 5 = 0 11. x 2 + 11x + 24 = 0 12. x 2 + 15x + 36 = 0

13. t2 + 19t + 34 = 0 14. t2 + 18t + 65 = 0 15. t2 − 7t − 44 = 0

16. s2 + s − 30 = 0 17. m2 + 5m − 36 = 0 18. q2 − 5q − 66 = 0

19. c2 − 11c + 18 = 0 20. x2 − x − 1 = 0


Quadratic Equations • 99

Answers
1. (x + 2)(x + 4) 2. (x + 2)(x + 8) 3. (x + 3)(x + 9) 4. (x + 3)(x + 5)
5. (x + 2)(x + 5) 6. (x + 6)(x − 3) 7. (x + 5)(x − 2) 8. (x − 4)(x − 7)
9. (x + 3)(x + 1) 10. (x − 5)(x − 1) 11. (x + 3)(x + 8) 12. (x + 12)(x + 3)
13. (t + 17)(t + 2) 14. (t + 13)(t + 5) 15. (t − 11)(t + 4) 16. (s + 6)(s − 5)
17. (m + 9)(m − 4) 18. (q − 11)(q + 6) 19. (c − 9)(c − 2) 20. Does not factorise

Solution by ‘Completing the Square’


As shown earlier, expressions like (x − 4)2 , x 2 and (x + 1)2 are perfect squares.

If x 2 = 5, then x = ± 5.
√ √
If (x − 4)2 = 5, then x − 4 = ± 5 and therefore x = +4 ± 5.
√ √
If (x + 1)2 = 5, then x + 1 = ± 5 and therefore x = −1 ± 5.

So if the quadratic equation can be rearranged so the LHS of the equation is a perfect
square and the RHS is a number, then the roots can be found, as in the examples above.
The process of rearranging the equation into a perfect square is called ‘completing the
square’.

The method is shown by using the following example.

Example

Solve 2x 2 + 5x = 3 by ‘completing the square.

The process is as follows:

1. Rearrange the equation so all the terms are on the same side of the equals sign and
that the x 2 term is positive

Hence 2x 2 + 5x − 3 = 0

2. Make the coefficient of the x 2 term 1. In this example, this is done by dividing
everything by 2. Hence
2 2 5 3
x + x− =0
2 2 2
5 3
i. e. x 2 + x − = 0
2 2
100 • Mathematics

3. Rearrange the equation so the x 2 and x terms are on the LHS of the equals sign and
the constant is on the RHS. So

5 3
x2 + x =
2 2

4. Take the coefficient of x, half it and square, the answer. Add this value to both sides
5 5
of the equation. In this case the coefficient is , halving it gives and squaring it
 2 2 4
5
gives . So
4
 2  2
5 5 3 5
x2 + x + = +
2 4 2 4
 2  
5 5 5 2
The LHS is now a perfect square, that is, x 2 + x + = x+
2 4 4
5. Calculate the value of the RHS. So
 
5 2 3 25 24 + 25 49
x+ = + = =
4 2 16 16 16

  
5 49 5 7
6. Taking the square root of both sides gives x + = →x+ =±
4 16 4 4
5 7
7. Solving this simple equation gives x = − ±
4 4

5 7 2 1
i. e. x=− + = =
4 4 4 2
5 7 12
and x = − − = − = −3
4 4 4

1
So the roots of 2x 2 + 5x = 3 are and −3.
2
Example

Solve 2x 2 + 9x + 8 = 0 by completing the square.

2x 2 + 9x + 8 = 0
9
x2 + x + 4 = 0
2
9
x 2 + x = −4
2
Quadratic Equations • 101

 2  2
9 9 9
x2 + x + = −4 +
2 4 4
 2
9 81
x+ = −4 +
4 16
 2
9 −64 + 81
x+ =
4 16
 2
9 17
x+ =
4 16

9 17
x+ =±
4 16
= ±1. 031
x = −2. 25 ± 1. 031
x = −1. 22 or − 3. 28

Answers given are correct to 3 significant figures.

Example

Solve 4. 6x 2 + 3. 5x − 1. 75 = 0 by completing the square.

Answers to be given correct to 3 decimal places.

4. 6x 2 + 3. 5x − 1. 75 = 0
4. 6x 2 + 3. 5x = 1. 75
3. 5 1. 75
x2 + x=
4. 6 4. 6
3. 5 1. 75
x2 + x=
4. 6 4. 6
   2
3. 5 3. 5 2 1. 75 3. 5
x2 + x+ = +
4. 6 9. 2 4. 6 9. 2

 
3. 5 2
The LHS is now a perfect square so x + = 0. 5251654
9. 2

3. 5 √ 3. 5
x+ = 0. 5251654 = ±0. 7246830 → x = − ± 0. 7246830
9. 2 9. 2
That is, x = 0. 344 or − 1. 105
102 • Mathematics

Questions
Solve the following equations using the method of ‘completing the square’.

Give the answers correct to 3 decimal places.

1. x 2 + 4x + 1 = 0 2. 2x 2 + 5x − 4 = 0 3. 3x 2 − x − 5 = 0
4. 5x 2 − 8x + 2 = 0 5. 4x 2 − 11x + 3 = 0 6. 2x 2 + 5x = 2

Answers

1. −3. 732, −0. 268 2. −3. 137, 0. 637 3. 1. 468, −1. 135
4. 1. 290, 0. 310 5. 2. 443, 0. 307 6. −2. 851, 0. 351

Solution of Quadratic Equations by


Formula
The general form of a quadratic equation is ax 2 + bx + c = 0, where a, b and c are
constants.

If the same procedure is applied to this equation as that applied to Examples 5, 6 and 7
the result is known as the quadratic formula and is given by:

−b ± b2 − 4ac
x=
2a
Example

Using the quadratic formula, solve x 2 + 2x − 8 = 0.

Comparing x 2 + 2x − 8 = 0 with ax2 + bx + c = 0 gives a = 1, b = 2 and c = −8.




−b ± b − 4ac
2 −2 ± (2)2 − 4 × 1 × (−8)
Substituting these values into x = gives x =
2a 2×1
√ √
−2 ± 4 + 32 −2 ± 36 −2 ± 6 −2 + 6 −2 − 6
So x = = = so x = or = 2 or −4
2 2 2 2 2
(The equation could have been solved by factorisation as x 2 + 2x − 8 = (x + 4) (x − 2) So
(x + 4) (x − 2) = 0 gives x = −4 or x = 2.)
Quadratic Equations • 103

Example

Using the quadratic formula, solve 3x 2 − 11x − 4 = 0.

This gives a = 3, b = −11 and c = −4.



√ √
− (−11) ± (−11)2 − 4 × 3 × (−4) 11 ± 121 + 48 11 ± 169 11 ± 13
So x = = = =
2×3 6 6 6
24 2 1
This gives x = or − , i.e. x = 4 or −
6 6 3
Example

Using the quadratic formula, solve 4x 2 + 7x + 2 = 0, giving the answers correct to 2


decimal places.

This gives a = 4, b = 7 and c = 2.


√ √ √
−7 ± 72 − 4 × 4 × 2 −7 ± 49 − 32 −7 ± 17 −7 ± 4. 123
So x = = = =
2×4 8 8 8
−7 + 4. 123 −7 − 4. 123
x= = −0. 36 or x = = −1. 39
8 8
Example
x+2 3
Using the quadratic formula, solve + = 7, giving the answers correct to 4
4 x−1
significant figures.
x+2 3
Multiplying throughout by 4(x − 1) gives 4 (x − 1) × + 4 (x − 1) × =
4 x−1
4 (x − 1) × 7

So
(x − 1) (x + 2) + 12 = 28 (x − 1)
x 2 + 2x − x − 2 + 12 = 28x − 28
So x 2 − 27x + 38 = 0

Putting a= 1, b = −27 and c = 38 into the formula gives x =


− (−27) ± (−27)2 − 4 × 1 × 38
2×1
√ √
27 ± 729 − 152 27 ± 577 27 ± 24. 0208
So x = = =
2 2 2
51. 0208 2. 9792
This gives x = = 25. 51 or = 1. 490.
2 2
104 • Mathematics

Questions
Solve the following equations using the quadratic formula.

Give the answers correct to 3 decimal places.

1. 2x 2 + 5x − 4 = 0 2. 5. 76x 2 + 2. 86x − 1. 35 = 0 3. 2x 2 − 7x + 4 = 0

3 5 x+1
4. 4x + 5 = 5. (2x + 1) = 6. = x−3
x x−3 x−1

Answers

1. 0.637, −3. 137 2. 0.296, −0. 792 3. 2.781, 0.719


4. 0.443, −1. 693 5. 3.608, −1. 108 6. 4.562, 0.438

Practical questions involving quadratic equations

Example

The area of a rectangle is 23.6 cm2 and its width is 3.10 cm shorter than its length.
Determine the dimensions of the rectangle, correct to 3 significant figures.

Let the length of the rectangle be L cm. Then the width is (L − 3. 10) cm.

Area = length × width = L × (L − 3. 10) = 23. 6

So L2 − 3. 10L − 23. 6 = 0

Using the formula:




− (−3. 10) ± (−3. 10)2 − 4 (1) (−23. 6) 3. 10 ± 9. 61 + 94. 4 3. 10 ± 10. 20
L= = =
2×1 2 2

13. 30 7. 10
So L = = 6. 65 cm or − = −3. 55 cm.
2 2
Length cannot be negative so L = 6. 65 cm and width = 6. 65 − 3. 10 = 3. 55 cm.

Check: 6. 65 × 3. 55 = 23. 6075 = 23. 6 correct to 3 significant figures.


Quadratic Equations • 105

Example

Calculate the diameter of a solid cylinder which has a height of 82.0 cm and a total
surface area of 2.0 m2 .

Total surface area = curved surface area + area of both circular ends.

So A = 2πrh + 2πr2 where r = radius and h = height.

Since the area is in m2 the height should be in metres. Therefore h = 0. 82 m.

So 2. 0 = 2πr (0. 82) + 2πr2 → 2πr2 + 2πr (0. 82) − 2. 0 = 0

Dividing by 2 throughout gives πr2 + πr (0. 82) − 1. 0 = 0

Using the quadratic formula with a = π, b = 0. 82π and c = −1 gives


 
− (0. 82π) ± (0. 82π)2 − 4π (−1) − (0. 82π) ± (0. 82π)2 + 4π
r= =
2π 2π

−2. 5761 ± 19. 2027
=
6. 2832

This gives either r = 0. 2874 m or r = −1. 1074 m.

So the radius of the cylinder is 0.2874 m, making the diameter 0.575 m, correct to 3
significant figures.

Example

The height, s metres, of a mass projected vertically upwards at time t seconds is


1
s = ut − gt2 , where u is the initial velocity and g is the acceleration due to gravity.
2
Take u as 30 m/s g as 9.81 m/s2 .

Determine how long the mass will take to reach a height of 16 m

(a) on the ascent,


(b) on the descent

When s = 16 m,
1
16 = 30t − (9. 81) t2
2
16 = 30t − 4. 905t2
4. 905t2 − 30t + 16 = 0
106 • Mathematics

Using the formula



√ √
− (−30) ± (−30)2 − 4 (4. 905) (16) 30 ± 900 − 313. 92 30 ± 586. 08
= =
2 (4. 905) 9. 81 9. 81
30 ± 24. 21
=
9. 81

54. 21
So either t = = 5. 53 s or t = 0. 59 s, correct to 2 decimal places.
9. 81
The answers are therefore (a) 0.59 s and (b) 5.53 s.

Example

Two ships sail from one port to another, a distance of 825 nautical miles, on the same
course, the speed of one ship being 4 knots faster than the other. The fast ship leaves
port 2 h after the slow ship and arrives at their destination 14 h sooner. Find the speeds
of the two ships.

Let x knots = speed of slow ship then (x + 4) knots = speed of fast ship.

Difference in times for the two ships to cover the journey = 2 + 14 = 16 h

An equation can be formed from the times taken on the journey:

Time taken by slow ship − Time taken by fast ship = 16 h


Distance
Time =
Speed
Distance travelled by slow ship Distance travelled by fast ship
therefore, − = 16
Speed of slow ship Speed of fast ship
825 825
So − = 16
x (x + 4)
Multiplying every term by x(x + 4) and simplifying,
825 825
· x · (x + 4) − · x · (x + 4) = 16 · x · (x + 4)
x (x + 4)
825 (x + 4) − 825x = 16x 2 + 64x
825x + 3300 − 825x = 16x 2 + 64x
3300 = 16x 2 + 64x
6x 2 + 64x − 3300 = 0

Dividing throughout by 16, x 2 + 4x − 206. 25 = 0


Quadratic Equations • 107

Solving this quadratic by formula:


−4 ± 42 − 4 × 1 × (−206. 25)
x=
2×1
√ √
−4 ± 16 + 825 −4 ± 841
= =
2 2
−4 ± 29 25 33
So x = = or − = 12. 5 or − 16. 5
2 2 2

The minus quantity is not realistic, the practical value of x is therefore 12.5.

Speed of slow ship = 12.5 knots, speed of fast ship = 16.5 knots.

Questions

Practical problems involving quadratic equations:

1
1. The angle a rotating shaft turns through in t seconds is given by θ = ωt + αt2 .
2
Determine the time taken to complete 4 radians if ω = 3. 0 rad/s and α = 0. 60 rad/s2 .
2. The power P developed in an electrical circuit is given by P = 10i − 8i2 , where I is the
current in amperes.
Determine the current necessary to produce a power of 2.5 W in the circuit.
3. The sag, x metres, in a cable stretched between two supports distance L metres apart
12
is given by L = + x.
x
Determine the sag when the distance between the supports is 20 m.
x2
4. The constant K is 1. 8 × 10−5 . Given that K = , determine x given that v = 10.
v (1 − x)
3x (20 − x)
5. The bending moment M at a point in a beam is given by M = , where x
2
metres is the distance from the point of support. Determine the distance x when the
bending moment is 50 Nm.
6. If two resistances, x and y, are connected in series their total resistance is x + y.
1 1 1
When connected in parallel their total resistance is such that = + .
R x y
Two resistors, when connected in series, have a total resistance of 40 ohms. When
connected in parallel their total resistance is 8.4 ohms. If one of these resis-
tors is Rx ohms: show that R2x − 40Rx + 336 = 0 and calculate the resistance
of each.
108 • Mathematics

Answers

1. 1.191 s 2. 0.345 A or 0.905 A 3. 0.619 m or 19.38 m


4. 0.0133 5. 1.835 m or 18.165 m 6. 12 ohms, 28 ohms

Simultaneous Equations – One Linear


and One Quadratic
Two simultaneous equations, one linear and the other quadratic, usually have two pairs
of solutions. Graphically, the equations give rise to a straight line and a curve. One
possibility is shown in Figure 5.1.

The solutions to the equations are the coordinates of the points of intersection.

If, however, the graphs resembled Figure 5.2, there would only be one pair of answers.

If the graph resembled Figure 5.3 there would be no solutions.

It must be noted that graphical means, unless performed by a computer, are gener-
ally inaccurate and time consuming, so an algebraic method is used. The form of the
equations defines the method to be used.

Example

(a) y = 2x 2 + 3x − 5 and y = 4x − 2
(1) y = 2x 2 + 3x − 5 and y = 4x − 2, so 2x 2 + 3x − 5 = 4x − 2
(2) Rearranging this equation gives: 2x 2 + 3x − 5 − 4x + 2 = 0

 Figure 5.1
Quadratic Equations • 109

 Figure 5.2

 Figure 5.3

(3) Resulting in the quadratic equation: 2x 2 − x − 3 = 0


(4) This factorises to give: (2x − 3) (x + 1) = 0
3
(5) Therefore x = or x = −1
2
3 3
(6) y = 4x − 2, so when x = , y = 4 × − 2 = 4
2 2
(7) x = −1, y = 4 × (−1) − 2 = −6
 
3
(8) The two pairs of answers are , 4 and (−1, −6)
2
 
−b ± b2 − 4ac
(9) Using the formula x =
2a
[a] In this case, a = 2, b = −1 and c = −3, giving

− (−1) ± (−1)2 − 4 × 2 × (−3)
x=
2×2
110 • Mathematics

 
+1 ± (+1 + 24) 1 ± (25) 1 ± 5 1 + 5 1−5 6
[b] So x = = = = and = and
4 4 4 4 4 4
−4
4
3
[c] This gives x = and x = −1, the question then continues as shown at (6)
2
above

(b) 2x 2 + y2 = 3 and x + y = 2

(1) The substitution method works best for this type of question.
(2) x + y = 2 means that y = 2 − x
(3) Substituting this into 2x 2 + y2 = 3 gives 2x 2 + (2 − x)2 = 3
(4) Expanding the brackets and collecting terms gives
 
(5) 2x 2 + (2 − x)2 = 3 ⇒ 2x 2 + 4 − 4x + x 2 = 3 ⇒ 3x 2 − 4x + 1 = 0
1
(6) This factorises to give (x − 1) (3x − 1) = 0 so x = 1 and x = (or use formula)
3
1 1 5
(7) y = 2 − x so when x = 1, y = 2 − 1 = 1, and when x = , y = 2 − =
  3 3 3
1 5
(8) The two pairs of answers are (1, 1) and ,
3 3

Before the section on the solution of cubic equations is discussed, it will be useful to go
over the work on ‘Polynomial division’.

Recap: Polynomial Division


A polynomial is an expression of the form f (x) = a + bx + cx 2 + dx 3 + K. Polynomial
division is sometimes required when solving equations with high powers.

Example

Divide 2x 2 + x − 3 by x − 1

2x 2 + x − 3 is called the dividend and x − 1 the divisor. The usual layout is shown below
with both the dividend and divisor shown in descending order of powers.
Quadratic Equations • 111

2x + 3
x − 1 2x 2 + x − 3
2x 2 − 2x
3x − 3
3x − 3
0

Reasoning

(1) Dividing the first term of the dividend by the first term of the divisor.
2x 2
(2) That is, gives 2x, which is put above the first term of the dividend as shown.
x
(3) The divisor is then multiplied by 2x, i.e., 2x(x − 1) = 2x 2 − 2x, which is then placed
under the dividend as shown.
(4) Subtracting gives 3x and the −3 is ‘dropped down’ to give 3x − 3.
(5) The process is then repeated, that is, the first term of the divisor, x, is divided into
3x, giving +3, which is placed above the dividend as shown.
(6) Then 3(x − 1) = 3x − 3 which is placed under the 3x − 3.
(7) The remainder on subtraction is 0, which completes the division.
 
(8) Therefore 2x 2 + x − 3 ÷ (x − 1) = (2x + 3)

A check can be made by multiplying x − 1 by 2x + 3.

For further examples see Chapter 2.

Function Notation
In most cases seen thus far an equation has usually been given as y = · · · , where the
expression following the equals sign usually is an equation in terms of x. In simple terms,
this is saying that y is a function of x, that is, the value of y is related to the value of x by
the equation.

This can be written as y = f (x) where f (x) = · · · .

For example, saying y = x 2 is the same as saying f (x) = x 2 .

If the value of y, when x = 3, has to be found it is much simpler to write ‘What is f (3)?’
than ‘Find the value of y when x = 3’.
112 • Mathematics

Questions

For f (x) = x 2 − 3x + 4, find:

1. f (1) 2. f (2) 3. f (−1) 4. f (0) 5. f (−3)

For f (x) = x 3 + 2x 2 + 3x + 1, find:

6. f (−1) 7. f (−2) 8. f (3) ÷ f (2) 9. f (1) ÷ f (0) 10. f (3) × f (−3)

Cubic Equations
A cubic equation is one which contains the cube of the unknown, so x 3 = 8 is a cubic
equation of the simplest form and is solved simply by taking the cube root of both sides.

Therefore x = 3 8 = 2.

However, the form usually associated with the title cubic equation contains either or
both of the first and second powers of the unknown as well as its cube.

Example

x 3 + 2x 2 − x = 2.

One method of solution is to bring all terms of the equation to one side of the equals
sign, leaving 0 on the other side. This expression can, hopefully, be resolved into its
three factors and each of these factors are then, in turn, equated to 0 which produces a
value of the unknown that will satisfy the given equation.

Example

x 3 + 2x 2 − x − 2 = 0

The three factors of this expression are: (x − 1), (x + 1) and (x + 2)

By equating each factor to 0, the three roots are obtained.

x−1= 0 ∴x=1
x+1=0 ∴ x = −1
x+2=0 ∴ x = −2
Quadratic Equations • 113

The roots of this equation are +1, −1 and −2, and any one of these values of x will
satisfy the given equation.

If the equation is not a very easy one like the above, the factors will not be readily seen.

The following theorem proves to be very useful.

The Factor Theorem


A statement of the theorem is

‘if x = α is a root of the equation f (x) = 0, then (x − α) is a factor of f (x)’

Example

Factorise x 3 − 7x − 6 and use the answers to solve the cubic equation x 3 − 7x − 6 = 0.

Solution

Let f (x) = x 3 − 7x − 6

If x = 1, then f (1) = 13 − 7(1) − 6 = −12


If x = 2, then f (2) = 23 − 7(2) − 6 = −12
If x = 3, then f (3) = 33 − 7(3) − 6 = 0

Since f (3) = 0, (x − 3) is a factor of f (x).

At this point there is a choice of methods to follow.

Either f (x) can be divided by (x − 3) to give x 2 + 3x + 2, and this can be factorised on


sight to give the other two factors (x + 2) and (x + 1).

Alternatively, other values of x can be tried. As x gets larger, the cubed term predomi-
nates and f (x) gets larger. It therefore makes sense to try negative values of x.

If x = −1, f (−1) = (−1)3 − 7 (−1) − 6 = −1 + 7 − 6 = 0

So (x = −1) = (x + 1) is a factor of f (x).

If x = −2, f (−2) = (−2)3 − 7 (−2) − 6 = −8 + 14 − 6 = 0

So (x = −2) = (x + 2) is a factor of f (x).


114 • Mathematics

Therefore x 3 − 7x − 6 = (x − 3) (x + 1) (x + 2) which gives the solutions to f (x) = 0 as


x = 3, −1 and −2.

Questions
Use the factor theorem to factorise:

1. x 2 + 2x − 3 2. x 3 + x 2 − 4x − 4 3. 2x 3 + 5x 2 − 4x − 7
4. 2x 3 − x 2 − 16x + 15 5. x 3 + 4x 2 − x − 6 6. x 3 − 2x 2 − x + 2

Answers

1. f (1) = 0; f (−3) = 0 so (x − 1) (x + 3)

2. f (−1) = 0; f (−2) = 0; f (2) = 0 so (x + 1) (x + 2) (x − 2)


 
3. f (−1) = 0; no other integers so (x + 1) 2x 2 + 3x − 7

4. f (1) = 0; f (−3) = 0; f (2. 5) = 0 so (x − 1) (x + 3) (2x − 5)

5. No integer values so does not factorise

6. f (−1) = 0; f (2) = 0; f (1) = 0 so (x + 1) (x − 2) (x − 1)

In such cases, attempt to get one solution by trial and error and from this get the first fac-
tor. Dividing the cubic equation by this factor will produce a quadratic equation which
can be solved by factors or by quadratic formula. The following examples show how this
is done.

Example

Find the roots of the equation: x 3 − x 2 = 5. 75x − 7. 5.

Re-arrange with all terms on one side, in descending powers of x

x 3 − x 2 − 5. 75x + 7. 5 = 0

Find the first root by trial

Try x = 1, 13 − 12 − 5. 75 × 1 + 7. 5 = 1 − 1 − 5. 75 + 7. 5 = 1. 75
Try x = 2, 23 − 22 − 5. 75 × 2 + 7. 5 = 8 − 4 − 11. 5 + 7. 5 = 0 so this is one solution.

x = 2 satisfies the equation, therefore x − 2 must be a factor.


Quadratic Equations • 115

Divide this factor into the cubic expression, this will produce a quadratic whose roots
can be found.

x 2 + x − 3. 75
x−2 x 3 − x 2 − 5. 75x + 7. 5
x 3 − 2x 2
+1x 2 − 5. 75x + 7. 5
+1x 2 − 2x
−3. 75x + 7. 5
−3. 75x + 7. 5
0

Equate the resulting quadratic to 0 and, if the factors cannot be seen, solve by formula,
x 2 + x − 3. 75 = 0.

So a = 1, b = 1 and c = −3. 75

−1 ± (1)2 − 4 × 1 × (−3. 75)
x=
2×1
√ √
−1 ± 1 + 15 −1 ± 16
= =
2 2

3 5
So x = and −
2 2

x 2 + x − 3. 75 = 0

3 5
The roots of the given equation are 2, and − .
2 2
The result could be checked by substituting each of these roots into the given cubic
equation.

Example
3
Solve the following equation: 2x 2 = 17 +
x
Multiply throughout by the LCM, which is x, to eliminate the fraction:

2x 3 = 17x + 3
116 • Mathematics

Arrange all terms on left-hand side and equate to 0,

2x 3 − 17x − 3 = 0

The roots must multiply together to give −3, so choose the factors of 3 as first choices.

Find the first root by trial, try x = 1,

2 × 13 − 17 × 1 − 3 = 2 − 17 − 3 = −18

Try x = 3,
2 × 33 − 17 × 3 − 3 = 54 − 51 − 3 = 0

Hence x = 3 is a root and x − 3 must be a factor.

Divide the cubic equation by this factor to obtain a quadratic which can be solved,

2x 2 + 6x + 1
x − 3 2x 3 − 0x 2 − 17x − 3
2x 3 − 6x 2
+ 6x 2 − 17x − 3
+ 6x 2 − 18x
+x − 3
+x − 3
0

The other two roots can be found by equating the resulting quadratic to zero and
 
−b ± b2 − 4ac
solving by using the formula x =
2a
 
−6 ± 62 − 4 × 2 × 1
In this case, a = 2, b = 6 and c = 1, giving x =
2×2
 
−6 ± (36 − 8) −6 ± (28) −6 ± 5. 2915
So x = = = = −0. 1771 and −2. 8229
4 4 4
Therefore, the roots of the given cubic equation are 3, −0. 1771 and −2. 8229.

Note the last two examples.

In the former the last term of the cubic equation is 7.5 and easy factors of this number
which come readily to mind are 1 and 7.5, 2 and 3.75, 3 and 2.5, 5 and 1.5; all being
Quadratic Equations • 117

either positive or negative, and the first root of this equation, by trial, should be one of
these.

Trial figures for the first root should therefore be chosen with this in mind.

If the first root is not a number which can readily be found by trial as in the examples,
the process can be quite labour intensive and the equation would probably be more
easily solved by plotting an accurate graphical.

Test Examples 5
1. Find the value of x in each of the following equations:
(i) (2x + 8)(3x − 5) = 0
(ii) (0. 5x − 10)(0. 25x + 5) = 0
(iii) (5x + 0. 5)(4x + 0. 8) = 0

2. Solve the following equations by the method of factorising:


(i) 3x 2 + 2x − 33 = 0
(ii) 4x 2 − 17x + 4 = 0
(iii) 12x 2 + 10x − 12 = 0

3. Solve the following equations by the method of completing the square:


15
(i) x 2 − x − =0
4
(ii) 3x 2 + 2x − 1 = 0
(iii) 4x 2 − 9x + 2 = 0

4. Solve the following equations by the quadratic formula:


(i) 3x 2 − 2x + 0. 25 = 0
(ii) 5x 2 + 4x − 5. 52 = 0
(iii) 10x 2 − x − 0. 2 = 0

5. Find the value of x when: log (0. 5x) = 2 × log (x − 6)

6. Find the value of b in the equation: 6b4 − 2. 46b2 + 0. 24 = 0

7. Find the value of V in the equation: V 2.8 − 5. 1V 1.4 + 5. 6 = 0


118 • Mathematics

8. Find the values of x and y in the following simultaneous equations:

x 2 − xy + 2y2 = 1
x + 2y = 8

9. In 5 h less time than it takes a certain ship to travel 330 nm, another ship which is
31/2 knots faster can travel 4 nm further. What are the speeds of the two ships?

10. The area of a rectangle is 76 cm2 and the perimeter is 350 mm.
Find the length and breadth.

11. Find the roots of the following cubic equation: 2x 3 + 3x 2 − 17x = 30.

16 3
12. Find the values of x to satisfy the following equation: 2x − 2− = 3.
x x
GRAPHS
6
A graph is a diagram which shows the relationship between two quantities.

Graphs are usually plotted on squared paper (metric).

Taking the related quantities to be x and y, Figure 6.1 shows the elements of plotting
graphs.

Both quantities vary in value throughout the graph and the value of one depends upon
the value of the other. In practice there are often cases where the values of one of the
quantities are entirely dependent upon how the other is varied.

Take for example the stretching of a spring, if a load is suspended on the spring a certain
amount of stretch takes place, if a heavier load is suspended a greater stretch occurs, so
the amount of stretch depends on the magnitude of the load. This is called Hooke’s Law.

In this example, the load would be referred to as the independent variable and the
stretch as the dependent variable.

In the drawing of a graph of a function of x such as x 2 − 2x + 5, let the value of this be


denoted by y and so y = x 2 − 2x + 5. For a series of chosen values of x the values of y are
calculated to obtain a number of plotting points to enable the graph to be drawn.

Choose the values of x and the values of y depend upon these, hence x is the
independent variable and y the dependent variable.

The larger the scale to which the graph is drawn the more accurate the results obtained
when reading points from the graph, therefore the scale chosen should be as large as
possible depending upon the highest and lowest values to be represented.

The various values of x and y (or other similar pairs of variable quantities) are plotted
thus, referring to Figure 6.1.

The point of intersection of the horizontal and vertical base lines, marked 0, is the com-
mon 0 point for both quantities and therefore represents 0 value for both x and y.
120 • Mathematics

4 y

Positive values of y
3

1
x
–6 –4 –2 2 4 6

Negative values of y
Negative values of x –1 Positive values of x

–2

–3

–4

 Figure 6.1

Positive values of x are measured horizontally to the right from 0 and negative val-
ues are measured horizontally to the left. The horizontal base line is referred to as the
xx axis. Horizontal measurements are called abscissae (singular: abscissa). Positive val-
ues of y are measured vertically above 0 and negative values are measured vertically
downwards from 0. The vertical base line is the yy axis. Vertical measurements are called
ordinates.

Plotting of Straight-Line Graphs


Consider an equation of the form y = a + bx where a and b are any constant quantities
and may be positive or negative.

Example

Let a = 2 and b = 1. 5, so y = 2 + 1. 5x

For various values of x between the limits of, say, x = −2 and x = +4, the y values are
calculated as:

When x = −2, y = 2 − 3 = −1
When x = −1, y = 2 − 1. 5 = +0. 5
When x = 0, y = 2 + 0 = +2
Graphs • 121

10 y

6
y = 2 + 1.5x

x
–2 –1 1 2 3 4

–2

 Figure 6.2

When x = +1, y = 2 + 1. 5 = +3. 5


When x = +2, y = 2 + 3 = +5
When x = +3, y = 2 + 4. 5 = +6. 5
When x = +4, y = 2 + 6 = +8

These co-ordinates can be plotted and a graph drawn through the plotted points as in
Figure 6.2.

Note

(i) The graph is a straight line, and with this knowledge the graph of any equation of
the form y = a + bx may be plotted with two pairs of values only to give two points
through which to draw the straight line. (Although it is best to use three in case a
mistake has been made in the calculations.)
(ii) From left to right the graph slopes upwards and the value of b determines the
gradient of the slope.
In this case, for every increase of 1 in x value there is an increase of 1.5 in the y value;
if b had been greater than 1.5 the slope would have been greater and vice-versa.
The graph slopes upwards because b is positive, if b had been negative the graph
would have sloped downwards.
(iii) The value of y when x is 0 is 2, and this is the value of a.
It is the intercept on the y axis.
122 • Mathematics

16 y
y=4+x
14
12
10
y = 3 + 0.5x
8
6
4
2 y = 2 – 0.25x
x
1 2 3 4 5 6 7 8 9 10 11 12
–2 y = –0.5x
–4
–6 y = –2 – 0.5x
–8

 Figure 6.3

Now consider the equations,

• y =4+x
• y = 3 + 0. 5x
• y = 2 − 0. 25x
• y = −0. 5x
• y = −2 − 0. 5x

All of these are of the general form y = a + bx.

Graphs of these equations are plotted in Figure 6.3 between the limits x = 0 and x = 12.

Examine these graphs carefully.

Determining the Equation to a


Straight-Line Graph
Straight-line graphs are examples of direct or indirect proportion.

Another way the equation of a straight line can be written is y = mx + c where y is the
‘vertical’ variable and x is the ‘horizontal’ variable.

The value of m gives the slope of the graph and the value of c gives the intercept on the
y axis that is, the line where x = 0.
Graphs • 123

If the graph slopes from bottom left to top right, the slope is positive; if it slopes from
top left to bottom right the slope is negative.

To calculate the slope once the graph has been drawn:

• draw a right-angled triangle using the straight line as the hypotenuse;


• calculate the lengths of the vertical and horizontal sides (not their physical lengths
but what their lengths represent on the scales on the axes);
• find the value of m by dividing the vertical figure by the horizontal.

If the graph does not have the vertical axis at x = 0, the value of c cannot be read off the
scale but has to be calculated.

• Re-arrange the equation to give c = y − mx.


• Choose any point on the straight line and find its x and y co-ordinates.
• Substitute these values along with the value of the slope, m, into the formula to find
the value of c.

Example

The following table shows the cost of printing circuit boards (Figure 6.4).

Number printed 5 10 15 25 30
Cost (pence) 35 60 85 135 160

175 Cost, y
Cost of printing circuit boards
150
x
125
B
100

75

50
x
25
A Number of printed, x
–5 5 10 15 20 25 30

 Figure 6.4
124 • Mathematics

Represent the information graphically using appropriate scales.

(i) Where does the line cross the x and y axes?


(ii) What is the equation of the line?

(i) By accurate plotting, the values where the line crosses the axes are (−2, 0) and
(0, 10) for the x and y axis respectively.
(ii) To find the slope, m, choose any two points, A and B say.
The co-ordinates of A are (5, 35) and B (25, 135).
The vertical difference is 135 − 35 = 100.
The horizontal difference is 25 − 5 = 20.
Vertical difference 100
The slope is m = = =5
Horizontal difference 20
The intercept has already been found, i.e., c = 10
The equation of the line is therefore y = 5x + 10
(Check the equation by choosing a point, say (15, 85). 85 = 5 × 15 + 10)

Questions

Calculate the equation of the straight line that best represents the data given.

1. The table gives a set of readings obtained when a mass of x (g) is hung from a spring,
causing it to extend to a length y (cm):

x (g) 10 40 80 100 120 150


y (cm) 116 123 144 152 160 173

Answer: y = 0. 44x + 107


2. The electrical resistance of various lengths of wire are given in the following table:

Length x (mm) 110 120 150 160 190 210


Resistance y (ohms) 3.3 3.6 4.5 4.8 5.7 6.3

Answer: y = 0. 03x
Graphs • 125

3. The following is a comparison table between temperature measured in degrees


Fahrenheit and degrees Celsius:

◦ C(x) 10 25 50 60 75 100
◦ F(y) 50 77 122 140 167 212

Answer: y = 1. 6x + 32
4. The table shows the length of a metal rod at different temperatures. It is assumed
that the equation is L = m · t+ c

t (◦ C) 0 10 20 30 40 50 60
L (mm) 100 100.02 100.04 100.06 100.08 100.10 100.12

Answer: L = 0. 02t + 10

Example

Find the equation of a straight line graph which passes through the points (2, 4) and
(10, 7).
Vertical difference 7−4 3
The slope is m = = = = 0. 375.
Horizontal difference 10 − 2 8

The intercept c = y − mx where (x, y) is one of the points, say (2, 4).

So c = 4 − 2 × 0. 375 = 4 − 0. 75 = 3. 25.

The equation is therefore y = 0. 375x + 3. 25.

There are many practical applications of determining the equation to a given graph
such as when performing a series of experiments on a machine, an engine, or a
piece of material under a strength test, to ascertain how one quantity varies with
another.

For instance, various loads can be hung from the lifting hook of a lifting machine and
the effort required to lift each load is found experimentally and tabulated. A graph is
then drawn showing how the effort varies with the load. The equation to the graph is
determined and this is the law of the particular machine.
126 • Mathematics

Example

In an experiment on a certain machine the following data were observed:

Load, m 20 40 60 80 100 120


Effort, F 4.5 6.1 7.8 8.9 10.4 12.1

If the law of this machine expressing the relationship between effort applied (F) and
load lifted (m) can be expressed as the linear equation F = a + bm, plotting a graph rep-
resenting the above experimental values with a horizontal scale of load will determine
the linear law of this machine.

After plotting the experimental values a straight line is drawn as near as possible
through these points as shown in Figure 6.5. Those points not exactly on this line
are probably due to irregularities of the machine or errors of observation during the
experiment.

The value of a reads 3

The value of b is the increase of effort per unit increase of load.

Choosing two points on the line, P1 and P2 , Q to P2 measures 7.5, P1 to Q measures 100,
therefore b = 7. 5 ÷ 100 = 0. 075.

Hence the linear law of this machine is F = 3 + 0. 075m.

15 F

Load (m) against effort (F )

P2
F = 0.075m + 3
10

7.5

5
Q
P1 100

m
20 40 60 80 100 120 140

 Figure 6.5
Graphs • 127

Graphical Solution of Simultaneous


Linear Equations
The method of solving simultaneous equations by graphical means can best be demon-
strated by an example:

To find the values of x and y which satisfy the equations,

2x + 5y = 34
and 4x − 3y = 16

Find the value of y in the equation

2x + 5y = 34
5y = 34 − 2x
y = 6. 8 − 0. 4x (i)

Find the value of y in the second equation

4x − 3y = 16
−3y = 16 − 4x
4 16
y= x− (ii)
3 3
Two points are required for each line so for y = 6. 8 − 0. 4x

when x = 0, y = 6. 8 − 0 = 6. 8
when x = 10, y = 6. 8 − 4 = 2. 8

which gives the points (0, 6.8) and (10, 2.8)


4 16
For y = x −
3 3
16
when x = 0, y = −
3
4 16 36 16 20
when x = 9, y = × 9 − = − =
3 3 3 3 3
   
16 20
which gives the points 0, − and 9,
3 3
128 • Mathematics

8 y

6 y = 6.8 – 0.4x

x
2 4 6 8 10 12

–2
y = 1.33x – 5.33

–4

–6

 Figure 6.6

The points x = 0 and x = 10 for the first equation, and x = 0 and x = 9 for the second
equation were chosen simply because they appeared to be easy figures for substituting
and to produce reasonably sized graphs. x = 0 is obviously the first choice for a plotting
point of any graph, the other point could be any value of x but it should be chosen with
a view to produce simple figures.

The graph is now plotted as shown in Figure 6.6.

At the point of intersection of the lines the values are x = 7 and y = 4 and these are the
only values which are true for both equations.

Hence, x = 7 and y = 4.

Graphical Solution of Quadratic


Equations
Graphs of equations containing x to the first power are all straight lines.

Graphs of equations containing x to other powers, such as x 2 (quadratic equations)


and x 3 (cubic equations) are curves, and obviously more than two plotting points are
Graphs • 129

necessary as a guide to the drawing of a curve; the more plotting points the more
accurate the curve can be drawn.

As in previous cases the equation to be solved is first simplified, all terms are brought
to one side in order of descending powers of the unknown, say x, leaving 0 on the side.
Replace the nought by y, that is, y = the given expression and plot a graph for a series
of values of x.

Where this graph intersects the x axis (the line y = 0) are the values of x which will satisfy
the equation.

The values of y in proximity to these points therefore change from positive to nega-
tive or from negative to positive, the trial values of x for calculating the plotting points
for the curve should therefore be chosen with this is in mind. The following worked
examples will clarify this explanation.

Example

Find, graphically, the values of x which solve the equation, x 2 + 2x = 5. 5x − 1. 96

Simplify and bring all terms to one side, so

Let y = x 2 − 3. 5x + 1. 96

Calculate values of y for selected values of x

When x = 0, y = 0 − 0 + 1. 96 = +1. 96
When x = 1, y = 1 − 3. 5 + 1. 96 = −0. 54
When x = 2, y = 4 − 7 + 1. 96 = −1. 04
When x = 3, y = 9 − 10. 5 + 1. 96 = +0. 46
When x = 4, y = 16 − 14 + 1. 96 = +3. 96

Note that the value of y changes sign between x = 0 and x = 1, and again between x = 2
and x = 3, therefore the two values of x will be obtained from the graph at these two
intersections of the x axis and hence there is no need to plot the graph beyond these
limits.

The larger the scale the more accurate will be the reading of the values of x. Therefore
the graph should be plotted to the largest scale possible on the paper.
130 • Mathematics

3 y

x
1 2 3

–1

–2

 Figure 6.7

Results of greater accuracy could be obtained by calculating values of y for a few points
between x = 0 and x = +1, and also between x = +2 and x = +3 and drawing to a
larger scale only those two parts of the curve which cross the x axis.

By drawing the graph as shown in Figure 6.7, when y = 0, x = 0. 7 and 2.8, the graph is
concave upwards (i.e. +x 2 ).

Example

Solve x 2 − 0. 8x − 3. 84 = 0

Let y = x 2 − 0. 8x − 3. 84

When x = 0, y = 0 − 0 − 3. 84 = −3. 84
When x = 1, y = 1 − 0. 8 − 3. 84 = −3. 64
When x = 2, y = 4 − 1. 6 − 3. 84 = −1. 44
When x = 3, y = 9 − 2. 4 − 3. 84 = +2. 76

Note that the value of y changes sign from negative to positive between x = 2 and
x = 3, therefore there is no need to proceed further in this direction. The other value
where change of sign takes place, that is, from positive to negative, must be when x is a
negative quantity.
Graphs • 131

Proceeding then in this direction,

When x = −1, y = 1 + 0. 8 − 3. 84 = −2. 04


When x = −2, y = 4 + 1. 6 − 3. 84 = +1. 76

Knowing that y = 0 between x = −1 and −2, there is sufficient data to plot the graph. It
may be of assistance to clear up the above by tabulating these results before attempting
to plot them. Thus

x −2 −1 0 +1 +2 +3
y +1. 76 −2. 04 −3. 84 −3. 64 −1. 44 +2. 76

Plotting the graph as in Figure 6.8 and reading the values of x when y = 0, x = 2. 4 or
−1. 6

Another method of solving quadratic equations, and one which can often be applied to
solve cubic and more complicated equations is demonstrated by the following example.

Example
7
To solve x 2 − 4x + =0
4
 
7
This can be written as x 2 − 4x − =0
4

3 y

1
x = – 1.6 x = 2.4 x
–2 –1 1 2 3
–1

–2

–3

–4

–5

 Figure 6.8
132 • Mathematics

 
7 7
So let y1 = x2
and let y2 = 4x − then x2 − 4x − =0 becomes y1 − y2 = 0 or,
4 4
more importantly, y1 = y2 .

In other words the solutions are the x ordinates of the intersection of the graphs of y1
and y2

Graph plotting points are found for each part:

y1 = x 2 (i)

When x = 0, y1 = 0
When x = 1, y1 = 1
When x = 2, y1 = 4
When x = 3, y1 = 9
When x = 4, y1 = 16

7
y2 = 4x − (ii)
4

This is a straight line equation and two plotting points only are required plus one for
checking purposes.
7
When x = 0, y2 = −
4
7 25
When x = 2, y2 = 8 − =
4 4
7 57
When x = 4, y2 = 16 − =
4 4
7
The curve of y1 = x 2 and the straight line representing y2 = 4x − are now plotted as
4
shown in Figure 6.9.

From the graph it is seen that the curves intersect when x = 0. 5 and x = 3. 5.
7
This means x 2 − 4x + = 0 when x = 0. 5 and x = 3. 5.
4
Always check results by substituting them into the original equation to see if they are
correct.
Graphs • 133

18 y

16
7
14 y2 = 4 x −
4
12

10

6
y1 = x 2
4

1 2 3 4 5
–2

–4

x = 0.5 x = 3.5

 Figure 6.9

Graphical Solution of Simultaneous


Quadratic and Linear Equations
These are dealt with in a similar manner as the last example.

Two quadratic equations could be involved, or one quadratic and one linear equation.

Example

Find the values of x and y which satisfy the equations

y = 12 + 3x − 0. 5x 2 and
y = 14 − 1. 25x

The extreme values of x to be taken for calculating the plotting points are chosen by
making estimates to where the two graphs cross.
134 • Mathematics

Taking the linear equation first, being a straight line graph only three points are needed:

y = 14 − 1. 25x

when x = 0, y = 14 − 0 = 14
when x = 5, y = 14 − 6. 25 = 7. 75
when x = 10, y = 14 − 12. 5 = 1. 5

Taking the quadratic: y = 12 + 3x − 0. 5x 2

when x = 0, y = 12 + 0 − 0 = 12
when x = 2, y = 12 + 6 − 2 = 16
when x = 4, y = 12 + 12 − 8 = 16
when x = 6, y = 12 + 18 − 18 = 12
when x = 8, y = 12 + 24 − 32 = 4
when x = 9, y = 12 + 27 − 40. 5 = −1. 5

This appears to be sufficient. The graph is convex upwards because the x 2 term is
negative.

18 y
16
14
P1
12
10
8
6
4
P2
2
x
1 2 3 4 5 6 7 8 9 10 11 12
–2
–4

 Figure 6.10
Graphs • 135

Drawing the graph as in Figure 6.10, the points of intersection P1 and P2 produce the
following values of x and y:

x = 0. 5 and y = 13. 375 or


x=8 and y=4

Determination of laws
One case has already been shown, that of determining the law connecting effort and
load in a lifting machine. This relation was expressed by a straight line equation and the
law found quite simply.

In experimental work when two variables are thought to be connected, a set of mea-
surements can be taken, plotted as a graph and examined to see if such a link exists. If
a link does exist, the equation of the link can be found and used to determine further
values.

If the points, when plotted, lie on a straight line then the relationship is, y = a + bx,
where b is the slope and a the intercept on the y axis. However, in the majority of
cases the points will not lie on a straight line but on a curve. In such cases the non-
linear equation can be modified to produce a linear one, enabling the constants in the
equation to be found.

Some of the more common types are given below. If it suspected that the link is:

1. y = ax 2 + b
Plotting y against x 2 should result in a straight line where the slope is a and the
intercept is b.
a
2. y = + b
x
1
Plotting y against should result in a straight line where the slope is a and the
x
intercept is b.
3. y = a · x n
Taking logarithms (usually base 10, but not essentially) gives:
 
log(y) = log a · x n so
 
log(y) = log(a) + log x n so
log(y) = log(a) + n. log (x)
136 • Mathematics

Plotting log(y) against log(x) should result in a straight line where the slope is n and
the intercept is log(a).
4. y = a · bx
Taking logarithms (usually base 10, but not essentially) gives:
 
log(y) = log a · bx so
 
log(y) = log(a) + log bx so
log(y) = log(a) + x. log(b)

Plotting log(y) against x should result in a straight line where the slope is log(b) and
the intercept is log(a).
5. y = A · ebx
Taking natural logarithms gives:
 
ln(y) = ln A · ebx so
ln(y) = ln(A) + bx. ln(e) so
ln(y) = ln(A) + bx

Plotting ln(y) against x should result in a straight line with slope b and intercept ln(A).

One typical example is that of a mass of a gas being expanded or compressed in a cylin-
der. The law connecting the variation of pressure as the volume of the gas is increased
or decreased by the movement of the piston is: p · V n = constant, C.

Writing this equation in log form (as in 3. above):

log P + n log V = log C

and transposing to express log p in terms of the other quantities:

log P = log C − n log V

It is now reduced to a straight line equation similar to y = a + bx

the variable log p taking the place of the variable y

the variable log V taking the place of the variable x

the constant log C taking the place of the constant a

the constant n taking the place of the constant b


Graphs • 137

A graph of log p against log V is now plotted and the values of n and C are determined
from the slope and intercept.

Example

The following ordinates and abscissae were measured from part of the expansion curve
of an indicator diagram off an I.C. engine, where p is the pressure and V the volume of
the gases in the cylinder.

If the law of expansion can be expressed by p · V n = C estimate the value of n.

p 28 24 16.6 13 9.7 6.8 4.7


V 0.6 0.7 0.9 1.1 1.4 1.8 2.4

Tabulating the values of p and V with their respective logarithms:

p log(p) V log(V)
28.0 1.4472 0.6 −0.2218
24.0 1.3802 0.7 −0.1549
16.6 1.2201 0.9 −0.0458
13.0 1.1139 1.1 0.0414
9.7 0.9868 1.4 0.1461
6.8 0.8325 1.8 0.2553
4.7 0.6721 2.4 0.3802

The graph is now plotted as shown in Figure 6.11. Note that the lowest value of log p is
0.6721, the graph can be drawn to a larger scale by starting with a value of log p just a
little lower than this, say 0.6, instead of commencing with 0 origin.

Choosing two points on the graph,

decrease in log (p) 0. 65


n= = = 1. 3
increase in log (V) 0. 5

Therefore n = 1. 3

Thus, log p = log C − 1. 3 log V

In the original equation p · V 1.3 = C


138 • Mathematics

1.5 log(p)

1.4

1.3

1.2

1.1
0.65
1

0.9

0.8
0.5
0.7
log(V )
0.6
0

 Figure 6.11

Curve Sketching
It is often useful to sketch the graph by inspection of the algebraic equation.

From previous work, the general form y = a + bx:

this is a straight line graph, +b slopes upwards, −b slopes downwards, the greater the
value of b the steeper the slope, a is the intercept on the y axis (when x = 0).

For the quadratic equation, general form is y = ax 2 + bx + c:

this is a parabolic graph, +a convex upwards, the greater the a value the steeper the
slope (or the ‘narrower’ the curve):
b
The line of symmetry of the parabola is the straight (vertical) line x = − :
2a
If the parabola
√ intersects the x axis, the roots of the equation (i.e. when y = 0) are at the
b2 − 4ac
values to the left and right of the line of symmetry.
2a

−b ± b2 − 4ac
In other words, at the values x = (see section on solution of quadratic
2a
equations).
Graphs • 139

Test Examples 6
1. On the same set of axes, plot the following four graphs represented by the
equations:
(i) y = 2 + x
(ii) y = 12 − 1. 5x
(iii) y = −1 − 0. 5x
(iv) y = −4 + 1. 25x
all between the limits of x = 0 and x = 12.
2. A straight line passes through the pair of points (−2, 14.5) and (8, −3).
From it derive the equation to the graph:
3. Plot a graph using the following values and find the equation of the graph.

x −2 −1 0 1 2 3
y 10 7 4 1 −2 −5

4. The following data were taken during an experiment on a small turbine where P
represents the power developed and m the rate of consumption of steam.
Assuming that the relationship between P and m can be represented by the
straight line equation m = a + bP, draw a straight line as near as possible through
the plotted points of the experimental results, estimate the values of a and b and
hence the law connecting p and m.

P 20 25 30 35 40 45 50
m 220 265 315 365 410 455 505

5. Find, graphically, the values of x and y which satisfy the simultaneous equations,

3x + 5y = 23 and, 5x − 2y = 12. 5

6. Find, graphically, the values of p and q in the simultaneous equations,

5p − 2q = 5. 6 and
2p − 3q = −4. 8

21
7. Find, graphically, the value of x in the equation, x 2 − 5x + = 0.
4
140 • Mathematics

8. Draw the graph of y = 0. 5x 2 − 2x − 6 between the values of x = −4 and x = +8.


From the graph read the values of x in the following equations:
(i) 0. 5x 2 − 2x − 6 = 0
(ii) 0. 5x 2 − 2x − 4 = 0
(iii) 0. 5x 2 − 2x − 1 = 0
(iv) 0. 5x 2 − 2x = 0
(v) 0. 5x 2 − 2x + 1 = 0
9. On the same set of axes draw graphs of y1 = x 2 and y2 = 3. 5 + 2. 5x between the
values of x = −2 and x = +4; find the values of x in the equation x 2 −2. 5x −3. 5 = 0.
10. Find, by graphical means, the values of x and y which satisfy the simultaneous
equations,

y = 0. 4x 2 − 3x + 2 and
y = 1. 4x − 2

11. Using graphical means only and taking values of x between 0 and +6, solve the
following pair of simultaneous equations for x and y:

y 2 = 16x and
y = 5 + 8x − 2x 2

Note: y2 = 16x can be expressed as y = ±4 x
Suggested scales: x axis 2 cm = 1 unit, y axis 2 cm = 4 units.
12. By drawing a graph of y = e−x and y = x 2 solve the equation:

x 2 · ex = 1

Note: plot values between x = 0 and x = 1.


TRIGONOMETRY
7
AND GEOMETRY
An angle is the corner of two joining lines and the magnitude of an angle is measured
in either degrees or radians.

Measurement of Angles
The most common unit of angle measurement that is known is probably the degree.
There are 360◦ in a circle and so a degree is one three-hundred-and-sixtieth part of a
circle.

The symbol for a degree is ◦ and so there are 360◦ in a circle.

Figure 7.1 shows a quarter of a circle which is 90◦ and is termed a right-angle, an angle
which is less than 90◦ (Figure 7.2) is called an acute angle, greater than 90◦ but less than
180◦ (Figure 7.3) is an obtuse angle, and greater 180◦ (Figure 7.4) is a reflex angle.

Right angle
90°

 Figure 7.1
142 • Mathematics

Acute angle
<90°

 Figure 7.2

90° < Obtuse angle < 180°

 Figure 7.3

Reflex angle
>180°

 Figure 7.4

One sixtieth part of a degree is termed one minute and the sixtieth part of a minute is
one second.

Symbols are used to represent degrees, minutes and seconds.

60 s = 60 = 1 min; 60 min = 60 = 1◦ ; 360◦ = 1 circle.

An angle of 35 degrees 23 minutes and 15 seconds is written as 35◦ 23 15 .

Notation depends on the circumstance: some situations require angles to be given in


decimal form accurate to, say, 3 decimal places; others require answers correct to the
nearest tenth of one minute, for example 35◦ 25.7 ; and others may require an accuracy
correct to the nearest second, for example 35◦ 23 15 .

Circular measure

A radian is the angle subtended at the centre of a circle by an arc equal in length to the
radius (Figure 7.5).
Trigonometry and Geometry • 143

Therefore 2π radians = 360◦


180◦ ∼ π ∼
1 radian = = 57. 3◦ or 57◦17 45 1◦ = = 0. 01745 radians
π 180

r A
θ

 Figure 7.5

θ is in radians

Length of arc, s = r · θ
1 2
Area of sector, A = ·r ·θ
2
Similarly, if a wheel of 0.3 m radius turns through 4 rad in 1 s, then a point on the rim
moves at a rate of 4 × 0. 3 = 1. 2 m/s.

In symbols:

If ω = angular velocity, v = linear velocity and r = radius then v = ωr

The angular velocity of a rotating part of a mechanism is often more conveniently


expressed in rad/s rather than the more practical rev/min (rpm).

To convert from one to the other

Suppose that N = velocity in rev/min and ω = velocity in rad/s


2πN
So N revolutions per minute = N × 2π = 2πN rad/min = rad/s
60
2πN
Therefore ω = .
60
144 • Mathematics

Example

Express in radians and degrees the angles subtended at the centre of a circle of 50 mm
radius, by arc length of, (i) 50, and (ii) 140 mm respectively.

(i) Arc length of 50 mm:


s 50 180
Since S = rθ , θ= = = 1 rad, 1 rad = ≈ 57. 3◦
r 50 π
(ii) Arc length of 140 mm:
s 140 180
θ= = = 2. 8 rad, 2. 8 rad = 2. 8 × ≈ 160. 4◦
r 50 π
Example

A flywheel of 1 m diameter is rotating at 120 rev/min.

(i) Express this in rad/s,


(ii) Find the linear velocity, in m/s, of a point on the rim.
2πN 2π × 120 240π
(iii) ω = = = = 4π rad/s.
60 60 ‘60
It is acceptable to leave the answer in terms of π but as a number ω ≈ 12. 57 rad/s.
(iv) v = ωr = 4π × 0. 5 ≈ 6. 28 m/s (Remember the diameter is 1 m)

Questions

1. Express as radians, to 4 decimal places:


(a) 36◦ (b) 42◦ 24 (c) 86◦ 45 (d) 228◦ 16
2. Express the following angles, stated in radians, in degrees and minutes:
(a) 1.8 (b) 0.7942 (c) 3.2106 (d) 1.7763
3. Express in degrees:
π 2π 5π 9π
(a) (b) (c) (d)
4 5 9 8
4. Express in radians, as fractions of π:
(a) 80◦ (b) 54◦ (c) 135◦ (d) 165◦
5. The pendulum of a longcase clock is 50 cm long and swings through an arc of 8
cm. Through what angle does the pendulum swing?
6. Find the area of the sector of a circle of diameter 12 cm bounded by two radii and
an arc length of 10 cm.
7. The mass of a circular brass disc is 280 g. The mass of a sector sheared out of it is
63 g. Find the angle of the sector.
8. An electric motor runs at 1500 rpm. What is the angular speed in radians per
second?
Trigonometry and Geometry • 145

9. A 20 cm diameter flywheel is rotating at 5400 rpm. What is its angular speed and
what is the linear speed of a point on its rim?
10. A car with a 50 cm diameter wheel is travelling at 72 km/h. What is the angular
speed of rotation of the wheels and how many rpm does this represent?
11. The cross-section of a piece of corrugated plastic used for roofing consists of a
series of circular arcs, each 8 cm across and 2 cm deep. Each sheet is 80 cm wide. If
the sheet could be flattened, how wide would it be then?

8 cm

2 cm

12. A tramline AXYB consists of two straight portions AX and YB each 500 m long, the
angle between them being 132◦ , connected by a circular arc of radius 80 m. What
is the total length of the line from A to B?
X
A
Y

Answers

1. (a) 0.6284 (b) 0.7399 (c) 1.514 (d) 3.983

2. (a) 103◦ 6 (b) 45◦ 31 (c) 183◦ 57 (d) 101◦ 46

3. (a) 45◦ (b) 72◦ (c) 100◦ (d) 202.5◦

4π 3π 3π 11π
4. (a) (b) (c) (d)
9 10 4 12

5. 9◦ 10 6. 30 cm2 7. 81◦ 8. 157 rad/s

9. 566 rad/s, 56.6 m/s 10. 80 rad/s, 764 rpm 11. 92.7 cm 12. 1067 m
146 • Mathematics

Trigonometric Ratios
A right-angled triangle is one which contains an angle of 90◦ , the longest side (oppo-
site the right angle) is termed the hypotenuse, the other two sides are termed the
opposite and adjacent depending upon which of the other two angles are under
consideration.

Considering angle θ (Figure 7.6), side AC is opposite the angle and therefore referred to
as such, the other side BC is the adjacent.

A
e
us

Opposite
ten
po
Hy

θ
B C
Adjacent

 Figure 7.6

α
e
us

Adjacent
ten
po
Hy

B C
Opposite

 Figure 7.7

If angle α is being considered (Figure 7.7), BC is the opposite and AC is the adjacent.
Trigonometry and Geometry • 147

The ratios of the lengths of the sides of a right-angled triangle are expressed by sine
(abbreviated sin), cosine (abbreviated cos) and tangent (abbreviated tan), as follows:

opposite
sine of angle =
hypotenuse
adjacent
cosine of angle =
hypotenuse
opposite
tangent of angle =
adjacent

Referring to Figure 7.6,

AC
sin θ =
AB
BC
cos θ =
AB
AC
tanθ =
BC

These are the ratios most often used.

Another three ratios are the reciprocals of the three above: namely the cosecant (cosec),
the secant (sec) and the cotangent (cot) where

1 AB
cosec θ = =
sin θ AC
1 AB
sec θ = =
cos θ BC
1 BC
cotθ = =
tanθ AC

Every angle has its own value of sine, cosine and tangent.

The sum of the angles in any triangle is 180◦, therefore if one angle of a right-angled
triangle is 30◦ the other must be 60◦ , and if one angle is 45◦ the other must be 45◦ .

A calculator will give the values of these ratios for any given angle.

Example

sin 30◦ = 0. 5
opposite
This means that = 0. 5 (from Figure 7.8)
hypotenuse
148 • Mathematics

opposite
If the hypotenuse is taken as, say, 200 cm, then = 0. 5
200
and so opposite= 0. 5 × 200 = 100 cm

200
100

30°

 Figure 7.8

adjacent
Similarly cos 30◦ = 0. 8660 which means that = 0. 866,
200
giving adjacent = 173. 2 cm

60°
200
100

30°

173.2

 Figure 7.9

As the base angle is 30◦ , the angle at the top must be 60◦ .
173. 2
From Figure 7.9 it can be seen that sin 60◦ = = 0. 866 = cos 30◦
200
100
Similarly cos 60◦ = = 0. 5 = sin 30◦
200
Trigonometry and Geometry • 149

Alternative Notation for the


Right-Angled Triangle, ABC
Using A as the reference angle:

e)
est sid
long

Opposite, a
(
e, c
tenus
Hypo

A C
Adjacent, b

Hypotenuse: the side AB opposite the right angle, denoted c


Opposite: the side BC opposite to the angle A, denoted a
Adjacent: the side AC adjacent to the angle A, denoted b

Note: That is, capital letters for angles, small case letters for sides opposite the angle.

Other terms used when describing angles

Complementary angles
If the sum of any two angles is 90◦ they are said to be complementary angles.

For example, 60◦ is the complementary angle to 30◦ , 50◦ is complementary to 40◦ , 70◦
is complementary to 20◦ , and so on.

In all such cases the sine of an angle is equal to the cosine of its complement.

The notes referring to Figure 7.11 show that the sine of 30◦ is the same as the cosine of
60◦ , and cos 30◦ is the same as sin 60◦ .
150 • Mathematics

α°

θ°

 Figure 7.10

In general, referring to Figure 7.10,

sin θ = cos α = cos (90 − θ)


cos θ = sin α = sin (90 − θ)
tanθ = cot α = cot(90 − θ)

Supplementary angles

If the sum of two angles is 180◦ they are said to be supplementary to each other.

Thus 80◦ is the supplementary angle of 127◦ , and so on.

In order that angles can be more easily understood it is useful to know the following
basic facts about straight lines and angles.

Properties of Angles and Straight Lines


(i) The total angle on a straight line is 180◦ . Angles A and B are called adjacent angles
(being next to each other). They are also supplementary.

A B

(ii) When two straight lines cross, the opposite angles are equal.

α β α=β
χ=δ
δ
Trigonometry and Geometry • 151

Parallel lines

When two parallel lines are cut by a transversal the following facts are true:

• the corresponding angles are equal,


a=l:b=m:c=p:d=q:

a
b

d
c
l
m
q p

• the alternate angles are equal,


d=m:c=l:
• the interior angles are supplementary,
d + l = 180◦ : c + m = 180◦

(a) An acute-angled triangle has all its angles less than 90◦ .
(b) A right-angled triangle has one of its angles equal to 90◦ .
(c) An obtuse-angled triangle has one angle greater than 90◦ .
(d) A scalene triangle has all three sides of different length and hence all three angles
are different.
(e) An isosceles triangle has two sides and two angles equal.
(f ) The three angles add up to 180◦ .
(g) The longest side is opposite the largest angle and the shortest side opposite the
smallest angle.
(h) An equilateral triangle has all three sides the same length and all three angles equal.
The angles are therefore 60◦ .

Theorem of Pythagoras
This theorem refers only to right-angled triangles.

Pythagoras’ Theorem states:


152 • Mathematics

‘In a right-angled triangle, the square on the hypotenuse is equal to the sum of the squares
on the other two sides.’

c
b

B C
a

 Figure 7.11

This can be expressed algebraically as

AB2 = AC2 + BC2 or c2 = a2 + b2

Pythagoras’ Theorem is generally used to find the length of one side of a right-angled
triangle when the other two sides are known (Figure 7.11).

Examples
(i) (ii)

x 10
5 7

12 x

x 2 = 52 + 122 102 = x 2 + 72
= 25 + 144 x 2 = 102 − 72
= 169 = 100 − 49

x = 169 = 51

= 13 x = 51
= 7. 14

It follows that if three given sides of a triangle obey Pythagoras’ Theorem the triangle
must be a right-angled triangle.
Trigonometry and Geometry • 153

This may be shown by a small square of side c contained in a larger square of side (a + b)
arranged as in Figure 7.12.
Base × Perpendicular height
The area of a right-angled triangle is
2
ab
In the triangles of sides a, b and c, the area of one triangle =
2
ab
So the area of four triangles = 4 × = 2ab
2
The area of large square = (a + b)2 = a2 + 2ab + b2 and the area of small square = c2 .

c
a

b a

 Figure 7.12

Area of small square = Area of large square − Area of 4 triangles

c2 = (a + b)2 − 2ab = a2 + 2ab + b2 − 2ab = a2 + b2

Example

Find the side marked x.

5 x

3 10

Let the height of the triangles be h.


154 • Mathematics

Then 52 = 33 + h2 so h2 = 52 − 32 = 25 − 9 = 16

Also x 2 = 102 + h2 = 100 + 16 = 116



Therefore x = 116 = 10. 77 to 2 decimal place accuracy.

Questions

In questions 1 to 8, find the value of the length, x.

1. 2.
x
x 5
6

8 7

3. 4.
9.4 12.5
x
x

3.4 8.4

5. 6.

15 7 x
x

x 3 10

7. 8. 16
33

8 8
6
x
56

9. Calculate the length of the diagonal of a square with sides of 10 cm.


Trigonometry and Geometry • 155

Answers

1. 10 2. 8.60 3. 8.76 4. 9.26


5. 10.61 6. 11.83 7. 10.58 8. 63.00
9. 14.14

Examples

Referring to Figure 7.13.

A B

 Figure 7.13

1. If angle A = 42◦ and side AC = 12 cm, what is the length of side BC?
Since BC is the side opposite to angle A and AC is the hypotenuse, the sine ratio is
used.
BC
= sin 42◦
AC
So BC = AC × sin 42◦ = 12 × 0. 6691 cm = 8. 03 cm

2. If angle C = 17. 8◦ and A = 5 cm, find the length of side AC.


AB is the side opposite ∠C and AC is the hypotenuse, so the sine ratio is used.
AB
= sin C
AC
AB 5 5
So AC = = ◦
= = 16. 4 cm
sin C sin 17. 8 0. 3057
The two previous examples have allowed a side to be found given a side and an
angle.
It is also possible to find an angle given two sides.
To do this the inverse trigonometric ratios must be used.
156 • Mathematics

The inverse ratios are written sin−1 (. . .), cos−1 (. . .) and tan−1 (. . .)
In some texts they are called arcsin(. . .), arcos(. . .) and arctan(. . .).
3. Side AC = 11 cm and side BC = 8 cm. What are angles A and C?
Since BC is the side opposite angle A and AC is the hypotenuse
BC 8
sin A = = = 0. 7273 to 4 decimal places.
AC 11
So A = sin−1 0. 7273 = 46. 7◦

Since the three angles add up to 180◦, angle C = 180◦ − (A + 90◦) = 43. 3◦ .

Questions
1. Find the length of the sides marked x in the following diagrams:

(i) (ii)

20
18
x

35°
x 42°

2. Find the size of the angles marked y in the following diagrams:

(i) (ii)
θ°

12
5 3

θ° 8

3. In the triangle ABC, ∠B = 90◦ , ∠C = 26. 35◦ and b = 13. 4 cm. Calculate the length of
side c of the triangle.
4. In the triangle ABC, ∠C = 90◦ , ∠A = 69. 3◦ and a = 3. 4 cm. Calculate the length of
side c of the triangle.
5. An equilateral triangle has a vertical height of 20 cm. Calculate the length of the
equal sides.
6. Calculate the length of the equal sides of an isosceles triangle whose altitude
(vertical height) is 15 cm and whose equal angles are 48.6◦ .
Trigonometry and Geometry • 157

7. Find the values of the letters in the following table (AB is the hypotenuse):

Angle A Angle B Angle C Side a Side b Side c

(i) A 90◦ 15 cm 25 cm

(ii) A 90◦ 38 cm 42 cm

(iii) 15◦ 90◦ a 40 cm

(iv) 63◦ 90◦ a 15 cm

(v) A 90◦ 12 cm 23 cm

(vi) 19◦ 90◦ a 32.5 cm

(vii) 26◦ 90◦ 39 cm c

(viii) B 90◦ 955 mm 1000 mm

Answers

1. (i) 16.38 (ii) 12.04

2. (i) 36.9◦ (ii) 33.7◦

3. c = 5. 95

4. c = 3. 63

5. 23.09 cm

6. 20 cm

7. (i) A = 31. 0◦ (ii) A = 42. 1◦ (iii) a = 10. 72 cm (iv) 7.64 cm

(v) 58.6◦ (vi) a = 30. 73 cm (vii) 43.4◦ (viii) 5.7◦

Examples Where Trigonometry can


be Used
(i) From a point 52 m away from the foot of a vertical building, the angle of elevation of
the top of the building is 20◦ . What is the height of the building?
158 • Mathematics

The angle of elevation of an object is the angle through which the eye is raised above
the horizontal to see the object.
The angle of depression is the angle below the horizontal when looking down on an
object.

A diagram of the situation is crucial.

20°
52 m

h
∴ tan 20 =
52
∴ h = 52 × tan20 = 52 × 0. 3640
So h = 18. 93 m to 2 decimal places

Bearings
Bearings refer to the direction of one point from another point. Sometimes they are
given in the form north-east (NE), west (W) etc. but usually they are given as three-figure
numbers.

The bearing represents the angle turned from North in a clockwise direction.
Trigonometry and Geometry • 159

Some examples are shown below:


(a) (b) N (c)
N N

270°
045°
200°
(Due west)

To solve some questions involving bearings it is necessary to have a suitable diagram


showing angles and distances clearly, calculating some angles using basic geometry,
before even considering Pythagoras’ Theorem or basic trigonometry.

Example

A ship sails for a distance of 15 km on a bearing of 250◦. How far south from the original
position is the ship?
N

S
250°
20°
d
15 km

S is the start position and F the final position.

Let d represent the distance south of S.


d
sin 20 = ⇒ d = 15 × sin20 = 15 × 0. 3420 = 5. 13 km
15

Questions

1. A ship is 3 km from the foot of a 150 m high cliff. What is the angle of depression of
the ship from the cliff top?
160 • Mathematics

2. A square has sides of length 7 cm. Find the distance from the mid-point of one of
the sides to either end of the opposite side.
3. From the top of a 90 m high cliff the angle of depression of a boat at sea is 15.3◦ .
How far is the boat
(i) from the cliff?
(ii) from the observer?
4. What is the angle of elevation of the top of a tree 8 m high from a point on the
ground 9.2 m away from the foot of the tree?
5. Find the sides of the right-angled triangle ABC if the sides of the square are 8 cm.

20°

6. A man walks 3 km from A to B on a bearing of 020◦ and then 2 km from B to C on a


bearing of 110◦ . What is the distance and bearing of C from A?

Answers

1. d The angle of depression = d. This is the


150 m same as angle x.
Cliff
150
x tanx = = 0. 05 ⇒ x = d = 2. 9◦
3000 m Ship 3000
2. 7

3.5  √
d
d= (7)2 + (3. 5)2 = 61. 25 = 7. 83 cm

3. 15.3° x = 15. 3◦
90 90
90 m d tan15. 3 = ⇒b= = 329 m
Cliff b tan15. 3
x
90 90
b Boat sin 15. 3 = ⇒d= = 341 m
d sin 15. 3
Trigonometry and Geometry • 161

4.

8
8m
tanx = = 0. 8696 ⇒ x = 41◦
9. 2

9.2 m

5.
h
h tan20 = ⇒ h = 2. 912 ⇒ height =
20° 8
10. 912
8
8
tan20 = ⇒ d = 21. 98 ⇒ base = 29. 98
8 8 d
20° By Pythagoras’ Theorem, hypotenuse =
8
d 31.9 cm

6.

110°
The shaded angle is 90◦ .
B 2
tan(a) = ⇒ a = 33. 7◦ ⇒ Bearing =
3
033. 7
2 km √ √
20°
C d = 22 + 32 = 13 = 3. 61 km
3 km

a d

Coordinate Systems
The usual system of coordinates (x, y) is called Cartesian or rectangular coordinates. The
term ‘rectangular’ is used because the x value and the y value represent the base length
(x) and the height (y) of a rectangle (Figure 7.14).

Another way of identifying the point is to use Polar coordinates where the coordinates
represent a distance and angle.
162 • Mathematics

(x, y)

 Figure 7.14

∠r, θ

 Figure 7.15

By comparing Figures 7.14 and 7.15 it can be seen that


 y
• r= x 2 + y2 and θ = tan−1
x
• x = r · cos θ and y = r · sin θ

These formulae allow conversion between one system and the other.

Example

In rectangular coordinates a point is described as being at (3, 4).

What are its Polar coordinates (angle in degrees)?


 √ √
r = 32 + 42 = 9 + 16 = 25 = 5
 
4
θ = tan−1 = tan−1 (1. 333) = 53. 13◦
3
Therefore ∠5, 53. 13◦

Example

In Polar coordinates a point is described as being ∠ 8, 30◦


Trigonometry and Geometry • 163

What are its rectangular coordinates?

x = 8 cos 30◦ = 8 × 0. 866 = 6. 93


y = 8 sin 30◦ = 8 × 0. 5 = 4
Therefore (6. 93, 4).

Calculators have these conversion functions as standard. It is a very useful tool so check
how your particular calculator operates.

Sines, cosines and tangents should be quite familiar but how exactly do these functions
change for different angles?

Graphs of each of the three functions will help to explain.

Imagine a rod, length 1 m, fixed at one end, and free to rotate in a circle about that
fixed end. Initially the rod is held horizontally pointing to the right. As the rod rotates
anticlockwise the angle the rod is to the horizontal increases, as does the height of the
end of the rod above the initial position. If the height of the end of the rod is plotted
against the angle the rod makes with the horizontal, a smooth curve is formed. This is
the sine wave. (Heights below the initial position are negative.) Once the rod has made
one complete rotation the curve is repeated. The wave is said to be periodic.
90°

h h
α
180° 0°
0° α 90° 180° 270°

270°

If instead of the vertical height above the horizontal being measured and plotted, the
horizontal distance from the vertical is plotted, the resulting curve is that of the cosine
wave.
1

0 360°
90° 180° 270°

–1
164 • Mathematics

sin θ
It can be shown that tan θ = .
cos θ
This has an interesting effect on the graph because when θ = 90◦ , cos θ = 0, and so
tan θ does not exist, as it is not possible to divide by 0.

The full graph looks like the following:


10

0° 90° 180° 270° 360°


–2
–4
–6
–8
–10

It is obvious that this graph is periodic but repeats after 180◦ whereas the sine and
cosine curves repeat after 360◦.

If the rod is rotating at a fixed angular velocity, ω rad/s, the above graphs can be plotted
where time is the horizontal variable, not angle.

The Equation of a Sine Wave


The general formula for a sine wave is: R · sin(ωt + α).

where R is the amplitude, ω is the angular velocity and α is the phase shift relative to
sin(ωt).

From these values the frequency, periodic time and phase angle may be found since
ω
• the frequency f = Hz,

2π 1
• the periodic time T = or s and
ω f
180◦
• the phase angle is α ×
π
Trigonometry and Geometry • 165

Example

The current in a circuit at any time t seconds is given by i = 75. 0 sin(100 πt + 0. 320)
amperes. Find the amplitude, periodic time, frequency and phase angle, relative to
sin(100 πt).

Solution

(i)
• Amplitude = 75.0 amperes
2π 2π
• Periodic time = = = 0. 02 s
ω 100π
ω 100π
• Frequency = = = 50 Hz
2π 2π
• Phase angle = 0. 32 radians = 18. 3◦ (leading).

Questions

1. An alternating voltage is given by v = 63. 0 sin(250πt + 0. 240) V.


Find the amplitude, periodic time, frequency and phase angle, relative to
sin(250 πt).
2. The instantaneous value of current in an alternating-current circuit at any time t is
given by i = 200. 0 sin(50πt − 0. 683) amperes.
Find the amplitude, periodic time, frequency and phase angle, relative to sin(50 πt).
3. The current in an a.c. circuit is given by i = 40 sin(100πt − 0. 32) amperes.
Find the amplitude, the periodic time, the frequency and the phase angle, relative
to sin(100 πt).

Answers

1. 63, 125 Hz, 8 ms, 13.8◦

2. 200, 40 ms, 25 Hz, −39. 1◦

3. 40, 20 ms, 50 Hz, −18. 3◦

Sinusoidal Waveforms
Here are some examples of sine waves and their equations:

1. y = sin(x)
360◦
Amplitude: 1 Period: 360◦ Frequency: =1 Phase angle: 0◦
360◦
166 • Mathematics

4 y

1
x
–90 90 180 270 360
–1

–2

–3

–4

 Figure 7.16

2. y = 3sin(x)
360◦
Amplitude: 3 Period: 360◦ Frequency: =1 Phase angle: 0◦
360◦

4 y

1
x
–90 90 180 270 360
–1

–2

–3

–4

 Figure 7.17

3. y = 3sin(2x)
360◦
Amplitude: 3 Period: 180◦ Frequency: =2 Phase angle: 0◦
180◦
Trigonometry and Geometry • 167

4 y

1
x
–90 90 180 270 360
–1

–2

–3

–4

 Figure 7.18

4. y = 3sin(x + 45◦ )
360◦
Amplitude: 3 Period: 360◦ Frequency: =1 Phase angle: +45◦
360◦

4 y

2
+ 45°
1
x
–90 90 180 270 360
–1

–2

–3

–4

 Figure 7.19

5. y = 3sin(x − 45◦ )
360◦
Amplitude: 3 Period: 360◦ Frequency: =1 Phase angle: −45◦
360◦
168 • Mathematics

4 y

2
– 45°
1
x
–90 90 180 270 360
–1

–2

–3

–4

 Figure 7.20

6. y = 3sin(2(x − 30◦)) or y = 3sin(2x − 60◦ )


360◦
Amplitude: 3 Period: 180◦ Frequency: =2 Phase angle: −60◦
180◦

4 y

1
– 30°
x
–90 90 180 270 360
–1

–2

–3

–4

 Figure 7.21

7. y = 3sin(2(x + 60◦)) or y = 3sin(2x + 120◦ )


360◦
Amplitude: 3 Period: 180◦ Frequency: =2 Phase angle: +120◦
180◦
Trigonometry and Geometry • 169

4 y

1 + 60°
x
–90 90 180 270 360
–1

–2

–3

–4

 Figure 7.22

It is very important to remember that equations whose variable is t (time) must be calculated
taking the angles as radians and not degrees.

Example

The current in a circuit at any time t seconds is given by i = 75. 0 sin(100πt + 0. 320)
amperes.

Find:

(i) the amplitude, periodic time, frequency and phase angle, relative to sin(100 πt).
(ii) the value of the current when t = 0;
(iii) the value of the current when t = 6 ms, that is, 0.006 s;
(iv) the time when the current first reaches 50.0 amperes;
(v) the time when the current is a maximum.

Sketch one cycle of the waveform.

Solution

(i) Amplitude = 75. 0 amperes


2π 2π
Periodic time = = = 0. 02 s
ω 100π
ω 100π
Frequency = = = 50 Hz
2π 2π
170 • Mathematics

Phase angle = 0. 32 rad = 18. 3◦ (leading).


 
α 0. 32
In addition = = 0. 00102 s = 1. 02 ms . This will be needed later!
ω 100π

(ii) When t = 0,

i = 75. 0 sin(0 + 0. 320)


= 75 sin(0. 320)
= 75 × 0. 314567
= 23. 6 amperes

(iii) When t = 0. 006 s,

i = 75. 0 sin(100π × 0. 006 + 0. 320)


= 75 sin(2. 205)
= 75 × 0. 80557
= 60. 4 amperes

(iv) When i = 50 amperes,

50 = 75 sin(100πt + 0. 320)
0. 66667 = sin(100πt + 0. 320)
100πt + 0. 320 = sin−1 (0. 66667)
100πt + 0. 320 = 0. 7297
0. 7297 − 0. 320
t=
100π
= 0. 0013 s
= 1. 3 ms

(v) At a maximum i = 75

sin(100πt + 0. 320) = 1
100πt + 0. 320 = 1. 5708
1. 5708 − 0. 320
t=
100π
= 0. 00398 s
= 3. 98 ms
Trigonometry and Geometry • 171

80 i, current
(amps)
60

40 This is the same time as the calculation


for the time to reach a maximum 18.98 ms
20
t, time (ms)
M B A
5 10 15 20
–20 Quarter wave Half wave
3.98 ms
–40
8.98 ms
–60

–80

1.02 ms

 Figure 7.23

There is a way of checking whether everything has been drawn correctly.


α
For the graph in Figure 7.23, = 1. 02 ms is the time shift for one cycle of the waveform.
ω
Therefore the time value for point A is 20 − 1. 02 = 18. 98 ms.

Between points A and B there is a half wave.

Since the periodic time is 20 ms, a half wave must take 10 ms and so B is at time 8.98 ms.

Similarly there is a quarter wave between B and M (the time for a maximum current).

A quarter wave takes 5 ms and so M is at time 8. 98 − 5 = 3. 98 ms.

The answer to (v), the time to a maximum works out at 3.98 ms, totally independent of
the graphical answer.

Questions

1. An alternating voltage is given by v = 63. 0 sin(250πt + 0. 240) V.


Find:

(i) the amplitude, periodic time, frequency and phase angle, relative to sin(250 πt).
172 • Mathematics

(ii) the value of the voltage when t = 0;


(iii) the value of the voltage when t = 2 ms;
(iv) the time when the voltage first reaches 30.0 V;
(v) the time when the voltage is first a maximum.
Sketch one cycle of the waveform.
2. The instantaneous value of current in an a.c. circuit at any time t is given by
i = 200. 0 sin(50πt − 0. 683) amperes. Find:
(i) the amplitude, periodic time, frequency and phase angle, relative to sin(50 πt).
(ii) the value of the current at t = 0;
(iii) the value of the current at t = 10 ms;
(iv) the time that the current first reaches 100.0 amperes;
(v) the time that the current first reaches −58.0 amperes;
(vi) the time in the first cycle where the current is a maximum.
Sketch the curve showing all the relevant details.
3. The current in an a.c. circuit is given by i = 40 sin(100πt − 0. 32) amperes.
Find: the amplitude, the periodic time and frequency, the phase angle, relative to
sin(100 πt) and sketch one cycle of the waveform.

Solutions

1. (i) 63, 125 Hz, 8 ms, 13.8◦ , 2. (i) 200, 40 ms, 25 Hz, −39. 13◦ , 3. (i) 40
α α
= 0. 3 ms = −4. 3 ms
ω ω

(ii) 14.98 (ii) −126.2 (ii) 20 ms, 50 Hz

(iii) 61.2 (iii) 155.1 (iii) −18. 3◦

(iv) 0.33 ms (iv) 7.7 ms i0 = −12. 6

(v) 1.7 ms (v) 2.5 ms tmax = 6. 01

α
(vi) 14.3 ms = −1. 01 ms
ω
Trigonometry and Geometry • 173

200
q2
150

100

50
t, time (s)
0.005 0.01 0.015 0.02 0.025 0.03 0.035 0.04
–50
q3
–100 q1

–150

–200

 Figure 7.24

Addition of Sine Waves of Equal


Frequency
If the two waves y = 12 sin(50πt) and y = 18 cos(50πt) are added the result is a third
sine wave (see Figure 7.25)

25 y

20 12sin(50πt ) + 18cos(50πt )

15

10

5 12sin(50πt )
t (ms)
5 10 15 20 25 30 35 40
–5

–10

–15
18cos(50πt )
–20

–25

 Figure 7.25
174 • Mathematics

The resultant wave 12 sin(50πt) + 18 cos(50πt) is obviously just another sine wave.

Its equation is therefore y = R · sin(ωt + α)

It is necessary to find the amplitude, R, and the phase shift, α, of the resultant wave
compared with the sine wave.

Theory
If the two waves a · sin(ωt) and b · cos(ωt) are added the resulting
 wave can be written
√ −1 b
in the form R · sin(ωt + α) where R = a2 + b2 and α = tan .
a
Example

Taking the waves 12 sin(50πt) and 18 cos(50πt): they are of the same frequency since
both have angular velocity 50π rad/s

The resultant amplitude R = 122 + 182
√ √
So R = 144 + 324 = 468 = 21. 633 to 3 decimal place accuracy.
 
18
In radians, since the variable is time, α = tan−1 = tan−1 (1. 5)
12
Therefore α = 0. 983 rad

The resultant equation is therefore y = 21. 633 sin(50πt + 0. 983)

In practice, if a and b, in that order, are put through the calculator’s rectangular to polar
conversion facility then the answers produced are R and α (remember that the angles
are in radians).

Therefore 12 sin(50πt) + 18 cos(50πt) = 21. 63 sin(50πt + 0. 9828)

[Keep R to 2 decimal places and α to 3 decimal places.]

Questions
Find the single wave when the following pairs of waves are added.
Trigonometry and Geometry • 175

Answers

1. 3 sin(t) 4 cos(t) 5 sin(t + 0. 9273)

2. 4 sin(2t) 6 cos(2t) 7. 21 sin(2t + 0. 9828)

3. 12 sin(5t) 5 cos(5t) 13 sin(5t + 0. 3948)

4. 34 sin(50πt) 67 cos(50πt) 75. 13 sin(50πt + 1. 1012)

5. 100 sin(5πt) 45 cos(5πt) 109. 66 sin(5πt + 0. 4229)

Relationships Between Sine, Cosine


and Tangents of Angles
Referring to Figure 7.26

θ
B C

 Figure 7.26

AC BC
sin θ = and cos θ =
AB AB
sin θ AC BC AC AB AC AC
So = ÷ = × = but = tan θ
cos θ AB AB AB BC BC BC
176 • Mathematics

sin θ
Therefore tanθ = (i)
cos θ
By Pythagoras’ Theorem (AB)2 = (AC)2 + (BC)2

(AB)2 (AC)2 (BC)2


Dividing every term by (AB)2 gives = +
(AB)2 (AB)2 (AB)2
 2  2
AC BC
Therefore 1 = +
AB AB
AC BC
but sin θ = and cos θ =
AB AB
Therefore

1 = sin2 θ + cos2 θ which is usually written as

sin2 θ + cos2 θ = 1 (ii)

In the same manner, the links between the reciprocal functions can be found.
(AB)2 (AC)2 (BC)2
(AB)2 = (AC)2 + (BC)2 and dividing every term by (AC)2 gives = +
(AC)2 (AC)2 (AC)2

Therefore cosec2 θ = 1 + cot2 θ (iii)

(AB)2 (AC)2 (BC)2


(AB)2 = (AC)2 + (BC)2 and dividing every term by (BC)2 gives = +
(BC)2 (BC)2 (BC)2

sec2 θ = tan2 θ + 1 (iv)

Example

Without using calculator or tables, find the cosine and tangent of an angle whose sine
is 0.8.
sin2 θ + cos2 θ = 1 so cos2 θ = 1 − sin2 θ
  √ √
Therefore cos θ = 1 − sin2 θ = 1 − (0. 8)2 = 1 − 0. 64 = 0. 36 = 0. 6
sin θ 0. 8
Also tanθ = = = 1. 333 correct to 3 decimal places.
cos θ 0. 6
Be careful in how you write the powers of trigonometric functions.

Example

sin θ × sin θ is written as sin2 θ , not sin θ 2


Trigonometry and Geometry • 177

sin θ 2 means the sine of θ 2

Similarly, cos2 θ , tan2 θ etc.

Identities
An identity is an equation that is true for all values of the quantities involved in the
equation.

Example

x 2 − y2 = (x − y)(x + y)
(x + y)2 = x 2 + 2xy + y2

are examples of identities because they are true for any values of x and y.

A trigonometric identity is an equation that is true for any angle, thus the four expres-
sions derived in the previous section:
sin θ
tan θ =
cos θ
sin2 θ + cos2 θ = 1
cosec2 θ = 1 + cot2 θ
sec2 θ = tan2 θ + 1

are all trigonometric identities because they are true for any value of angle θ and
many more can be proved from knowledge of these fundamental identities and the
trigonometric ratios. A few simple examples are shown below.

Examples

(i) Prove that


1 + cos θ
cosec θ + cot θ =
sin θ
Simplifying the left-hand side of the equation:
1 cos θ
cosec θ + cot θ = +
sin θ sin θ
1 + cosθ
=
sin θ
178 • Mathematics

(ii) Prove that


sec A − cos A
= tanA
sin A

Simplifying the left-hand side of equation:

1
sec A − cos A − cos A
= cos A
sin A sin A
1 cos2 A

= cos A cos A
sin A
1 − cos2 A
= cos A
sin A
1 − cos2 A
=
cos A sin A
sin2 A
=
cos A sin A
sin A
=
cos A
= tanA

(iii) Prove that


tanθ
sin θ = 
1 + tan2 θ

Simplifying the right-hand side of equation:

tanθ tanθ
 =√
1 + tan2 θ sec2 θ
tan θ
=
sec θ
sin θ 1
= ÷
cos θ cos θ
sin θ cos θ
= ×
cos θ 1
= sin θ

Note: See also Compound and Double Angles in Chapter 8.


Trigonometry and Geometry • 179

Introduction to Hyperbolic Functions


There are three main functions, corresponding to the sine, cosine and tangent of
trigonometry.

ex − e−x
• hyperbolic sine of x, sinh(x) =
2
ex + e−x
• hyperbolic cosine of x, cosh(x) =
2
ex − e−x
• hyperbolic tangent of x, tanh(x) =
ex + e−x

Most calculators allow these functions to be evaluated by pressing the ‘hyp’ key before
‘sin’, ‘cos’ or ‘tan’.

Questions

Evaluate, correct to 4 significant figures.

1. (i) sinh(0. 64) (ii) sinh(2. 182)


2. (i) cosh(0. 72) (ii) cosh(2. 4625)
3. (i) tanh(0. 65) (ii) tanh(1. 81)
x 
4. A wire hangs so that its shape is described by y = 50 cosh .
50
Evaluate, to 4 significant figures, the value of y when x = 25.
 
L
5. The length l of a heavy cable hanging under gravity is given by l = 2c · sinh .
2c
when c= 40 and L = 30.
Find the value of l 
6. 3d
6. V = 0. 55L · tanh
2 is a formula for velocity of waves over the bottom of
L
shallow water, where d is the depth and L is the wavelength.
If d = 8. 0 and L = 96, calculate V.

Answers

1. (i) 0.6846, (ii) 4.376


2. (i) 1.271, (ii) 5.910
3. (i) 0.5715, (ii) 0.9478
180 • Mathematics

4. 5638
5. 30.71
6. 5.042

Hyperbolic Identities
There are several ‘links’ between hyperbolic functions.

Some of them are:

• cosh(x) + sinh(x) = ex
• cosh(x) − sinh(x) = e−x
• cosh2(x) – sinh2(x) = 1. Note that cos2(x) + sin2(x) = 1

Be careful with the signs.

Solving Equations Using Hyperbolic


Functions
a · sinh(x) + b · cosh(x) = c where a, b and c are constants can be solved by following
these steps:
 x   x 
e − e−x e + e−x
1. Change sinh(x) to and cosh(x) to .
2 2
2. Arrange the equation into the form p · ex + q · e−x + r = 0, where p, q and r are
constants.

3. Multiply each term by ex , which gives p · (ex )2 + r · ex + q = 0 since (ex ) · e−x = 1.
4. Solve the resulting quadratic equation by factorisation or formula.
5. This gives values for ex , so take natural logarithms (ln) to find the values of x.

Example

Solve the equation 2. 6 cosh(x) + 5. 1 sinh(x) = 8. 73.


Trigonometry and Geometry • 181

Solution
   
ex + e−x ex − e−x
2. 6 + 5. 1 = 8. 73
2 2
1. 3ex + 1. 3e−x + 2. 55ex − 2. 55e−x = 8. 73
3. 85ex − 1. 25e−x − 8. 73 = 0
 2
3. 85 ex − 8. 73ex − 1. 25 = 0

This quadratic equation has a = 3. 85, b = −8. 73 and c = −1. 25 which, on substitution
into the quadratic formula, gives ex = 2. 4027 or −0. 1351

Therefore x = ln(2. 4027) or ln(−0. 1351)

Therefore x = 0. 8766 to 4 significant figures as ln(−0. 1351) has no solution.

Example
 x 
A chain hangs in the form given by y = 40 cosh . Determine, correct to 4 significant
40
figures, (a) y when x is 25, and (b) x when y = 54. 30

Solution
 x   
25
(a) Since y = 40 cosh , y = 40 cosh = 40 cosh(0. 625) = 40 × 1. 20175 =
40 40
48. 07  
25  x  54. 30
(b) When y = 54. 30, 54. 30 = 40 cosh ⇒ cosh = = 1. 3575
40 40 40
Following the procedure 1–4 given previously:
 x/40 
e + e−x/40
= 1. 3575 ⇒ ex/40 + e−x/40 = 2. 715 ⇒ ex/40 + e−x/40 − 2. 715 = 0
2

Multiplying by ex/40 gives (ex/40 )2 − 2. 715ex/40 + 1 = 0


Using a = 1, b = −2. 715 and c = 1 in the quadratic formula gives ex/40 = 2. 2756 or
x
0.43945. So = ln(2. 2756) or ln(0. 43945).
40
Therefore x = 40 ln(2. 2756) = 32. 89 or 40 ln(0. 43945) = −32. 89

Questions
In questions 1–5 solve the given equations correct to 4 decimal places.

1. sinh(x) = 1 [0.8814]
2. 2 cosh(x) = 3 [±0. 9624]
3. 3. 5 sinh(x) + 2. 5 cosh(x) = 0 [−0. 8959]
182 • Mathematics

4. 2 sinh(x) + 3 cosh(x) = 5 [0. 6389, −2. 2484]


5. 4 tanh(x) − 1 = 0 [0.2554]
6. A flexible cable suspended between two horizontal points hangs in the form of a
catenary, the
 xequation
 of the curve being given by
y = c cosh − 1 , where y is the sag of the cable, x the horizontal distance from
c
the midpoint to one end and c is a constant.
Determine the sag when c = 20 and 2x = 16 [1.622m]

Answers

1. 0.8814 2. ±0. 9624 3. −0.8959


4. 0.6389, −2.2484 5. 0.2554 6. 1.622 m

Addition of sinh(ax) and cosh(ax)


Example

Find the number of sinh and the number of cosh functions that add together to give
3e2x + 5e−2x .

Solution

Suppose there are A sinh functions and B cosh functions.


   2x 
e2x − e−2x e + e−2x
So A · sinh(2x) + B · cosh(2x) = A · +B·
2 2
   
2x A B −2x B A
=e + +e −
2 2 2 2

Equating this to 3e2x + 5e−2x gives

A B
+ =3
2 2
Addition of these two equations gives B = 8; subtraction gives A = −2

B A
− =5
2 2

Therefore 3e2x + 5e−2x = −2 sinh(2x) + 8 cosh(2x)


Trigonometry and Geometry • 183

Basic Hyperbolic Trigonometric


Identities
sinh(A ± B) = sinh A · cosh B ± coshA · sinh B
cosh(A ± B) = coshA · cosh B ± sinh A · sinh B

If B = A, sinh 2A = 2 sinh A · cosh A and cosh 2A = cosh2 A + sinh2 A

Questions
Solve the following equations for x:

4 1
1. sinh(x) = 2. tanh(x) =
3 2

3. sinh(x) + 4 = 4 cosh(x) 4. 7 + 2 cosh(x) = 6 sinh(x)

5. 2 sinh(x) + 6 cosh(x) = 9 6. 5 cosh(x) + sinh(x) = 7

7. 2 cosh(2x) + 3 cosh(x) − 8 = 0 8. 4 cosh(x) − e−x = 3

9. 20 cosh(2x) − 21 sinh(x) = 200

Answers

1. 1.0986 2. 0.5493 3. 0, 0.511


4. 1.386 5. −1.386, 0.693 6. −1.099, 0.693
7. ±0.693 8. −0.693, 0 9. −1.386, 1.609

Latitude and Longitude


Figure 7.27 represents the earth, centre O: N and S are the north and south poles.

• All circles on the earth’s surface centred at O are great circles.


• All great circles passing through the poles are meridians of longitude.
184 • Mathematics

C
Q
P

θ
W O E

Y
X

 Figure 7.27

• All circles whose plane is at right angles to the line NOS are parallels of latitude: of
these only the equator is a great circle.
• The longitude of any point on the earth’s surface is measured by its angular displace-
ment east or west of the meridian which passes through Greenwich.
• The latitude of any point is given by its angular displacement, θ , north or south of the
plane of the equator.

In Figure 7.28, if NPXS represents the meridian of Greenwich and NQYS another merid-
ian, the circle centre C through P and Q a parallel of latitude, and the circle WXYE the
equator, then the longitude of Q is the angle PCQ (or XOY), and the latitude of Q is the
angle θ , QOY (or POX).

If, for example, PCQ = 16◦ and QOY = 54◦ , then the longitude of Q is 16◦ E and its
latitude is 54◦N.

The metre was originally defined as one ten-millionth part of the quadrant of the
meridian which passes through Paris. If R is the earth’s radius, then
1
of 2πR = 107 m
4
Trigonometry and Geometry • 185

∴ 2πR = 4 × 107 m
∴ 2πR = 4 × 104 km

The circumference of the Earth is therefore 40,000 km.

If P is in latitude θ and r is the radius of the circle of latitude, then r = R · cos θ .

This relationship, together with the value of R, will help in the calculation of distances
on the earth’s surface.

r P
C θ

R
O
X

 Figure 7.28

Example

Find the distance, measured along the parallel of latitude, between two places in
latitude 42◦ N, whose longitudes are 23◦ W and 17◦ E.

Solution

The differences in longitude of the two places (A and B) = ACB = XOY = 23◦ + 17◦ = 40◦

The radius of the small circle, the parallel of latitude, r = R · cos 42◦
40 1
Therefore, arc AB of circle of longitude = × 2πr = × 2πR cos 42◦
360 9
1
AB = × 4 × 104 × cos 42◦ km ≈ 3300 km
9
186 • Mathematics

N N

C
r Y
A
C 42
A B B
R r
42
O O, C 40°
Y O
X
A
X

S S

 Figure 7.29

Longitude and Time


The earth rotates on its axis once every 24 h spinning from west to east. This means that
a difference of one degree of longitude causes a difference of four minutes of time, and
that places to the west reach noon later than places to the east.

The nautical mile

One nautical mile (nm) is defined as the length of an arc on a great circle which subtends
an angle of 1 at the earth’s centre.

Hence 360 × 60 nm = 4 × 104 km. Therefore 1 nm = 1.852 km

A speed of 1 nm/h is called one knot: this unit of speed is almost universally used by
seamen and airmen.

Example

P and Q are two points on the earth’s surface in latitude 56◦ N: their longitudes are 25◦E
and 95◦ E respectively. Take 2πR = 4 × 104 km. Find the distance PQ.

(a) in km along the parallel of latitude,


(b) in km along a great circle,
(c) in nautical miles along a great circle.
Trigonometry and Geometry • 187

Solution
N N
(i) (ii)
r
C P
Q
C 56
P
R
O 56
Y O
X

C S S O

(iii) (iv)

35° 35° θ θ
r R

P Q P Q

70 7 7
(a) In (iii), arc PQ =× 2π · r = × 2π · R cos(56◦ ) = × 4 × 104 × cos(56◦)
360 36 36
So PQ ≈4350 km
(b) From (iii) PQ = 2 · r · sin(35) = 2 · R cos(56) · sin(35)
From (iv) PQ = 2R · sin(θ )

∴ sin (θ ) = cos (56) · sin (35) ⇒ θ = sin−1 (0. 3207) = 18◦ 42

Therefore ∠POQ = 2θ = 37◦24 = 37. 4◦


37. 4 37. 4
So in (iv), arc PQ = × 2π · R = × 4 × 104 ≈ 4160 km
360 360
(c) ∠POQ = 37◦ 24 = 2244 ⇒ arc PQ = 2244 nm
As a check, 2244 × 1. 852 ≈ 4160 km.

Questions

Take 2πR = 4 × 104 km. Answers should be to 3 significant figures.

1. On a globe of radius 15 cm find the length of the parallel of latitude 62◦ S.


2. Two places on the same meridian have latitudes 23◦ S and 41◦ S. What is their
distance apart, measured along the meridian?
188 • Mathematics

3. As in question 2, but the latitudes are 23◦ S and 41◦ N.


4. Two places on the equator have longitudes 63◦ E and 132◦E. How far apart are they,
measured along the equator?
5. As in question 4, but the places have longitudes 132◦E and 126◦ W.
6. Two places on the same meridian are 3940 km apart. If one of them is in latitude
18◦ N and the other is in the southern hemisphere what is the latitude of the other?
7. What is the length of the Arctic Circle (latitude 66◦ 30 N)? How far from the North
Pole is any point on this circle?
8. Two places are both in latitude 37◦ S and their longitudes are 34◦W and 29◦E. What
is their distance apart, measured along the parallel of latitude?

Answers

1. 44.24 cm 2. 2000 km 3. 7111.1 km 4. 7666.7 km


5. 11333.3 km 6. 17.5◦S 7. 15950 km 2611 km 8. 5590.4 km

Test Examples 7
1. (i) Express the following angles in rad: 114◦ 36 and 286◦ 30 .
(ii) Express in (a) radian and degree, the angles subtended by arc lengths of 10 m,
and 30.4 m respectively, on a circle of 10 m diameter.
2. A disc flywheel is rotating at a speed of 10.52 rad/s. Calculate the linear velocity, in
m/s, of points on the wheel at radii of 100 and 500 mm respectively. Find the linear
velocity in m/s of the rim at 2 m diameter when rotating at 125 rev/min.
3. In a right-angled triangle ABC, length AC is 36 mm, length BC is 27 mm, and the
angle at C is 90◦ . Calculate the length of the hypotenuse AB and the sine, cosine
and tangent of the angle at B.
4. Find, without the use of calculator or tables, the sine and tangent of an angle whose
cosine is 0.4924.
5. Write down the sine, cosine and tangent of the following angles:
10◦ 33 46◦ 55 150◦ 47 201◦ 21 287◦ 14
Sketch sine and cosine curves for angles between 0◦ and 360◦.
6. The following refer to angles between 0◦ and 360◦:
(a) find the angles whose sines are: 0.3783, −0.7005,
Trigonometry and Geometry • 189

(b) find the angles whose cosines are: 0.9687, −0.8769,


(c) find the angles whose tangents are: 0.2010, −3.2006,
7. If θ = 80◦ , find the values of:
sin θ cos θ sin 2θ cos 2θ sin2 θ cos2 θ
8. Find the values of θ between 0◦ and 360◦ which satisfy the equation:

cos2 θ − sin2 θ = 1

9. If a = 2 sin θ , and b = 5 cos θ , find the values of θ for angles between 0◦ and 180◦
when:
a2 2b2
= −4
2 5
10. The velocity of the piston in a reciprocating engine is given by:
 
sin 2θ
v = ωr sin θ + m/s
2n
and the acceleration is given by:
 
cos 2θ
a = ω r cos θ +
2
m/s2
n

where ω = angular velocity of crank in rad/s,


r = throw of crank (1/2 stroke) in m,
θ = angle of crank past top centre, in deg,
n = ratio of connecting rod length to crank length.

Calculate
(i) the velocity, and (ii) the acceleration of the piston of an engine of 1 m stroke, con-
necting rod length 2 m, at the instant the crank is 80◦ past top centre and running
at 150 rev/min.
11. The Bernoulli equation may be written as

P1 v12 P2 v22
+ + h1 = + + h2
ρ 2g ρ 2g

If (h1 − h2 ) = 2, v12 − v22 = 8. 4, P1 = 350, ρ = 10, transpose the formula in a
suitable way to find the value of the pressure P2 · (g = 9. 81)
12. The increase in resistance of strip conductors due to eddy currents at power
frequencies is given by:
 
α · t sinh(αt) + sin(αt)
λ=
2 cosh(αt) − cos(αt)
190 • Mathematics

Calculate λ, correct to 5 significant figures, when α = 1. 5 and t = 1. 2


13. The signal to noise ratio is given by the formula:
 
A X
= 10 × log10
B Y
where X is signal power and Y is noise power in watts.
A
Determine Y given = 15 and X = 2.
B
14. The signal to noise ratio is given by the formula:
 
S Psignal
= A · ln
N Pnoise
where P is signal and noise power in watts.
S
Determine Pnoise correct to 3 significant figures, given = 4. 0, A = 2. 0 and
N
Psignal = 2. 3
15. If 6ex − 3e−x = A · cosh(x) + B · sinh(x) find the values of A and B.
16. A polytropic process is one in which the pressure and volume of a gas are related
by the formula p · v n = c, where c is a constant. The temperatures (T) and specific
volumes (v) at the beginning and end of such a process are related by the formula
 1−n
T2 v2
=
T1 v1
Find an expression for the index, n, in terms of T1 , T2 , v1 and v2 .
Calculate the value of n for such a process in which the specific volume of a vapour
is 0.0292 m3 kg−1 at 211 K and 0.00399 m3 kg−1 at 300 K.
17. The increase in resistance of strip conductors due to eddy currents at power
frequencies is given by:
 
α. t sinh(αt) + sin(αt)
λ=
2 cosh(αt) − cos(αt)
Calculate λ, correct to 5 significant figures, when α = 1. 8 and t = 1. 25

R U2
18. If θt − θi = · ln find the value of U2 (to 2 decimal places) given that
J U1
θt = 2. 5, θi = 1. 5, R = 0. 415, J = 0. 3, U1 = 50

Circle Theorems
Useful to know
Trigonometry and Geometry • 191

Figure 7.30

(1) The angle subtended at the centre of a circle is always twice the angle subtended at
the circumference.

(2) If AB forms a diameter then the angle θ is always 90◦ .

 Figure 7.30

Figure 7.31

(3) The angles made in the same segment are equal.

 Figure 7.31
192 • Mathematics

Definition
Figure 7.32

A cyclic quadrilateral is a four-sided figure whose vertices (corners) all lie on the
circumference of the same circle.

Supplementary
(Add to 180°)

 Figure 7.32

Figure 7.33

(4) Chords of equal length are equidistant from the centre of the circle.

 Figure 7.33
Trigonometry and Geometry • 193

Figure 7.34

(5) If two chords intersect, the product of the parts of one chord is equal to the product
of the parts of the other.

d
c
b

 Figure 7.34

Figure 7.35

(6) The last result is true even if the chords intersect outside the circle, that is, a · b = c · d

c
d

 Figure 7.35
194 • Mathematics

Figure 7.36

(7) Extended chords are called secants. If a tangent and secant intersect as shown then

AO · OB = OC 2

O
C

 Figure 7.36

Figure 7.37

(8) If a radius is drawn to meet a tangent at its point of tangency the two lines meet at
right angles.

Radius Tangent

 Figure 7.37
SOLUTION OF
8
TRIANGLES
As seen in the previous chapter, the ratios of the lengths of the sides of a right-angled
triangle are expressed by sine, cosine and tangent, as follows:
opposite
sine of angle =
hypotenuse
adjacent
cosine of angle =
hypotenuse
opposite
tangent of angle =
adjacent

Example

A light on a cliff is viewed from a position at sea, the angle of elevation being 32◦ 15 . At a
point 100 m further away from the cliff and in direct line with the first observation point,
the light is viewed again and the angle of elevation is now 20◦ 44 . Find (a) the height of
the light above sea level, and (b) the horizontal distance from the second observation
point to the light.
CD  
Referring to Figure 8.1, = tan 32◦ 15
x
Therefore CD = x × 0. 63095 (i)

CD  
Similarly = tan 20◦ 44
100 + x

Therefore CD = (100 + x) × 0. 37853


CD = 37. 853 + 0. 37853x (ii)
196 • Mathematics

D Light

32°15´ 20°44´

C x B 100 m A

 Figure 8.1

Equating (i) and (ii) gives

0. 63095x = 37. 853 + 0. 37853x


0. 25242x = 37. 853
37. 853
x=
0. 25242
x = 149. 96 m

Therefore AC = 100 + 149. 96

Horizontal distance = 249.96 m

From (i): CD = x × 0. 63095 = 149. 96 × 0. 63095 = 94. 62 m

Inverse Trigonometric Functions


The calculator will only give the principal value, α.

Consider sin(x) = 0. 5

So x = sin−1 (0. 5) = 30◦

However, it can be seen that there is another angle that also has a sine of 0.5.

An accurate graph will give this angle as 150◦, but how can it be found without an
accurate plot?
Solution of Triangles • 197

1.2 y
1
0.8
0.6
0.4
0.2
x
–120 –90 –60 –30 30 60 90 120 150 180 210 240 270 300 330 360
–0.2
–0.4
–0.6
–0.8
–1
–1.2

 Figure 8.2

Answer: because the graph is symmetrical about the x = 90◦ line, the second angle is
simply 180◦ minus the angle given by the calculator.

So for sin−1 (x).

The second value for 0◦ ≤ x ≤ 360◦ is given by 180◦ − α, where a is the principal value.

To find the second value for the angle in the range 0◦ ≤ x ≤ 360◦ a different ‘rule’ is
needed for cosine and tangent.
cos−1 (x)

The second value for 0◦ ≤ x ≤ 360◦ is given by 360◦ − α.

tan−1 (x)

The second value for 0◦ ≤ x ≤ 360◦ is given by α + 180◦.

If values of x>360◦ are required they are found by adding 360◦ to the values already
obtained.

Example
sin(x) = 0. 6

From the calculator x = 36. 9◦ , and the alternative value is 180 − 36. 9, that is, 143.1◦.

sin(x) = −0. 3

From the calculator x = −17. 5◦, and the alternative is 180 − (−17. 5)) = 197. 5.
198 • Mathematics

The next values are −17. 5 + 360 and 197. 5 + 360. That is, 342.5◦ and 557.5◦ . Therefore
the required solutions are 197.5◦ and 342.5◦.

cos(x) = 0. 34.

The calculator gives 70.1◦ and the alternative solution is 360◦ − 70. 1◦ , that is, 289.9◦.

Cosine Rule and the Sine Rule for


Non-Right-Angled Triangles
The Cosine rule applies if you are given either

(i) two sides and the included angle, for example, b, c and A; or
(ii) three sides.

The rule(s)

(i) a2 = b2 + c2 − 2 × b × c × cos(A)
b2 + c2 − a2
(ii) cos(A) =
2×b×c
The Sine rule applies when you are given a ‘matching pair’ of values and one other
dimension.

The rule

sin(A) sin(B) sin(C)


(i) = = , if you are finding an angle,
a b c
a b c
(ii) = = , if you are finding a side.
sin(A) sin(B) sin(C)

Remember that there is the possibility of two values for the angle when the sine function
is used.

It is preferable to use the Cosine rule (ii) if at all possible.


Solution of Triangles • 199

Plane Trigonometry – Sine and


Cosine Rules
1. The triangle XYZ has y = 11 cm, z = 15 cm and X = 55◦.

55°

11 cm
15 cm

Using the cosine rule x 2 = y2 + z2 − 2 · y · z · cos(X)



Therefore x = 112 + 152 − 2 × 11 × 15 × cos(55) = 12. 5 cm.

(The exact value of x should be either written down or, more preferably, stored in the
calculator’s memory.)
x 2 + y 2 − z2
Re-arranging the cosine rule gives cos(Z) =
2·x·y
12. 52 + 112 − 152
Therefore cos (Z) = = 0. 19142
2 × 12. 5 × 11
Therefore Z = cos−1 (0. 19142) = 78. 96◦
x 2 + z2 − y2 12. 52 + 152 − 112
Similarly cos(Y) = = = 0. 69421
2·x·z 2 × 12. 5 × 15
Therefore Y = cos−1 (0. 69421) = 46. 0◦ .

Example

The vertical post of a jib crane is 10 m long. The angle between jib and post is 40◦, and
between jib and tie the angle is 45◦ . Find the length of the tie and the length of the jib.
200 • Mathematics

TIE 45°
B
Post = 10 m

JIB

40°

 Figure 8.3

a c c · sin (A)
Referring to Figure 8.3, = so a =
sin(A) sin(C) sin(C)
10 × 0. 6428
Therefore a = = 9. 090 m
0. 7071
Thus B = 180◦ − (40◦ + 45◦) = 95◦
b c c · sin (B)
Also = so b =
sin(B) sin(C) sin(C)
10 × 0. 9962
Therefore b = = 14. 088 m
0. 7071
Length of TIE = 9. 09 m

Length of JIB = 14. 09 m

Example

Two ships leave the same port at the same time on courses which diverge at 29◦ . When
one ship had travelled 40 nm the two ships were then 21 nm apart, find how far the
other ship had travelled from port.
a c c · sin (A)
Referring to Figure 8.4, = so sin (C) =
sin(A) sin(C) a
40 × 0. 48481
Therefore sin (C) = = −0. 92345
21
So C = 67◦ 26 or 180◦ − 67◦ 26 = 112◦ 34

Therefore there are two possible answers to this problem, as illustrated in Figure 8.4.
Solution of Triangles • 201

21 nm
C
a
b 21 nm
a

29°
A c
40 nm B
Port

 Figure 8.4

If C = 67◦ 26 ,

B = 180◦ − (29◦ + 67◦26 ) = 83◦ 34


a b a · sin (B)
But = so b =
sin(A) sin(B) sin(A)
21 × 0. 99370
Therefore b= = 43. 04 (i)
0. 48481

If C = 112◦34 ,

B = 180◦ − (29◦ + 112◦34 ) = 38◦26


21 × 0. 62160
Therefore b= = 26. 93 (ii)
0. 48481

For the two ships to be 21 nm apart and one having travelled 40 nm, the other ship has
travelled either 43.04 or 26.93 nm.

Example

A ship going due North at 16 knots runs into a 5 knots current moving North-East. Find
the resultant speed and the direction of the ship.

Reference Figure 8.5, Angle A = 90◦ + 45◦ = 135◦

a2 = b2 + c2 − 2 × b × c × cos(A)
= 162 + 52 − 2 × 16 × 5 × cos(135)
= 256 + 25 − 160 × (−0. 7071)
= 349. 14
202 • Mathematics

c=5 B

A
135°

b = 16

 Figure 8.5


Therefore a = 349. 14 = 19. 85 knots
a c c · sin(A)
Now = so sin (C) =
sin(A) sin(C) a
5 × 0. 70711
Therefore sin (C) = = 0. 17811
19. 85
So C = 10◦ 16

Resultant speed = 19.85 knots on a bearing of 010◦16 (10◦16 East of North).

Application of Trigonometry
Questions
1. Two voltage phasors are shown in Figure 8.6. If z = 60 V and w = 90 V, calculate
the value of their resultant (i.e. the length AB) and the angle the resultant makes
with z.
2. Two powerboats, A and B, leave a port at the same time. A sails at a steady speed
of 52 km/h on a bearing of 212◦ and B at 38 km/h on a bearing of 156◦. Find their
distance apart after 21/2h.
3. A room 9 m wide has a span roof which slopes at 32◦ on one side and 41◦ on the
other. Find the length of the roof slopes to the nearest centimetre.
Solution of Triangles • 203

w = 90 V

30°
A
z = 60 V

 Figure 8.6

75°
X Z

 Figure 8.7

4. A jib crane consists of a vertical stanchion AB, 5.2 m in length, the inclined jib BC,
12.8 m in length, and a tie AC. The angle BAC is 122◦.
Calculate (a) the length of the tie and (b) the inclination of the jib to the vertical.
5. A reciprocating engine mechanism is shown in Figure 8.7, where XY represents the
rotating crank, YZ is the connecting rod and Z is the piston which moves horizontally
along the line XZ.
If the rotating crank is 1.24 cm in length and the connecting rod is 6.48 cm, calculate
for the position shown (a) the inclination of the connecting rod to the vertical and
(b) the distance XZ.
6. Figure 8.8 shows the jib of a crane, consisting of three members plus the pulley wire
supporting the load. Find the length of the strut AC and determine its angle from the
vertical.
7. A crank mechanism of a petrol engine is shown in Figure 8.9. Arm OA is 12.0 cm long
and rotates clockwise about O. The connecting rod AB is 35.0 cm long and end B is
constrained to move horizontally.
(i) For the position shown above, determine the angle between AB and the hori-
zontal.
(ii) Calculate the length of OB.
204 • Mathematics

a=9m
B
110°
c = 2.7 m b Load

 Figure 8.8

A
35.0 cm
B 12.0 cm
57°
O

 Figure 8.9

8. PQ and QR are the phasors representing the alternating currents in two branches of
a circuit. Phasor PQ is 20.0 A and is horizontal. Phasor QR (which is joined to the end
of PQ to form a triangle PQR) is 29.0 A and is at an angle of 32◦ to the horizontal.
Determine the resultant phasor PR and the angle it makes with PQ.

Answers

1. 145 V, 18.1◦ 2. 110 km


3. 6.17 m, 4.99 m 4. (a) 9.26 m (b) 37.9◦
5. (a) 10.7◦ (b) 6.689 cm 6. 10.24, 55.7◦
7. (a) 16.7◦ (b) 40.1 cm 8. 47.2, 19.0◦

Example

The three sides of a triangle measure 8, 6 and 4 cm respectively. Find the angles
(Figure 8.10).
Solution of Triangles • 205

6 cm
b 4 cm
a

c A
B
8 cm

 Figure 8.10

b2 + c2 − a2
cos A =
2bc
4 + 82 − 62
2
=
2×4×8
= 0. 6875

Therefore angle A = 46◦ 34

(The alternative angle for A is 180◦ − 46◦34 = 133◦26 . This cannot be the case as angle
C is larger than angle A because side c is longer than side a. This would make angle C
larger than 133◦ 26 making the angles in the triangle greater than 180◦.)
a b b · sin (A)
= so sin (B) =
sin(A) sin(B) a
4 × 0. 7262
So sin(B) = = 0. 4841
6
Therefore angle B = 28◦ 57

This gives angle C = 180◦ − (46◦34 + 28◦57 ) = 180◦ − 75◦31

Therefore angle C = 104◦29

Summarising the formulae given so far for solving triangles, the sine, cosine and tangent
ratios of sides are used for right-angled triangles, and either the sine rule or cosine rule for
other triangles.

The sine rule is easier to calculate than the cosine rule but there is the problem with
each inverse sine calculation resulting in two possible values for the angle.

This problem never occurs with the cosine function and so the cosine rule may be
preferable.
206 • Mathematics

The cosine rule must be used in the situations where

• two sides and the angle between them (the included angle) are given, or
• three sides are given.

Areas of Triangles
1
Area of triangle = (base × perpendicular height)
2
Referring to Figure 8.11,

Perpendicular height = a × sin (C)


1
So Area = (b × a × sin C)
2
ab · sin C
Therefore Area =
2
In general terms this means that ‘the area is half the product of two sides and the sine of
the angle between these sides’.

An easy formula to apply and a very useful one.

Another important formula for finding the area of a triangle is one which calculates the
area directly from the three sides of the triangle:

Area = s (s − a) (s − b) (s − c) where a, b and c are the sides of the triangle and
(a + b + c)
s=
2

a
Perp. c
height

C b
A
Base

 Figure 8.11
Solution of Triangles • 207

Example

The three sides of a triangle measure 8, 10 and 14 cm respectively, find the area
enclosed.

(a + b + c) (8 + 10 + 14)
s= = = 16
2 2
s − a = 16 − 8 = 8
s − b = 16 − 10 = 6
s − c = 16 − 14 = 2

 √ √
Area = s (s − a) (s − b) (s − c) = 16 × 8 × 6 × 2 = 1536 = 39. 2 cm2 , correct to
1 decimal place.

Equilateral triangle

An equilateral triangle is one which has three equal sides and three equal angles
(Figure 8.12).

Each angle is therefore 60◦ .



3
Perpendicular height = Side × sin 60◦ = Side ×
2
1
Area of triangle = (Base × Perpendicular height)
2

60°
Si
e

de
Sid

60° 60°
Side

 Figure 8.12
208 • Mathematics

 √ 
3
Side × Side × √
2 3
Therefore Area = = × (Side)2 ≈ 0. 433 × (Side)2
2 4

Isosceles triangle

An isosceles triangle is one which has two equal sides and two equal angles.

In Figure 8.13 angles A and C are equal, therefore the lengths of the sides a and c are
equal.

It is thus an easy matter to calculate the perpendicular height or other data required to
find the area by any of the general formulae.
a
c

A C

 Figure 8.13

Compound Angles

In some problems it is often convenient to express the relationship between the


trigonometrical ratios of a compound angle and those of the two single component
angles.

Referring to Figure 8.14,

Area of CED = area of CFD + area of CFE


e · [d · sin (A + B)] g · [e sin (A)] · g · [d sin (B)] ·
= +
2 2 2
Solution of Triangles • 209

e
a
g
A F
C c
B

b
d

 Figure 8.14

ed
Dividing throughout by ,
2
g g
sin (A + B) = · sin A + · sin B
d e
= cos B · sin A + cos A · sin B

Usually written as,


sin (A + B) = sin A · cos B + cos A · sin B (i)

By a similar process it can be shown that,

sin (A − B) = sin A · cos B − cos A · sin B (ii)

Again referring to Figure 8.14 and applying the cosine rule,

e2 + d 2 − c 2
cos C =
2ed
e2 + d2 − (a + b)2 e2 + d2 − a2 − 2ab − b2
cos (A + B) = =
2ed 2ed
but, by Pythagoras, e2 − a2 = g2 and d2 − b2 = g2

2g2 − 2ab g2 ab g g a b
Therefore cos (A + B) = = − = × − ×
2ed ed ed e d e d

Therefore cos (A + B) = cos A · cos B − sin A · sin B (iii)

By a similar kind of process it can also be shown that,

cos (A − B) = cos A · cos B + sin A · sin B (iv)


210 • Mathematics

The tangent of angle being the sine divided by its cosine, the relationship for the
tangent of a compound angle can be obtained by dividing sin(A + B) by cos(A + B),

sin (A + B) sin A · cos B + cos A · sin B


tan(A + B) = =
cos(A + B) cos A · cos B − sinA · sin B

Dividing all terms, top and bottom, by cos A cos B,


sin A · cos B cos A · sin B
+
tan(A + B) = cos A cos B cos A cos B
cos A · cos B sin A · sin B

cos A cos B cos A cos B
Therefore
tan A + tanB
tan (A + B) = (v)
1 − tanA · tanB
and by a similar process,
tan A − tanB
tan (A − B) = (vi)
1 + tanA · tanB
In summary

sin (A ± B) = sin A · cos B ± cos A · sin B


cos (A ± B) = cos A · cos B ± sin A · sin B
tan A ± tanB
tan(A ± B) =
1 ± tanA · tan B

Double Angles
Formulae expressing the relationship between the ratios of angles and double angles
are useful and can be obtained from those above by letting A = B;

sin (A + B) = sin A · cos B + cos A · sin B gives sin 2A = 2 sin A cos A


cos (A + B) = cos A · cos B − sin A · sin B gives cos 2A = cos2 A − sin2 A (#)
tan A + tanB 2 tanA
tan (A + B) = gives tan 2A =
1 − tanA · tanB 1 − tan2 A

Also, since cos2 A + sin2 A = 1

then cos2 A = 1 − sin2 A and sin2 A = 1 − cos2 A


Solution of Triangles • 211

Substituting for cos2 A and sin2 A in turn into (#) gives:


 
cos 2A = cos2 A − sin2 A = 1 − sin2 A − sin2 A

Therefore cos 2A = 1 − 2 sin2 A


 
cos 2A = cos2 A − sin2 A = cos2 A − 1 − cos2 A

Therefore cos 2A = 2 cos2 A − 1

Note: Also see Identities in Chapter 7.

Test Examples 8
1. The top of a vertical mast is viewed from a position at 15 m from its base on level
ground and the angle of elevation measured to be 45◦ 34 . Find the height of the
mast.
2. A boat is sighted from a point on a cliff 95 m above the sea, the angle of depression
of the line of view being 14◦ 25 . Find
(i) the horizontal distance from cliff to boat, and
(ii) the distance from observation point to boat.
3. A right-angled triangular plate has one angle of 28◦ 37 and the length of the
hypotenuse is 120 mm. Find the lengths of the other two sides and the area of
the triangle.
4. The top of a flagstaff is viewed from a point on a level ground at some distance
from the foot of the staff, and the angle of elevation is 48◦ 30 . At another point
10 m further away from the foot, the staff top is viewed again and this time the
angle of elevation is 37◦ 38 . Find the distance from the foot of staff to the first
observation point and also the height of the flagstaff.
5. The length of the sides of a cube is 60 mm. Find the length of the diagonal across
the face and the length of cross diagonal from one corner to the opposite corner
passing through centre of the cube.
6. The length of the vertical post of a jib crane is 15 m. The angle between jib and
post is 35◦ 30 and between tie and post the angle is 105◦ 30 . Calculate the length
of the jib and tie.
7. In a reciprocating engine of 800 mm stroke, the connecting rod is 1600 mm long.
Find how far the crosshead has moved from the top of its stroke when the crank is
35◦ past its top dead.
212 • Mathematics

8. The fuel valve of a diesel engine closes when the piston has moved one-tenth of its
stroke, find the angular position of the crank from top dead centre when the valve
closes.
9. A ship going due South at 17 knots runs into a 4 knots current running 50◦ East of
North. Find the resultant speed and direction of the ship.
10. Two ships approach a port, their courses converging at an angle of 23◦ . At a certain
time one ship is twice as far from port as the other and their distance apart is 32
nm; find how far each ship is from port.
11. Two ships are 50 nm apart and going towards the same port at the same speed
of 18 knots, on courses which converge at 73◦ 39 . If one ship will arrive in port
half-an-hour before the other, find their distances from port.
12. Three sides of a triangle measure 16.4, 10.2 and 9.8 m respectively. Find the angles.
13. Two sides of a triangle measure 6.5 and 7.5 m respectively and the angle included
between these two sides is 46◦ 51 . Find the area of the triangle.
14. Find the area in cm2 of a triangular plate of sides 71, 42 and 53 mm long
respectively.
15. The area of an equilateral triangle is 57.27 cm2 . Find the length of its sides.
MENSURATION
9
OF AREAS
Units
Areas are measured in (unit)2 , e.g. m2 , cm2 .

It is often necessary to change units. The conversion factors are given in the table below:

Linear Area
1 cm = 10 mm 1 cm2 = 102 = 100 mm2
1 m = 100 cm 1 m2 = 1002 = 10000 cm2
1 m = 1000 mm 1 m2 = 10002 = 1000000 mm2
1 km = 1000 m 1 km2 = 10002 = 1000000 m2

In general, if the linear units are in the ratio 1 : r, the areas are in the ratio 1 : r2 .

It follows that, for example, as


1
1 cm2 = 100 mm2 then 1 mm2 = cm2 = 0. 01 cm2 = 10−2 cm2
100

A rectangle is a four-sided figure whose opposite sides are parallel and equal in length
to each other (Figure 9.1). Each of the angles is 90◦ .
214 • Mathematics

Height

Base

 Figure 9.1

Area of rectangle = base × height.

A parallelogram is a four-sided figure whose opposite sides are parallel and equal in
length to each other.

It therefore follows that opposite angles are equal, one pair of opposite angles being
obtuse and the other pair acute and supplementary to the obtuse angles.

It may be considered as a rectangular framework leaning over to one side as in Figure 9.2
where it can be seen that the outer triangular area (shown dotted) at one end is equal
to the inner triangular area at the other and therefore the area of the parallelogram is
equal to that of a rectangle of the same base and same perpendicular height.

Perpendicular
height

Base

 Figure 9.2

Area of parallelogram = Base × Perpendicular height.

Also, if a diagonal is drawn from one corner to the opposite corner, it will bisect the
parallelogram into two equal parts, each part being a triangle, the area of each being
half that of the parallelogram (Figure 9.3).
Mensuration of Areas • 215

Perpendicular
height

Base

 Figure 9.3

1
Area of triangle = × Base × Perpendicular height
2
ab · sin C
Area of triangle =
2
 (a + b + c)
Area of triangle = s (s − a) (s − b) (s − c) where s =
2
A rhombus is a special kind of parallelogram. It is a diamond-shaped, four-sided
figure with all sides of equal length and opposite sides parallel to each other (see
Figure 9.4). The diagonals of a rhombus are perpendicular to each other and their inter-
section forms right angles; the diagonals bisect each other and each bisects its corner
angles.

 Figure 9.4

The area of a rhombus is half of the product of its diagonals.

A trapezium is a four-sided figure, of which only two sides are parallel. Referring to
Figure 9.5, if a and b are the respective lengths of the two parallel sides and h is the
perpendicular height, then,
216 • Mathematics

(a + b) h
Area of trapezium = Average length × Perp. height =
2

a+b
h
2

 Figure 9.5

The equivalent rectangle of the trapezium is shown in dotted lines.

Polygons

A polygon is a figure bounded by more than four straight sides.

A regular polygon has all its sides equal in length and all its angles equal; any regular
polygon is therefore made up of as many equal triangles as there are sides and the
method of finding the area of a polygon is to first find the area of one triangle and then
multiply by the number of triangles which make up the polygon (Figure 9.6).

Special names are given to some regular polygons as follows:

Number of sides Name


5 Pentagon
6 Hexagon
7 Heptagon
8 Octagon
9 Nonagon
10 Decagon

A polygon with n sides is made up from n triangles.


360◦
Central angle of each triangle, α =
n
Mensuration of Areas • 217

α
r r

 Figure 9.6

If the radius of the circle that surrounds the polygon is r, then r is also the slant height
of each triangle.

Note that all the triangles are isosceles.


ab · sin C r × r × sin α r2 · sin α
Area of each triangle = =
2 2 2
n · r2 · sin α
As there are n triangles the area of polygon
2
It is, however, also convenient to express the area of the more common polygons in
terms of the length of the ‘sides’ or ‘flats’.
180 − α
If the central angle is α, then the base angle β = (as the triangle is isosceles the
2
base angles are equal).

Side

 Figure 9.7

h
From Figure 9.7 = tanβ
Side/2
218 • Mathematics

(Side) tan β
So perpendicular height h =
2
1 (Side) tan β (Side)2 · tan β
The area of the triangle is therefore × (Side) × =
2 2 4
n · (Side)2 · tan β
and the area of the polygon is
4
360
If there are n sides then α = for the most common polygons.
n

Number of sides α β
5 72◦ 54◦
6 60◦ 60◦
8 45◦ 671/2◦
10 36◦ 72◦

Therefore the areas of these polygons are:

Number of sides Area


5 1. 7205 × (side)2
6 2. 5981 × (side)2
8 4. 8284 × (side)2
10 7. 6942 × (side)2

The circle

The circumference is the outer rim of the circle, an arc is part of the Circumference, other
common terms are illustrated in Figure 9.8.

Circumference = π × diameter
= πd
= π × 2 × radius
= 2πr

where π = 3. 142 to 3 decimal places.


Mensuration of Areas • 219

Sector
Chord
Arc

Diameter

Segment

Radius

Tangent

 Figure 9.8

The definition of π is the result when the circumference of a circle is divided by its diameter.

The area of a circle is given by the formula Area = π · r2 .

Many formulae are just stated, but in some cases, they can be proved. In other words a
reason can be given for the way they look.

Take the circle as an example.

A circle can be considered to be a regular polygon with an infinitely large number


of sides. Therefore the circle is made up from a great number of small triangles with
vertical height r.
r × Base
Each triangle has an area equal to and so the area of the circle is equal to the
2
area of all the triangles.
 
r × Base
Therefore, area of circle = sum of areas of the triangle = sum of =
2
 r × Base 
2

 r × Base  r  
but = {Base} where {Base} = Circumference
2 2
r
So Area × 2π · r = π · r2
2
 2
d d π · d2
Also, since r = , area = π · =
2 2 4
It is often more practical to use the formulae containing d, rather than r, because it is
easier in reality to measure the diameter of a circle than its radius.
220 • Mathematics

Annulus or circular ring

Area of annulus = Area of outer circle − Area of inner circle

 Figure 9.9

Area = π · R2 − π · r2
 (see Figure 9.9)
= π R2 − r2
or
D 2 d 2
Area = π · −π ·
2 2
π 2
= D − d2
4
By factorisation R2 − r2 = (R + r)(R − r) and D2 − d2 = (D + d)(D − d)

Use of the factors makes calculations of annular areas much quicker.

Example

Find the effective under-face area of a reciprocating pump piston 41.5 mm diameter if
the piston rod diameter is 8.5 mm.
π 2 π
Area of annulus = D − d2 = (D + d) (D − d)
4 4
π
Area = (41. 5 + 8. 5) (41. 5 − 8. 5)
4
π
= × 50 × 33
4
= 1296 mm2 to the nearest whole number
Mensuration of Areas • 221

Sector of a Circle
A sector of a circle is shown in Figure 9.10.

Arc

Area

r
θ

 Figure 9.10

If θ is the angle at the centre, in degrees,

area of circle of 360◦ = π · r2


π · r2
area of sector of 1◦ =
360
θ · π · r2
area of sector of θ ◦ =
360
θ πd θ πr
Similarly arc length = =
360 180
Further, if the angle θ at the centre be measured in radians,

area of circle of 2π radians = π · r2


π · r2 r2
area of sector of 1 radian = =
2π 2
θ · r2
area of sector of θ radians =
2
Similarly arc length = rθ
222 • Mathematics

Segment of a Circle
Referring to Figure 9.10,

Area of
segment

 Figure 9.10

Area of segment = Area of sector − Area of triangle


θ · r2
From previous section, area of sector =
2
where θ is the centre angle in radians.
ab · sin C
Area of triangle = where, in this case,
2
a = r, b = r and C is the centre angle θ in radians.
r2 · sin θ
Therefore area of triangle =
2

Area of segment = area of sector − area of triangle


r2 · θ r2 · sin θ
= −
2 2
r 2
= · [θ − sin θ] , θ in radians
2

Example

Calculate the area of segment which subtends an angle of 150◦ at the centre of a circle
of 150 mm diameter.
Mensuration of Areas • 223

π 5π
Converting θ to radians, θ = 150 × =
180 6
Also sin 150◦ = 0. 5
r2
Area of segment = · [θ − sin θ]
2
 
752 5π
= − 0. 5
2 6
= 5957mm2

Ellipse
Circumference = π× average diameter (see Figure 9.11)

Minor
axis
d

Major axis
D

 Figure 9.11

D+d
Therefore C = π
2
π ·D·d
Area =
4

Surface of a Cylinder
Imagine the curved surface of a cylinder being unrolled as in Figure 9.12: it can be seen
that the curved surface area of a cylinder A = πd · h or A = 2π · r · h.
224 • Mathematics

Circumference = π·d

 Figure 9.12

Surface of a Sphere
The curved surface area of a sphere is equal to the curved surface area of its circum-
scribing cylinder, that is, a cylinder of equal diameter and height.

Referring to Figure 9.13,

 Figure 9.13

Curved surface area = π × diameter × height

Therefore area = π · d2 or area = 4π · r2

The curved surface area of a segment of the sphere or any such sliced portion is equal
to the curved surface area of the corresponding slice of the circumscribing cylinder.

Referring to Figure 9.14,

Curved surface area of segment = π · d · h = πd × h


Mensuration of Areas • 225

 Figure 9.14

Surface area of a cone

If a cone has a base of radius r and the length of a side (i.e. the slant height) is L then the
curved surface area = π · r · L.

Theorem of Pappus, or Guldinus

This theorem is one of the most useful to employ in finding areas and volumes of objects
of circular shapes, or if the area or volume is known it can be used to determine the
position of the centroid, or centre of gravity (mass), of many sections.
226 • Mathematics

With regard to areas the theorem states:

If a line, lying totally on one side of a fixed axis, is rotated about that axis in its own plane,
it will sweep out a surface area equal to the length of the line multiplied by the distance
moved by its centre of gravity.

Axis Axis

L L
Area swept out
r

 Figure 9.15

Consider a ‘line’ such as a straight piece of wire of length L (Figure 9.15), positioned at a
distance r from an axis parallel to the wire.

If the wire is moved around through a complete circle about the axis, a surface area like
a thin cylinder will be swept out.

The centre of gravity of the piece of wire is at its midpoint.

The distance moved by the centre of gravity in one revolution is the circumference of
the cylinder.

Therefore, distance moved = 2π · r

So the area swept out = Length of line × Distance moved by centre of gravity = L×2π ·r

Thus area swept out = 2π · r · L

Surface area of a frustum of a cone (or pyramid)

The frustum of a cone (or pyramid) is the shape remaining when the top section of the
cone (or pyramid) is removed by a cut parallel to the base (Figure 9.16).

The only measurements available are the radius (or diameter) of the top surface, the
radius (or diameter) of the base surface, the height of the frustum and the length of the
sloping side.
Mensuration of Areas • 227

h L

 Figure 9.16

(R + 2)
Curved surface area of frustum = L × 2π × = π · L · (R + r)
2

Surface area of a torus

Consider a circle of radius r whose centre is R units from a fixed vertical axis (Figure 9.17).

When this circle is swept through a complete revolution about the axis, the shape
created is called a torus (basically, a doughnut).

The surface area of this torus, that is the area exposed to the air is given by the formula:

curved surface area = 4π 2 · R · r.

r R

 Figure 9.17

If the circle was replaced by an ellipse of major axis D and minor axis d, curved surface
area π 2 · R · (D + d).
228 • Mathematics

Similar figures

Areas of similar figures vary as the square of their corresponding linear dimensions.

Similar figures mean that they are of the same shape and proportions, although their
sizes are different.

d 2d

 Figure 9.18

Consider two circles, one of diameter d, the other twice as big in diameter = 2d, as in
Figure 9.18.

π · d2
Area of small circle =
4
π · (2d)2 π · 4d2 π · d2
Area of large circle = = =4·
4 4 4

Therefore the ratio of their areas is 1 : 4, that is 1 : 22

Therefore multiplying a linear dimension, for examples, diameter, by 2 increases the


area by a factor of 4.

So if lengths are increased by a factor of 3, areas are increased by a factor of 9.

In general, if the linear units are in the ratio 1 : r, the areas are in the ratio 1 : r2 .

Example

A hexagonal plate is cut out of a thin sheet of metal. Due to an error in marking off, the
sides were all made 10% longer than intended.

What is the percentage of error in area?

Ratio correct dimensions to wrong = 100 : 110 = 1 : 1. 1


Mensuration of Areas • 229

Therefore lengths are a factor of 1.1 too large.

So area will be a factor of 1.12 too large, that is, a factor of 1.21

Therefore the area is 21% too large.

Irregular figures

There are occasions when an area cannot be calculated precisely because it is not a
regular shape. In other words it is not made up of triangles, rectangles, etc.

To find the area of an irregular shape an approximate method has to be used. One such
method is Simpson’s First Rule.

In essence the area is divided up into an even number of equally spaced strips and a
formula applied to estimate the area. (The more strips, the more accurate the estimate:
doubling the number of strips will increase the accuracy by a factor of 16.)

Simpson’s First Rule


Simpson’s First Rule is based on the fact that a parabola y = ax 2 + bx + c can be found
so that it passes through three points (Figure 9.19). So, in the following diagram, the
curve AD can be represented as a parabola which then allows the area of ABCD to be
approximated as:
h
Area of ABCD ≈ (1 · y0 + 2 · y1 + 1 · y2 )
3

D
y0 y1 y2

B C
h h

 Figure 9.19

This rule can be applied repeatedly to adjacent pairs of areas to approximate the area
of larger shapes such as WXYZ (Figure 9.20).
230 • Mathematics

Z
1 2 3
y0 y1 y2 y3 y4 y5 y6

X Y
h

 Figure 9.20

h
Area 
1 =≈ (1 · y0 + 4 · y1 + 1 · y2 )
3
h
Area 
2 ≈ (1 · y2 + 4 · y3 + 1 · y4 )
3
h
Area 
3 ≈ (1 · y4 + 4 · y5 + 1 · y6 )
3
1 + Area 
Area WXYZ = Area  2 + Area 
3

h h
≈ (1 · y0 + 4 · y1 + 1 · y2 ) + (1 · y2 + 4 · y3 + 1 · y4 )
3 3
h
+ (1 · y4 + 4 · y5 + 1 · y6 )
3
h
≈ (1 · y0 + 4 · y1 + 2 · y2 + 4 · y3 + 2 · y4 + 4 · y5 + 1 · y6 )
3

This method can be extended to any even number of strips, each of width h.

It is easiest to set the calculations in a table, the values 1, 4, 2, 4, . . . being called the
‘Simpson multipliers’.

Waterplane area

When calculating the area of a waterplane it is usual to divide the length of the ship into
about 10 equal sections. It is convenient to make the measurements from the centre-
line to the ship side, giving half ordinates. The answer from Simpson’s rule can then be
doubled to give the waterplane area.
Mensuration of Areas • 231

Example

The equally spaced half ordinates of a watertight flat 27 m long are 1.1, 2.7, 4.0, 5.1, 6.1,
6.9 and 7.7 m respectively. Calculate the area of the flat.

Solution

Half ordinate Multiplier Product for area


1.1 1 1.1
2.7 4 10.8
4.0 2 8.0
5.1 4 20.4
6.1 2 12.2
6.9 4 27.6
7.7 1 7.7
Total 87.8

As there are 7 ordinates there must be 6 equal intervals.


27
Therefore h = = 4. 5 m
6
h 4. 5
Area = × Total = × 87. 8 = 132. 2 m2
3 3
The waterplane is therefore 132. 2 × 2 = 264. 4 m2 .

Example

The ordinates measured from the centreline to the hull across a ship at the load water
line are: 0.2, 9, 15.5, 20, 21.5, 20.5, 18.5, 12.5 and 1.3 m respectively, and the length is
180 m. Find the water plane area.

Solution

Half ordinate Multiplier Product for area


0.2 1 0.2
9.0 4 36.0
15.5 2 31.0
20.0 4 80.0
232 • Mathematics

Half ordinate Multiplier Product for area


21.5 2 43.0
20.5 4 82.0
18.5 2 37.0
12.5 4 50.0
1.3 1 1.3
Total 360.5

As there are 9 ordinates there must be 8 equal intervals.


180
Therefore h = = 22. 5 m
8
h 22. 5
Area = × Total = × 360. 5 = 2703. 75 m2
3 3
The waterplane is therefore 2703. 75 × 2 = 5407. 5 m2 .

Test Examples 9
1. In a parallelogram ABCD, the opposite parallel sides AD and BC are each 100
mm long, the other sides are each 60 mm long, and the diagonal AC is 140 mm.
Calculate the angles, the short diagonal, the perpendicular height and the area.
2. The sides of a rhombus are each 32 mm long and the length of the long diagonal
is 48 mm. Calculate the angles, the length of the short diagonal and the area.
3. In a trapezium ABCD, the two parallel sides are AB and CD, and their lengths are
100 and 60 mm respectively. Side BC is perpendicular to the parallel sides and its
length is 50 mm.
Find the area of the trapezium (cm2 ).
4. The lengths of the sides of a four-sided figure ABCD are, in m, AB = 1, BC = 2,
CD = 1. 5, DA = 3. 5, and the angle BCD is 117◦ 17 . Find the area of the figure.
5. The length of the sides of a regular hexagonal plate is 80 mm. The plate is cut par-
allel to one of its sides and this reduces the area by 10%, calculate the thickness of
the piece cut off.
6. An octagonal plate, the sides of which are each 30 mm long, has a circular hole
50 mm diameter cut out of it. Find the net area of the plate in mm2 .
Mensuration of Areas • 233

7. Find the length of the sides and the area of the largest equilateral triangular plate
that can be cut out of a circular plate 120 mm diameter.
8. The outer and inner diameters of the collar of a single-collar thrust shaft are 755
and 415 mm respectively, and the effective area of contact with the thrust pads is
0.7 of the face of the collar. Calculate (i) the effective area of contact, in m2 , and
(ii) the total force on the collar, in kN, when the thrust pressure is 2000 kN/m2 .
9. Find the area, in cm2 , of the smaller segment of a circle of 200 mm diameter if the
length of the chord is 180 mm.
10. Find the diameter of a solid hemisphere whose total surface area (including the flat
circular base) is 58.9 cm2 .
11. The ball of a Brinell hardness testing machine is 10 mm diameter. Calculate the
depth and curved surface area of an indentation in a material under test when the
surface diameter of the indentation is 5 mm.
12. It is required to make a hollow cone out of thin flat sheet steel, the base diameter of
the cone to be 150 mm and the perpendicular height 125 mm. Find the dimensions
of the sector to be cut out of the sheet to make this cone.
13. A lampshade has the form of a frustum of a cone, the diameters at the base and
top are 320 and 180 mm respectively and the perpendicular height is 170 mm.
Calculate the curved surface area.
14. A circular ring made of round bar is 640 mm outside diameter and 440 mm inside
diameter. Calculate the surface area to be painted.
15. An equilateral triangular plate has sides 125 mm long, and another similarly shaped
plate has sides 175 mm long. By what percentage is the larger plate greater in area
than the smaller plate?
16. Regularly spaced semi-ordinates measured transversely across a ship at the load
water line are as follows: 0.1, 3, 5.85, 7.2, 8.1, 8.4, 8.4, 8.25, 8.1, 7.5, 6.3, 3.75 and
0.5 m respectively, and the length is 150 m. Find the area of the waterplane by
Simpson’s rule.
10
VOLUME – MASS,
CENTRE OF
GRAVITY, MOMENT
Volume (V) is the result of the product of three dimensions measured in similar units.
Units m3 , mm3 and, often, cm3 .

Linear Volume
1 cm = 10 mm 1 cm3 = 103 = 1, 000 mm3
1 m = 100 cm 1 m3 = 1003 = 106 = 1, 000, 000 cm3
1 m = 1000 mm 1 m3 = 10003 = 109 = 1, 000, 000, 000 mm3
1 km = 1000 m 1 km3 = 10003 = 109 = 1, 000, 000, 000 m3

1000 cm3 = 1litre so 1 cm3 = 1millilitre (ml)

In general, if the linear units are in the ratio 1 : r, the volumes are in the ratio 1 : r3

It follows that, for example, as


1
1 cm3 = 1, 000 mm3 then mm3 = cm3 = 0. 001 cm3 = 10−3 cm3
1, 000

Mass (m) is the quantity of matter possessed by a body and is proportional to the
volume and the density of the body.
Volume – Mass, Centre of Gravity, Moment • 235

It is a constant quantity, that is, the mass can only be changed by adding more matter
or taking matter away.

The S.I. unit for mass is the kilogram (kg).

(1 kg = 1, 000 grams (g); 1, 000 kg = 1 tonne)

Density (ρ) is a measure of the mass per unit volume.

The unit of density is kg/m3 .

Other units are: g/cm3 for solid materials,


g/ml for liquids,
g/l for gases.

In some cases t/m 3 (tonne per m 3 ) and kg/1 (kg per litre) may be used.

The formula connecting these three quantities is:

Mass = Volume × Density


m =V ·ρ

The density of pure water may be taken as 1000 kg/m3

(This is equal to 1 tonne/m3 , 1 kg/l and 1 g/ml.)

Units must be consistent throughout, such as,

Mass [kg] = Volume [m3 ] × Density [kg/m3 ]

or
Mass [kg] = Volume [cm3 ] × Density [g/cm3 ]

Relative density (specific gravity) of a substance is the ratio of the mass of a volume
of the substance to the mass of an equal volume of pure water. In other words, it is the
ratio of the density of the substance to the density of pure water.

Prisms
A regular prism is a bar of regular cross-section, some examples are given in
Figure 10.1.
236 • Mathematics

 Figure 10.1

In all these cases,


Volume = Area of cross-section × Length
Hence, to find the volume of a prism, calculate the area of the end and multiply this by
length (or height) of the prism.

Example

A brass bar 250 mm long has a constant hexagonal cross-section measuring 90 mm


across the face from one corner to the opposite corner. Find (i) the volume of the bar,
(ii) the mass in kg if the density of brass is 8.4 g/cm3 .

90 mm

45 mm

45 mm

60°
45 mm

 Figure 10.2

Side of each equilateral triangle = 45 mm (see Figure 10.2)


Perpendicular height = 45 × sin 60◦
1
Area of each equilateral triangle = × Base × Perpendicular height
2
Volume – Mass, Centre of Gravity, Moment • 237

1
= × 45 × 45 sin60◦
2
= 876. 85 mm2

Therefore area of end face = 6 × 876.85 = 5261.1 mm2

Volume of prism = Area × Length

Therefore V = 5261. 1 × 250 = 1315275 mm3 = 1315 cm3 (nearest whole number)

Mass (m) = Volume (V) × Density (p)

Therefore m = 1315 × 8. 4 = 11046 g

So mass = 11.05 kg to 2 decimal places.

Pyramids
A pyramid is a body standing on a triangular, square or polygonal base, its sides tapering
to a point at the apex, some examples are illustrated in Figure 10.3. The cone may be
considered as a pyramid with a circular base.

 Figure 10.3

The volume of a pyramid is one-third of the volume of its circumscribing prism.


238 • Mathematics

Therefore, the volume of a cone is one-third of the volume of a solid cylinder having the
same height and base diameter as the cone.
π · r2 · h
Therefore, volume of a cone = .
3
Similarly, the volume of a square pyramid is one-third the volume of a bar of square base
with height equal to that of the pyramid.

In all cases:
1
Volume of pyramid = (Area of base × Perpendicular height)
3

Oblique Prisms and Pyramids


If the prism or pyramid be imagined as being made up of a number of discs or lamina-
tions and pushed over to one side, it can be seen by reference to Figure 10.4 that the
same rule for finding the volumes of regular prisms or regular pyramids can be applied
provided the perpendicular height is used.

 Figure 10.4
Volume – Mass, Centre of Gravity, Moment • 239

Frustums
A frustum of a cone or pyramid is the bottom piece left, after a portion has been sliced
off the top (Figure 10.5).

The volume can be found by subtracting the volume of the sliced-off top part from the
volume of the complete cone.

h L

 Figure 10.5

Example

Find the volume of a frustum of a cone if the radius of the base is 5 cm, the radius of the
top surface is 3 cm and the frustum has a height of 10 cm.

3 cm

10 cm

10 cm
240 • Mathematics

Looking at the frustum in cross-section gives:

10

It is necessary to project the sides to form the original cone and then find the height of
the projected section, h.

10

h h + 10
By similar triangles =
3 5
So 5h = 3 (h + 10) ⇒ 3h = 5h + 30 ⇒ 2h = 30

Therefore h = 15 cm

The volume of the whole cone is

π · r2 · h π × 52 × (15 + 10)
VW = = = 654. 50 cm3
3 3
The volume of the top cone is

π · r2 · h π × 32 × 15
VT = = = 141. 37 cm3
3 3
So the volume of the frustum is

654. 50 − 141. 37 = 513. 13 cm3


Volume – Mass, Centre of Gravity, Moment • 241

Sphere

4π · r3 π · d3
Volume of sphere = =
3 6
Volume of hollow sphere = Volume of outer sphere − Volume of inner sphere

π · D3 π · d 3 π  3 
Therefore V = − = D − d3
6 6 6
where D = outer diameter and d = inner diameter.

Example

A solid lead cone, 40 mm diameter at the base and 120 mm perpendicular height is to
be melted down and cast into a hollow sphere of 10 mm uniform thickness. Find the
inside and outside diameters of the sphere.

Solution

Working in centimetre:

Let d = inside diameter, then (d + 2) cm = outside diameter.


π 3 
Volume of hollow sphere = D − d3
6
π 
= (d + 2)3 − d3
6
π 3 
= d + 6d2 + 12d + 8 − d3
6
π 2 
= 6d + 12d + 8
6
π · r2 · h
Volume of cone =
3
π × 22 × 12
=
3
= 16π cm3

Therefore
π 2 
6d + 12d + 8 = 16π
6
6d2 + 12d + 8 = 96
6d2 + 12d − 88 = 0
3d2 + 6d − 44 = 0
242 • Mathematics

Solving this quadratic:



−6 ± 62 − 4 × 3 × (−44)
d=
2×3

−6 ± 564
=
6
−6 ± 23. 749
=
6
= 2. 958 and − 4. 958

Obviously d has to be positive, so the inner diameter is 2.96 cm and the outer diameter
is 4.96 cm, both to 2 decimal places.

Theorem of Pappus applied to volumes

With regard to volumes the theorem states:

If an area, lying totally on one side of a fixed axis, is rotated about that axis in its own
plane, it will sweep out a volume equal to its area multiplied by the distance moved by
its centre of gravity, or centroid.

Axis

r
R

 Figure 10.6

In the majority of cases areas are being swept around one complete revolution and the
resultant volumes are referred to as ’solids of revolution’.

For example, consider a flat circular disc of radius r, its centre being at R from the axis as
in Figure 10.6. If this area is swept around the axis through one complete revolution, it
Volume – Mass, Centre of Gravity, Moment • 243

will sweep out a solid ring of circular section (a torus), the mean radius of the ring being
R, and the radius of the cross-section of the material being r.

A practical example of this is the volume of a ring, therefore:

Volume swept out = Area × Distance moved by the centroid


Volume of ring = π · r2 × 2πR = 2π 2 · R · r2

Now consider an elliptical area of major diameter D, minor diameter d, swept around
through one revolution, R being the radius from the axis to the centre of the ellipse
(Figure 10.7).

Axis

R
d

 Figure 10.7

The volume swept out is the shape of a circular lifebuoy of elliptical cross-section.

Volume swept out = Area × Distance moved by the centroid


π π2
Volume of lifebuoy = D·d×π ·R = D·d·R
4 4

It is best to work from first principles making use of this theorem rather than applying
formulae.

Example

The inside diameter of a solid, circular, cork lifebuoy is 500 mm and the section is
elliptical 160 mm major diameter by 120 mm minor diameter.

Find (i) its volume in m3 , (ii) its mass if the density of cork is 240 kg/m3 .
244 • Mathematics

Referring to Figure 10.7, working in metres:

D = 160 mm = 0. 16 m
d = 120 mm = 0. 12 m
(Internal diameter) (Major axis)
R= + = 250 + 80 = 330 mm = 0. 33 m
2 2
Volume swept out = Area × Distance moved by the centroid

π
Volume = × 0. 16 × 0. 12 × 2π × 0. 33
4
= 0. 031267 m3
Mass = Volume × Density
= 0. 031267 × 240
= 7. 504 kg

Force, weight and centre of gravity

Force is that which produces or tends to produce motion in a body.

The unit of force is the newton (N).

It may be defined as ‘the force required to give unit acceleration (a gain of velocity of 1 m/s
every second the force is applied) to unit mass (one kg)’.

The weight (W) of a body is the gravitational force on the mass of that body, that is, the
force of attraction exerted on the body by the earth.

If a body is allowed to fall freely, it will fall with an acceleration of 9.81 m/s2 , this is
termed gravitational acceleration and is represented by g.

Since 1 N of force will give 1 kg of mass an acceleration of 1 m/s2 , then the force (N) to
give m kg of mass an acceleration of 9.81 m/s2 is m × 9. 81. Hence, at the earth’s surface,
the gravitational force on a mass of m kg is mg newtons, therefore:
 
Weight (N) = Mass (kg) × g m/s2
W = m·g

The centre of gravity of a body is that point through which the whole weight of the mass
can be considered as acting (Centre of Mass).
Volume – Mass, Centre of Gravity, Moment • 245

For instance, imagine a body to be compressed in volume into one tiny particle with-
out losing any mass, the position of this small heavy particle would be at the centre of
gravity of the body to have the same effect.

If the body was suspended from this point, or supported on it, it would balance perfectly
without tilting.

When dealing with an area instead of a solid, an area theoretically has no mass, therefore
it is not strictly correct to use the term centre of gravity, in such cases the term centroid
can be used.

Parallelogram

A parallelogram would balance if laid on a knife edge along either one of its two diag-
onals, therefore its centre of gravity (for a plate) or centroid (for a plane area) is at the
intersection of its diagonals as shown in Figure 10.8.

 Figure 10.8

Trapezium

The centroid of a trapezium is at the intersection of EF and HG as shown in Figure 10.9,


found graphically as follows:

Join EF (the midpoints)


Produce AD to G, length DG being equal to BC
Produce BC to H, length BH being equal to AD
Join HG
246 • Mathematics

The intersection of EF and HG is the centroid.

B E C
H

G
A F D

 Figure 10.9

Triangle

The centroid of a triangle lies on a line joining a corner of the triangle with the midpoint
of its opposite side, at a point at one-third of the height from that side (see Figure 10.10).

h h
3

 Figure 10.10

Pyramid

The centre of gravity of a pyramid or cone is at one-quarter of the height above the base
(see Figure 10.11).
Volume – Mass, Centre of Gravity, Moment • 247

h
4

 Figure 10.11

Hemisphere

The centre of gravity of a hemisphere is at three-eighths of the radius above the


diameter.

Volume of the frustum of a cone

Let r = radius at top, R = radius at bottom and h = perpendicular height. See


Figure 10.12.

 Figure 10.12

π ·h  2 
Total volume = R + R · r + r2
3
D d π ·h  2 
As R = and r = , Volume = D + D · d + d2
2 2 12
248 • Mathematics

Similar solids

Areas of similar solids vary according to the cube of their corresponding linear
dimensions.

Similar solids mean that they are of the same shape and proportions, although their
sizes are different.

d 2d

 Figure 10.13

Consider two spheres, one of diameter d, the other twice as big in diameter = 2d, as in
Figure 10.13.

π · d3
Volume of small sphere =
6
π · (2d)3 π · 8d3 π · d3
Volume of large sphere = = =8·
6 6 6

Therefore the ratio of their areas is 1 : 8, that is 1 : 23 .

Therefore multiplying a linear dimension, for example, diameter, by 2 increases the


volume by a factor of 8.

So if lengths are increased by a factor of 3, volumes are increased by a factor of 27.

In general, if the linear units are in the ratio 1 : r, the areas are in the ratio 1 : r3

It follows that, if the solids were made of the same material, their masses would also
vary in the same ratios.
Volume – Mass, Centre of Gravity, Moment • 249

Simpson’s first rule applied to volumes

The procedure of finding the volume of an irregular object by Simpson’s rule is the same
as for the area of an irregular figure, it is merely a matter of substituting cross-sectional
areas for ordinates.

Example

The immersed cross-sectional areas through a ship 180 m long, at equal intervals, are 5,
118, 233, 291, 303, 304, 304, 302, 283, 171 and 0 m2 respectively.

Calculate the displacement of the ship in sea water of 1.025 tonne/m3 .

Solution

Cross-sectional areas Multiplier Product for volume


5 1 5
118 4 472
232 2 466
291 4 1164
303 2 606
304 4 1216
304 2 608
302 4 1208
283 2 566
171 4 684
0 1 0
Total 6995

180
Common interval h = = 18 m
10
h 18
Volume of displacement = × Total = × 6995 = 41970 m3
3 3
Displacement = Volume of displacement × Density
= 41970 × 1. 025 = 43019 tonne
250 • Mathematics

Example

A casting of light alloy, 750 mm long, has a variable cross-sectional area throughout its
length. At regular distances of 125 mm apart, starting from one end, the sectional areas
are, 12.2, 17.5, 23.2, 27.9, 21.0, 11.2 and 0 cm2 respectively. Find the volume and its mass
if the density of the material is 3.2 g/cm3 .

Solution

Cross-sectional areas Multiplier Product for volume


12.2 1 12.2
17.5 4 70.0
23.2 2 46.4
27.9 4 111.6
21.0 2 42.0
11.2 4 44.8
0 1 0
Total 327.0

Common interval h = 12. 5 cm


h 12. 5
Volume = × Total = × 327. 0 = 1362. 5 cm3
3 3
Mass = Volume × Density = 1362. 5 × 3. 2 = 4360 g = 4. 36 kg

Example

Plot the graph xy = 6 between the limits x = 2 and x = 6.

If the area under this graph is rotated about its x axis through one complete revolution,
calculate the volume swept out by Simpson’s rule.

Solution
6
x·y = 6 ⇒ y =
x
Plotting points for values of x between the limits x = 2 and x = 6,
Volume – Mass, Centre of Gravity, Moment • 251

x 2 3 4 5 6
y 3 2 1.5 1.2 1

4 y

x
0
1 2 3 4 5 6

 Figure 10.14

The graph is shown in Figure 10.14. When the area under this graph is rotated about its
x axis through one revolution, the volume swept out appears as shown in Figure 10.15.

 Figure 10.15

The y ordinates of the graph become the radii of the solid at regular intervals along its
length. Putting the cross-sectional areas at these regular intervals through Simpson’s
rule as in the previous example:
252 • Mathematics

Radii r Cross-sectional areas = π · r2 Multiplier Product for volume


3 9π 1 9π
2 4π 4 16π
1.5 2. 25π 2 4. 5π
1.2 1. 44π 4 5. 76π
1 1π 1 1π
Total 36. 26π

Common interval between ordinates = 1


1
Volume = × 36. 26π = 37. 97 units3
3

Flow of liquid through pipes

Volume flow (V) is the volume of a fluid flowing past a given point in unit time (m3 /s).

The velocity or speed of flow is the ’length’ of liquid which passes in a given time.

For example, if the velocity of the liquid is 2 m/s it means that a column of the liquid 2 m
long passes every second.

So

   
Volume flow m3 /s = Cross-sectional area of the liquid m2 × Velocity (m/s) .

Therefore V = A · v

Mass flow (m) is the mass of fluid flowing past a given point in unit time.

Since density is the mass per unit volume, then the mass flow is the product of the
volume flow and the density. So

   
Mass flow (kg/s) = Volume flow m3 /s × Density kg/m3

Therefore m = V · ρ
Volume – Mass, Centre of Gravity, Moment • 253

Example

Oil of density 0.85 g/ml flows full bore through a pipe 50 mm diameter at a velocity of
1.5 m/s. Find the quantity flowing (i) in m3 /h, (ii) kg/h, (iii) tonne/h.

Solution
Diameter of pipe = 0.05 m

Velocity = 1.5 m/s = 1. 5 × 3600 = 5400 m/h


   
(i) Volume flow m3 /h = Area m2 × Velocity (m/h)

π · (0. 05)2
V= × 5400
4
= 10. 603 m3

(ii)

Density = 0. 85g/ml
 
= 0. 85 × 106 g/m3
 
0. 85 × 106
= kg/m3
1000
= 850 kg/m3
   
Mass flow (kg/h) = Volume flow m3 /h × Density kg/m3
Therefore m = 10. 603 × 850 = 9012. 4 kg
(iii) m = 9.0124 tonne/h

Flow through valves

When a liquid flows out of the open end of a pipe, the maximum quantity of liquid
escaping depends upon the area of the bore of the pipe end.

The flow can be restricted by a cover over the pipe end so that the area of escape is less
than the area of the pipe bore.

Referring to Figure 10.16, the area of escape is the surface area of a cylinder whose
diameter is the same as that of the pipe and whose height is the ‘Lift’. In other words,
the area of escape is that of a rectangle whose length is the circumference of the pipe
(π · d) and width is the lift.
254 • Mathematics

Lift

 Figure 10.16

The maximum effective lift will be when this area of escape is equal to the area of the
bore. (In other words, when the amount of water flowing in the pipe has the same area
through which to escape.)

Therefore, Area of escape = Area of bore,

π · d2
(π × d) × lift =
4
d
lift =
4

So the maximum effective lift is one-quarter of the valve diameter. See following
example for practical use.

If the lift is more than this, no more liquid can flow through than that which is allowed
by the area of the bore of the seat, but if the lift is less than one-quarter of the diameter,
the area of escape is less than the area of the bore, and the quantity of liquid escaping
depends upon the area p × d×lift.

Example

Calculate the quantity of water flowing, in litre/minute through a valve 100 mm diame-
ter when the lift is 15 mm and the velocity of the water is 3 m/s, assuming that the wings
of the valve take up one-sixth of the circumference.
Volume – Mass, Centre of Gravity, Moment • 255

One-sixth of the area is obstructed by the wings, this leaves five-sixths of the area for
the water to flow through.

Working in metres:

5
Area of escape between valve and pipe = × (π × 0. 1) × 0. 015 m2
6
5
Volume of water = × π × 0. 1 × 0. 015 × 3 m3 /s
6
As 1 m3 = 1000 litres,
5
the volume of water = × π × 0. 1 × 0. 015 × 3 × 1000 × 60
6
= 706. 86 litre/min.

Centres of gravity by first moments

When an object is subjected to a force which acts in line with its centre of mass there
is no turning effect produced. However, if the force acts along any other line, the force
will have a turning effect.

The greater the perpendicular distance between the line of action of the force and the
centre of mass or pivot, the larger the turning effect (see Figure 10.17).

The turning effect is called the turning moment of a force and can be found by:

Moment = Force × Perpendicular distance from force to pivot.

The unit of measurement of a moment is the newton metre (Nm)

Perpendicular
distance between
force and pivot d

Pivot

 Figure 10.17
256 • Mathematics

The moment of a force F about the pivot is F × d.

Turning moments act in either a clockwise or anticlockwise direction. If an object is bal-


anced then the sum of all the clockwise moments equals the sum of all the anticlockwise
moments.

Example

Find the missing force X which is needed to hold the following balanced beam in
equilibrium (Figure 10.18). Find also the total force on the pivot, R.

4 kN 8 kN X

2m 2m R
3m

Space diagram

 Figure 10.18

Solution
For equilibrium:

Clockwise turning moments about R = Anticlockwise turning moments about R

(X × 3) = (4 × 4) + (8 × 2)
3X = 8 + 16
3X = 24
So X = 8 kN

For equilibrium:

Total upward force on beam = Total downward force on beam

So R =4+8+X
R = 4 + 8 + 8 = 20 kN
Volume – Mass, Centre of Gravity, Moment • 257

Centre of gravity of compound shapes

Consider a piece of plate cut to shape shown in Figure 10.19.

Imagine this plate supported horizontally on one single support: call this support the
fulcrum. The fulcrum (F) must be positioned exactly at the centre of gravity of the plate if
the plate is to be perfectly balanced because ’the centre of gravity is that position through
which the whole weight can be considered as acting’.

Now take moments about the end O: in other words, imagine the plate is temporarily
hinged at this end. For perfect equilibrium the moments of all the forces tending to
turn the plate clockwise around the hinge must be equal to the moments of the forces
tending to turn the plate anticlockwise about the hinge.

Let W1 , W2 and W3 represent the weights of the top, centre and bottom parts.

Let X1 , X2 and X3 represent the distances of the centres of gravity of these parts from 00.

Let X represent the position of the fulcrum (F) from 00, and as previously stated this is
the centre of gravity of the whole plate.

Moments about 00:

Clockwise moments = Anticlockwise moments

(W1 × X1 ) + (W2 × X2 ) + (W3 × X3 ) = F × X

Since the total upward force Fmust be equal to the total downward force,

then F = W1 + W2 + W3 .

Therefore

(W1 × X1 ) + (W2 × X2 ) + (W3 × X3 ) = (W1 + W2 + W3 ) × X


(W1 × X1 ) + (W2 × X2 ) + (W3 × X3 )
Therefore X =
(W1 + W2 + W3 )

Moments of weights
In other words : X=
Weights

As previously explained, weight = mass × g, and since g is constant it can be cancelled


from numerator and denominator to give:
Moments of masses
X=
Masses
258 • Mathematics

Similarly, if the same material is used throughout the shape, then Mass = Volume ×
Density. Therefore the density can be cancelled out giving:

Moments of volumes
X=
Volumes

Furthermore, if the plate is of uniform thickness, Volume = Area × Thickness, and


therefore the thickness can be cancelled giving:

Moments of areas
X=
Areas

So for convenience take moments of masses, moments of volumes or moments of areas.


The above expressions provide methods of finding the position of the centre of gravity of
objects and figures made up from regular shapes.

Moments as used above are usually referred to as ’first moments’ to distinguish them
from ‘second moments’ which are used in applied mechanics.

120

20

80

100 150
CG

Y
30

160

 Figure 10.19

Example: To find the position of the centre of gravity of the plate illustrated in
Figure 10.19 which is symmetrical and of uniform thickness throughout, the dimensions
shown being all in mm.

Working in centimetre:

Area of top flange = 12 × 2 = 24 cm2


Volume – Mass, Centre of Gravity, Moment • 259

Area of centre web = 10 × 8 = 80 cm2

Area of bottom flange = 16 × 3 = 48 cm2

Distance of centre of gravity of top flange from base = 14 cm

Distance of centre of gravity of centre web from base = 8 cm

Distance of centre of gravity of bottom flange from base = 1. 5 cm

Taking moments about base,


Moments of areas
Y=
Areas
(24 × 14) + (80 × 8) + (48 × 1. 5)
=
24 + 80 + 48
336 + 640 + 72
=
152
1048
Therefore Y = = 6. 89 cm above the base.
152
Example

A hole 30 mm diameter is bored through a solid disc 90 mm diameter, the centre of the
hole being 25 mm from the centre of the disc (Figure 10.20).

Find the position of the centre of gravity of the disc after the hole has been cut out.

90 mm

25 mm
30 mm

20 mm

 Figure 10.20

Moments about base, working in centimetre:


Moments of areas
y=
Areas
260 • Mathematics

π · × 92 π · × 32
× 4. 5 − ×2
4 4 π
= cancel by
π · × 92 π · × 32 4

4 4
 2   3 
9 × 4. 5 − 3 × 2
=
92 − 32
346. 5
=
72
= 4. 8125 cm

Therefore centre of mass is 48.125 mm from the base, or 3.125 mm above the disc
centre.

(Alternatively, moments can be taken about centre of disc.)

Note: In this example area is lost by boring the hole, therefore the summation of areas
is the net area obtained by subtracting the area of the hole from the area of the disc;
also, the summation of moments of areas is the difference between the moments of
areas of the disc and hole.

In each of the above two cases, it is obvious that the centre of gravity lies on the vertical
centre line because the figures are symmetrical, therefore it is sufficient to calculate the
position of the centre of gravity in one direction only.

For figures that are not symmetrical, it is necessary to express the position of the centre
of gravity in two directions at right angles to each other, say from the base and from
one side; this is done by taking moments about these two datum lines separately.

Irregular figures

Simpson’s rule can be employed to find the moment of an irregular area in a similar
manner to which it is applied in finding the area.

Moment of area using Simpson’s Rule

This is found in the longitudinal direction by multiplying each of the ordinates by its
distance from the axis and then using Simpson’s rule.
Volume – Mass, Centre of Gravity, Moment • 261

Example

The distances from the centre-line to the hull for a 240 m long ship were taken at inter-
vals of 30 m, the measurements starting from the aft of the vessel and numbered 0 to 8
inclusive.

The values were:

Position number 0 1 2 3 4 5 6 7 8
Half-width 0 12.2 15.8 16.0 16.0 15.9 13.9 9.8 0

Find the position of the centre of gravity of the vessel.

(Moments are taken from the amidships position.)

Solution

Half-width, A=w× Position from


Position w Multiplier multiplier midships, p M = A × p
0 (aft) 0 1 0 −4 0
1 12.2 4 48.8 −3 −146.4
2 15.8 2 31.6 −2 −63.2
3 16.0 4 64.0 −1 −64.0
4 (amid) 16.0 2 32.0 0
5 15.9 4 63.6 1 63.6
6 13.9 2 27.8 2 55.6
7 9.8 4 39.2 3 117.6
8 (fore) 0 1 0 4 0
Total 309.0 −36.8

240
Common interval h = = 30 m
8
h 30
Area = A × × 2 (for both sides) = 309. 0 × × 2 = 6180 m2
3 3
Excess moment = −36. 8 (aft)
262 • Mathematics

h
×M×h×2 M×h
Centre of gravity from amidships = 3 =
h A
×A×2
3
−36. 8 × 30
Therefore centre of gravity = = −3. 57 m (aft of amidships)
309. 0
Example

Take a right-angled triangle of 80 mm base and 48 mm height as illustrated in


Figure 10.21, to find the area and position of centroid from base by Simpson’s rule.

Taking a regular shape such as this allows a comparison to be made with results
calculated from formulae.

20

40
48 mm
60

80 mm

 Figure 10.21

Ordinate Multiplier Area = Distance of M =A×p


Ordinate × ordinate
Multiplier from base
0 1 4.8 0
0
2 4 3.6 28.8
8
4 2 2.4 19.2
8
6 4 24 1.2 28.8
8 1 0 0
8
Total 48 76.8

4. 8
Common interval h = = 1. 2 cm
4
Volume – Mass, Centre of Gravity, Moment • 263

48 × 1. 2
Area = = 19. 2 cm2
3
76. 8 × 1. 2
Moment of area about base = = 30. 72 cm3
3
Moment of area 30. 72
Distance of centroid from base = = = 1. 6 cm
Area 19. 2
From Figure 10.10 the position of the centroid is a distance of one-third of the height
from the base. As the height of the triangle is 4.8 cm the centroid will be 1.6 cm from
the base, a value which matches that obtained by Simpson’s rule.

Further, if this triangle is swept through one complete revolution about its base it will
sweep out a cone with dimensions 4.8 cm radius of base and 8 cm perpendicular height.
π × 4. 82 × 8
Therefore the volume of this cone = = 193. 02 cm3
3
By Theorem of Pappus:

Volume swept out = Area × Distance moved by the centroid

Therefore Volume = 19. 2 × 2π × 1. 6 = 193. 02 cm3

Shift of centre of gravity due to shift of loads

Consider a system composed of loads which weigh w1 , w2 and w3 as shown in


Figure 10.22, the centre of gravity of each being h1 , h2 and h3 respectively from the
base.

w3

d
w2 w2
w3

h2
w1 h3 w1
h1

 Figure 10.22
264 • Mathematics

Let y1 be the distance of the centre of gravity of the whole system from the base, then:

Moments of weights
y1 =
Weights
w1 h1 + w2 h2 + w3 h3
=
w1 + w2 + w3

If w3 is lifted into the position shown, through a height of d, the new centre of gravity
of the whole system from the base, represented by y2 , is:
w1 h1 + w2 h2 + w3 (h3 + d)
y2 =
w1 + w2 + w3
w1 h1 + w2 h2 + w3 h3 + w3 d
=
w1 + w2 + w3
The shift of the centre of gravity of the whole system in the direction measured from
the base is y2 − y1 which is:
w1 h1 + w2 h2 + w3 h3 + w3 d w1 h1 + w2 h2 + w3 h3
y2 − y1 = −
w1 + w2 + w3 w1 + w2 + w3
w3 d
=
w1 + w2 + w3
In words this is:
Weight shifted × Distance moved
Shift in centre of gravity =
Total weight

As shown previously, weight can be represented by mass, so


Mass shifted × distance moved
Shift in centre of gravity =
Total mass

Test Examples 10
1. An I-section steel girder of 150 mm overall depth has unequal flanges, the top
flange is 100 mm wide by 12 mm thick and the bottom flange is 140 mm wide
by 14 mm thick. The centre web is 10 mm thick. Considering the flanges as rectan-
gular in section by neglecting radii and fillets, calculate the mass in kg/m run if the
density of the material is 7.86 g/cm3 .
2. A hollow steel shaft, 400 mm outside diameter and 200 mm inside diameter, has
a coupling 75 mm thick and 760 mm diameter at each end, and the overall length
Volume – Mass, Centre of Gravity, Moment • 265

is 6 m. Neglecting fillets and coupling bolt holes, find the mass of the shaft in tons
taking the density of steel as 7. 86 × 103 kg/m3 .
3. A cylinder and sphere and base of a cone are all the same diameter, and the heights
of the cylinder and cone are each equal to the diameter of the sphere. Find the ratio
of the volumes of the cylinder and sphere relative to the volume of the cone.
4. A piece of flat steel plate, having a mass of 6.5 kg/m2 , is cut to the shape of a sector
of a circle of radius 180 mm and subtended angle at the centre 240◦ , and the sector
is rolled into a cone. Find (i) the mass of material used, (ii) the diameter of the base
of the cone, (iii) the perpendicular height of the cone, (iv) the capacity of the cone
in litres.
5. An object is constructed by brazing the base of a solid cone to the flat surface of
a solid hemisphere, the diameter of the base of the cone and the diameter of the
hemisphere both being 60 mm, and the perpendicular height of the cone 50 mm.
Find the mass of the object if the density of the materials is 8.4 g/cm3 .
6. A hollow lead sphere has a uniform thickness of 10 mm and its mass is 3 kg. Taking
the specific gravity of lead as 11.4, find its outside diameter.
7. A hole 24 mm diameter is bored centrally through a sphere 51 mm diameter. Cal-
culate the volume of the drilled sphere in cm3 and its mass if the density of the
material is 7. 86 × 103 kg/m3 .
8. A tapered hole is bored through a right circular cone, concentric with the axis of
the cone. The base diameter of the cone is 64 mm and the perpendicular height is
60 mm. The diameter of the hole at the base of the cone is 28 mm and the diameter
where it breaks through the surface of the cone is 16 mm. Calculate the volume
and mass of the remaining hollow frustum, taking the density of the material as
8.4 g/cm3 .
9. The lengths of the sides of the base of a regular hexagonal pyramid is 25 mm
and the perpendicular height is 60 mm. Find the volume in cm3 . If this pyramid
is cut through a plane parallel to its base at half the height, find the volume of the
remaining frustum.
10. A vessel in the form of a hollow cone with vertex downwards is partially filled
with water. The volume of the water is 200 cm3 and the depth of the water is
50 mm. Find the volume of water which must be added to increase the depth to
70 mm.
11. The diameter of the base of a hollow cone is 300 mm and its perpendicular height
is 500 mm. It is partly filled with water so that when resting on its base the depth
of the water is 250 mm. If the cone is inverted and balanced on its apex, what will
then be the depth of the water?
266 • Mathematics

12. The surface area of a solid sphere is 1.5 times the surface area of a smaller sphere,
and the difference in their volumes is 10 cm3 . Find the volume and diameter of the
smaller sphere.
13. The diameters of a barrel are 395 mm at each end, 477 mm at quarter and
three-quarter lengths, 500 mm at mid-length. The total length is 581 mm. Using
Simpson’s rule calculate the capacity of the barrel in litres.
14. A water trough has a regular isosceles triangular section, the angle at the bottom
being 80◦ . Calculate the volume flow of water along the trough, in m3 /h, when the
depth of the water in the trough is 180 mm and it is flowing at a velocity of 0.5 m/s.
15. Find the height of the centre of gravity of a frustum of a cone which is 80 mm
diameter at the base, 60 mm diameter at the top and 40 mm perpendicular height.
DIFFERENTIAL
11
CALCULUS
(DIFFERENTIATION)
The development and application of the differential calculus may be explained by the
relation between equations and their graphs.

The meaning of the gradient at any point on such a graph is of particular importance in
this context and it is therefore dealt with first.

The symbol Δ represents a change in a quantity.

So, for example,

Temperature change ΔT = T2 − T1

Change in time Δt = t2 − t1

Change in angle Δθ = θ2 − θ1

Change in distance Δx = x2 − x1

Change in velocity Δv = v2 − v1

The symbol δ means a small but measurable change in a quantity such as δT, δt,
δθ etc.
268 • Mathematics

Gradient of a Straight Line


It will be recalled, from previous work on graphs, that the gradient (or slope) of a line is
change in y
the between any two points on the line.
change in x

y B

δy Δy
δx

Δx
x

 Figure 11.1

For the line shown in Figure 11.1,


Change in y Δy
Gradient = = between A and B.
Change in x Δx
δy Δy
This would be the same for a small change. =
δx Δx
Δy δy
The ratio is the same as the ratio and the ratio is the gradient of the straight line.
Δx δx

Infinitesimally Small Changes ‘d’


The symbol d is used to denote a change that is infinitesimally small. In other words, the
points A and B are essentially the same point. On the graph the ratios are all the same.
The value is the same at any point on a straight-line graph.
dy δy Δy
= =
dx δx Δx
This ratio holds true even when the changes approach zero.
Differential Calculus (Differentiation) • 269

The following examples are given by way of revision, to show how the gradient of a line
is calculated.

Example

Plot the graph of the equation y = 3x between the limits x = 0 and x = 4 and find the
gradient of the graph.

15 y

10

Change in y = 12
5

Change in x = 4 x
1 2 3 4

 Figure 11.2

Referring to Figure 11.2,

Change in y
Gradient =
Change in x
12
=
4
=3

Note: The change in y is three times the change in x.

Thus the gradient indicates the rate of change of one quantity, with respect to another
related quantity.

Example

Plot the graph of the equation y = 2 − 0. 25x between the limits x = 0 and x = 8. Find
the gradient of this graph.

The graph is shown in Figure 11.3.


270 • Mathematics

2.5 y

2
Change in y = 2

1.5

0.5

Change in x = 8 x
1 2 3 4 5 6 7 8

 Figure 11.3

Change in y
Gradient =
Change in x
−2
=
8
= −0. 25

(Note that the value of y changes from 2 to zero, that is, the change in y is −2.)

Two important points are noted from the previous examples:

(i) If y increases as x increases the gradient is positive


(ii) If y decreases as x increases the gradient is negative.

Zero Gradient
The gradient of a horizontal straight line is zero, since the change in y is zero for any
given change in the value of x (Figure 11.4).
y
y=c
c

 Figure 11.4
Differential Calculus (Differentiation) • 271

Gradient of a Curve
The gradient of a curve is not constant, but is changing from point to point along the
length of the curve.

The gradient of a curve at any given point is the gradient of the tangent to the curve at
that point P.

atP
P
r ve
cu
o
n tt
nge
Ta
a c

 Figure 11.5

Referring to Figure 11.5,

Gradient of the curve at P = Gradient of the tangent at P

The gradient of a curve may be obtained by drawing the tangent and measuring the
gradient, or mathematically, by the differential calculus.

Differentiation from First Principles


From Figure 11.5 it was seen that the gradient of the tangent at P was required, in order
to determine the gradient of the curve at P.

Chord PA is now constructed, as shown (Figure 11.6).

δ is used, for convenience, to express mathematically the change in the value of y and x
between points P and A.

Thus, δx, pronounced ‘delta ex’, means a small change in the value of x.
272 • Mathematics

δy
P
δx
P
at
e
rv
cu
t to
en
ng
Ta

 Figure 11.6

Similarly, δy means the corresponding change in the value of y. From Figure 11.6,
δy
Gradient of chord PA =
δx
The gradient of this chord is obviously quite different to the gradient of the tangent to
the curve.

However, if point A is now allowed to move along the curve towards point P, it is seen
(Figure 11.6) that the slope of the chord more and more resembles the slope of the
tangent as A gets closer to P (Figure 11.7).

P δy

δx
P
at
e
rv
cu
to
e nt
ng
Ta

 Figure 11.7

In fact, when point A gets very close to point P, the slope of the chord and the tangent
are virtually the same.

As A gets very close to P, so length δx is becoming very small, until it finally approaches
zero value.

That is, δx approaches zero value (denoted by δx → 0).


Differential Calculus (Differentiation) • 273

From the above reasoning, it follows that, when δx has become infinitely small, the ratio
δy/δx has reached a special value, or limit, where it represents the gradient of the curve
at point P.
δy
In order to indicate that this is a special value of it is identified by changing the
δx
δy dy
notation from to . This relationship is expressed mathematically as:
δx dx
δy dy
lim it =
δx→0 δx dx
dy
The term is called the differential coefficient of y with respect to x and the process of
dx
dy
finding is called differentiation.
dx
Example

From first principles, derive an expression for the gradient of the curve y = x 2 at any
point on the curve.

The sketch of the graph of y = x 2 is shown in Figure 11.8.

Point P(x, y) represents any point on the curve.

A
(x + δx, y + δy)

(x, y) x

 Figure 11.8

Chord PA is drawn as shown.

The co-ordinates for point P are (x, y).

Since A is very close to P, the co-ordinates for point A are (x + δx, y + δy)

Now, the equation of this curve is y = x2 (i)


274 • Mathematics

Therefore, at point P, y = x 2 and, at point A, y + δy = (x + δx)2

So y + δy = x 2 + 2x · δx + (δx)2 (ii)

Subtracting equation (i) from equation (ii) gives

(y + δy) − y = x 2 + 2x · δx + (δx)2 − x 2

Therefore δy = 2x · δx + (δx)2 (iii)

δy δy 2x · δx + (δx)2
Therefore the gradient is therefore =
δx δx δx
2x · δx + (δx)2 2x · δx (δx)2
But = + = 2x + δx
δx δx δx
If point A now moves very close to point P, so length δx will approach zero value.
δy
So lim it = 2x
δx→0 δx

dy
Therefore = 2x
dx
Hence, at any point on the curve y = x 2 the gradient of the curve is 2x.

Example

Calculate the gradient of the curve y = x 2 at the points x = 3 and x = −2


dy
From the previous example, any point on the curve has gradient = 2x
dx
When x = 3, gradient = 2 × 3 = 6

When x = −2, gradient = 2 × −2 = −4

These results can be confirmed by drawing tangents to the curve y = x 2 at x = 3 and


x = −2

The gradient of each of these tangents, when measured, should be reasonably close to
the calculated values.

Example

From first principles, find the differential coefficient of y, with respect to x, for the
equation y = 3x 3 .
Differential Calculus (Differentiation) • 275

(The differential coefficient or gradient may be calculated without actually drawing a


graph, although a sketch of the graph is often useful when solving problems.)
 
y = 3x 3 , therefore (y + δy) = 3 (x + δx)3 = 3 x 3 + 3x 2 (δx) + 3x · (δx)2 + (δx)3

So (y + δy) = 3x 3 + 9x 2 (δx) + 9x · (δx)2 + 3 (δx)3

It follows that (y + δy) − y = 3x 3 + 9x 2 (δx) + 9x · (δx)2 + 3 (δx)3 − 3x 3

So δy = 9x 2 (δx) + 9x · (δx)2 + 3 (δx)3

δy 9x 2 (δx) + 9x · (δx)2 + 3 (δx)3


and = = 9x 2 + 9x · (δx) + 3 (δx)2
δx δx
δy
Therefore lim it = 9x 2
δx→0 δx
So for the curve y = 3x 3 the differential coefficient is 9x 2

General Rule for Differentiating


a Power
From the previous section, the following results were obtained:
dy dy
For the curve y = x 2 , = 2x and for the curve y = 3x 3 , = 3x 2
dx dx
From the form of these results (which is confirmed by solving further examples), the
following general rule is noted:

dy
• when y = a · x n the differential coefficient − n · a · x n−1
dx
dy
So, if y = 9x 4 (i.e. a = 9 and n = 4), = 4 × 9x 4−1 = 36x 3
dx
This is true for any value of n.

Example

Differentiate the following equations with respect to x:

(a) y = x8 (b) y = 2x −3 (c) y = −7x −4


1 x3
(d) y=x (e) y = (f ) y =
x 12
276 • Mathematics

Answers
dy
(a) y = x8 = 8x 7
dx
dy 6
(b) y = 2x −3 = −3 × 2x −3−1 = −6x −4 = − 4
dx x
dy 28
(c) y = −7x −4 = −4 × −7x −4−1 = 28x −5 = 5
dx x
dy
(d) y = x or y = x 1 = 1 · x 1−1 = 1 · x 0 = 1
dx
1 dy 1
(e) y= = x −1 = −1 × x −1−1 = −1x −2 = − 2
x dx x
x3 dy x 3−1 3x 2 x 2
(f ) y= =3· = =
12 dx 12 12 4

Differential Coefficient of a
Constant
As shown previously, when y is a constant, the resulting graph has zero gradient (see
Figure 11.4).
dy
That is, if y = a (where a is any constant), = 0.
dx

Differential Coefficient of a Sum of


Terms
The differential coefficient of a sum of terms is obtained by differentiating each term
separately.

Example
dy
If y = x 3 − 2x 2 + 4x − 9, then = 3x 2 − 4x + 4
dx
Differential Calculus (Differentiation) • 277

Example
x3 3
Differentiate the equation y = − 2 + 3x with respect to x.
3 x
x3 3 x3
Rearrange the equation: y = − 2 + 3x = − 3x −2 + 3x
3 x 3
dy = x 2 3x 2 6
Therefore = − (−2) × 3x −2−1 + 3 = x 2 + 6x −3 + 3 = x 2 + 3 + 3
dx 3 x
Example

Differentiate the equation s = 3t2 − 20t + 40 with respect to t.

(Note: Symbols s and t represent two related quantities, as do x and y.)


ds
s = 3t2 − 20t + 40 so = 6t − 20
dt

Second Differential Coefficient


In some cases, having differentiated an expression once, it is necessary to differentiate
a second time.
d2 y
The notation is used to denote the second differentiation of an expression.
dx 2
Example

Consider the equation y = x 4


dy
Differentiating once: = 4x 3
dx
d2 y
Differentiating twice: = 12x 2
dx 2

Example

Determine the second differential coefficient of the following equation:

y = x 3 − 4x 2 + 3x − 7
dy
= 3x 2 − 8x + 3
dx
d2 y
= 6x − 8
dx 2
278 • Mathematics

Distance, Velocity and Acceleration


For a moving body:
Distance travelled
Average velocity =
Time taken

Using symbol v for velocity, s for distance and t for time, this equation may be expressed
δs
in calculus notation: average velocity = .
δt
If the velocity at a given instant (the instantaneous velocity) is required, the time interval
δt must be very small, that is, δt must approach zero value.
δs ds
Thus: lim it =
δt→0 δt dt
Hence, at any given time t, the instantaneous velocity of a moving body may be
ds
expressed as: v =
dt
Change in velocity
Similarly: Average acceleration =
Time taken
By the same reasoning as that above, the instantaneous acceleration a is given by the
dv
expression: a = .
dt
Example

The distance s moved by a body in time t is given by the formula s = t3 − 3t2 .

Find expressions for the velocity and acceleration of the body at any instant.

s = t3 − 3t2
ds
v= = 3t2 − 6t
dt
dv
a= = 6t
dt
Note: The expression for the acceleration of the body is seen to be the second differen-
tial equation of the original equation relating distance s to time t.
ds d2 s
s = t3 − 3t2 = 3t2 − 6t = 6t
dt dt2

Example

A body moves s metres in t seconds according to the relationship s = t3 − 7t2 − 3.


Differential Calculus (Differentiation) • 279

(a) Derive expressions for the velocity and acceleration of the body at any instant.
(b) Use these expressions to find the velocity and acceleration of the body after 5 s.

Answer

(a) s = t3 − 7t2 − 3 so v = 3t2 − 14t and a = 6t − 14


(b) When t = 5, v = 3 (5)2 − 14 × 5 = 5 m/s and a = 6 × 5 − 14 = 16 m/s2

Example

The distance (displacement) s moved by a body in time t is given by the expression:

s = 40t − 5t2

Calculate (a) the velocity after 2 s; (b) the time taken for the body to come to rest.

(a) s = 40t − 5t2 so v = 40 − 10t


When t = 2 s, v = 40 − 20 = 20 m/s
(b) When the body comes to rest, v = 0
40 40
Therefore 40 − 0t = 0 so t = =4s ∴t=
10 10

Maxima and Minima


An important application of differential calculus involves functions which have maxi-
mum or minimum values.

The graph shown below (Figure 11.9) represents a function which has a local maximum
value of y at point A and a local minimum value of y at B.

Such points are called turning points and have zero gradient, that is, at a turning point
dy
= 0.
dx
At a maximum value the gradient changes from positive to negative, so the gradient is
decreasing. Therefore the rate of change of the gradient is negative.
 
dy
d
d (gradient) dx d2 y
The rate of change of the gradient is given by = = 2.
dx dx dx
280 • Mathematics

dy
=0

ient
dx

g ra d
Ne
ga

itive
tiv
e

Po s
gr
ad
ie
nt

nt
radie
ive g

dy
=0
Posit

dx

 Figure 11.9

d2 y
So if < 0, the turning point is a maximum.
dx 2
d2 y
It follows that if > 0, the turning point is a minimum.
dx 2
d2 y
The case where = 0 will be looked at later in the text.
dx 2
Example

The curve y = 3x − x 2 + 1 has a turning point.

(a) Find the value of x at which this occurs.


(b) Determine whether the turning point is a maximum or minimum value.
(c) Find the value of y at the turning point.

Answer
dy
(a) y = 3x − x 2 + 1 so = 3 − 2x
dx
dy
At a turning point =0
dx
Therefore 3 − 2x = 0 giving x = 1. 5
d2 y
(b) = −2 and so the turning point is a maximum.
dx 2
(c) To find the maximum value of y, substitute x = 1. 5 in the original equation:
Differential Calculus (Differentiation) • 281

d2 y d2 y
Returning to the value of : if = 0 the curve has a point of inflexion. The gradient
dx 2 dx 2
does not change from negative to positive, or vice-versa, so this point is stationary.

A point of inflexion resembles the following diagram (Figure 11.10).

 Figure 11.10

Example

(a) Determine the values of x which give maximum or minimum values of y in the
equation
y = x 3 − 6x 2 + 9x
(b) Calculate these maximum or minimum values of y.

Answer
dy d2 y
(a) = 3x 2 − 12x + 9 and = 6x − 12
dx dx 2
dy
For maximum or minimum values =0
dx
Therefore 3x 2 − 12x + 9 = 0 or x 2 − 4x + 3 = 0
This factorises to give (x − 3)(x − 1) = 0
Therefore the turning points occur when x = 1 and x = 3.
d2 y
(b) When x = 1: = (6 × 1) − 12 = −6 :
dx 2
When x = 1: y = (1)3 − 6 (1)2 + 9 × 1 = 4: therefore ‘maximum’ at (1, 4)
d2 y
When x = 3: = (6 × 3) − 12 = 6:
dx 2
When x = 3: y = (3)3 − 6 (3)2 + 9 × 3 = 0: therefore ‘minimum’ at (3, 0)

Example

A rectangular sheet of metal measures 16 cm by 10 cm.


282 • Mathematics

From each corner a square of side x cm is removed, and the flaps formed are bent up to
make an open box.

Find its maximum volume.

16 cm

x cm

x cm

10 cm

x cm

(16 – 2x) cm

(10 – 2x) cm

Solution

The volume of the ‘box’ is length × width × height

Therefore volume = (16 − 2x) (10 − 2x) x = 160x − 52x 2 + 4x 3 cm3


dV
So = 160 − 104x + 12x 2 = 0 at a turning point
dx
Cancelling by 4 gives 3x 2 − 26x + 40 = 0
20
Solving, using the quadratic formula or by factorising, gives and 2.
3
Differential Calculus (Differentiation) • 283

20 10
The first value can be rejected since 10 − 2 × =− cm and, obviously a length
3 3
cannot be a negative value.

This gives the solution as x = 2.

The dimensions of the box are therefore:


Length = 16 − 2 × 2 = 12 cm
Width = 10 − 2 × 2 = 6 cm
Height = 2 cm
giving a volume of 12 × 6 × 2 = 144 cm3
d2 V
Now = −104 + 24x = −104 + 24 × 2 = −56 when x = 2, making the turning point
dx 2
a maximum.

Questions

1. 80 metres of fencing is used to enclose a rectangular area, using a straight wall for
one side. Find the maximum area that can be enclosed.
2. Panels of length 2 m are used to enclose a rectangular area 512 m2 . If one side of
the pen is a straight wall and needs no fencing, find the minimum number of panels
needed.
3. Sheet metal is used for making an oil-tank, open at the top, on a square base. If the
volume of the tank is to be 4 m3 , find the least area of sheet metal required.

Solutions

1. A = (80 − 2x) · x = 80x − 2x 2


dA
= 80 − 4x = 0 at turning point
dx

d2 A
Therefore 4x = 80 and so x = 20 2 = −4 and since this is obviously negative, the
dx
turning point is a maximum.

x x

80 – 2x

The maximum area is A = (80 − 2 × 20) 20 = 40 × 20 = 800 m2


284 • Mathematics

2. Let the total length of panels be L


Then L = 2x + y
512
But x · y = 512 and so y =
x

x 512 m2 x

Therefore L = 2x + 512x −1

dL
= 2 − 512x −2 = 0 at a turning point
dx
512
= 2 so 512 = 2x 2 giving x 2 = 256
x2

So x = 16 m
d2 L
= 1024x −3 which is positive when x = 16 so this is a minimum value.
dx 2
512
If x = 16, y = = 32 making L = 64 m.
16
As each panel is 2 m wide the requirement is for 32 panels.
4
3. Volume gives x 2 · h = 4 so h = 2
x
4 16
Surface area A = 4xh + x 2 = 4x · 2 + x 2 = + x2
x x
dA
So A = 16x −1 + x 2 giving = −16x −2 + 2x
dx
16 16
So 2x − 2 = 0 giving 2x = 2 so 2x 3 = 16
x x
Therefore x 3 = 8 giving x = 2
4
When x = 2, h = 2 = 1
2
d2 A 32
2
= 32x −3 + 2 = 3 + 2 > 0 giving a minimum value for A.
dx 2
Therefore A = 4 × 2 × 1 + 22 = 8 + 4 = 12 m2 .
Differential Calculus (Differentiation) • 285

Differentiation of sin(x) and cos(x)


For any angle x and any value n:
dy
If y = sin (x) , then = cos (x)
dx
dy
If y = sin (nx) , then = n · cos (nx)
dx
dy
If y = cos (x) , then = − sin (x)
dx
dy
If y = cos (nx) , then = −n · sin (nx)
dx

Note: In calculus, angles are always measured in radians.

These differential coefficients can be demonstrated by reference to the sketch of the


graphs of sin(x) and cos(x) (Figure 11.11).

Gradient = 0
y = cos(x)
y = sin(x)

Gradient = –1

π 2π

 Figure 11.11

For example,
π
when x = rad, the gradient of the sine curve is zero:
2
π 
this gives the point , 0 on the curve of the differential.
2
286 • Mathematics

when x = π rad, the gradient of the sine curve is −1:

this gives the point (π, −1) on the curve of the differential.

If this procedure is continued for different values of x on the sine curve, the points
dy
plotted form the cosine curve, showing that if y = sin (x), then = cos (x).
dx
Similarly, the gradient at any point on the cosine curve is numerically equal to the
negative of the corresponding value of the sine curve.

Example

Differentiate the following expressions with respect to x:

(a) y = sin(3x)
(b) y = 2 sin(5x)
(c) y = 4 sin(x) − 3 sin(2x)
(d) y = 4 cos(2x)
(e) y = 6 sin(4x) + 5 cos(3x)

Answers
dy
(a) = 3 cos (3x)
dx
dy
(b) = 10 cos (5x)
dx
dy
(c) = 4 cos (x) − 6 cos (2x)
dx
dy
(d) = −8 sin (2x)
dx
dy
(e) = 24 cos (4x) − 15 sin(3x)
dx

Differentiation of ln(x) and ex

dy
if y = ex , then = ex
dx
dy
if y = enx , then = n · enx
dx
Differential Calculus (Differentiation) • 287

dy 1
if y = ln (x) , then =
dx x
dy 1
if y = ln (nx) , then =
dx x

dy
Note carefully the last statement: the value of n does not appear in for y = ln(nx) but
dx
it does occur in all other cases.

This can be explained as follows.

Using the laws of logarithms: ln (nx) = ln (n) + ln (x)


d d d
Therefore: [ln (nx)] = [ln (n)] + [ln (n)]
dx dx dx
d
but ln(n) is a constant and so [ln (n)] = 0
dx
d d 1 1
Therefore [ln (n)] + [ln (n)] = 0 + =
dx dx x x
Naperian logarithms (In) have special applications, for example, when finding the work
done by an expanding gas.

Example

Obtain the second differential coefficient of the equations y = ln (x) and y = ex .

Answer
dy 1
If y = ln (x), then = = x −1
dx x
d2 y 1
So = (−1) × x −1−1 = −x −2 = − 2
dx 2 x
dy
if y = ex , then = ex
dx
d2 y
so 2 = ex
dx

Function Notation
In an equation such as y = 2x or y = 5x 3 + 3x, the value of y obviously depends upon
the value chosen for x. Hence, y is said to be a function of x and the general expression
288 • Mathematics

for such a relationship is:


y = f (x)

Functional notation may also be used to indicate the differentiation process, using the
symbols shown below:
y = f (x)
dy
= f  (x)
dx
d2 y
= f  (x)
dx 2

Example

If y = 5x 3 + 3x then

f (x) = 5x 3 + 3x
f  (x) = 15x 2 + 3
f  (x) = 30x

Text Examples 11
1. Differentiate the following equation with respect to x:
(a) y = x 3 + 3x 2 − 9x + 4
2x 3 7
(b) y = − 2 +x
√ 3 x
5
(c) y = x 3 + 1
(d) y = 3 cos (x) + 2 sin(x)
2. Calculate the gradient of the curve y = x 2 + 3x − 7 when x = 3 and x = −2.
3. The displacement s metres of a body from a fixed point is given by the equation
s = 20t − 5t2 + 4 where t is the time in seconds. Find:
(a) The velocity after 2 s.
(b) The displacement when the velocity is zero.
(c) The acceleration.
4. Determine the gradient of a tangent to any point in the curve
1
y = 4x +
x
Differential Calculus (Differentiation) • 289

Show that there are two points where the gradient is zero.
5. Find the co-ordinates of the point on the graph of y = 3x 2 − x + 2 where the
gradient is equal to −7.
6. The angle θ radian through which a shaft has turned after time t seconds is given
by
t2
θ = 2 + 16t −
2
Find
 the angular velocity after2 s and the time for the shaft to come to rest.

Note: angular velocity ω =
dt
7. (a) Determine the second differential coefficient of the expression y = x 3 + 3x +
ln (x) with respect to x.
(b) Determine the second differential coefficient of the expression y = 3 cos (θ ) −
7 sin (θ ) + θ with respect to θ
8. Determine the maximum value of y in the equation y = 12x + 3x 2 − 2x 3 .
9. For a beam of length l, the bending moment M at any distance x from one end is
given by:
ωLx ωx 2
M= −
2 2
where ω is the uniform load per unit length.
Show that bending moment is a maximum at the centre of the beam.
10. Differentiate the
 following equations:
1 4
(a) t = 2. 1 × 3 2 − √
θ 5
θ
a · un − 1
(b) z =
c
(c) y = 3x · x 2 − 4
(d) y = 2 cos (θ ) + 5
x3
11. Determine the minimum value of f (x) for the equation f (x) = − 2x 2 + 3x + 1.
3
12. Power (P) and Voltage (V) of a lamp are related by P = a · V b where a and b are
constants. Find an expression for:
(a) the rate of change of power with voltage and
(b) power per volt at 100 volts, when a = 0. 5 × 10−10 and b = 6.
13. A line of length L is to be cut up into four parts and put together as a rectangle.
Show that the area of the rectangle will be a maximum if each of its sides is equal
to one quarter of L (i.e. a square).
290 • Mathematics

14. Determine the second differential coefficient of:


(a) f (θ ) = cos (θ ) − ln(θ )
(b) f (t) = a · t2 + 2 · ln(t)
(c) f (x) = 5ex
15. A body moves so that its displacement (distance) x metres, which it travels from a
certain point O, is given by: x = 0. 2t2 + 10. 4 where t is the time in seconds.
Find the velocity and acceleration
(a) 5 s after the body begins to move and
(b) when the displacement is 100 m.
16. Angular displacement (θ rad) from rest of a revolving wheel is given by:
θ = 2. 1 − 3. 3t + 4. 8t2 where t is the time in seconds.
Find the angular velocity and angular acceleration after 1.5 s.
17. Verify that the equation f (x) = x 5 − 5x has a maximum and a minimum value and
determine the value of x and f (x) at these points.
dy
18. The gradient function of y = a · x 2 + bx + c is = 4x + 2.
dx
The function has a minimum value of 1. Find the values of a, b and c.
INTEGRAL
12
CALCULUS
(INTEGRATION)
Integration may be considered as reversing the process of differentiation.

That is, given the differential coefficient of a function, we are required to find the original
function.

The symbol is used to denote the integration process. This is the old-fashioned letter
’S’ and the reason for its use becomes evident when integration is used to find the ’sum’
of a number of quantities.

Constant of Integration
Consider the three equations below:

y = x 2 , y = x 2 + 3, y = x 2 + 7

dy
For each of these equations = 2x, which equation is the integral of 2x?
dx
Obviously, when reversing the differentiation process (i.e. integrating), provision must
be made for the possibility of a constant in the original equation.

This is achieved by adding a constant, C, called the constant of integration.


292 • Mathematics


Therefore 2x dx = x 2 + C

which may be interpreted as: (the integral of) 2x (with respect to x) = x 2 + C.

Every integral must be concluded with ‘d#’ where # is the letter corresponding to the
variable in the equation.

General Rule for Integration of


Powers of x
Consider the following differential equations:

x2 dy
If y = , =x
2 dx
x3 dy
If y = , = x2
3 dx
x4 dy
If y = , = x3
4 dx
From these equations, the following integrals are obtained:

x2
x dx = +C
2

x3
x 2 dx = +C
3

x4
x 3 dx = +C
4
From these cases it can be seen that the general rule is ‘add 1 to the power, then divide by
this new power’.

Expressing this mathematically, a general rule for integration is obtained, which


includes a constant of integration:

x n+1
x n dx = +C
n+1
Note: there is an important exception to this rule.
1
The integral of x −1 or cannot be integrated using this rule as the new power would
x
be 0, and division by 0 is impossible.
Integral Calculus (Integration) • 293

Definition

1
dx = ln (x) + C
x
Example

Evaluate the following integrals:


   −4 
x
(a) x 7 dx (b) 3x 5 dx (c) dx (d) 8 dx
3

Solutions

x8
(a) x 7 dx =+C
8

3x 6 x6
(b) 3x 5 dx = +C = +C
6 2
 −4
x x −3 x −3 1
(c) dx = +C =− +C =− 3 +C
3 3× (−3) 9 9x

(d) 8 dx = 8x + C

This last result may be confirmed by differentiating the answer, or by applying the
general rule, as shown:
  
    x1
8 dx = (8 × 1) dx = 8 × x 0 dx as x 0 = 1 = 8 × + C = 8x + C
1

Integration of a Sum of Terms


The integral of a sum of terms is equal to the sum of their separate integrals.

Example
   
 2 
3x + 7x − 10 dx = 3x 2 dx + 7x dx − 10dx

7x 2
= x3 + − 10x + C
2
Note: The constants which should be added to each separate term are combined as a
single constant.

Example

 
Evaluate the integral: x 2 − 3x 3 + 2 dx
294 • Mathematics


 2  x 3 3x 4
x − 3x 3 + 2 dx = − + 2x + C
3 4
Example

Integrate the expression 3t2 + t + 1 with respect to t.



 2  t2
3t + t + 1 dt = t3 + + t + C
2
Example
1
Integrate the expression 3x 2 − 6x + with respect to x.
2
  
2 1 x
3x − 6x + dx = x 3 − 3x 2 + + C
2 2
Questions

Work out the following:

  
   
1. x 5 dx 2. x 3 − 2x dx 3. x 3 + 6 dx
  
−3
 
4. x dx 5. 3x − 5x + 6 dx
2
6. (x + 1) (x + 2) dx
      
1 1+x x + x4
7. √ dx 8. dx 9. dx
x x4 x3
      
1 + y3   1
10. dy 11. y4 − 2y3 − 9 dy 12. y 1+ dy
y2 y

Answers
x6 x4 x4
1. +c 2. − x2 + c 3. + 6x + c
6 4 4
x −2 5x 2 x 3 3x 2
4. − +c 5. x3 − + 6x + c 6. + + 2x + c
2 2 3 2

x 1/2 √ x −3 x −2 1 1
7. +c =2 x+c 8. − − +c=− 3 − 2 +c
1/2 3 2 3x 2x
x2 x2 1 y2 1
9. −x −1 + +c = − + c 10. − +c
2 2 x 2 y
x 5 y4 y2
11. − − 9y + c 12. +y+c
5 2 2
Integral Calculus (Integration) • 295

Evaluating the Constant of Integration


The value of the constant of integration, for a given function, can be calculated,
provided a corresponding pair of values of x and y are known.

Example

The gradient of a curve is 4x + 5.

If the curve passes through the point (x = 0, y = −4), find the equation of the curve.
dy
Gradient of curve = 4x + 5
dx
   
dy
Therefore y= dx = (4x + 5) dx =2x 2 + 5x + C (i)
dx

To find the constant of integration C, substitute x = 0, y = −4, in equation (i):

−4 = 2 × (0)2 + 5 × 0 + C

Therefore C = −4

The equation of the curve is y = 2x 2 + 5x − 4

Example

The curve of a graph has a gradient of 10x − x 2

If the curve passes through the point (3, 52), find the equation of the graph.
dy
Gradient of curve = 10x − x 2
dx
   
dy   x3
Therefore y= dx = 10x − x 2 dx = 5x 2 − + C (ii)
dx 3

To find the constant of integration C, substitute x = 3, y = 52, in equation (ii):

(3)3
52 = 5 × (3)2 − +C
3
Therefore 52 = 45 − 9 + C so C = 16
x3
The equation of the curve is y = 5x 2 − + 16
3
296 • Mathematics

Integration of sin(x), cos(x), ln(x) and ex

The following differential coefficients are known:


dy
if y = sin (x) , then = cos (x)
dx
dy
if y = cos (x) , then = − sin (x)
dx
dy 1
if y = ln (x) , then =
dx x
dy
if y = ex , then = ex
dx
Reversing the process, the following integrals are obtained:

cos(x)dx = sin(x) + C

sin (x) dx = − cos (x) + C
  
1
dx = ln (x) + C
x

ex dx = ex + C

When x is scaled by a value, for example, y = sin(3x) or in general, y = sin(nx) the


following integrals are obtained:

sin(nx)
cos(nx)dx = +C
n

cos (nx)
sin (nx) dx = − +C
n
  
1 ln (x)
dx = +C
nx n

enx
enx dx = +C
n

Example

Determine the integral [2 cos (x) − sin (x)] dx

[2 cos (x) − sin (x)] dx = 2 sin (x) − cos (x) + C = 2 sin (x) + cos (x) + C
Integral Calculus (Integration) • 297

Example

Determine the integral [cos (2θ) + 5 sin(3θ)] dθ with respect to θ .

sin (2θ) 5 cos (3θ )
[cos (2θ) + 5 sin (3θ)] dθ = − +C
2 3

Example
 
 5
Determine the integral dx
x
     
5 1
dx = 5 × dx = 5 ln (x) + C
x x

Definite Integration: Area Under


a Curve

Theory

In Figure 12.1, the curve represents the sketch of a function f (x). The area abc may be
divided into a large number of strips, or elements, one of which is shown.

Let A = area abc


δA = area of one strip

If δx is made very small, the effect of the curve at the top of the strip becomes negligible
and the strip can then be considered to be a rectangle of height y and width δx.

Therefore the area of the strip, δA = y · δx


δA
Dividing both sides by δx gives =y
δx
dA δA
Letting δx reduce to 0 gives = lim it =y
dx δx→0 δx
Integrating both sides of this expression with respect to x:
   
dA
dx = (y) dx
dx

Therefore A = y dx between the limits x = a and x = b.
298 • Mathematics

a b x
δx

 Figure 12.1

This is written using the notation:


b
A= y dx
a

To evaluate the integral:

• Integrate f (x) (the constant, C, can be omitted).


• The integral is inserted in square brackets, with upper and lower limits of x placed
after the second bracket with the lower limit at the bottom of the bracket and the
upper limit at the top.
• Evaluate this expression using the top value, b, and call it B.
• Evaluate this expression using the bottom value, a, and call it A.
• Work out B − A.
• C can be ignored because it would automatically cancel when B − A is calculated.

Example

Find the area between the curve y = x 2 , the x axis and the ordinates x = 2 and x = 4
(Figure 12.2).
4  4    
x3 43 23 64 8 56
x 2 dx = = − = − = = 18. 67
3 2 3 3 3 3 3
2

When an integral has limits, it is called a definite integral.


Integral Calculus (Integration) • 299

y = x2

2 4 x

 Figure 12.2

60

50

40

30

20

10

1 2 3 4 5 6

 Figure 12.3

Example

Find the area enclosed by the curve y = x 2 + 2x + 1, the x axis, and the ordinates x = 2
and x = 5 (Figure 12.3).
5
 
Area = x 2 + 2x + 1 dx
2
 5
x3
= + x2 + x
3 2
300 • Mathematics

   3 
53 2
Area = + 52 + 5 − + 22 + 2
3 3
   
125 8
= + 25 + 5 − +4+2
3 3
125 8
= + 30 − − 6
3 3
117
= + 24
3
= 63 units2

One important result from definite integration is that areas of a graph that appear below
the x axis have a negative value.

Questions

1. Find the area under the curve y = x + 3x 2 between x = 1 and x = 2.


1
2. Find the area under the curve y = 2 between x = 1 and x = 2.
x
3. Find the area under the curve y = 2x − x 2 between x = 0 and x = 2.

4. Find the area under the curve y = x between x = 1 and x = 4.

Answers
2  2    
 2
 x2 4 1 3 1
1. A = x + 3x dx = + x3 = +8 − + 1 = 10 − = 8
2 1 2 2 2 2
1
2  
2 1 1
2. A = x −2 dx = −x −1 1
= − − (−1) =
2 2
1
2  2  
  x3 23 8 4 1
3. A = 2x − x dx = x 2 −
2 2
= 2 − −0= 4− = = 1
3 0 3 3 3 3
0
4   
√ 2 3 4
2 3 14 2
4. A = x dx = x 2 = 42 −1 = =4
3 1 3 3 3
1

The following questions involve the calculation of areas but also require an application
of algebra to their solution.

Questions

1. The diagram below shows the graph of y = x 2 − 4


Integral Calculus (Integration) • 301

(a) Find the co-ordinates of P, Q and R.


(b) Find the area of the region enclosed by the curve and the x axis.

x
P Q

Answers
(a) At the points P and Q, y = 0
Therefore x 2 − 4 = 0 so x 2 = 4
Therefore x = ±2, giving P at (−2, 0) and Q at (2, 0)
At R, x = 0, so y = −2 giving R at (0, −2)
 3 1
1  2  x (1)3 (−1)3
(b) Area = x − 4 dx = − 4x = − 4 (1) − − 4 (−1)
−1 3 3 3
   −1 
1 1 22 22
Therefore area = − 4 − − + 4 = − , so actual area =
3 3 3 3
2. (a) Sketch the curve y = x(3 − x).
(b) Find the area of the region enclosed by the curve and the x axis.

Answers
(a) 3 y
2

1
x

–1 1 2 3
–1

–2

–3

3 3  3  
  3x 2 x 3 27 27 9
(b) Area = x (3 − x) dx = 3x − x 2
dx = − = − −0 =
2 3 0 2 3 2
0 0
302 • Mathematics

3. The curve y = 9 − x 2 and the line y = 2x + 6 intersect at P and Q.


(a) Find the co-ordinates of P and Q.
(b) Find the area of the region enclosed by the curve and the x axis.
y

x
P

Answers

(a) At P and Q 9 − x 2 = 2x + 6 so 0 = x 2 + 2x − 3 = (x + 3) (x − 1)
Therefore at P, x = −3, giving y = 0. So P(−3, 0)
At Q, x = 1, giving y = 8. So Q (1, 8)
1 1 1
   2 
(b) Area = 9 − x dx − (2x + 6) dx =
2
−x − 2x + 3 dx
−3 −3 −3
 3 1  
x 1 (−3)3
Area = − − x + 3x 2 = − −1+3 − − − (−3)2 + 3 (−3)
3 −3 3 3
 
5 22
Area = − (9 − 9 − 9) =
3 3
1
4. The diagram shows the curve y = √ and the line y = x.
x
The line and the curve intersect at M.
(a) Find the coordinates of M.
(b) Find the area of the shaded region.

x
0 4
Integral Calculus (Integration) • 303

Answers
1 √
(a) At M, √ = x so x · x = 1 giving x = 1
x
When x = 1, y = 1 so M (1, 1)
1 4  2 1  3/2 4
√ x 2x
(b) Area = x dx + x dx = +
2 0 3 1
0 1
     
1 2×8 2×1 1 14 31
Area = − (0) + − = + =
2 3 3 2 3 6
1
5. The diagram shows the curve y = 2x +
x2
There is a minimum point at M
(a) Find the coordinates of M.
(b) Find the area of the shaded region.

x
0 2

Answers
dy
(a) For a minimum value =0
dx
1
y = 2x + → y = 2x + x −2
x2
dy
Therefore = 2 − 2x −3 = 0 at a turning point
dx
2
2− =0
x3
2
2= 3
x
2x 3 = 2
304 • Mathematics

x3 = 1
√3
x= 1
x=1

1
When x = 1, y = (2 × 1) + =3
12
Therefore M is the point (1, 3)
2 
  2 1 2
(b) Area = 2x + x −2 dx = x 2 − x −1 1
= x2 −
x 1

1     
1 1 1 7
Area = 2 −
2 − 1 −
2 = 4− − (0) = = 3. 5
2 1 2 2
Definite integration of other functions is performed in exactly the same manner.

There is one critical point that must be remembered – any operations involving Calculus
and trigonometric functions must be performed using radian measure.

Example

Evaluate the definite integral sin (x) dx
0


sin (x) dx = [− cos (x)]π0 = (− cos (π)) − (− cos (0)) = (− (−1)) − (− (1)) = 1 + 1 = 2
0

Questions

Calculate the values of:

2  2 1 π/2

 x2 + x3 2 cos(x)
1. x3 + 4 dx 2. dx 3. dx 4. 3 sin(4φ)dφ
x 3
1 0 0 0

π/3
   π/2
 π   0.5
2t t
5. 2 sin dt 6. sin(2t)dt 7. cos dt 8. (1 + 0. 6 cos(0. 2t))dt
3 2
π/6 π/2 π/2 −0.2

π 1 2 1  
1
9. (sin(x) − sin(3x))dx 10. ex dx 11. e−2t dt 12. eθ/3 − θ/3 dθ
e
0 0 1 0.5

2   3 3 1
  4
13. 1 + 2e0.3x dx 14. 4 ex + e−x dx 15. 2e−0.4t dt 16. dx
5e1.4x
1 2 0 −1
Integral Calculus (Integration) • 305

Answers

1. 7.75 2. 4.67 3. 0.561 4. 0


5. 0.521 6. 0 7. 0.586 8. 1.12
9. 1.33 10. 1.72 11. 0.0585 12. 0.253
13. 4.15 14. 2.18 15. 0.811 16. 0.732

Integration as a Summation
In the previous section, it was shown that the area under a curve could be determined
by dividing the area into very narrow strips.

a b x
δx

 Figure 12.4

One such strip is shown in Figure 12.4. The width of the strip may be represented by
symbol dx if its width approaches 0 value.

Area of one strip = y × dx

The total area abc could be obtained by adding together all these small areas, such as
the one shown.

That is, area abc = the summation of all the areas such as y · dx between the limits x = a
and x = b.
306 • Mathematics

b
Therefore Area = y dx
a

Thus, the symbol may be interpreted as meaning the summation of the quantities to
which it is applied.

Volume of a solid of revolution

If a curve is rotated about the x axis as shown (Figures 12.5 and 12.6), the shape
generated is called a solid of revolution.

dx

 Figure 12.5

dx

 Figure 12.6
Integral Calculus (Integration) • 307

The area under the curve is divided into a large number of elemental strips, such as the
one shown (Figure 12.5).

By rotating this element about the x axis, a solid disc is generated (Figure 12.6).

Volume of one disc = Cross-sectional area × Thickness


= π · y2 · dx

The total volume is the summation of the volumes of all such discs between the limits
x = a and x = b.
b
Therefore Volume = πy2 dx
a

Since the constant factor π is not affected by the integration process, the equation may
be written as:
b
Volume of a solid of revolution = π y2 dx
a

Example

The curve y = x 2 is rotated about the x axis between the limits x = 1 and x = 4.
Determine the volume of the solid of revolution produced (Figure 12.7).

 Figure 12.7

Solution b 4 4
 2  4
V= π · y2 dx = π · x 2 dx = π x dx
a 1 1
 4
x5

5 1
308 • Mathematics

   
45 15
V =π − = 204. 8π = 642. 8 cubic units (1 decimal place)
5 5

Example

Calculate the volume generated by rotating the curve y = x 3 about the x axis between
x = 0 and x = 2.

Solution
b 2 2  7 2  
2
 2  6 x 27 128π
V= π · y dx = π · x 3 dx = π x dx = π =π = units3
7 0 7 7
a 0 0

So the volume of revolution is 57.4 units3 to 1 decimal place.

Example
π · r2 · h
Use integral calculus to prove that the volume of a cone is where r is the radius
3
of the base and h is the vertical height (Figure 12.8).

 Figure 12.8

r r
The gradient of the sloping line is and so it has equation y = x
h h
h  h h
r 2 r2 2 π · r2
Volume of revolution = π x dx = π x dx = x 2 dx
h h2 h2
0 0 0
 h   
π · r2 x 3 π · r2 h3 π · r2 h3 π · r2 h
Therefore volume = = · − (0) = 2 · =
h2 3 0 h2 3 h 3 3
Integral Calculus (Integration) • 309

Distance and velocity by integration

In the previous chapter, it was shown that the relation between distance s, time t and
velocity v could be expressed as:
ds
=v
dt
Integrating both sides of this equation with respect to t, gives
   
ds
dt = vdt
dt


Therefore s= vdt (i)

Similarly the relationship between acceleration, velocity and time was defined as
dv
=a
dt

so it follows that v= a dt (ii)

Example

The velocity v of a body after time t is given by the expression v = 10 − 2t.

If displacement (distance) s = 24 when t = 2, obtain an expression for distance s in terms


of t.

Solution
 
s= v dt = (10 − 2t) dt = 10t − t2 + C

but when t = 2, s = 24; therefore 24 = (10 × 2) − 22 + C

This gives 24 = 20 − 4 + C and so C = 8

Therefore s = 10t − t2 + 8

Example

The velocity v m/s after t seconds for a body is given by v = 4 + 7t.


310 • Mathematics

Find the distance travelled by the body in the interval from t = 0 to t = 5 s.


5  5  
7t2 7 × 52 175
s= (4 + 7t) dt = 4t + = (4 × 5) + − (0) = 20 + = 107. 5 m
2 0 2 2
0

Example

A body moves such that its acceleration, a m/s2 after time t seconds is given by
a = 18 − 2t.

(a) Derive an expression for the velocity v m/s of the body, given that v = 20 m/s when
t = 0.
(b) Use the expression to find the velocity of the body after 3 s.

Solution
 
(a) v = adt = (18 − 2t) dt = 18t − t2 + C
but when t = 0 v = 20; therefore 20 = 0 − 0 + C, so C = 20 and v = 18t − t2 + 20
(b) When t = 3, v = (18 × 3) − 32 + 20 = 54 − 9 + 20 = 65 m/s.

Test Examples 12
1. Evaluate the following integrals:

 4 
(a) x − x 2 − 8x + 5 dx
  
4 1
(b) + dx
x2 x
3
 
(c) 2x − 3x 2 − 1 dx
1
dy
2. A curve has a gradient of = x 2 + x − 2.
dx
If the curve passes through the point x = 2, y = 5, find its equation.
3. Determine the integrals:

(a) (3 cos (x) − 2 sin(x) + 4) dx

(b) (4 cos (x) − sin (x) + x) dx
Integral Calculus (Integration) • 311

π/2
(c) cos (x) dx
0

4. Find the area between the curve y = x 3 −4x 2 + 3x and the x axis between the limits
x = 1 and x = 3.
5. The curve shown below (Figure 12.9) was plotted during the isothermal expansion
of a gas, following the law pV = C, between volume
  V1 and V2 .
V2
Show that the area under the graph = pV · ln
V1

p
pV = C

V
V1 V2

 Figure 12.9

6. The acceleration a m/s2 of a body is given by the equation a = 6t.


(a) Obtain an expression for velocity v after t seconds given that v = 0 when t = 0.
(b) Calculate the average velocity during the period t = 2 to t = 3.
(c) Calculate the instantaneous velocity at t = 2. 5.

7. The velocity v m/s of a body after time t seconds is given by the equation

v = 3t2 + 8t + 12.

Find the displacement of s metres for the body after 10 s, given that s = 10 when
t = 0.
x
8. A cone is generated by rotating the line y = about the x axis from x = 0 to x = 6.
2
Calculate the volume of the cone.
9. Find the volume generated by rotating about the x axis that part of the curve
y = x 2 − x which lies between its intersections with the x axis.
312 • Mathematics

10. Determine the following integrals:



(a) (x + 1) (x + 2) dx
  
1 1
(b) − 2 dx
x x
2

(c) (a + 2b) (x + 1) dx
1

(d) 3ex dx

11. Evaluate the area enclosed by the curve f (x) = 0. 06x 2 + 10, the x axis and the
ordinates x = 6 and x = 8.
12. A gas is compressed from a volume of V1 to a volume of V2 according to the law
p · V n = C, where p is the pressure and C is a constant.
Show that the area under the compression curve on a pressure–volume diagram,
between the stated volume limits, is given by:

C 1 1
Area = − −
n−1 V2n−1 V1n−1

13. Determine the following integrals:


  
1 1
(a) sin (θ ) − dθ
3 2

(b) sin (x) dx
0
π/2
(c) (5 cos (x) + 3 sin (x) − x) dx
0

x2 y2
14. Find the volume of the ellipsoid formed by rotating the ellipse: + = 1, of
a2 b2
major axis 2a and minor axis 2b, about major axis.
15. A particle is projected witha horizontal
 velocity (u) into a resisting medium so that
dv
at time (t) its acceleration equals a constant (k) multiplied by its velocity (v).
v dt
Show that: ln = −kt
u
16. Sketch the curve and find an expression for the area between limits x = 0 and x = 1,
b
the curve y = and the x axis. Evaluate this area when a = 1 and b = 2.
(x + a)
Integral Calculus (Integration) • 313

17. Evaluate the following integrals:



3
(a) dx
x
 3
x
(b) dx
a

 3 
(c) 4z + 3z2 + 2z + 1 dz

(d) (2 cos (θ ) − 5 sin (θ )) dθ
18. A particle P starts from rest at O with a velocity of 5 m/s and moves along a straight
line OX with an acceleration of −2t2 at time t seconds after leaving O.
Describe the motion after 3 s.
13
ADVANCED
CALCULUS:
METHODS OF
DIFFERENTIATION
Differentiation – The Product Rule
When two separate functions of a variable are multiplied they can only be differentiated
if the product rule is applied.

The Product Rule


Let y = u · v where u and v are functions of the same variable, x, for instance.

Then to differentiate y the following rule applies:


dy dv du
=u· +v·
dx dx dx
or sometimes
dy
= u · dv + v · du
dx
Advanced Calculus: Methods of Differentiation • 315

Example (set all questions this way to aid memory)

Let y = x 3 · sin (x)

Solution
u = x3 v = sin(x)
dy
du dv so = x 3 · cos(x) + 3x 2 · sin(x)
= 3x2 = cos(x) dx
dx dx
If possible the answer should be simplified as far as possible.
dy
So = x 2 (x · cos(x) + 3 sin(x))
dx

Questions
Differentiate the following:

1. y = 3x 2 · cos(x) 2. y = sin(x) · cos(x) 3. y= x · cos(x)
   
4. y = 3x 2 + 2 · x 2 − 2 5. y = 5e2x · sin(x) 6. y = e7x · cos(x)

7. y = x 3 · ln(2x) 8. y = 3 x · e4x 9. y = ex · ln(x)
  √
10. y = e2x 4x 2 − 4x + 1 11. y = 3 x 7 · ln(5x) 12. y = 6e5x · sin(x)

Answers

dy
1. = 6x cos(x) − 3x 2 sin(x) = 3x (2 cos(x) − x sin(x))
dx
dy
2. = cos2 (x) − sin2 (x)
dx
dy 1 − 1 √ cos(x) − 2x sin(x)
3. = x 2 cos(x) − x sin(x) = √
dx 2 2 x
dy    
4. = (6x) x 2 − 2 + 3x 2 + 2 (2x) = 6x 3 − 12x + 6x 3 + 4x = 12x 3 − 8x
dx
dy
5. = 10e2x sin(x) + 5e2x cos(x) = 5e2x (2 sin(x) + cos(x))
dx
dy
6. = 7e7x cos(x) − e7x sin(x) = e7x (7 cos(x) − sin(x))
dx
316 • Mathematics

dy 1
7. = 3x 2 ln(2x) + x 3 · = x 2 (3 ln(2x) + 1)
dx x
dy √ 3 1 3e4x (8x + 1)
8. = 3 x4e4x + x − 2 · e4x = √
dx 2 2 x
 
dy 1 1
9. = ex ln(x) + ex · = ex ln(x) +
dx x x
dy    
10. = 2e2x 4x 2 − 4x + 1 + e2x (8x − 4) = 2e2x 4x 2 − 1
dx

The Quotient Rule


When two functions of the same variable are divided the quotient rule of differentiation
has to be applied. The quotient rule is:

du dv
u dy v · dx − u · dx dy v · du − u · dv
If y = then = or sometimes =
v dx v2 dx v2

Example
3x 3
Let y =
sin(x)

Solution
u = 3x 3 v = sin(x)

so
du dv
= 9x 2 = cos(x)
dx dx

dy sin(x) · 9x 2 − 3x 3 · cos(x)
Therefore =
dx sin2 (x)
dy 3x 2 · (3 sin(x) − x · cos(x))
Or, if possible, =
dx sin2 (x)
Advanced Calculus: Methods of Differentiation • 317

Questions
Differentiate the following:

4x 2x − 1 3e2x 3x 4 + 2x 2 − 1
1. y = 2. y = 3. y= 4. y =
x2 − 1 3x 2 + 5x 4x 2 − 3 4e5x

4 sin(x) 6 cos(x) x3 4e6x
5. y = 6. y = 3 7. y= 8. y =
5x 2 + 2x x +4 cos(x) sin(x)
√  
ln(6x) 3 x+x x2 3 ln 52 x
9. y = 10. y = 7 11. y= 12. y = 2
6 sin(x) 2 ln(4x)
3 sin(x) x + 2x

Answers
 2   
dy x − 1 4 − 4x(2x) −4x2 − 4 4 x2 + 1
1. =  2 = 2 = −  2
dx x2 − 1 x2 − 1 x2 − 1
 2 
dy 3x + 5x 2 − (2x − 1) (6x + 5) 6x2 + 10x − 12x2 − 4x + 5
2. =  2 =  2
dx 3x2 + 5x 3x2 + 5x

dy −6x2 + 6x + 5
So =  2
dx 3x2 + 5x
 2   
dy 4x − 3 6e2x − 3e2x (8x) 6e2x 4x2 − 4x − 3
3. =  2 =  2
dx 4x2 − 3 4x2 − 3
   
dy 4e5x 12x3 + 4x − 20e5x 3x4 + 2x2 + 1
4. =  2
dx 4e5x
 3   
dy 12x + 4x − 5 3x4 + 2x2 + 1 −15x4 + 12x3 − 10x2 + 4x − 5
= 5x
=
dx 4e 4e5x
 2 
dy 5x + 2x 4 cos(x) − 4(10x + 2) sin(x)
5. =  2
dx 5x2 + 2x
 3      
dy x + 4 (−6 sin(x)) − 6 cos(x) 3x2 dy −6 x3 + 4 sin(x) + 3x2 cos(x)
6. =  2 So =  2
dx x3 + 4 dx x3 + 4
√ 3 
dy x 2 cos(x) + x. sin(x) dy 4e6x (6 sin(x) − cos(x))
7. = 8. =
dx cos2 x dx sin2 x
 
3
dy 1
sin(x) − ln(6x) cos(x) dy √
2 x
+ 1 ln(4x) − √3 + 1
x
9. = x
10. =
dx 6 sin2 x dx 2 (ln(4x)
7 2

 5x 
dy 3x(2 sin(x) − x cos(x)) dy 3(x + 2) − 6(x + 1) ln
11. = 12. =  2
2
dx 9 sin2 (x) dx x2 + 2x
318 • Mathematics

The function of a function rule (chain rule)

The type of function defined as ‘a function of a function’ is probably best looked at with
an example.

The function y = (4x − 3)7 can be differentiated by first multiplying (4x − 3) by itself
seven times and then differentiating each term in turn. This is a long process and is,
more often than not, subject to error. In this type of function a substitution is made.

Example

Let u = 4x − 3, then instead of y = (4x − 3)7 the equation is y = u7 .

Solution
dy dy du
The rule for differentiating by substitution is = ×
dx du dx
In other words, differentiate the ‘substituted’ function, differentiate the substitution,
multiply these two answers and, finally, replace the substitution by the original expres-
sion.
dy du
From the above substitution, = 7u6 and = 4.
du dx
dy
Therefore, = 7u6 × 4 = 28u6 = 28 (4x − 3)6
dx

Questions
Differentiate

 5  
1. y = sin (6x + 1) 2. y = 3t 4 − 2t 3. y= 4x2 + x − 3

4. y = (x + 1)4 5. y = (1 − 2x)7 6. y = sin2 (x)

1  
7. y= 8. y = 6 cos3 (x) 9. y= 4x3 + 2x2 − 5x
(2x − 1)6
1 3
10. y= 11. y = cos7 (x) 12. y=  2 
2x2 − 3x + 1 3x − 1
 3 5
13. y = 2x2 + 3x + 1 2 14. y= sin (3x) 15. y = (cos (6x)) 3
 √ 
16. y = sin 3x2 17. y = cos (sin (2x)) 18. y = sin x

 
19. y = (sin (3x) + cos (2x))4 20. y= 3
x3 − 5x2 − 6x + 4
Advanced Calculus: Methods of Differentiation • 319

Answers

dy dy   4
1. = 6 cos(6x + 1) 2. = 5 · 12t3 − 2 3t4 − 2t
dx dt

dy 8x + 1 dy
3. = √ 4. = 4(x + 1)3
dx 2 4x 2 + x − 3 dx

dy dy
5. = −14(1 − 2x)6 6. = 2 sin(x) cos(x)
dx dx

dy −12 dy
7. = 8. = −18 cos2 (x) · sin(x)
dx (2x − 1)7 dx

dy 12x 2 + 4x − 5 dy 4x − 3
9. = √ 10. = − 2
dx 2 4x 3 + 2x 2 − 5x dx 2x − 3x + 1
2

dy dy 18x
11. = −7 cos6 (x) · sin(x) 12. = − 2
dx dx 3x 2 − 1
dy 3  1/2 dy 3 cos (3x)
13. = (4x + 3) 2x 2 + 3x + 1 14. =
dx 2 dx 2 sin (3x)

dy dy  
15. = −10 (cos (6x))2/3 · sin (6x) 16. = 6x · cos 3x 2
dx dx
√ 
dy dy cos x
17. = −2 sin (sin (2x)) · cos (2x) 18. = √
dx dx 2 x
dy
19. = 4 [sin (3x) + cos (2x)]3 · [3 cos (3x) − 2 sin (2x)]
dx

dy 3x 2 − 10x − 6
20. =  
dx  2
x 3 − 5x 2 − 6x + 4
3
2

Differentiating inverse trigonometric functions

There is more than one way of approach, but this is possibly ‘easier’ to follow.

Example

Differentiate y = sin−1 (x)


320 • Mathematics

Solution
y = sin−1 (x) means sin(y) = x or that x = sin (y) · · · · · · (#)
dx cos (y) dy 1
So = cos(y) = , inverting both sides gives =
dy 1 dx cos(y)
However, this gives the differential as a function of y, not x.

One of the standard trigonometric identities is cos2 (y) + sin2 (y) = 1

Re-arranging gives cos2 (y) = 1 − sin2 (y) ⇒ cos(y) = 1 − sin2 (y)



Substituting for sin(y) [see (#) above] gives cos(y) = 1 − x2
dy 1
Therefore, if y = sin−1 (x), =√
dx 1 − x2
Example

Differentiate y = sin−1 (5x)

Solution
sin (y)
y = sin−1 (5x) means sin (y) = 5x ⇒ x=
5
dx cos(y) dy 5
So = and so =
dy 5 dx cos(y)

As in the above example cos(y) = 1 − sin2 (y)



Substituting for sin(y) gives cos(y) = 1 − (5x)2 = 1 − 25x 2

dy 5
Therefore, if y = sin−1 (5x), =√
dx 1 − 25x 2
Example

Differentiate y = cos−1 (3x)

Solution
cos (y)
y = cos−1 (3x) means cos (y) = 3x ⇒ x=
3
dx − sin(y) dy 3
So = and so =−
dy 3 dx sin(y)

Re-arranging cos2 (y) + sin2 (y) =1 gives sin2 (y) =1 − cos2 (y) ⇒ sin(y) = 1 − cos2 (y)

Substituting for cos(y) gives sin(y) = 1 − (3x)2


Advanced Calculus: Methods of Differentiation • 321

dy 3
Therefore, if y = cos−1 (3x), = −√
dx 1 − 9x 2
Example

Differentiate y = cosh−1 (x)

Solution

y = cosh−1 (x) means x = cosh(y)


dx dy 1
So = sinh(y), therefore =
dy dx sinh(y)

One of the standard hyperbolic identities is cosh2 (y) − sinh2 (y) = 1

Re-arranging gives sinh2 (y) = cosh2 (y) − 1 ⇒ sinh(y) = cosh2 (y) − 1



Substituting for cosh(y) gives sinh(y) = x2 − 1
dy 1
Therefore, if y = cosh−1 (x), =√
dx x −1
2

Questions

Find the first derivative of 1. y = cos−1 (x) 2. y = sinh−1 (x)


Find the second derivatives of 3. y = cos−1 (x) 4. y = cosh−1 (x)

Answers

dy 1 dy 1
1. = −√ 2. =√
dx 1 − x2 dx x2 + 1
dy x dy x
3. =−  3 4. =−  3
dx dx
1 − x2 x2 + 1

Methods of Integration

Integration by substitution

Functions which require integrating are not usually in the form where they can be
looked up in a set of tables. However, there is a way around this problem if a suitable
substitution is made.
322 • Mathematics

An important rule to follow is that once the substitution has been made the original
variable is replaced entirely.

The following examples show the method.

Example

Find cos(5x + 2) dx

Solution
du du
Let u = 5x + 2; then = 5, that is, du = 5dx ⇒ dx =
dx 5
du
Substituting into the original integral gives cos(u)
5
1 1 1
So cos(u)du = sin(u) + c = sin(5x + 2) + c
5 5 5

Example

Find 3x(x 2 + 2)6 dx

Solution
du du
Let u = x 2 + 2; then = 2x ⇒ dx =
dx 2x
du 3
Substituting gives 3x(u)6 = u6 du as the x’s cancel.
2x 2
3 u7 3
Therefore the integral is · +c= · (x 2 + 2)7 + c
2 7 14

Example
et
Find dt
3 + et

Solution
du du
Let u = 3 + et ; then = et ⇒ dt = t
dt e
et du du
Substituting gives · = = ln(u) + c = ln(3 + et ) + c
u et u
Advanced Calculus: Methods of Differentiation • 323

Questions
Integrate by substitution:

1. sin(3x + 2) 2. 2 cos(4t + 1) 3. 4 sin(6x − 3)

1
4. (9x + 5)8 5. 4e7t−1 6. 5t(t2 − 1)7
12
  6x + 2
7. x(3x 2 + 4)8 8. (4x 2 − 1) 4x 3 − 3x 9.  5
3x 2 + 2x − 1

Answers

1
1. u = 3x + 2 ⇒ I = cos(3x + 2) + c
3
1
2. u = 4t + 1 ⇒ I = − sin(4t + 1) + c
2
2
3. u = 6x − 3 ⇒ I = cos(6x − 3) + c
3
1
4. u = 9x + 5 ⇒ I = (9x + 5)9 + c
972
4
5. u = 7t − 1 ⇒ I = e7t−1 + c
7
5 2 8
6. u = t2 − 1 ⇒ I = t −1 +c
16
1  2 9
7. u = 3x 2 + 4 ⇒ I = 3x + 4 + c
54
√ 2 2 3
8. u = 3x 2 + 2 ⇒ I = u du = 3x + 2 2 + c
3
√ du 2  3 3
9. u = 4x 3 − 3x ⇒ I = u = 4x − 3x 2 + c
3 9
1
10. u = 3x 2 + 2x − 1 ⇒ I = u−5 du = −  4 + c
4 3x + 2x − 1
2

If a substitution is unavailable, and for the most case these are quite obvious, the next
approach is ‘integration by parts’.
324 • Mathematics

Integration by parts

The formula for integration by parts is:

u dv = uv− v du

It is best demonstrated by an example.

Example

Find 3x · cos (x) dx

Solution

As far as possible, always let the variable ‘u’ equal the term that includes the ‘x’.

In this case u = 3x

so the term ‘dv’ is whatever follows the ‘x’ expression:

so for this example dv = cos(x)

Now it is necessary to determine ‘du’ and ‘v’

• to determine ‘du’, differentiate ‘u’,


• to determine ‘v’ integrate ‘dv’.

So du = 3 and v = sin(x)

These terms are now inserted into the formula u dv = uv − v du giving

3x · sin(x) − sin(x) · 3dx

It is now necessary to determine the final integral.

sin(x) · 3 dx = 3 sin(x) dx = −3 cos(x)

Substituting this expression into the formula gives 3x · sin(x) − (−3 cos(x))

So 3x · cos (x) dx = 3x · sin(x) + 3 cos(x) + C


Advanced Calculus: Methods of Differentiation • 325

Recap

To determine 3x · cos (x) dx

∫3x.cos(x)dx
u dv

u = 3x dv = cos(x)
du = 3 v = sin(x)

3x · cos (x) dx = 3x · sin(x) − sin(x) · 3 dx = 3x · sin(x) + 3 cos(x) + C

Example

Find x · e2x dx

Solution
If x · e2x dx is compared with the left-hand side of the rule it is possible to put

u = x and dv = e2x
1
So du = 1 and v = e2x
2
These expressions are substituted into the rule giving
1 1 2x 1 1
x · e2x dx = x· e2x − e · 1dx = x · e2x − e2x + C
2 2 2 4
The main problem lies in deciding which part of the expression is ‘u’ and which is ‘dv’.

• Always choose the ‘u’ function so that it will become simpler on differentiation.
• Always choose the ‘dv’ function so that it can be integrated easily.

Questions
Using integration by parts, find the following indefinite integrals and evaluate the
definite integrals:

1. x · ex dx 2. 3x · ln(5x) dx 3. 2x · e3x dx
326 • Mathematics

π
2 π 2

4. x · cos(x) dx 5. x · sin(x) dx 6. ln(x) dx Hint: ln(x) = 1 × ln(x)


0 π 1
2
1 π 2
−x

7. x·e dx 8. (π − x) · cos(x) dx 9. x · ln(x) dx
0 0 1

Answers

3 2 2 2
1. x · ex − ex + c 2. x (2 ln(5x) − 1) + c 3. x · e3x − e3x + c
4 3 9

4. 0.571 5. 2.142 6. 0.386

7. 0.265 8. 2 9. 0.494

A third procedure is to use partial fractions. This involves splitting an algebraic fraction
into its component parts and integrating each in turn.

The forms of partial fractions are summarised as:

Denominators containing Expression example Form of partial fraction

f (x) A B C
Linear factors + +
(x + a) (x − b) (x + c) x+a x−b x+c
f (x) A B C
Repeated linear factors + +
(x + a)3 (x + a) (x + a)2 (x + a)3
f (x) Ax + B D
Quadratic factors    +
ax 2 + bx + c (x + d) ax 2 + bx + c (x + d)

Integration using partial fractions

Once the partial fractions have been obtained the following integrals have to be used
to obtain the final solution:
1 1
• dx = · ln (ax + b)
ax + b a
1 1 
−1 x
• dx = · tan
x 2 + a2 a a
Advanced Calculus: Methods of Differentiation • 327

f  (x)
• dx = ln [f (x)]
f (x)

An example may help to explain the third integral:


cos (x)
If f (x) = sin(x) then f  (x) = cos(x) and so dx = ln [sin (x)]
sin (x)
Example
11 − 3x
Determine dx (linear factors)
(x − 1) (x + 3)

Solution
11 − 3x A B A(x + 3) + B(x − 1)
≡ + ≡
(x − 1) (x + 3) (x − 1) (x + 3) (x − 1)(x + 3)

Hence 11 − 3x ≡ A(x + 3) + B(x − 1)

Let x = 1, then 8 = 4A, from which A = 2.

Let x = −3, then 20 = −4B, from which B = −5


 
11 − 3x 2 5
Hence dx = − dx = 2 ln (x −1) − 5 ln (x + 3) + c
(x − 1) (x + 3) (x − 1) (x + 3)
 
(x − 1)2
or ln + c, by the laws of logarithms.
(x + 3)5

Example
2x + 3
Determine dx (repeated linear factor)
(x − 2)2

Solution
2x + 3 A B A(x − 2) + B
≡ + ≡
(x − 2)
2 (x − 2) (x − 2)2
(x − 2)2

Therefore

2x + 3 ≡ A(x − 2) + B

Let x = 2, then 7 = B

Choose any other x value

Let x = 4, then 11 = 2A + B, then 4 = 2A, then A = 2.


 
2x + 3 2 7 7
So dx = + dx = 2 ln(x − 2) − +c
(x − 2)2 (x − 2) (x − 2)2 (x − 2)
328 • Mathematics

 
7
dx is determined using the substitution u = (x − 2)
(x − 2) 2

Example
2
Determine dx
(x − 1)(x 2 + 1)

Solution
2 A (Bx + C)
= + 2 
(x − 1)(x 2 + 1) (x − 1) x +1

 
So 2 = A x 2 + 1 + (Bx + C) (x − 1)

Let x = 1, then 2 = A(1 + 1), then 2 = 2A, then A = 1

Let x = 0, then 2 = A(1) + (C)(−1), then 2 = A − C, then 2 = 1 − C, then C = −1

Comparing the coefficients of x2 : 0 = A + B, then B = −1


2 1 (−x − 1) 1 x 1
So dx = + 2  dx = − 2 −  dx
(x − 1)(x 2 + 1) (x − 1) x +1 (x − 1) x +1 x +1
2

2 1  
Therefore dx = ln (x − 1) − ln x 2 + 1 − tan−1 (x) + C
(x − 1)(x 2 + 1) 2

Questions

By the use of partial fractions find the following integrals and evaluate the definite
integrals.

4
5x − 3 x
1. dx 2. dx
(x − 3)(x + 3) (x − 1)(x − 2)
3
1.5
9x 2 + 34x + 29 3x 2 − 7x + 2
3. dx 4. dx
(x + 1)(x + 2)(x + 3) (2x − 1)(x + 1)(x − 1)
1.1
5
6 − 15x + 7x 2 2x + 4
5. dx 6. dx
2x(1 − x)(2 − 3x) (1 + x)(x − 1)(2x + 1)
2
Advanced Calculus: Methods of Differentiation • 329

Answers
5x − 3 A B
1. ≡ + ⇒ 5x − 3 = A(x + 3) + B(x − 3)
(x − 3)(x + 3) x − 3 x + 3

Choose values of x to eliminate a bracket


Let x = 3 so 12 = 6A A=2
Let x = −3 so −18 = −6B B = 3
 
2 3
Therefore I = + dx = 2 ln(x − 3) + 3 ln(x + 3) + c
x−3 x+3
x A B
2. ≡ + ⇒ x = A(x − 2) + B(x − 1)
(x − 1)(x − 2) x − 1 x − 2
Let x = 2 so 2=B
Let x = 1 so 1 = −A A = −1
4 
−1 2
Therefore I = + dx = [− ln(x − 1) + 2 ln(x − 2)]43
x−1 x−2
3
I = (− ln(3) + 2 ln(2)) − (− ln(2) + 2 ln(1)) = 3 ln(2) − ln(3) − 2 ln(1) = 0. 981

9x 2 + 34x + 29 A B C
3. ≡ + +
(x + 1)(x + 2)(x + 3) x + 1 x + 2 x + 3

So 9x 2 + 34x + 29 = A(x + 2)(x + 3) + B(x + 1)(x + 3) + C(x + 1)(x + 2)


Let x = −1 so 4 = A(1)(2) 2A = 4 A=2
Let x = −2 so −3 = B(−1)(1) −B = −3 B = 3
Let x = −3 so 8 = C(−2)(−1) 2C = 8 C=4
 
2 3 4
Therefore I = + + dx
x+1 x+2 x+3
Integral = 2 ln(x + 1) + 3 ln(x + 2) + 4 ln(x + 3) + c

3x 2 − 7x + 2 A B C
4. ≡ + +
(2x − 1)(x + 1)(x − 1) 2x − 1 x + 1 x − 1

So 3x 2 − 7x + 2 = A(x + 1)(x − 1) + B(2x − 1)(x − 1) + C(2x − 1)(x + 1)


  
1 3 3 1
Let x = so − = A − A=1
2 4 2 2
Let x = −1 so 12 = B(−3)(−2) 6B = 12 B=2
Let x = 1 so −2 = C(1)(2) 2C = −2 C = −1
1.5 
1 2 1
So I = + − dx
2x − 1 x + 1 x − 1
1.1
330 • Mathematics

 1.5
1
I= ln(2x − 1) + 2 ln(x + 1) − ln(x − 1) = 2. 8723 − 3. 8776 = −1. 005
2 1.1

6 − 15x + 7x 2 A B C
5. ≡ + +
2x(1 − x)(2 − 3x) 2x 1 − x 2 − 3x
So 6 − 15x + 7x 2 = A(1 − x)(2 − 3x) + B(2x)(2 − 3x) + C(2x)(1 − x)
Let x = 0 6 = A(1)(2) 2A = 6 A=3
Let x = 1 −2 = B(2)(−1) −2B = −2 B = 1
Let x = 2 4 = A(−1)(−4) + B(4)(−4) + C(4)(−1)
4 = 4A − 16B − 4C
4 = 12 − 16 − 4C so 4C = 12 − 16 − 4 = −8 C = −2
 
3 1 2 3 2
I= + − dx = ln(x) − ln(1 − x) + ln(2 − 3x) + c
2x 1 − x 2 − 3x 2 3
2x + 4 A B C
6. ≡ + +
(1 + x)(x − 1)(2x + 1) 1 + x x − 1 2x + 1
So 2x + 4 = A(x − 1)(2x + 1) + B(1 + x)(2x + 1) + C(1 + x)(x − 1)
Let x = −1 2 = A(−2)(−1) 2A = 2 A=1
Let x = 1 6 = B(2)(3) 6B = 6 B=1
    
1 1 3 3
Let x = − 3=C − 3=C − C = −4
2 2 2 4
5‘  
1 1 4  5
I= + − dx = ln(1 + x) + ln(x − 1) − 2 ln(2x + 1)
1 + x x − 1 2x + 1 2
2
I = (ln(6) + ln(4) − 2 ln(11)) − (ln(3) + ln(1) − 2 ln(5)) = 0. 503

Integration of trigonometric products

The four basic identities are:


1
sin(A) cos(B) = [sin(A + B) + sin(A − B)] (1)
2
1
cos(A) sin(B) = [sin(A + B) − sin(A − B)] (2)
2
1
cos(A) cos(B) = [cos(A + B) + cos(A − B)] (3)
2
1
sin(A) sin(B) = − [cos(A + B) − cos(A − B)] (4)
2
Advanced Calculus: Methods of Differentiation • 331

The method is to use the above identities to rewrite trigonometric products into
trigonometric additions/subtractions.

Example

Find 8 sin(5t) cos(2t) dt

Solution
1
Using (1) 8 sin(5t) cos(2t) dt = 8× [sin (7t) + sin(3t)] dt
2
1
So 8 × [sin (7t) + sin (3t)] dt = 4 [sin (7t) + sin (3t)]dt
2
 
cos (7t) cos (3t)
=4 − − +C
7 3

Questions

1. Find 5 cos(6t) · cos(2t) dt

2. Find 5 sin(7t) cos(3t) dt

π/4

3. Evaluate cos(4x) cos(2x) dx


0

4. Find sin(8t) cos(t) dt

π/3

5. Evaluate cos(5x) cos(2x) dx


0

6. Find sin(2t) cos(t) dt


π

7. Evaluate cos(5x) cos(2x) dx


0

8. Find sin(3t) cos(t) dt

π/2

9. Evaluate cos(2x) cos(3x) dx


0
332 • Mathematics

Answers
   
5 sin(8t) sin(4t) 5 cos(10t) cos(4t)
1. + +C 2. − − +C
2 8 4 2 10 4
 
1 cos(9t) cos(7t)
3. 0.167 4. − − +C
2 9 7
 
1 cos(3t)
5. 0.062 6. − − cos(t) + C
2 3
 
1 cos(4t) cos(2t)
7. 0 8. − − +C
2 4 2

9. 0.6

Applications of Integration

Basic formulae

If y is a function of x and the range is a ≤ x ≤ b then the following formulae apply:

Areas and volumes of revolution


b
Area : A = y dx
a

y
y = f(x)

Area A

x
0 x=a x=b

 Figure 13.1
Advanced Calculus: Methods of Differentiation • 333

With reference to Figure 13.1, the volume of revolution, V, obtained by rotating Area A
through one revolution about the x axis is given by:
b

V= π · y2 dx
a

If a curve x = f (y) is rotated 360◦


about the y axis between limits y = c and y = d then
the volume generated is given by:
d

V= π · x 2 dy
c

Example

The curve y = x 2 + 4 is rotated about the x axis between the limits x = 1 and x = 4.
Determine the volume of the solid of revolution produced (Figure 13.2).

 Figure 13.2

Solution
b 4 4
 2  
V= π · y dx =
2
π · x + 4 dx = π
2
x 4 + 8x 2 + 16 dx
a 1 1
 4
x5 8x 3
=π + + 16x
5 3 1
       
45 8 · 43 1 5 8 13
V =π + + 16 · (4) − + + 16 (1) = 420. 6π cubic units.
5 3 5 3

Questions

1. The curve xy = 3 is rotated one revolution about the x axis between the limits x = 2
and x = 3. Determine the volume of the solid produced.
334 • Mathematics

2. The curve y = 2x 2 + 3 is rotated about


(a) the x axis between the limits x = 0 and x = 3, and
(b) the y axis, between the same limits.
Determine the volumes generated in each case.

Answers

1. 1.5π cubic units 2. (a) 329.4π cubic units


(b) 81π cubic units

Centroids of plane areas

The centroid of an area is the point at which the total area is considered to be situated
for calculation purposes. (If the area was a thin lamina the centroid would indicate the
position of the centre of mass.)

If x and y represent the coordinates of the centroid of area A, then:

b b
1
x · y dx y2 dx
2
a a
x= and y =
b b

y dx y dx
a a

Example

Find the position of the centroid of the area bounded by the curve y = 3x 2 , the x axis
and the ordinates x = 0 and x = 2.

Solution
2 2
 2
x · 3x 2 dx 3x 3 dx 3x 4
0 0 4 12
x= = = 0
= = 1. 5
2 2
[x 3 ]20 8
3x 2 dx 3x 2 dx
0 0
Advanced Calculus: Methods of Differentiation • 335

2 2
1  
2 2 1  2
3x dx 9x 4 dx 9 x5 9 32
2 2
2 5 0 · 18
y= 0
2
= 0
2
= = 2 5 = = 3. 6
8 8 5
3x 2 dx 3x 2 dx
0 0

Therefore the centroid lies at (1.5, 3.6)

Example

Find the position of the centroid of the area enclosed by the curve y = 5x − x 2 , the x axis
and the ordinates x = 0 and x = 5

Solution 5 5
     5
x · 5x − x 2 dx 5x 2 − x 3 dx 5x 3 x 4
625

3 4
x= 0
= 0
= 0
= 12 = 2. 5
5 5
5x 2 x 3 5 125
 2
  2
 −
5x − x dx 5x − x dx 2 3 0 6
0 0

and
5 5
1  
2 2 1    5
5x − x dx 25x 2 − 10x 3 + x 4 dx 1 25x 3 10x 4 x 5
2 2 − +
0 0 2 3 4 5 0
y= = =
5 5 125
   
5x − x 2
dx 5x − x 2 dx 6
0 0
 
1 25 · (125) 10(625) 3125
· − +
2 3 4 5
y= = 2. 5
125
6
Therefore the centroid lies at (2.5, 2.5).

Questions

For questions 1 and 2, find the position of the centroids of the areas bounded by the
given curves, the x axis and the given ordinates.

1. y = 3x + 2, x = 0 and x = 4.
2. y = 5x 2 , x = 1 and x = 4.
3. Find the position of the centroid of a sheet of metal formed by the curve y = 4x − x 2 ,
the x axis and the lines x = 0 and x = 4.
336 • Mathematics

4. Find the co-ordinates of the centroid of the area which lies between the curve
y = x 2 − 2x, the x axis and the lines x = 0 and x = 2.

Answers

1. (2.5, 4.75) 2. (3.036, 24.36) 3. (2, 1.6) 4. (1, −0.4)

Second moment of area

It is the measure of resistance to the bending of a structural member. The further the
majority of the mass of a body is from its centroid the more resistive it is to bending.

If Ix and Iy represent second moments of area about the x-axis and y-axis respectively,
then:
b b
1
Ix = 3
y dx and Iy = x 2 · ydx
3
a a

Example

Calculate the second moment of area about the y axis of the function
1
y= for values of x between 3 and 4.
(x + 2) (x + 1) (x − 1)

Solution
4 x2 4 4/3 1/2 1/6

Moment = dx = − + dx
3 (x + 2) (x + 1) (x − 1) 3 (x + 2) (x + 1) (x − 1)

(The evaluation of the partial fractions is not shown here.)

Therefore
 4
4 1 1
Moment = · ln (x + 2) − · ln (x + 1) + · ln (x − 1)
3 2 6 3
   
4 · ln(6) ln(5) ln(3) 4 · ln(5) ln(4) ln(2)
= − + − − +
3 2 6 3 2 6
= 0. 199 to 3 decimal places
Advanced Calculus: Methods of Differentiation • 337

Second moment of mass (moment of inertia)

If the mass per unit volume of the volume of revolution generated by the rotation of
area A about the x axis is m, then the moment of inertia of the solid about the x axis is:
b
1
Ix = y4 dx
2
a

Theorem of Pappus

If x and y represent the coordinates of the centroid of area A, then:

Volume of revolution obtained by rotating area A about the x axis = 2πyA


Volume of revolution obtained by rotating area A about the y axis = 2πxA

Example

From an earlier example, the position of the centroid of the area bounded by the curve
y = 3x 2 , the x axis and the ordinates x = 0 and x = 2 was found to be x = 1. 5, y = 3. 6
and the area enclosed was 8 unit2 .

Therefore

Volume of revolution obtained by rotation about the x axis = 2π × 3. 6 × 8


= 180. 96 unit3
Volume of revolution obtained by rotation about the y axis = 2π × 1. 5 × 8
= 75. 40 unit3

The area between curves

The area enclosed between the curves y = f1 (x) and y = f2 (x) (the shaded area in the
diagram, Figure 13.3) is given by:
b b b
shaded area = f2 (x)dx − f1 (x)dx = [f2 (x) − f1 (x)] dx
a a a
338 • Mathematics

y = f2(x)

y = f1(x)
1 2 3 4 x
x=a x=b

 Figure 13.3

Example

Determine the area enclosed between the curves y = x 2 + 1 and y = 7 − x

Solution
It is first of all necessary to find the points of intersection of the two curves (see Figure
13.4).

12 y

x
–4 –3 –2 –1 1 2 3

 Figure 13.4
Advanced Calculus: Methods of Differentiation • 339

At these points x 2 + 1 = 7 − x y =7−x

Therefore x 2 + x − 6 = 0 ⇒ (x + 3) (x − 2) = 0 ⇒ x = −3 and x = 2
2   2  
Shaded area = (7 − x) − x 2 + 1 dx = 6 − x − x 2 dx
−3 −3
  3 2
  
x2 x 4 8 9 −27
Therefore area = 6x − − = 12 − − − −18 − −
2 3 −3 2 3 2 3
   
22 27 125
= − − =
3 2 6

Mean and root mean square values

With reference to Figure 13.5,


b
1
mean value: y = ydx
b−a
a
⎧ ⎫

⎨ b

 1
r.m.s. value:  y2 dx
⎩b − a ⎭
a

y
y = f(x)

x
0 x=a x=b

 Figure 13.5

Example

A sinusoidal voltage has equation y = 50 sin(t). Find the mean and r.m.s. values over half
a cycle.
340 • Mathematics

Solution
Half a cycle means that a = 0 and b = π.
π
1 1
Mean value y = 50 sin(t)dt = [−50 cos(t)]π0
π −0 π
0

50 50 100
Therefore y = [(− cos(π) − (− cos(0))] = [− (−1) − (−1)] =
π π π
The mean value is 31.83 to 2 decimal places.
⎧ ⎫ 
 π ⎧ π

⎨ 1 ⎬ ⎨ 2500 ⎬
 
The r.m.s. value is  (50 sin(t))2 dt =  sin2 (t)dt
⎩π −0 ⎭ ⎩ π ⎭
0 0

Now sin2 (t)dt cannot be found directly, but the table of integrals gives
 
1 sin(2t)
sin2 (t)dt = t −
2 2
π      
1 sin (2π) 1 sin (0) π
So sin (t) dt =
2
π− − 0− =
2 2 2 2 2
0
$
2500 π √
Therefore the r.m.s. value = · = 1250 = 35. 36 v.
π 2
2 1
Note: For a sine wave, mean value = × peak value; r.m.s. = √ × peak value
π 2
Example

Calculate the r.m.s. value of y = 250 sin (25πt) for the first 2 ms.

Solution
The r.m.s. value is
⎧ ⎫ 
 ⎧ ⎫
⎨ 0.002
⎬ ⎨ 0.002

 1 
 (250 sin(25πt))2 dt =  31250000 sin2 (25πt)dt
⎩ 0. 002 − 0 ⎭ ⎩ ⎭
0 0

1
The trigonometric identity gives sin2 (25πt) = (1 − cos (50πt))
2
0.002      
1 sin (50πt) 0.002 1 sin (0. 1π)
So sin (25πt) dt=
2
t− = 0. 002 − − (0)
2 50π 0 2 50π
0
Advanced Calculus: Methods of Differentiation • 341

0.002

So sin2 (25πt) dt = 0. 000016368


0
√ √
Therefore the r.m.s. value = 31250000 × 0. 000016368 = 511. 5 = 22. 62

Questions

1. The velocity, v ms−1 , of a particle is given by the equation v = 4t + 2, where t is the


time in seconds. Find the average value of v between t = 2 and t = 4 s.
2. Find the r.m.s. value of the function y = x (3 − x) for values of x from 0 to 3.
3. A current i = 5 sin(t) A flows in an electrical circuit. Determine the mean and r.m.s.
values, over the range t = 0 to t = π s.

Answers
b 4  4
1 1 1 4t2 1  4  4
1. y = ydt = (4t + 2) dt = · + 2t = · 2t2 + 2t 2 = t2 + t 2
b−a 4−2 2 2 2 2
a 2
   
mean value = 42 + 4 − 22 + 2 = 20 − 6 = 14 ms−1
3 3
 
2
2. The squared value = [x (3 − x)] dx = x 2 (3 − x)2 dx
0 0
3
 
= x 2 9 − 6x + x 2 dx
0
3 3 
 
9x 3 6x 4 x 5
9x − 6x + x dx =
2 3 4
− +
3 4 5 0
0  
9 × 27 6 × 81 243
= − + − (0)
3 4 5
8. 1
So the squared value = 8.1, giving the mean value as = 2. 7
3−0

The r.m.s. value is therefore 2. 7 = 1. 643 to 4 significant figures.
π
1 1
3. y = 5 sin(t)dt = [−5 cos(t)]π0
π −0 π
0
5 5 10
So y = [(− cos(π) − (− cos(0))] = [− (−1) − (−1)] =
π π π
342 • Mathematics

Therefore the mean value is 3.18 to 2 decimal places.


⎧ ⎫ 
 π ⎧ ⎫
⎨ 1 ⎬ ⎨ 25 π ⎬
 
The r.m.s. =  (5 sin(t))2 dt =  sin2 (t)dt
⎩π − 0 ⎭ ⎩π ⎭
0 0
 
1 sin(2t)
But sin2 (t)dt = t−
2 2
π      
1 sin (2π) 1 sin (0) π
So sin2 (t) dt = π− − 0− =
2 2 2 2 2
0 $
25 π √
Therefore the r.m.s. value = · = 12. 5 = 3. 54 V, to 2 decimal places.
π 2

Questions

Determine the root mean square values of the following functions between the speci-
fied limits:

1. y = x 2 from x = 1 to 3. 2. y = x from x = 0 to 2.
3. y = sin(x) + 2 from x = 0 to 2π 4. y = sin(2x) from x = 0 to π
5. y = ex from x = −1 to +1

Answers

1. 4.92 2. 1.15 3. 5.317 4. 0.707 5. 1.35

Application of Differentiation

Rates of change

Example

If the volume of a spherical balloon increases by 4 cm3 each second, what is the rate of
growth of the radius?
Advanced Calculus: Methods of Differentiation • 343

Solution
4
If the radius of the balloon is r, then the volume V = π · r3
3
dV
The rate of change of volume with respect to time, that is, is 4 cm3
dt
dV dV dr dV
The Chain rule gives: = · where = 4π · r2
dt dr dt dr
dr dr 4 1
Therefore 4 = 4π · r2 × and so = = cm/s
dt dt 4π · r2 π · r2
Example

A container in the shape of a right circular cone of height 20 cm, and base radius 2 cm is
catching the drips from a tap leaking at the rate of 0.2 cm3 /s. Find the rate at which the
surface area of water is increasing when the water is halfway up the cone.

Solution
Suppose the height at any time is h cm, and that the radius of the surface of the water
at that time is r cm.

20
r

 Figure 13.6

r h h
By similar triangles (Figure 13.6), = , and so, r =
2 20 10
π · h2 dA
The surface area of the water A = π · r2 = and it is necessary to find when
100 dt
h = 10 cm.
344 • Mathematics

dA dA dh
By the Chain rule: = ·
dt dh dt
dA 2πh dA 2πh dh
But = and so = · (1)
dh 100 dh 100 dt

1 π · h3
The volume of water, V = π · r2 · h =
3 300
dV 3πh2 dV dV dh 3π · h2 dh
Therefore = and so = · = ·
dh 300 dt dh dt 300 dt

dV 3π · h2 dh 3π · h2 dh dh 20
Therefore = · ⇒ 0. 2 = · ⇒ = (2)
dt 300 dt 300 dt dt πh2
dA 2π · h 20 2
From equation 1 and equation 2 = · =
dt 100 π · h2 5h
dA 2
and when h = 10 cm, = = 0. 04 cm2 /s.
dt 50

Questions
1. The area of the surface of a sphere is 4π · r2 , r being the radius. Find the rate of
change of the area in cm2 /s when r = 2 cm, given that the radius increases at the
rate of 1 cm/s.
2. The volume of a cube is increasing at the rate of 2 cm3 /s. Find the rate of change of
the length of the side of the base when its length is 3 cm.
3. The area of a circle is increasing at a rate of 3 cm2 /s. Find the rate of change of the
circumference when the radius is 2 cm.
4. At a given instant the radii of two concentric circles are 8 and 12 cm. The radius of
the outer circle increases at the rate of 1 cm/s and that of the inner at 2 cm/s. Find
the rate of change of the area enclosed between the two circles.
5. A hollow right circular cone is held vertex downwards beneath a tap leaking at the
rate of 2 cm3 /s. Find the rate of rise of water level when the depth is 6 cm given that
the height of the cone is 18 cm and its radius is 12 cm.
1
6. An ink blot on a piece of paper spreads at the rate of cm2 /s.
2
Find the rate of increase of the radius of the circular blot when the radius is 1/2 cm.
7. A hemispherical bowl is being filled with
 water at auniform rate. When the height
1
of the water is h cm the volume is π r · h2 − h3 cm3 , r cm being the radius of
3
the hemisphere. Find the rate at which the water level is rising when it is halfway to
the top, given that r = 6 cm and that the bowl fills in 1 min.
Advanced Calculus: Methods of Differentiation • 345

8. An inverted right circular cone with a vertical angle of 120◦ is collecting water from
a tap at a steady rate of 18π cm3 /min. Find
(i) the depth of the water after 12 min,
(ii) the rate of increase of the depth at this instant.

9. From the formula v = (60s + 25) the velocity, v, of a particle can be calculated
when its distance, s, from the origin is known.
Find the acceleration when v = 10.
 
1 2 dx dy
10. If y = x − , find when x = 2 given =1
x dt dt

Answers

2 3
1. 16π cm2 /s 2. cm/s 3. cm/s 4. Decreasing, 8π cm2 /s
27 2
1 1 4 1
5. cm/s 6. cm/s 7. cm/s 8. 6 cm, cm/min
8π 2π 45 6
4
9. 30 10.
15

Test Examples 13
Determine the following integrals:

1. x · ex dx 2. 2x · e3x dx
π
5x − 3
3. x · sin(x)dx 4. dx
(x − 3)(x + 3)
π
2
9x 2 + 34x + 29 7x − 2x 2
5. dx 6. dx
(x + 1)(x + 2)(x + 3) (2 − x)2 (1 + x)
1 4
  x
7. x· 1 − x 2 dx 8.   dx
0 3
3
x2 − 7
π

9. sin (θ ) · cos2 (θ ) dθ 10. 5 sin(7t) cos(3t)dt


0
346 • Mathematics

π/4

11. cos(4x) cos(2x)dx


0
12. (i) Determine the value of y for the area enclosed between the curve y = x 2 + 1,
the ordinates x = 1 and x = 3 and the x axis.
(ii) Calculate the volume of revolution when this shape is rotated by 360◦ about the
x axis (see Figure 13.7).

12 y

10

2
x
1 2 3 4

 Figure 13.7

13. (i) Determine the area enclosed by the two curves y = x 2 and y = (8x) (see
Figure 13.8).
(ii) If this area is rotated through 360◦ about the x axis determine the volume of the
solid of revolution produced.

3
y = x2

2 y = √(8x)

1 2
–1

 Figure 13.8
Advanced Calculus: Methods of Differentiation • 347

14. A particle moves in such a way that the displacement, s, from a certain point is
given by the formula s = 2t − 1. 4 ln(0. 5 + 4t)
(i) Determine expressions for the velocity and acceleration of the particle.
(ii) Calculate the displacement and acceleration of the particle when the velocity is
0, giving the answer correct to 3 decimal places.
14
STATISTICS
In general, the purpose of statistics is to take the evidence contained in a numerical
record of certain events and to present particular features of the data which can then
be used in a management context.

Definitions
• A population is the group from which the data is collected.
• A sample is a part of the population.
• An individual is a member of the population (this does not necessarily mean a person).
• A variable is a feature characteristic of any individual that differs from individual to
individual.
• A variable that differs in quantity is called a quantitative variable, for example, height.
• A variable that differs in description is called a qualitative variable for exampe, colour.
• A discrete variable is one that can only take certain values within its range, for example,
the number of men making a ship’s crew.
• A continuous variable is one that can take any value within its range, for example, the
length of a ship may be measured to any degree of accuracy.

Tabular Presentation of Data


The advantages are:

• the required figures can be located more readily,


• comparisons can be made more easily,
• patterns may be revealed,
Statistics • 349

• tables are easier for further calculations,


• tables take up less space.

It should always be borne in mind that when presenting a report some explanation
should accompany it, pointing out the most significant features.

Points worth remembering are as follows:

• Have the purpose of the table clearly in mind from the start. Typical aims may be:
◦ to show the original figures in an orderly manner,
◦ to show a distinct pattern in the data,
◦ to summarise the information,
◦ to publish statistics that others may wish to use, for example, Lloyd’s statistics.
• Aim at simplicity so the table is easy to understand.
• Always give the table a full explanatory heading.
• State the units of measurement.
• State the source of the data.
• Make sure that headings are unambiguous.

An example illustrating some of these points is shown in Table 14.1.

Table 14.1 The number and gross tonnage of trading vessels of over 100 gross
registered tons in ‘flag of convenience’ fleets, 1954–1976
1954 1976
Country of Gross tonnage Gross tonnage
registration Number in million tons Number in million tons
Liberia 245 2.4 2, 600 73.5
Panama 595 4.1 2, 680 15.6
Honduras 130 0.4 57 0.1
Cyprus — –.– 765 3.1
Singapore — –.– 722 5.5
Lebanon — –.– 136 0.2
Somalia — –.– 255 1.8
Total 970 6.9 7, 215 99.8
World total 32, 358 97.4 65, 887 372.0
Percentage of the world total 3 7 11 27
Source: Lloyd’s Register of Shipping.
350 • Mathematics

The original purpose of Table 14.1 was to show the growth in the number and tonnage
of vessels in the main countries considered as offering the facility of flag of convenience
over the period 1954–1976.

Frequency Distributions
The most popular form of tabular presentation of statistical data is the use of a fre-
quency distribution. This is a table which shows, for each separate value or group of
values of the variable, the number of times which it occurs.

Table 14.2 Frequency distribution of the number of days in a month,


for Liverpool, with 0.3 mm or more of rainfall
Number of days in a month with
0.3 mm or more of rain Number of months
13 5
14 0
15 3
16 0
17 2
18 1
19 1

Total 12

Table 14.3 Frequency distribution of the number of ships passing


the Felixstowe Ledge buoy per hour in a survey from 1100 to 1800
Time Number of ships
1100 but before 1200 4
1200 but before 1300 9
1300 but before 1400 5
1400 but before 1500 7
1500 but before 1600 3
1600 but before 1700 6
1700 but before 1800 9

Total 43
Statistics • 351

Table 14.4 Number of stars with given sidereal hour angles


Sidereal hour angles Number of stars of magnitude less than 3
0◦ –29◦ 3

30 –59 ◦ 3
60◦ –89◦ 4

90 –119 ◦ 6
120◦ –149◦ 7

150 –179 ◦ 7
180◦ –209◦ 3

210 –239 ◦ 4
240◦ –269◦ 6

270 –299 ◦ 7
300◦ –329◦ 4

330 –359 ◦ 6
Total 60

Examples

Obviously the choice of classes depends on the particular circumstances but there are
a few general hints.

• Make the classes of equal size where possible.


• Make the class intervals multiples of numbers in which people are used to thinking,
such as, 5, 10, 100 etc. and avoid such numbers as 13, 17 etc.
• The number of classes should usually be between 5 and 20.
• If there are a few extreme values at either end of a distribution it is better to put them
together in an open-ended class rather than having a series of classes with very few
observations in them.

Remember that these are only a guide and do not necessarily have to be adhered to.

Pictorial Representation of Data


There are a few general rules.

• Unless you have the deliberate intention of misleading it is important to give the
correct visual impression and to avoid optical illusions. So, if a graph is drawn, it is
important to state the scale used on each axis.
352 • Mathematics

• The graph should have a clear comprehensive title.


• The variable being measured should be on the horizontal axis with the results of the
measurement, or frequencies, on the vertical axis.
• The axes should be labelled.
• The source of the graph should be stated.
• It is important to keep any form of visual representation simple, so that overcrowding
is avoided.

Histograms
These are used to represent frequency distributions, and consist of a series of touching
rectangles with bases on a horizontal axis. Each rectangle represents a value or class in
the distribution and its height is drawn so that the area of the rectangle is proportional
to the frequency it represents. Thus the total area of the histogram represents the total
frequency in the distribution.

Figure 14.1 shows the histogram of the sidereal hour angle data given in Table 14.4.

The labelling of the classes is important since this is a discrete frequency distribution
and yet the histogram demands a series of touching rectangles drawn on a continuous
scale. It is as though each rectangle is extended by 1/2◦ in each direction, thus the second

Frequency
8

Sidereal hour angle


30 60 90 120 150 180 210 240 270 300 330 360

 Figure 14.1
Statistics • 353

Number of ships passing Felixstowe Ledge buoy per hour


10

8
7

5
4

3
2

1
Time of day
0
1100 1200 1300 1400 1500 1600 1700 1800

 Figure 14.2

one is drawn from 291/2◦ to 391/2◦ . In a continuous distribution there is no need for this
extension and it is most usual to label the end of the classes.

Hence Figure 14.2 shows the histogram for the ships’ arrival patterns shown in
Table 14.3.

Frequency Polygons
Instead of a histogram, which is rather tedious to draw, a frequency polygon may be
used, as shown in Figure 14.3. This is just a straight line graph joining the data points
together. The points are plotted at the midpoint of the class, for example, the point for
the data representing 13.00 to 14.00 is plotted at 13.30.

Cumulative Frequency Curves (Ogives)


This plots the running total of the frequencies against bits variable.

The data representing the sidereal hour angles (Table 14.4) becomes Table 14.5.
354 • Mathematics

Number of ships passing Felixstowe Ledge buoy per hour


10

1
Time of day
0
1100 1200 1300 1400 1500 1600 1700 1800

 Figure 14.3

Frequency
60

50

40

30

20

10
Sidereal hour angle
30 60 90 120 150 180 210 240 270 300 330 360

 Figure 14.4

Table 14.5
Hour angles <29◦ <59◦ <89◦ <119◦ <149◦ <179◦ <209◦ <239◦ <269◦ <299◦ <329◦ <359◦

Cum. freq 3 6 10 16 23 30 33 37 43 50 54 60

Pie Charts
A pie chart is a circle divided by radial lines into sections where the area of each section
is proportional to the quantity it represents. It can be shown that the angle of the
section is proportional to the area.
Statistics • 355

The ‘flag of convenience’ data for 1954 can be used to demonstrate this.

Number of ships Central angle

130
Honduras 130 × 360◦ = 48◦
970
245
Liberia 245 × 360◦ = 91◦
970
595
Panama 595 × 360◦ = 221◦
970

Total 970

Flag of covenience

Honduras
11%

Panama
25%

Liberia
64%

Statistical Averages
There are many ‘averages’ in statistics, each one being calculated differently. Therefore
it makes no real sense to speak of ‘The average is . . . unless the particular average is
known.

However, when the word ‘average’ is used, it generally applies to the arithmetic
mean.

There are three averages in common use:

• the arithmetic mean (abbreviated to ‘the mean’),


356 • Mathematics

• the median and


• the mode.

The Arithmetic Mean


For a given set of numerical data the mean is the total of all the items in the data set
divided by the number of items in the set. If the data items are labelled ‘x’ and there are
n items in the set then the mean

x 
x= , where x means ‘the total of all the data’
n

Example
2+5+8+5+6+9+3+4+1
The mean of 2, 5, 8, 5, 6, 9, 3, 4 and 1 is
9
43
So the mean, x = = 4. 7
9
Suppose that instead of a list of numbers, like those above, the data is in the form
of a frequency distribution. The mean is still defined as before but needs rather more
calculation.

Example

Consider the following:

x 6 7 8 9 10 11 12
f 2 4 7 7 5 4 1

The table states that the value ‘6’ occurred twice, so the input into the grand total is 12.
The value ‘7’ occurred 4 times, adding 28 to the grand total etc.

The total of the frequencies givesthe number of data items in the distribution so the
f ·x
formula for the mean is now x =  .
f
The following tabular method should always be used:
Statistics • 357

x f f ·x
6 2 6 x 2 = 12
7 4 7 x 4 = 28
8 7 8 x 7 = 56
9 7 9 x 7 = 63
10 5 10 x 5 = 50
11 4 11 x 4 = 44
12 1 12 x 1 = 12
 
f = 30 f · x = 265


f · x 265
So x =  = = 8. 83
f 30

Advantages

• It is widely understood and the basic calculation is straightforward.


• I makes use of all the data in the group and it can be determined with mathematical
precision.
• It can be determined when only the total value and the number of items are known.

Disadvantages

• A few items of very high or very low value may make the mean appear unrepresenta-
tive of the distribution.
• It may not correspond to an actual value and this may make it appear unrealistic.
• When there are open-ended class intervals, assumptions have to be made which may
not be accurate.

The second disadvantage refers to the following situation. Suppose the data above
referred to marks scored in a test, does a mean of 8. 83 make sense?
358 • Mathematics

The Median
The median is the middle item in the data set when the data has been arranged into
numerical order (either smallest to largest or largest to smallest).

If there is an odd number of data items the middle element can be found exactly but if
there is an even number the median is taken as being midway between the middle two
data items.

Example

Find the median of 3, 8, 2, 7, 4, 8, 5, 6, 5, 9, 5.

Rearranging into order gives 2, 3, 4, 5, 5, 5, 6, 7, 8, 8, 9.

The easiest way of determining which data item is the median is to add 1 to the number
of data items in the set and divide by 2.
11 + 1
There are 11 items, so the middle item is number = 6.
2
The median is therefore 5.

For a grouped frequency distribution a different approach is needed.

Consider the earlier example.

x f Cum. freq.
6 2 2
7 4 6
8 7 13
9 7 20
10 5 25
11 4 29
12 1 30

The cumulative frequency column has been added as the data is already in numerical
order. There are 30 items so the median is the 151/2th value; that is, midway between
the 15th and 16th.
Statistics • 359

From the cumulative frequency column the 13th value is 8 and the 20th is 9; so the 15th
and 16th are both 9. Therefore the median is 9.

For a grouped frequency distribution a similar approach is adopted.

Example

Consider the following data:

Age 0–10 10–20 20–30 30–40 40–50 50–60 60–70 70–80 80–90
Frequency 31 39 34 33 27 28 24 17 7

This gives the following cumulative frequencies:

Age 0–10 10–20 20–30 30–40 40–50 50–60 60–70 70–80 80–90
Cum. freq. 31 70 104 137 164 192 216 233 240

The 240 data items imply that the median is the 1201/2th value. The items required are
in the age category 30–40. However, it is not possible to identify the actual values as the
actual data is not known, only that they are in the category 30–40.

To gain a better estimate of the median in this situation a cumulative frequency curve
is needed.

The method is detailed below:

• Construct the cumulative frequency table.


• Draw the cumulative frequency curve; it gives a better estimate than the polygon.
• Calculate which data item is the median. (In the example above the median is the
1201/2th value.)
• Find this position on the vertical scale.
• Draw a horizontal line from this position until it meets the curve.
• Draw a vertical line down from this position and read off the median from the
horizontal scale.

The cumulative frequency curve for the ‘Age’ data is as follows:


360 • Mathematics

Cumulative frequency
240

200

160

120

80

40

Age
10 20 30 40 50 60 70 80 90

Advantages

• Extreme high or low values do not distort it as a representative average. Therefore it is


useful for describing distribution in areas such as wages where a few extreme values
would distort the arithmetic mean.
• It is straightforward to calculate even if not all the values are known, or where there
are irregular class intervals.
• It is often an actual value and even when it is not it may look representative and
realistic.

Disadvantages

• It gives the value of only one item. Other items are used to locate it but their values
do not influence it.
• In a continuous series, grouped in class intervals, the value of the median is only an
estimate.
• It is not suitable for further arithmetical calculations.

The Mode
The mode is the commonest data item. In the list of numbers used for the median
example the mode is 5 since 5 occurs three times.
Statistics • 361

In the ‘Age’ distribution the modal class is 10–20 as there are 39 items in this group.

Advantages

• It is commonly used.
• It is representative of the ‘typical’ item, being the most frequent.
• It is an actual value that is easy to determine and thus may appear sensible.

Disadvantages

• It may not be unique and can often be a matter of judgement.


• It does not include all the values of the distribution.
• It is not very useful if the distribution is widely dispersed.
• It is unsuitable for further calculations.

Calculation of the Mean of a Grouped


Frequency Distribution
The method is the same as for a frequency distribution but the ‘x’ values are taken as
being the mid-class values.

Example

To estimate the mean of the ‘Age’ data.

Age Frequency, f Mid-class value, x fx


0–10 31 5 155
10–20 39 15 585
20–30 34 25 850
30–40 33 35 1155
40–50 27 45 1215
(continued)
362 • Mathematics

Age Frequency, f Mid-class value, x fx


50–60 28 55 1540
60–70 24 65 1560
70–80 17 75 1275
80–90 7 85 595
 
f = 240 f · x = 8930

f · x 8930
An estimate of the mean is therefore x =  = = 37. 2 years.
f 240

Questions

Find the mean of the following frequency tables:

1. x 66 67 68 69 70 71 72 73
f 2 5 10 12 9 6 4 2

2. x 103 110 117 124 131


f 8 13 24 11 4

3. x 1 2 3 4 5 6
f 2 6 11 15 8 3

4. x 2 5 8 11 14
f 2 4 8 5 1

5. x 22 27 32 37 42
f 1 6 10 2 1

6. x 10 20 30 40 50 60
f 3 9 14 10 6 4

7. x 304 308 312 316 320


f 1 5 9 4 4

8. x 15 25 35 45 55 65
f 3 7 12 18 12 6
Statistics • 363

Answers

Question n x x
1 50 3465 50
2 60 6950 115.833
3 45 165 3.667
4 20 157 7.85
5 20 620 31
6 46 1570 34.130
7 23 7196 312.870
8 58 2500 43.103

Questions

For questions 1 and 2 calculate the mean:

1. Marks 1 2 3 4 5 6 7 8 9 10
Freq. 2 2 4 6 10 12 10 9 4 1

2. No. of persons/house 1 2 3 4 5 6 7 8
Freq. 56 124 161 148 73 25 8 5

For question 3 estimate the mean:

3. Class 1–5 6–10 11–15 16–20 21–25


Freq. 3 5 8 6 3

4. The mean of seven numbers is 15. When another number is added, the mean of
the eight numbers is 16. Find the eighth number.
5. The mean of seven numbers is 15. When three more numbers are added, the mean
of the ten numbers is 12. Find the mean of the three numbers.
6. The mean height of 100 boys is 1.70 m while the mean height of 50 girls is 1.64 m.
Find the mean height of the 150 children.
7. The mean weight of 100 children is 60 kg. If the mean weight of the 60 boys is 65
kg, find the mean weight of the 40 girls.
8. A motorist travels 100 km at an average speed of 90 km/h and returns at an average
speed of 80 km/h. Find his average speed over 200 km.
364 • Mathematics

9. A man can row at 8 km/h in still water. He rows upstream against a current of 2 km/h
and returns to his starting point. Find his average speed over the whole journey.
10. Write down five numbers that have a mode of 4, a median of 5 and a mean of 6.

Answers

1. n = 60 fx = 352 x = 5. 867

2. n = 600 fx = 1990 x = 3. 317

3. n = 25 fx = 330 x = 13. 2

x7 
4. = 15 ⇒ x7 = 105
7

x8 
= 16 ⇒ x8 = 128 ∴ 8th value = 23
8

x7 
5. = 15 ⇒ x7 = 105
7

x10 
= 12 ⇒ x10 = 120
10
The last three values therefore total 15 so their mean is 5.
 
b  g 
6. = 1. 7 ⇒ b = 170, = 1. 64 ⇒ g = 82
100 50
The total of all 150 children is therefore 170 + 82 = 252 m
252
Their mean is = 1. 68 m.
 150 
all  b 
7. = 60 ⇒ all = 6000 = 65 ⇒ b = 3900
100 60
 2100
g = 6000 − 3900 = 2100 so g = = 52. 5 kg
40
100
8. 100 km at 90 k/h takes h
90
100
100 km at 80 k/h takes h
80
 
100 100
So 200 km takes + h
90 80
200
The average speed =   = 84.71 k/h
100 100
+
90 80
Statistics • 365

9. Suppose he travels x km upstream.

x x
x km at 6 k/h takes h and x km at 10 k/h takes h
6 10
x x 
The total time for the 2x km is + h
6 10

x x 5x + 3x 8x 4x
+ = = =
6 10 30 30 15

2x 15 30x 30 15 1
Average speed = = 2x × = = = = 7 k/h
4x 4x 4x 4 2 2
15

Questions

Draw the cumulative frequency curves and estimate the median of the following
distributions:

1. Class interval 1–4 5–9 10–19 20–49 50–99 100–199


Frequency 2 10 15 24 14 5
[Median = 29.5]

2. Blood level 70–74 75–79 80–84 85–89 90–94 95–99


No. of people 40 91 165 248 200 156
[Median = approximately 87]

3. Class interval 71–79 80–88 89–97 98–106 107–115


Frequency 1 3 6 8 12
Class interval 116–124 125–133 134–142 143–151
Frequency 8 5 4 3
[Median = 111.5]

4. I.Q. 60–79 80–89 90–94 95–99


Frequency 9 29 51 57
I.Q. 100–104 105–114 115–129 130–149
Frequency 61 53 31 9
[Median = 99.8]
366 • Mathematics

Measures of Dispersion

Introduction

Example

Two cargoes are waiting to be loaded onto a ship, each one consists of 2,000 bags of
similar size. The bags in the two cargoes are the same shape and size and appear to
have little difference between them.

A sample of 5 bags is taken from each cargo.

From the first cargo the mass, in kilograms, of the bags are 148.3, 149.4, 149.6, 150.2 and
150.5, giving a mean mass of 149.6 kg.

From the second cargo the mass, in kilograms, of the bags are 138.4, 142.5, 149.6, 156.8
and 160.7, giving a mean mass of also 149.6 kg.

However, although both samples have the same mean mass, there is a difference in
variability between them, the first being much more consistent than the second.

By giving only one statistic, such as the mean, to describe the cargo does not convey
it’s make up. There is need for another statistic, one that gives a measure of spread or
dispersion.

As with measures of dispersion, there are several measures of dispersion depending on


the type of data being described.

Range

The simplest of these measures is the range which is the difference between the largest
and smallest of the data items.

So for the sample from the first cargo the range is 150. 5 − 148. 3 = 2. 2 kg while that
from the second cargo has a range of 160. 7 − 138. 4 = 22. 3 kg.

The difference between the two samples is immediately clear and, in this case, the range
is a good measure of dispersion as in each case the masses are spread regularly across
the range.
Statistics • 367

The problem with the range arises when there are ‘outliers’ in the data; in other words,
extreme values that are not typical to the rest of the figures.

Example

2, 3, 4, 5, 6, 7, 50

The range in this case is 48, as the data is dominated by the outlier 50. (The mean will
also be ‘corrupted’, 4.5 without the 50, 11 with it.)

In the vast majority of cases the range is found simply to give a snapshot of the data but
other measures, for example, the standard deviation, are needed for a more accurate
picture.

There are times when the range is the ideal measure of dispersion.

Consider the following:


Example

A bulk carrier has to be designed to carry cargoes of differing stowage factors, so that
its stability remains satisfactory at all times. These are:

Coal Stowage factor 1.3 m3 /tonne


Light grain Stowage factor 1.7 m3 /tonne
Iron ores Stowage factor 0.4 m3 /tonne
Manganese ore Stowage factor 0.5 m3 /tonne
Wheat Stowage factor 1.4 m3 /tonne
Slurry Stowage factor 0.8 m3 /tonne
Nitrates Stowage factor 1.0 m3 /tonne
Concentrates Stowage factor 1.5 m3 /tonne

The stability must be satisfactory when the ship is loaded with cargoes with the extreme
values of stowage factor, that is, for light grain with a stowage factor of 1.7 m3 /tonne
and iron ore with a stowage factor of 0.4 m3 / tonne and therefore the range of 1.3 m3 is
the measure of dispersion that is needed here.

Standard deviation

This is the measure of dispersion that is most commonly used and it is the most difficult
one to calculate.
368 • Mathematics

The definition of the standard deviation is ‘the square root of the arithmetic mean of the
squared deviations of the observations about the arithmetic mean of the distribution’

To understand this, it is best to look at a simple example.

Consider the set of 5 numbers x, {4, 5, 6, 9, 11}

Step 1: Find the arithmetic mean.



x 4 + 5 + 6 + 9 + 11 35
x= = = =7
5 5 5
Step 2: For each number xi , calculate its deviation di , from the arithmetic mean

That is, di = xi − x

So for
xi 4 5 6 9 11
di −3 −2 −1 2 4
Step 3: Square the deviations

So for
xi 4 5 6 9 11
di −3 −2 −1 2 4
di2 9 4 1 4 16

Step 4: Find the mean of the di2


 2
di 9 + 4 + 1 + 4 + 16 31
= = = 6. 2
5 5 5
This value is called the variance of the distribution and the final step is

Step 5: Find the square root of the variance



6. 2 = 2. 49 to 2 decimal places

The standard deviation is given the symbol σ where σ has the formula

1 
σ= · (xi − μ)2 where μ is the mean of the population.
n

Notation and formulae for the standard deviation

If ALL the data is being used in the calculations (i.e. the population) then the mean is
given the symbol μ and the standard deviation the symbol σ .
Statistics • 369

If only PART of the data is being used (i.e. a sample) then the mean is given the symbol
x and the standard deviation the symbol s.

1 
σ= · (xi − μ)2 is the formula created from the definition but it is not the easiest
n
one to use in practice.

Ungrouped data (that is, a list of numbers)


x2
σ= − μ2
n

1 
If a sample was being used the formula would be s = · (xi − x)2
n

Grouped data (that is, a frequency table)

 
f · x2 f ·x
σ=  − μ2 where μ = 
f f

f · x2
If a sample was being used the formula would be s =  − (x)2 where
f

f ·x
x= 
f
On its own, the mean is not enough to describe a set of data as it gives no feel as to the
dispersion of the data. The standard deviation is used to give this measure and a rough
rule of thumb is that 95% of the data lies within two standard deviations of the mean.

For ungrouped data the formulae are as follows:



x2
Standard deviation, σ = − (x)2 , where n is the number of data items.
n
(The variance of a set of data is simply σ 2 . )

Example

Find the mean and standard deviation of the following:

4, 5, 8, 3, 7, 6, 6, 7, 1, 4
370 • Mathematics

Set the data as follows:


x x2

x 51
4 16 x= = = 5. 1
n 10
5 25
8 64
 2 
x 301 √
3 9 σ= − (x)2 = − 5. 12 = 30. 1 − 26. 01
n 10

7 49 = 4. 09 = 2. 022
6 36 (The variance is therefore 4.09.)
6 36
7 49
1 1
4 16
Total 51 301

f ·x
For grouped data the formulae are slightly different. Namely mean x =  and
f
 2
fx
standard deviation σ =  − (x)2
f

Example

Find the mean and standard deviation of the following:

x 1 3 5 7 9 11 13
f 2 5 17 21 13 0 2

Again set out the working as follows:

x f x2 f ·x fx2
1 2 1 2 2
3 5 9 15 45
5 17 25 85 425
7 21 49 147 1029
9 13 81 117 1053
11 0 121 0 0
13 2 169 26 338
Total 60 392 2892


(fx) 392
x=  = = 6. 533 to 3 decimal places.
f 60
Statistics • 371

 2 
fx 2892 √
σ=  − (x) =
2
− (6. 533)2 = 5. 5199 = 2. 35 to 2 decimal places.
f 60

Questions

Find the standard deviation of the following frequency tables:

1. x 66 67 68 69 70 71 72 73
f 2 5 10 12 9 6 4 2

2. x 103 110 117 124 131


f 8 13 24 11 4

3. x 1 2 3 4 5 6
f 2 6 11 15 8 3

4. x 2 5 8 11 14
f 2 4 8 5 1

5. x 22 27 32 37 42
f 1 6 10 2 1

6. x 10 20 30 40 50 60
f 3 9 14 10 6 4

7. x 304 308 312 316 320


f 1 5 9 4 4

8. x 15 25 35 45 55 65
f 3 7 12 18 12 6

Answers
 
Question n x x2 x σ
1 50 3465 240269 50 1.7
2 60 6950 808488 115.833 7.579
(continued)
372 • Mathematics

 
Question n x x2 x σ
3 45 165 673 3.667 1.229
4 20 157 1421 7.85 3.107
5 20 620 19600 31 4.359
6 46 1570 61900 34.130 13.445
7 23 7196 2251856 312.870 4.407
8 58 2500 117850 43.103 13.191

In certain circumstances more than one measurement can be taken from an individual.
For example, if ships were being studied, their length and maximum tonnage may be
required. This is called bivariate data and allows questions like ‘Is there a link between
the variables being measured?’ This leads into the area of correlation and regression.

Correlation and Regression

Definitions (recap)

In statistics, mean has two related meanings:

• the arithmetic mean of a set of data, and


• the expected value of a random variable, which is also called the population mean.

For a data set, the mean is the sum of the observations divided by the number of obser-
vations. The mean is often stated along with the standard deviation: the mean describes
the central location of the data, and the standard deviation describes the spread.

The variance is one measure of statistical dispersion averaging the squared distance of
its possible values from mean. Whereas the mean is a way to describe the location of
a distribution, the variance is a way to describe its scale or degree of being spread out.
The unit of variance is the square of the unit of the original variable. The positive square
root of the variance, called the standard deviation, has the same units as the original
variable and can be easier to interpret for this reason.

Covariance is the measure of how much two variables change together (as distinct from
variance, which measures how much a single variable changes).
Statistics • 373

If two variables tend to vary together (i.e. when one of them is above its expected value,
then the other variable tends to be above its expected value too), then the covariance
between the two variables will be positive. On the other hand, if one of them is above
its expected value and the other variable tends to be below its expected value, then the
covariance between the two variables will be negative.

A trend line represents a trend, the long-term movement in time series data after other
components have been accounted for. It tells whether a particular data set (say GDP,
oil prices or stock prices) have increased or decreased over the period of time. A trend
line could simply be drawn by eye through a set of data points, but more properly their
position and slope is calculated using statistical techniques like linear regression. Trend
lines typically are straight lines.

1
–2 –1 0 1 2

Trend lines are sometimes used in business analytics to show changes in data over time.
This has the advantage of being simple. Trend lines are often used to argue that a par-
ticular action or event (such as training or an advertising campaign) caused observed
changes at a point in time. This is a simple technique; however, it suffers from a lack of
scientific validity in cases where other potential changes can affect the data.

Correlation (often measured as a correlation coefficient), indicates the strength and direc-
tion of a linear relationship between two random variables. In general statistical usage,
correlation or co-relation refers to the departure of two variables from independence.

A number of different coefficients are used for different situations. The best known is
the Pearson product-moment correlation coefficient, which is obtained by dividing the
covariance of the two variables by the product of their standard deviations.
374 • Mathematics

Correlation

It was suggested, for reasons of economy, that there was no need to have two weather
stations, A and B, covering a particular area. It was felt that the manning of station B
should be discontinued on the grounds that, if visibility is considered, the visibility in
the area of station B can reasonably be inferred from knowledge of the visibility from
station A. The visibility reported from the two stations A and B over a period was noted
on 12 occasions as follows:

Visibility (km)
Occasion Station A Station B
1 1.5 1.0
2 3.0 1.5
3 20.0 10.0
4 2.0 7.5
5 5.0 3.0
6 3.0 3.0
7 2.5 2.0
8 10.0 6.0
9 1.0 0.5
10 8.0 5.0
11 4.0 3.5
12 0.5 2.0

If the visibility from station A is plotted on the horizontal axis and that of station B on
the vertical axis, the following scatter diagram is created:
Scatter diagram to show the visibilty at two stations A and B
12
10
8
6
4
2
0
0 5 10 15 20 25

The main purpose of the diagram is to see if any visual link exists between the two
sets of data. A measurement of a linear relationship is the Product Moment Correlation
Coefficient, r (the PMCC). This is a value between +1 and −1 (+1 indicates that the data
Statistics • 375

form a perfect straight line with positive gradient, −1 indicates a perfect straight line
with negative gradient, while a value of 0 implies a total lack of link).

The PMCC is given by the formula


  
(x · y) − ( x) ( y)

r= 
   
n · (x)2 − ( x)2 n · (y)2 − ( y)2

Consider the visibility data where n = 12.

x y x·y (x)2 (y)2


1.5 1.0 1.5 2.25 1.0
3.0 1.5 4.5 9.0 2.25
20.0 10.0 200.0 400.0 100.0
2.0 7.5 15.0 4.0 56.25
5.0 3.0 15.0 25.0 9.0
3.0 3.0 9.0 9.0 9.0
2.5 2.0 5.0 6.25 4.0
10.0 6.0 60.0 100.0 36.0
1.0 0.5 0.5 1.0 0.25
8.0 5.0 40.0 64.0 25.0
4.0 3.5 14.0 16.0 12.25
0.5 2.0 1.0 1.25 4.0
    
x = 60. 5 y = 45. 0 x. y = 365. 5 (x)2 = 636. 75 (y)2 = 259. 0

12 × 365. 5 − 60. 5 × 45. 0


Therefore r =     = 0. 80 to 2 decimal
12 × 636. 75 − 60. 52 × 12 × 259. 0 − 45. 02
places.

The conclusion is that there is a direct linear relationship between the two stations and
that the relationship is reasonably high.

It is now reasonable to find the formula that links the two together.

This is done by calculating the equation of the line of best fit (this is technically called
the least squares regression line).
376 • Mathematics

Linear Regression

The equation of the regression line of y on x is y = b · x + a, where b is the slope and a is


the intercept on the y axis.

The value of the slope is:


  
n· (x · y) − ( x) · ( y)
b=  
n · (x)2 − ( x)2

and the intercept is found using:

a= y−b·x

Using the above data the value of b = 0. 42 and the value of a = 1. 64

So the regression line is y = 0.42x + 1.64

This line can then be plotted onto the scatter diagram giving:

Scatter diagram to show the visibilty at two stations A and B


12

10
y = 0.42x + 1.64
8
Station B

2
Station A
0
0 5 10 15 20 25

Either the graph or the equation can be used to predict values, for example, what would
the visibility at station B be if the visibility at station A was 14 km?

This is called interpolation as the data is being used to predict values within the range
of the data.

The graph (or equation) could be used to predict values outside of the data, for example,
what would the visibility at station B be if the visibility at station A was 24 km?

This is called extrapolation and must be viewed with some element of doubt as there is no
knowledge of what happens after a visibility of 20 km at station A.
Statistics • 377

Rank correlation

In order to calculate the PMCC the data has to be quantitative, that is, the data has to be
numerical.

Example

Consider the following situation:

In order to investigate the safety of navigation in a particular port, the ships using the
port over a period of two years were assigned to twelve classes (lettered A−L) which
were based on gross tonnage, nature of cargo and power of main engines. The inves-
tigator then assigned ranks to these classes in what he considered to be the order of
increasing potential danger resulting from vessel casualties. The relative frequency of
casualties, defined as the number of casualties per number of ships at risk over the
two-year period for each class, was as follows:

Rank for potential Relative frequency of


Class of ship danger of casualties casualties
B 1 Low 0.025
C 2 0.019
A 3 0.030
E 4 0.028
G 5 0.014
D 6 0.032
F 7 0.016
I 8 0.021
J 9 0.017
H 10 0.024
K 11 0.011
L 12 High 0.015

Is this evidence sufficient to confirm the suggestion that the risk of casualty decreases
as the potential danger of the consequences increases?
378 • Mathematics

The first step is to rank both variables in the same direction; say from 1 low to n high,
where n is the number of pairs, in this case 12. The second step is to find the difference,
di , between the ranks for each pair, and the third step is to calculate di2 in each case.

Spearman’s rank correlation coefficient, rs , is given by the formula:



6 d2
rs = 1 −  2 
n n −1
The necessary calculations are shown below:

Class of ship Danger rank Casualty frequency rank d d2


B 1 9 −8 64
C 2 6 −4 16
A 3 11 −8 64
E 4 10 −6 36
G 5 2 +3 9
D 6 12 −6 36
F 7 4 +3 9
I 8 7 +1 1
J 9 5 +4 16
H 10 8 +2 4
K 11 1 +10 100
L 12 3 +9 81
 2
d = 436

6 d2 6 × 466
rs = 1 −  2  =1− = −0. 52 to 2 decimal places.
n n −1 12 × (144 − 1)

The answer 0.52 does not indicate a very close relationship but the negative sign does
at least confirm the suggestion that, by and large, the risk of casualty decreases as the
potential danger of the consequences increases.

Worked example on linear regression and correlation

A mathematics lecturer was interested in the effect of remedial classes on the perfor-
mance of students who had originally failed an assessment. Eight students attended
Statistics • 379

remedial classes for a number of hours and their reassessment of the test gave the
following results.

Attendance at Percentage scored


Student remedial classes (h) in reassessment
A 2 60
B 12 90
C 8 80
D 10 85
E 6 75
F 4 70
G 12 95
H 14 90

(a) Plot a scatter diagram of the data to investigate whether a link appears to exist
between the attendance at remedial classes and the reassessment score. (Plot the
attendance time horizontally.)
(b) Calculate the value of the correlation coefficient for the data.
(c) Calculate the equation of the line of best fit which links attendance and attainment.
Draw this line on the scatter diagram.

Method – to be followed every time

Month, x %, y x2 y2 x·y
2 60 4 3600 120
12 90 144 8100 1080
8 80 64 6400 640
10 85 100 7225 850
6 75 36 5625 450
4 70 16 4900 280
12 95 144 9025 1140
14 90 196 8100 1260
    
Total x = 68 y = 645 x 2 = 704 y2 = 52975 x · y = 5820
380 • Mathematics

 
68 x y 645
• x= = = 8. 5 y= = = 80. 625
n 8 n 8
 2  2
x y
• Var(x) = − (x)2 Var(y) = − (y)2
n n
704 52975
= − (8. 5)2 = 15. 75 = − (80. 625)2 = 121. 4844
8 8
  
n · xy − ( x) · ( y)
• Correlation coefficient r = 
   
n · x 2 − ( x)2 n · y2 − ( y)2
8 × 5820 − 68 × 645
=    = 0. 9645
8 × 704 − 682 8 × 52975 − 6452
  

n· xy − ( x) · ( y)
Slope of regression line, b =  
n x 2 − ( x)2
8 × 5820 − 68 × 645
= = 2. 6786
8 × 704 − 682
• Intercept of regression line, a = y − b × x = 80. 625 − 2. 6786 × 8. 5 = 57. 86
• Equation of regression line is y = bx + a. Therefore, y = 2. 68x + 57. 9

If the situation under consideration


is such that the intercept must be 0 then the regression
x·y
line becomes y = bx where b =  2 .
x
Attendance versus Attainment
100

90
Precentage scored

y = 2.68x + 57.9
80

70

60

Attendance (h)
50
0 2 4 6 8 10 12 14 16 18

Questions
For questions 1 to 3 determine the equation of the regression line of y on x.

Find also the correlation coefficient, r.


Statistics • 381

1. x 14 18 23 30 50
y 900 1200 1600 2100 3800

2. x 2.7 4.3 1.2 1.4 4.9


y 11.9 7.10 33.8 25.0 7.50

3. x 6 3 9 15 2 14 21 13
y 1.3 0.7 2.0 3.7 0.5 2.9 4.5 2.7

4. The relationship between the voltage applied to an electrical circuit and the current
flowing is as shown:

Current (mA) 2 4 6 8 10 12 14
Voltage (V) 5 11 15 19 24 28 33

Assuming a linear relationship, determine the equation of the regression line of


voltage, y, on current, x, correct to 4 significant figures.
Plot the points on a scatter diagram and draw in the regression line.
Determine (i) the voltage when the current is 3 mA,
(ii) the current when the voltage is 40 v,
assuming the regression line is valid outside the data range.
Calculate the correlation coefficient.
5. In an experiment to determine the relationship between force and momentum, a
force, x, is applied to a mass, by placing the mass on an inclined plane, and the time,
y, for the velocity to change from u m/s to v m/s is measured. The results obtained
are as follows:

Force (N) 11.4 18.7 11.7 12.3 14.7 18.8 19.6


Time (T) 0.56 0.35 0.55 0.52 0.43 0.34 0.31

Determine the equation of the regression line of time on force, assuming a linear
relationship between the quantities, correct to 3 significant figures.
Calculate the correlation coefficient.

Answers

1. y = 80. 6x − 256, 0.9995


2. y = −6. 50x + 35. 9, −0. 9162
3. y = 0. 216x + 0. 0477, 0. 9919
382 • Mathematics

4. y = 2. 268x + 1. 117 (i) 7.92 V, (ii) 17.1 mA, 0.9988


5. y = −0. 0290x + 0. 881, −0. 9930

Probability
When an unbiased dice is rolled, each of the six faces has an equal chance of landing
uppermost. The throwing of 1, 2, 3, 4, 5 or 6 are called equiprobable events.

Suppose that we want to deal an ace from a pack of 52 cards. Each time we deal an ace
we sat that there has been a favourable outcome or a success. Since there are 4 aces in
the pack, the number of equiprobable events which produces a favourable outcome (or
the number of successes) is 4.

The probability of obtaining a favourable outcome is given by the formula:

Number of ways of choosing the equiprobable event


Probability =
Total number of possible choices

• All probabilities have a value between 0 and 1 inclusive.


• An event has a probability of 0 if it can never happen, and has a value of 1 if it is certain
to happen.
• The total probability covering all possible events is 1. That is,
probability of success + probability of failure = 1.

Example

One letter is chosen at random from the word ‘mathematics’.

What is the probability that the letter chosen will be:

(i) the letter a, (ii) the letter c, (iii) a vowel, (iv) a consonant?

(i) Since there are 11 letters in the given word there is a total of 11 equiprobable
2
events. There are 2 letter a’s in the word so the probability is .
11
1
(ii) There is only one letter c so the probability is .
11
4
(iii) There are 4 vowels so the probability is .
11
7
(iv) There are 7 consonants so the probability is .
11
Statistics • 383

Alternatively if a letter is not a vowel it must be a consonant. So the probability of a


vowel + the probability of a consonant = 1.
4 7
Therefore the probability of a consonant = 1 − = .
11 11

Questions

1. A fair dice is thrown. Calculate the probability that the result will be:
(i) a three, (ii) a score less than four, (iii) an even number.
2. A letter is chosen at random from the word ‘terrific’. Determine the probability that
it will be:
(i) an f, (ii) an r, (iii) a vowel, (iv) a consonant.
3. A bag contains 3 red counters, 5 blue counters and 2 green counters. A counter is
chosen at random from the bag. Find the probability that it will be:
(i) green, (ii) blue, (iii) red, (iv) not red.
4. There are 7 tomatoes in a bag. Four are red and the rest are green. A tomato is picked
at random from the bag. What is the probability that it is red?
5. Twenty discs, numbered from 1 to 20, are placed in a box and one disc is chosen at
random. Determine the probability that the number on the disc will be:
(i) even, (ii) a multiple of 3, (iii) more than 8, (iv) less than 8.
6. A card is drawn at random from a deck of 52 shuffled playing cards. Find the
probability that it will be:
(i) the king of hearts, (ii) a queen, (iii) an ace, king or queen,
(iv) the jack of diamonds or the ace of spades.
7. Two fair dice are thrown together and their scores added. Determine the probability
that the sum will be:
(i) 7, (ii) less than 5, (iii) more than 9.
8. From a shuffled pack of 52 playing cards 2 cards are dealt. What is the probability of:
(i) the first card dealt being an ace, (ii) the second card dealt also being an ace?

Answers
1 3 1 3 1
1. (i) (ii) = (iii) =
6 6 2 6 2
2 1 3 1 6 2
2. (i) (ii) (iii) = (iv) =
9 9 9 3 9 3
384 • Mathematics

2 1 5 1 3 3 7
3. (i) = (ii) = (iii) (iv) 1 − =
10 5 10 2 10 10 10
5
4.
8
10 1 5 1 8 2 10 1
5. (i) = (ii) = (iii) = (iv) =
20 2 20 4 20 5 20 2
1 4 1 12 3 2 1
6. (i) (ii) = (iii) = (iv) =
52 52 13 52 13 52 26
6 1 3 1 10 5
7. (i) = (ii) = (iii) =
36 6 36 12 36 18
4 1 3
8. (i) = (ii)
52 13 53

The Addition Law of Probability


If two events cannot happen at the same time, the events are said to be mutually exclu-
sive. For example, in a single roll of a fair dice a 3 or a 4 can be uppermost but not both
at the same time. Hence these are mutually exclusive events.

If p1 , p2 , p3 , . . . are the separate probabilities of a set of mutually exclusive events, the


probability of one of these events occurring is p1 + p2 + p3 + · · ·

Example

A dice with faces numbered 1 to 6 is rolled once. What is the probability of scoring a
three or a four?
1
Probability of scoring a three =
6
1
Probability of scoring a four =
6
1 1 2 1
Probability of scoring a three or a four = + = =
6 6 6 3
Sometimes the addition law is known as the OR law as we seek the probability of one
event or another event.

Notation
p (event A or event B) = p (A ∪ B)
Statistics • 385

The Multiplication Law of Probability


An independent event is one which has no effect on subsequent events. For instance, if
a dice is rolled twice, the result of the first roll does not have any influence on the result
of the second roll. Hence the two rolls of the dice are independent events.

The events of cutting an ace from a pack of cards and throwing a head in a single toss
of a fair coin are also independent events.

If p1 , p2 , p3 , . . . are the separate probabilities of a set of independent events then the


probability that all the events will occur is p1 × p2 × p3 × . . .

Example

If a coin is tossed and a card is dealt from a pack of 52 cards, calculate the probability
that the result will be a head and an ace.
4
Probability of dealing an ace =
52
1
Probability of throwing a head =
2
4 1 1
Probability of dealing an ace and throwing a head = × =
52 2 26
Example

A coin is tossed 4 times. What is the probability that each toss will result in a head?

The 4 tosses are independent events.


1 1 1 1 1
Therefore the probability = × × × =
2 2 2 2 16
The multiplication law of probability is sometimes called the AND law as we seek the
probability that one event and another event occur.

Notation
p (event A and event B) = p (A ∩ B)

The Probability Tree


Example

A coin is tossed three times. Find the probability that:


386 • Mathematics

(i) a head will occur on all three occasions,


(ii) only one head will occur in three tosses.

We can work out these probabilities using a probability tree.


H

H T
H

H
T
T

H
H
T

T
T H

(i) The branch of the tree needed to find the probability of three heads is shown below:
H

1 1 1 1
Probability = × × =
2 2 2 8
(ii) the branches of the tree required to find the probability of only one head on three
tosses is:
H T
T

H
T

T
T H
1 1 1 1 3
Each branch has probability × × = so the probability =
2 2 2 8 8
Statistics • 387

Conditional probability

Sometimes we want to find the probability that an event will occur given that a certain
condition has already been met.

Notation
p (A ∩ B)
p (event B occurs given that event A has already occurred) = p (B|A) =
p (A)

Example

In an analysis of case cargo damaged over a number of voyages, the following results
were obtained:

(i) In all, 50% were damaged due to inadequate ventilation and, for these, there was a
probability of 0.25 that they would be rejected, 0.75 that they would be kept.
(ii) In all, 15% were damaged due to weather effects and, for these, there was a
probability of 0.3 that they would be rejected, 0.7 kept.
(iii) In all were damaged during loading or discharging operations and, for these, there
was a probability of 0.6 that they would be rejected, 0.4 kept.

If a particular damaged case was rejected without a cause being stated, what is the
probability that this was due to inadequate ventilation?

Key: iv, inadequate ventilation; we, weather effects; do, discharging operations; r, reject;
k, keep.
0.25 0.125*
r rejected and iv

iv 0.75 k 0.375 event B = iv


0.5
r
0.3 0.045*
we
event A = rejected
0.15
0.7
p(A ∩ B )
k 0.105
do r p(B | A ) =
0.6 p( A )
0.21*
0.35 0.125
=
0.125 + 0.045 + 0.21
0.125
=
0.4 0.38
k 0.14
= 0.329
388 • Mathematics

The situation becomes a little bit more complicated when examples of the following
type have to be considered.

Suppose a manufacturer makes a certain item. On testing, these items are either perfect
or faulted but are indistinguishable to the eye.

Suppose that, of all the items manufactured, 10% are known to be faulty. These items
are packed, at random, in boxes of 10. What is the probability that a box, again chosen
at random, contains 3 faulty items?

This is an example of a Binomial probability. ‘Bi’ because there are two possibilities only
for each item; in this case, either perfect or faulty.

There is a fixed probability for each item being perfect or faulty, and a fixed number to
choose from.

These conditions allow the following method to be used.

Example

Ten resistors are chosen at random from a large batch.

It is known from experience that the probability of a resistor being defective is 10%, that
is, 0.1 (so the probability that the resistor being non-defective is 0.9).

What is the probability that the chosen batch contains 3 defective resistors?

There are 3 defective resistors so there are 7 non-defective resistors.

The probability of this event is (0. 1)3 × (0. 9)7 (10 independent events – see AND law)

However, this assumes the first 3 are defective and the next 7 are not defective

The number of ways of choosing r items from n items is denoted n Cr .


n!
The value of n Cr = , where n! = n × (n − 1) × (n − 2) × Λ × 2 × 1
r! × (n − r)!
For this example it is necessary to find how many ways 3 items can be chosen from 10.
10! 3628800
This is given by 10 C3 = = = 20
3! × 7! 6 × 5040
So the final probability is 20 × (0. 1)3 × (0. 9)7 = 0. 0096.

In general terms, if p is the probability of a ‘success’ and q (= 1 − p) is the probability of a


‘failure’, and X represents the number of successes, the probability of r successes from n trials
is given by:
p (X = r) = n Cr × pr × qn−r , r = 0, 1, 2, Λ, n
Statistics • 389

Questions
Problems 1 to 4 refer to a box containing a large number of capacitors of which 70% are
known to be within given tolerance limits.

1. When three capacitors are drawn at random, determine the probability that:
(i) two are not within,
(ii) two are within the given tolerance limit.
2. When six capacitors are drawn at random, determine the probability that:
(i) three are not within,
(ii) there are not more than two within the given tolerance limits.
3. When nine capacitors are drawn at random, determine the probability that:
(i) less than three are not within,
(ii) six are within the given tolerance limits.
4. When five capacitors are drawn at random, determine the probability that:
(i) more than four are not within,
(ii) there are two not within the tolerance limits.
5. A machine produces 10% defective components. In a sample of 5 drawn at random,
find the probability of having
(i) three defective,
(ii) two defective, and
(iii) no defective components.
6. The output of a machine has, on average, 94% perfect components. Determine the
probability that in a sample of four components more than one will be imperfect.
7. Resistors are packed in packets of ten and there are, on average, 2% defective. Find
the probability of finding two defective resistors.

Answers

1. (i) X = number outside tolerance p (X = 2) = 3 C2 × (0. 3)2 × (0. 7)1 = 0. 189


(ii) X = number within tolerance p (X = 2) = 3 C2 × (0. 7)2 × (0. 3)1 = 0. 441
2. (i) X = number outside tolerance p (X = 3) = 6 C3 × (0. 3)3 × (0. 7)3 = 0. 185
(ii) X = number within tolerance
Not more than 2 within ≡ 0 or 1 or 2 within
p (X = 2, 1 or 0) = 6 C2 × (0. 7)2 × (0. 3)4 + 6 C1 × (0. 7)1 × (0. 3)5 + 6 C0 × (0. 7)0 × (0. 7)6

= 15 × (0. 7)2 × (0. 3)4 + 6 × (0. 7)1 × (0. 3)5 + (0. 7)6
= 0. 0705
390 • Mathematics

3. (i) X = number outside tolerance


less than 3 not within ≡ 0 or 1 or 2 not within ≡ 0 or 1 or 2 without

p (X = 2, 1 or 0) = 9 C2 × (0. 3)2 × (0. 7)7 + 9 C1 × (0. 3)1 × (0. 7)8


+ 9 C0 × (0. 3)0 × (0. 7)9
= 36 × (0. 3)2 × (0. 7)7 + 9 × (0. 3)1 × (0. 7)8 + (0. 7)9
= 0. 463

(ii) X = number within tolerance p (X = 6) = 9 C6 × (0. 7)6 × (0. 3)3 = 0. 267


4. (i) X = number outside tolerance p (X = 5) = 5 C5 × (0. 3)5 × (0. 7)0 = 0. 0024
(ii) X = number outside tolerance p (X = 2) = 5 C2 × (0. 3)2 × (0. 7)3 = 0. 309
5. X = number defective
(i) p (X = 3) = 5 C3 × (0. 1)3 × (0. 9)2 = 0. 0081
(ii) p (X = 2) = 5 C2 × (0. 1)2 × (0. 9)3 = 0. 073
(iii) p (X = 0) = 5 C0 × (0. 1)0 × (0. 9)5 = 0. 5905

6. X = number defective
p(X > 1) = 1 − p(X = 0) − p(X = 1)
p (X > 1) = 1 − (0. 94)4 − 4 × (0. 94)3 × (0. 06) = 1 − 0. 7807 − 0. 1993 = 0. 02
7. X = number defective
p (X = 2) = 10 C2 × (0. 02)2 × (0. 98)8 = 0. 0153

Normal distribution

This is probably the most important probability distribution in statistics. It is often


referred to as the Gaussian distribution, particularly in the engineering world.

The graph of a normal distribution is shown below:

µ
Statistics • 391

The main characteristics are summarised below:

• It is a bell shaped curve, suggesting that there is a gradual fall-off in probability from
the highest point.
• It is symmetrical about the central point which is the mean value, μ.
• The mean = the median = the mode = μ in the diagram.
• The two tails get closer and closer to the base line but never touch or cross it, that is,
they are asymptotic.

Mathematically the equation of the normal distribution is given by the formula

1 2
p(X = x) = √ e−(x−μ) /2σ
2

σ 2π

where the mean of the distribution is μ and the variance is σ 2 .


 
This is written as x ∼ N μ, σ 2

There is no need to be able to manipulate this expression as the results are tabulated.

The characteristic shape of the curve remains the same regardless of the values of the
mean and variance.

Two curves with the same variance but different means are shown in Figure 14.5, while
two curves with the same mean but different variances are shown in Figure 14.6.

f (x)

x
µ1 µ2

 Figure 14.5
392 • Mathematics

f(x)

σ1 σ2

 Figure 14.6

Any normal distribution which has a mean of 0 and a variance of 1 is known as the
standard normal distribution. So if Z has a standard normal distribution then Z ∼ N(0, 1).
X −μ
The formula Z = transforms the X distribution into the Z distribution
σ
The equation of the standard normal curve is
1 −z2
f (z) = √ e /2

The standard normal distribution is important as the values of its cumulative function
have been tabulated.

Suppose the probability that Z is less than 1.52 is required. The answer is equal to the
shaded area in Figure 14.7.

Notation: The probability that the standard normal variable, Z, is less than a value, say z,
is written as Φ(z).

Therefore p (z ≤ 1. 52) = Φ (1. 52) = 0. 9357 (from tables).

The tables only give area to the left of a given positive value. However, the symmetry of
the curve can be used to answer other questions:

• To find the probability that z is greater than 1.52 the area to the right of 1.52 is needed.
This is just what remains when the area to the left of 1.52 is removed and since the
total area under the curve is 1, p (z ≥ 1. 52) = 1 − Φ (1. 52) = 1 − 0. 9357 = 0. 0643.
Statistics • 393

0.5 f(z)

0.4

0.3

0.2

0.1

z
–2 –1 1 2

1.52

 Figure 14.7

f (z)

z
a b

 Figure 14.8

• To find the probability that z is less than −1. 52: this area is the same as that required
in the above example. Therefore Φ (−1. 52) = 1 − Φ (1. 52).
In general Φ (−z) = 1 − Φ (z)
◦ The probability that z lies between two values, p (a < Z < b) = Φ (b) − Φ (a) (see
Figure 14.8).

Example

Coils of rope supplied by a particular manufacturer are quoted as having a nominal


length of 250 m. Tests indicate that the lengths are normally distributed with a mean
394 • Mathematics

length of 254 m and a standard deviation of 1.5 m. What is the probability that a
particular coil will be:

(a) shorter than 256 m


(b) longer than 257 m
(c) between 250 and 255 m in length?

Solution

Let X denote the random variable representing the lengths of coil.


 
Therefore X ∼ N 254, 1. 52

256 − 254
(a) p (X < 256) = p Z < = Φ (1. 33) = 0. 9082
1. 5
257 − 254
(b) p (X > 257) = 1 − p (X < 257) = 1 − p Z < = 1 − Φ (2) = 1 − 0. 9772
1. 5
= 0. 0228
250 − 254 255 − 254
(c) p (250 < X < 255) = p <Z< = p (−2. 67 < Z < 0. 67)
1. 5 1. 5)

p (−2. 67 < Z < 0. 67) = Φ (0. 67) − Φ (−2. 67)


= Φ (0. 67) − [1 − Φ (2. 67)]
= 0. 7486 − [1 − 0. 9962]
= 0. 7486 − 0. 0038 = 0. 7448

Questions
1. A machine tool producing pins must produce a diameter of 12 mm ± 0. 05 mm.
Continuous monitoring of the size by gauging equipment shows that the mean is
12.01 with a standard deviation of 0.03.
Assuming a normal distribution, what percentage is rejected?
2. The lifespan in hours of a mass produced light optical device is normally distributed
and has a mean of 1,400 with a standard deviation of 300.
What is the probability of one taken at random having a lifespan between 1,400 and
1,850 h?
What is the percentage that will last longer than 2,100 h?
If the guarantee is for 1,000 h what percentage will fail to meet the guarantee?
What lifetime should be guaranteed if 95% must be obtained?
3. A machine makes resistors with a nominal value of 22 kΩ and tolerance ±5%.
A sample of 1,000 produces a mean of 21.8 kΩ and a standard deviation of 0.8 kΩ.
Statistics • 395

Assuming the normal distribution is applicable; determine the percentage that is


likely to fall within the required values.

Answers
1. 11.4%
2. 43.3%, 1%, 9.1%, 907 h
3. 81.8%

Example

1. Ships travelling through the Corinth canal have speeds that follow a normal distri-
bution with a mean of 5 knots with a standard deviation of 1.5 knots.
 
That is, X ∼ N 5, 1. 52
(i) What is the probability that a ship chosen at random has a speed less than 6
knots?
(ii) What is the probability that a ship chosen at random is steaming at less than 3.5
knots?
(iii) What is the probability of a ship chosen at random steaming at more than 7
knots?
(iv) What is the probability that a ship is steaming at a speed between 4.5 and 6.5
knots?
Solution
6−5
(i) p (X < 6) = p Z < = p(Z < 0. 67) = Φ(0. 67) = 0. 749
1. 5
3. 5 − 5
(ii) p (X < 3. 5) = p Z < = p(Z < −1) = Φ(−1) = 1 − Φ (1) = 1 − 0. 813
1. 5
= 0. 1587
7−5
(iii) p (X > 7) = 1 − p (X < 7) = 1 − p Z < = 1 − Φ (1. 33) = 1 − 0. 908 =0. 092
1. 5
4. 5 − 5 6. 5 − 5
(iv) p (4. 5 < X < 6. 5) = p <Z< = p (−0. 33 < Z < 1) = Φ (1) −
1. 5 1. 5
Φ (−0. 33) = Φ (1) − (1 − Φ (0. 33)) = Φ (1) + Φ (0. 33) − 1 = 0. 8413 + 0. 6293 − 1
= 0. 4706

Questions

The following questions all relate to the speeds of ships passing through the Corinth
canal:

1. Mean = 4, s.d. = 1
(i) p(X < 6) (ii) p(X < 3) (iii) p(3 < X < 5)
396 • Mathematics

2. Mean = 10, s.d. = 3


(i) p(X < 8) (ii) p(X < 13) (iii) p(7 < X < 11) (iv) p(X > 8. 5)
3. Mean = 6, s.d. = 1.2
(i) p(4 < x < 7) (ii) p(X > 4. 5) (iii) p(X < 8. 3) (iv) p(X > 7. 8)

Answers

1. (i) Φ (2) = 0. 9772


(ii) Φ (−1) = 1 − Φ (1) = 1 − 0. 8413 = 0. 1587
(iii) Φ (1) − Φ (−1) = 0. 8413 − 0. 1587 = 0. 6826
2. (i) Φ (−0. 67) = 1 − Φ (0. 67) = 1 − 0. 7486 = 0. 2514
(ii) 0.8413
(iii) Φ (0. 33) − Φ (−1) = 0. 47
(iv) 1 − Φ (−0. 5) = 1 − (1 − Φ (0. 5)) = Φ (0. 5) = 0. 6915
3. (i) Φ (0. 83) − Φ (−1. 67) = 0. 7967 − 0. 0475 = 0. 7492
(ii) 1 − Φ (−1. 25) = 1 − (1 − Φ (1. 25)) = Φ (1. 25) = 0. 8944
(iii) Φ (1. 92) = 0. 973
(iv) 1 − Φ (1. 5) = 0. 0668

Normal probability table


z p(Z < z) z p(Z < z) z p(Z < z) z p(Z < z) z p(Z < z) z p(Z < z) z p(Z < z) z p(Z < z)
0.00 0.5000 0.50 0.6915 1.00 0.8413 1.50 0.9332 2.00 0.9772 2.50 0.9938 3.00 0.9987 3.50 0.9998
0.01 0.5040 0.51 0.6950 1.01 0.8438 1.51 0.9345 2.01 0.9778 2.51 0.9940 3.01 0.9987 3.51 0.9998
0.02 0.5080 0.52 0.6985 1.02 0.8461 1.52 0.9357 2.02 0.9783 2.52 0.9941 3.02 0.9987 3.52 0.9998
0.03 0.5120 0.53 0.7019 1.03 0.8485 1.53 0.9370 2.03 0.9788 2.53 0.9943 3.03 0.9988 3.53 0.9998
0.04 0.5160 0.54 0.7054 1.04 0.8508 1.54 0.9382 2.04 0.9793 2.54 0.9945 3.04 0.9988 3.54 0.9998
0.05 0.5199 0.55 0.7088 1.05 0.8531 1.55 0.9394 2.05 0.9798 2.55 0.9946 3.05 0.9989 3.55 0.9998
0.06 0.5239 0.56 0.7123 1.06 0.8554 1.56 0.9406 2.06 0.9803 2.56 0.9948 3.06 0.9989 3.56 0.9998
0.07 0.5279 0.57 0.7157 1.07 0.8577 1.57 0.9418 2.07 0.9808 2.57 0.9949 3.07 0.9989 3.57 0.9998
0.08 0.5319 0.58 0.7190 1.08 0.8599 1.58 0.9429 2.08 0.9812 2.58 0.9951 3.08 0.9990 3.58 0.9998
0.09 0.5359 0.59 0.7224 1.09 0.8621 1.59 0.9441 2.09 0.9817 2.59 0.9952 3.09 0.9990 3.59 0.9998
0.10 0.5398 0.60 0.7257 1.10 0.8643 1.60 0.9452 2.10 0.9821 2.60 0.9953 3.10 0.9990 3.60 0.9998
0.11 0.5438 0.61 0.7291 1.11 0.8665 1.61 0.9463 2.11 0.9826 2.61 0.9955 3.11 0.9991 3.61 0.9998
0.12 0.5478 0.62 0.7324 1.12 0.8686 1.62 0.9474 2.12 0.9830 2.62 0.9956 3.12 0.9991 3.62 0.9999
0.13 0.5517 0.63 0.7357 1.13 0.8708 1.63 0.9484 2.13 0.9834 2.63 0.9957 3.13 0.9991 3.63 0.9999
0.14 0.5557 0.64 0.7389 1.14 0.8729 1.64 0.9495 2.14 0.9838 2.64 0.9959 3.14 0.9992 3.64 0.9999
0.15 0.5596 0.65 0.7422 1.15 0.8749 1.65 0.9505 2.15 0.9842 2.65 0.9960 3.15 0.9992 3.65 0.9999
(continued)
Statistics • 397

Normal probability table (Continued)


z p(Z < z) z p(Z < z) z p(Z < z) z p(Z < z) z p(Z < z) z p(Z < z) z p(Z < z) z p(Z < z)
0.16 0.5636 0.66 0.7454 1.16 0.8770 1.66 0.9515 2.16 0.9846 2.66 0.9961 3.16 0.9992 3.66 0.9999
0.17 0.5675 0.67 0.7486 1.17 0.8790 1.67 0.9525 2.17 0.9850 2.67 0.9962 3.17 0.9992 3.67 0.9999
0.18 0.5714 0.68 0.7517 1.18 0.8810 1.68 0.9535 2.18 0.9854 2.68 0.9963 3.18 0.9993 3.68 0.9999
0.19 0.5753 0.69 0.7549 1.19 0.8830 1.69 0.9545 2.19 0.9857 2.69 0.9964 3.19 0.9993 3.69 0.9999
0.20 0.5793 0.70 0.7580 1.20 0.8849 1.70 0.9554 2.20 0.9861 2.70 0.9965 3.20 0.9993 3.70 0.9999
0.21 0.5832 0.71 0.7611 1.21 0.8869 1.71 0.9564 2.21 0.9864 2.71 0.9966 3.21 0.9993 3.71 0.9999
0.22 0.5871 0.72 0.7642 1.22 0.8888 1.72 0.9573 2.22 0.9868 2.72 0.9967 3.22 0.9994 3.72 0.9999
0.23 0.5910 0.73 0.7673 1.23 0.8907 1.73 0.9582 2.23 0.9871 2.73 0.9968 3.23 0.9994 3.73 0.9999
0.24 0.5948 0.74 0.7704 1.24 0.8925 1.74 0.9591 2.24 0.9875 2.74 0.9969 3.24 0.9994 3.74 0.9999
0.25 0.5987 0.75 0.7734 1.25 0.8944 1.75 0.9599 2.25 0.9878 2.75 0.9970 3.25 0.9994 3.75 0.9999
0.26 0.6026 0.76 0.7764 1.26 0.8962 1.76 0.9608 2.26 0.9881 2.76 0.9971 3.26 0.9994 3.76 0.9999
0.27 0.6064 0.77 0.7794 1.27 0.8980 1.77 0.9616 2.27 0.9884 2.77 0.9972 3.27 0.9995 3.77 0.9999
0.28 0.6103 0.78 0.7823 1.28 0.8997 1.78 0.9625 2.28 0.9887 2.78 0.9973 3.28 0.9995 3.78 0.9999
0.29 0.6141 0.79 0.7852 1.29 0.9015 1.79 0.9633 2.29 0.9890 2.79 0.9974 3.29 0.9995 3.79 0.9999
0.30 0.6179 0.80 0.7881 1.30 0.9032 1.80 0.9641 2.30 0.9893 2.80 0.9974 3.30 0.9995 3.80 0.9999
0.31 0.6217 0.81 0.7910 1.31 0.9049 1.81 0.9649 2.31 0.9896 2.81 0.9975 3.31 0.9995 3.81 0.9999
0.32 0.6255 0.82 0.7939 1.32 0.9066 1.82 0.9656 2.32 0.9898 2.82 0.9976 3.32 0.9995 3.82 0.9999
0.33 0.6293 0.83 0.7967 1.33 0.9082 1.83 0.9664 2.33 0.9901 2.83 0.9977 3.33 0.9996 3.83 0.9999
0.34 0.6331 0.84 0.7995 1.34 0.9099 1.84 0.9671 2.34 0.9904 2.84 0.9977 3.34 0.9996 3.84 0.9999
0.35 0.6368 0.85 0.8023 1.35 0.9115 1.85 0.9678 2.35 0.9906 2.85 0.9978 3.35 0.9996 3.85 0.9999
0.36 0.6406 0.86 0.8051 1.36 0.9131 1.86 0.9686 2.36 0.9909 2.86 0.9979 3.36 0.9996 3.86 0.9999
0.37 0.6443 0.87 0.8078 1.37 0.9147 1.87 0.9693 2.37 0.9911 2.87 0.9979 3.37 0.9996 3.87 0.9999
0.38 0.6480 0.88 0.8106 1.38 0.9162 1.88 0.9699 2.38 0.9913 2.88 0.9980 3.38 0.9996 3.88 0.9999
0.39 0.6517 0.89 0.8133 1.39 0.9177 1.89 0.9706 2.39 0.9916 2.89 0.9981 3.39 0.9997 3.89 0.9999
0.40 0.6554 0.90 0.8159 1.40 0.9192 1.90 0.9713 2.40 0.9918 2.90 0.9981 3.40 0.9997 3.90 1.0000
0.41 0.6591 0.91 0.8186 1.41 0.9207 1.91 0.9719 2.41 0.9920 2.91 0.9982 3.41 0.9997 3.91 1.0000
0.42 0.6628 0.92 0.8212 1.42 0.9222 1.92 0.9726 2.42 0.9922 2.92 0.9982 3.42 0.9997 3.92 1.0000
0.43 0.6664 0.93 0.8238 1.43 0.9236 1.93 0.9732 2.43 0.9925 2.93 0.9983 3.43 0.9997 3.93 1.0000
0.44 0.6700 0.94 0.8264 1.44 0.9251 1.94 0.9738 2.44 0.9927 2.94 0.9984 3.44 0.9997 3.94 1.0000
0.45 0.6736 0.95 0.8289 1.45 0.9265 1.95 0.9744 2.45 0.9929 2.95 0.9984 3.45 0.9997 3.95 1.0000
0.46 0.6772 0.96 0.8315 1.46 0.9279 1.96 0.9750 2.46 0.9931 2.96 0.9985 3.46 0.9997 3.96 1.0000
0.47 0.6808 0.97 0.8340 1.47 0.9292 1.97 0.9756 2.47 0.9932 2.97 0.9985 3.47 0.9997 3.97 1.0000
0.48 0.6844 0.98 0.8365 1.48 0.9306 1.98 0.9761 2.48 0.9934 2.98 0.9986 3.48 0.9997 3.98 1.0000
0.49 0.6879 0.99 0.8389 1.49 0.9319 1.99 0.9767 2.49 0.9936 2.99 0.9986 3.49 0.9998 3.99 1.0000
398 • Mathematics

Text Examples 14
1. Coils of rope supplied by a particular manufacturer are quoted as having a nominal
length of 250 m. Tests indicate that the coil lengths are normally distributed with a
mean length of 254 m and a standard deviation of 1.5 m.
(a) What is the probability that a particular coil will be shorter than 256 m?
(b) What percentage of the coils will be longer than 257 m?
(c) In a batch of 200 coils, how many would be expected to be between 250 and
255 m in length?
2. 5,000 ingots of metal have a mean mass of 16.5 kg and a standard deviation of 0.5 kg.
A random sample of 400 ingots, chosen without replacement, is taken.
(a) What is the standard error of x, the sample mean?
Find the probability that the sample will have a mean mass:
(b) between 16.495 kg and 16.506 kg
(c) more than 16.51 kg.
3. An analysis into the reason why marine engineers decide to leave the sea and take a
shore job was undertaken. The results are as follows:
(i) 20% disliked the work and, for these, there was a probability of 0.6 that they
would leave;
(ii) 50% felt they were underpaid and, for these, there was a probability of 0.4 that
they would leave;
(iii) 30% felt that long periods away from home was unsettling and, for these, there
was a probability of 0.9 that they would leave.
If a particular engineer left the sea without a cause being stated, what is the
probability that this was due to the fact of disliking the work?
4. Some results obtained from a tensile test on a steel specimen are shown below:

Tensile force (kN) x 4.8 9.3 12.8 17.7 21.6 26.0


Extension (mm) y 3.5 8.2 10.1 15.6 18.4 20.8

(i) Calculate the correlation coefficient between tensile force (x) and extension (y).
(ii) Plot the data on a suitably labelled scatter diagram and assuming a linear rela-
tionship, determine the equation of the regression line of y on x. Draw the
regression line on the scatter diagram.
SELECTION OF
15
EXAMINATION
STANDARD
QUESTIONS
1. Determine the second differential coefficients of:
2
(a) y = x 4 − + 7x − 9
x2
(b) y = ln (x) + sin (x)
2. (a) Prove that x can have any value to satisfy the equation:
 2  
1 1 2
x+ − x− =4
x x

(b) Find the values of  


1 1 1 2
(i) x 2 + 2 , (ii) x 4 + 4 , (iii) x given that x + =9
x x x
3. Water flows along a horizontal pipe of 100 mm internal diameter, the depth of
water in the pipe is 75 mm. Calculate the hydraulic mean depth d of the water
from the formula:

Cross-sectional area of water stream


d=
Perimeter of wetted pipe surface
400 • Mathematics

120
4. Given p = , where p and V represent the pressure (N/m2 ) and volume (m3 ) of a
V
given mass of gas at constant temperature, calculate by integration the work done
when the gas expands from V = 0. 5 m3 to V = 6. 5 m3 .
5. The volumetric analysis of a certain mixture is 68% water, 18% alcohol and the
remainder solids. Calculate
(a) how much water should be added to each litre of this mixture to reduce the
alcohol content to 15%,
(b) the percentage analysis of the new mixture.
6. Differentiate the following with respect to x:
Cx 3 b
(a) y = Rgx − (b) y = ax 3 + 2
64L x
Integrate the following:
2
 2.6 
(c) k − 1 dk
1

 2 
(d) 2v − 4 v dv

7. Sketch the graph of y = 11 + 7x− 2x 2 and use differential calculus to determine


the maximum value of y.
8. A component consists of a cone of base diameter 52.5 mm and perpendicular
height 45 mm, standing concentrically on the top of a solid cylinder 60 mm diam-
eter and 40 mm high. The base of this component is firmly fixed on to a flat
base-plate.
Calculate the total surface area of the component exposed above the base-plate.
9. (a) The distance s metres travelled by a body in time t seconds is given by:

s = t3 − 5. 5t2 − 4t + 68. 5.

Determine the time, distance travelled and acceleration of the body when its
velocity becomes 0.
(b) Find the following:

 √ 
(i) 3 x − 3x 2 dx
  
2
(ii) + 2 dx
x3
10. A roof has the form of a square pyramid, the base is 24 m2 and the length of the
ridge from corner of base to apex is 18 m. Find:
(i) the perpendicular height from base to apex, and
(ii) the total surface area of the roof.
Selection of Examination Standard Questions • 401

11. (a) Multiply 2x 3 − 3x 2 + 2x + 4 by x 2 − 3x + 5


(b) Divide 3x 3 + x 2 + 3x + 5 by x + 1
 3  2
(c) Multiply 2a2 b by 3ab2
 3  2
(d) Divide 3ab2 by ab3
12. Find the values of angle θ between 0◦ and 360◦ which will satisfy the equation:
tan(θ ) = cos(θ )
13. (a) Determine the second differential coefficients of:
(i) y = 4 cos (x) + ln (x)
4
(ii) y = 3x 3 + 2 + 6x − 2
x
16000
(b) The expression T = 1700 + + 0. 6x 2 is used to calculate the hoop tensile
x2
strength T of certain rotating discs. If x is the disc radius, use calculus to deter-
mine the value of x for which T is a minimum and the corresponding value of
T at that radius.
14. The following table gives corresponding values of x and y. Plot y against x and,
assuming they are connected by a straight line law, find the values of the constants
and state the law.

x −2.0 +2.1 +4.1 +5.9 +8


y −13.7 −5.9 −2.0 +1.7 +6

15. A sector of a circle is cut out of a piece of sheet metal, the radius of the sector being
102 mm and the angle at the centre 150◦ . The sector is then rolled into the shape
of a cone. Find the surface area of the metal used, the diameter of the base of the
cone and its perpendicular height.
16. Find the values of x and y in the simultaneous linear equations:

10y + 4b = −2x
3a + x = y

given that the same values of a and b in the above also satisfy the equations:

19a + 27b = 43
7a + 4b = −2

17. The rim of a flywheel is elliptical in cross-section with the major axis in the radial
direction. The major and minor axes of the rim section measures 300 and 200 mm
respectively, and the overall diameter of the wheel is 3.6 m. If the material is cast
iron of density 7. 21 × 103 kg/m3 , find the mass of the rim.
402 • Mathematics

18. Determine the gradient of a tangent to any point in the curve

x3 x2
y= −
4 2
and show that there are two points where the gradient is 0.
Determine the point on the curve where y is a minimum.
19. Factorise completely:
(i) 2x 2 − 2
(ii) 7x 2 + 9xy − 10y2
(iii) 14x 3 y2 + 22x 2 y3 − 12xy4
(iv) 3x 3 + 8x 2 y − xy2 − 10y3 given that (x − y) is one of the factors.
20. Using integration determine the difference in area bounded by the curves
x2 1 1
y = 10x 2 + x + 4 and y = − − , the x axis and the lines x = 1 and x = 5.
2 x 2
21. (a) Using integral calculus, determine the area bounded by the curve y = 2 cos (x)
π
and the x axis between the ordinates at x = 0 and x =
4
(b) A curve which passes through the point (1, −2) has a gradient of 3x 2 − 4.
Determine the equation of the curve.
22. (i) Solve the following quadratic equation by the method of ‘completing the
square’:
x 2 + 5x = 84

(ii) Find the value of x in the following quadratic equation by factorizing:

2x 2 − 3ax − bx = 0

23. Determine the second differential coefficient of:


1
(i) y = x 2 − 5x + 7 −
x
(ii) y = 2 cos (x) − 3 sin (x)
24. A block of brass is 80 mm long and has a varying cross-section throughout its
length. Commencing at one end, the cross-sectional areas are at regular distances
apart: 0, 2.9, 3.8, 4.1, 3.7, 2.7 and 1.1 cm2 respectively.
Find the volume by Simpson’s rule.
Determine the mass if the density of brass is 8.4 g/cm3 .
25. On a certain production machine, the cost C of manufacturing N articles is given
by C = a + bN where a and b are constants.
If it costs £8.50 for 300 articles, and £7.50 for 200.
(a) produce an equation to give the cost in pounds to manufacture any number of
articles,
Selection of Examination Standard Questions • 403

(b) find the cost to make 150,


(c) what is the minimum cost of running the machine?
26. Using integral calculus, determine the area bounded by the curve y = sin (x) +
cos (x)
for values of x between 30◦ and 60◦ .
2
27. A piece of wire 1 m long is cut into two pieces and each is bent into the form of
a square. Using differential calculus, determine the least possible sum of the two
areas.
28. A flat mild steel plate of elliptical shape measuring 420 mm across the major axis
and 370 mm across the minor axis is 45 mm thick. There is a groove around the
whole of the perimeter, 13 mm wide by 13 mm deep. Taking the density of steel as
7.86 g/cm3 , find the mass of the plate.
29. Two chords, 60 and 80 mm long respectively, are drawn parallel to each other inside
a circle. If the chords are 10 mm apart, find the radius of the circle.
30. A railway cutting, in the form of a trapezium, is 10 m wide at the bottom, 15 m
wide at the top, and 14 m deep. The cutting sweeps around on a circular arc, the
radius of the arc to the centre of the cutting is 805 m, and the angle subtended is
6◦ . Calculate the volume of earth removed, in m3 , in the making of the cutting.
31. In a triangle ABC the lengths of the sides AB and AC are 50 and 30 mm respectively
and the angle BAC is 20◦ . Calculate the length of the other side, the two remaining
angles and the area.
32. Draw graphs of the following simultaneous equations between the limits x = −2
and x = +3 and find the values of x and y,

x 2 − y = 1 and 2y − 3x = 4
78 7
33. Determine the area contained between the curves p = , p = 1.15 and the
V 1.2 V
ordinates V = 2 and V = 5.
34. A body moves S metres in t seconds where s = t3 − 6t2 + 9t + 3.
ds dv
Given that velocity v = and acceleration a = , calculate
dt dt
(i) the values of t at which the body is stationary
(ii) the values of the acceleration corresponding to the values of t determined
in (i).
x+2 x−1 7
35. Find the value of x in the equation: − =
x − 1 3x − 4 2
36. Calculate the area of the largest isosceles triangular plate with base angles of 75◦
that can be cut out of a circular plate having an area of 804.2 cm2 .
37. (a) Differentiate the following equations:
404 • Mathematics

 
(i) y = Ei − Ri2 with respect to i
 
c  
(ii) y = · V 1−n − p1−n with respect to v
1−n
(b) Evaluate the following integrals:
1
(i) 12x 7 dx
0
r
3  
(ii) π r2 − x 2 dx
2π · r3
0
38. Find the values of θ between 0◦ and 360◦ which will satisfy the equation:

2 sin2 (θ ) − 2 cos2 (θ ) − 2 = 0

2 × area of triangle
39. The radius of a circle inscribed in a triangle is given by: R =
perimeter of triangle
Calculate the radius of the circle inscribed in a triangle of sides 50, 60 and 70 mm
respectively.
40. Determine the value of x for which expression y = 4x 3 − 3x 2 − 18x is a maximum
or minimum, and calculate the corresponding values of y stating whether it is a
maximum or minimum.
41. A solid consists of right circular cone of 100 mm base diameter mounted concen-
trically on the flat surface of a hemisphere of equal diameter, the perpendicular
height of the conical part being also 100 mm.
Calculate the volume of the solid in cm3 and the curved surface area in cm2 .
If another similar solid was made with all linear dimensions double those of the
original what would be its volume and curved surface area?
42. Plot the graph of the equation y = x 2 − 8x + 7 between x = 0 and x = 9, using
intervals of unity. From the graph solve for x in each of the following equations:
(i) x 2 − 7x + 6 = 0
(ii) x 2 − 9x + 8 = 0
Suggested scales: x axis 2 cm = 1; y axis 2 cm = 5.
43. For a certain ship at deadweight displacement, an expression connecting the
 
power of the main engines and the speed of the ship is given by: P = V a + bV 2
where P = power of the engines in KW, V = speed of the ship in knots, a and b are
constants.
If the powers are 6024 and 13408 KW when the speeds are 12 and 16 knots
respectively, calculate the probable power when the speed is 14 knots.
44. Obtain the second differential coefficients of:
Selection of Examination Standard Questions • 405

x3 3
(i) y = − 3 + 3x 2 − 2x
3 x
(ii) y = 3 sin (x) + 4 cos (x)
45. A buoy is constructed by welding the base of a hollow cone to the flat base rim
of a hollow hemisphere, the common diameter of their bases being 2.44 m. The
greatest length of the buoy is 4.27 m. Calculate
(i) the curved surface area of the cone,
(ii) the angle subtended at the centre of the sector of a circle when the cone is
developed,
(iii) the curved surface area of the hemisphere,
(iv) the external volume of the buoy.
46. Solve for a, b, c and d in the following four simultaneous equations:

3a + 7b + 2c = 25
2a + 6b + c = 18
a + b + c + d = 14
2a + d = 9

47. A lens is flat on one side and convex on the other side, the convex surface being
part of a sphere. The diameter of the flat surface is 60 mm. The maximum thickness
of the lens is 4 mm and the minimum thickness is nil. Calculate the curved surface
area of the lens in cm2 .
dy
48. (a) If = cos (x) − 2 sin (x) determine an expression for y in terms of x, given that
dx
y = 3 when x = 0.
x2
(b) The area bounded by the curve y = , the line x = 4 and the x-axis between
4
0 and 4 is rotated once around the x-axis. Use integral calculus to determine
the volume of the solid generated.
49. Transpose the terms in the following equation to make k the subject:
⎡ ⎤1/2
⎢ w · u2 ⎥
P=⎢



k+a D
g· +
k (2a + k) + a2 Et
50. (a) If x varies directly as Z and inversely as y2 , calculate the percentage change in
x when Z is increased by 12% and y is decreased by 20%.
(b) The time for one beat of a pendulum varies as the square root of its length. If
a pendulum 174.4 mm long makes 105 beats in 44 s, calculate the length of a
similar pendulum to beat exactly once every second.
406 • Mathematics

51. Solve the following simultaneous equations to give the smallest values of the
angles x and y:

3 cos (x) + 4 sin (y) = 2 2 − 1. 5

5 cos (x) − 2. sin (y) = −3. 5

52. Show the height of a frustum of a right circular cone is one-third the height of the
complete cone when the volume of the frustum is 19/27 that of the complete cone.
53. (a) Determine the area bounded by the curve y = 2 sin (θ ) and the θ axis between
π 5π
the ordinates θ = and θ = .
6 6
(b) Evaluate:

(i) 3 cos (θ ) dθ
π/2

2
 
(ii) x 2 − 6x dx
1
3
 
(iii) x 4 − 7x + 8 dx
1
f1
54. Express V in terms of v, f1 and f2 in the following expression where K = :
f2
2
V +v
K=
V −v
Find the values of V when f1 = 12, f2 = 18 and v = 20.
55. Plot the graph y = x 2 + 2x − 8 between the limits x = −4 and x = +2.
If the area bounded by this curve and the x-axis is rotated through one revolution
about the x-axis, use Simpson’s rule to calculate the volume generated.
56. The total area of metal A needed to make a box of stated volume is given by the
16000
expression: A = x 2 + where x is the side of the square base.
x
Use calculus to find:
(i) the value of x for which A is a minimum,
(ii) the minimum value of A.
57. Three steel ball bearings, each 2 cm diameter, are placed inside a cylinder 6 cm
diameter, equally spaced around the bottom, then another ball bearing 4 cm
diameter is placed on top of these.
Calculate the volume of water, in ml, required to pour into the cylinder so that the
top ball is just submerged.
Selection of Examination Standard Questions • 407

If all linear dimensions were half of those given above, what would be the volume
of water required?
58. The distance from the centre to a corner of a regular pentagon is 3 cm, find the area
of the pentagon. If a piece is sliced off by a straight cut from one corner at an angle
of 30◦ to a side, find the ratio of the areas of the two pieces so formed.
59. (a) Find the angles between 0◦ and 360◦ that will satisfy the equation:

2 sin2 (θ ) − cos (θ ) = 1
(b) Find the angles between 0◦ and 360◦ that will satisfy the equation:

3 cos (2θ) + 2 cos (θ ) = 0

60. The resistance R (newton) to motion of a body and its velocity V (m/s) is tabulated
below:
V 5 10 15 20 25
R 10 25 50 85 130

Show that these values are connected by the law R = K + cV 2 , find the values of
the constants K and c and calculate the probable resistance to motion when the
velocity is 40 m/s.
61. Transpose the following to express k in terms of p and V:
k (p + V)
p=V+
k+p+V
Find the value of k when p = 9. 2 and V = 0. 042.
62. (a) Calculate the area of a trapezium inscribed in a semi-circle of radius 10 cm if
the length of each non-parallel side of the trapezium is equal to the radius of
the semi-circle.
(b) If this trapezium is rotated through one complete revolution about an axis
passing through the centre of, and perpendicular to, its long side, calculate
the volume of the solid frustum of a cone thus generated.
63. (a) Find the value of a, b and w in the simultaneous equations:

4a + 3b + 2w = 2
2a + 8b + 3w = 11
3a + 5b − 6w = −36

(b) Find the values of x and y in the simultaneous equations:

x 2 + y2 = 18. 5
x−y =1
408 • Mathematics

64. (a) Prove 1 + tan2 (θ ) = cos2 (θ ) − sin2 (θ )


tan (θ ) 1
(b) Prove sin (θ ) + sin (θ ) · cot2 (θ ) + =
cos (θ ) sin (θ ) · cos2 (θ )
65. Draw the graphs of the following equations between the limits x = −3 and x = 9
and find the area of the triangle enclosed by the graphs:

4y − x − 20 = 0
y−x+1=0
2y + x − 7 = 0

66. A hexagon of unit side is to have its area reduced by 20% by cutting it parallel to
one side. Calculate the thickness of the piece cut off and the distance from the
centre of the original hexagon to the corner of the new face.
67. Estimate the area between the curve y = e−2x and the x axis between the
limits x = 0 and x = +2 using Simpson’s rule, with eleven ordinates and hence
determine the length of the mean ordinate for this section of the curve.
68. The metal from the melting down of a solid cone 60 cm base diameter and 50 cm
slant height is to be cast into the form of a hollow sphere of 1.5 cm uniform
thickness. Calculate the outer and inner diameters.
69. Transpose for h in the following expression:
c h2
a=
2s d−h
Use the result to evaluate h when a = 1. 5, c = 15, s = 16 and d = 17.
70. Three points A, B and C lie on a horizontal ground. C is due North of A, and B is due
East of a point W which lies on the line joining A and C.
A vertical mast stands on point B and the angle of elevation to the top of the mast
from A is 20◦ , AB = 60 m, BC = 32 m, AC = 68 m.
(a) Prove that the angle at B is a right angle.
(b) Find the angles at A and C.
(c) Calculate the height of the mast.
(d) Find the angle of elevation from point W to the top of the mast.
71. Draw the graph y = x 2 + 1 between the limits x = −2 and x = +3 and use this to
solve the equation x 2 − x = 2.
r2
72. (a) Prove that the area of a segment of a circle is equal to (θ − sin (θ )), where
2
r = radius of circle and θ = angle subtended.
(b) A horizontal pipe is more than half full of water. The breadth of the water level
is 60 cm and the height from water-level to crown of pipe is 15 cm.
Selection of Examination Standard Questions • 409

Calculate
(i) the diameter of the pipe,
(ii) the cross-sectional area of the water,
(iii) litres of water contained over a length of 2 m.
W2 Z3
73. (a) Find the values of WZ, W 2 Z, and when W = x a+b and Z = x a−b .
Z W
(b) Find the values of x and y in the simultaneous linear equations:

4x + y = 10
2 7
+ =3
x y
74. In each of the following formulae, transpose and re-write in terms of k:

2ghDk2
(i) Q =  
d s2 − k2

1 1
(ii) F =
2π Lk
π (k − b)
(iii) L = (k + b) + + 2c
2 4c

75. The equation 7x 2 −28 + 7y2 = 0 represents the law of a circle. Use integral calculus
to determine the volume of revolution generated by the middle part of the circle
about its x axis between the limits x = +1 and x = −1.
Take each unit of x and y as representing 1 cm.
76. Show that, if x is numerically less than 1,

1 1 1×3 2 1×3×5 3
√ ≈1+ x+ x + x
1−x 2 2×4 2×4×6
Evaluate such an approximation for x = 0. 1.
77. Due to bad weather a ship’s speed is decreased by 20% for 8 h. The ship then runs
at normal speed for 10 h. In order to catch a tide the speed is then increased for 6 h.
Over the 24-h period the fuel consumption was found to be unchanged from the
normal consumption.
Given that the fuel consumption C is directly proportional to the cube of the ship’s
speed V, determine the percentage increase in the ship’s speed in the last 6 h.

x n+4 x 6n−7
78. (a) Simplify the expression: √ .
x 3n x 2n−1
(b) Find the value of x if: 10x = 20x−4 .
410 • Mathematics

79. (i) Determine the square root of 64−1 · a8 b12 .


(ii) Determine the cube root of 216a−12 · b0.6 .
(iii) Solve for a, b and c in the following simultaneous equations:

a−b+c= 3
2a + 8b − 3c = 56
7a + 3b + 3c = 69

80. The co-ordinates for two points on each of two straight line graphs are as follows:
first graph: (x1 , y1 ) = (−4, 6) and (x2 , y2 ) = (2, −10)
second graph: (x3 , y3 ) = (−5, −10) and (x4 , y4 ) = (2, 4)
(a) Draw the two straight line graphs using the same axes.
(b) Using the graphs determine:
(i) the algebraic expression for each straight line in the form y = ax + b,
(ii) the values of x and y which satisfy both expressions when considered as
simultaneous equations.

81. A tower 50 m high stands on the top of a hill which has a 20◦ incline. The angle
of depression from the horizontal from the top of the tower to a marker A on the
hill is 55◦ . At a point further down the slope in the same vertical plane is another
marker B at an angle of depression of 40◦ from the horizontal from the top of the
tower. Determine the distance between A and B.
82. A ship’s fuel consumption varies inversely as the calorific value of the fuel and varies
directly as the (ships speed in knots)3 .
The ship burns 60 tonne/day of fuel of calorific value 40 MJ/kg when travelling
at 20 knots. Determine the daily consumption when using fuel of calorific value
42 MJ/kg and steaming at 16 knots.
83. By plotting two graphs for unit values between x = 2 and x = 10 on the same axis
5
solve the equation: 0. 5 log (x) = .
2x
Suggested scales x axis 1 unit = 2 cm; y axis 1 unit = 10 cm.
84. Determine the area enclosed between the curves y = x 2 + 1 and y = 7 − x.
85. The sketch shows a diesel engine unit which has a stroke of 300 mm and a con-
necting rod AB of length 550 mm. Between the vertical and the position of the
connecting rod is an angle CAB. Twice during the downward movement of the
piston angle CAB equals 10◦ . Determine the piston travel between these piston
positions.
Selection of Examination Standard Questions • 411

x3 x2
86. For the function y = −
4 2
(i) show that there are two points on the curve where the gradient is 0;
(ii) determine the values of x and y at the point where the curve is a minimum.

π/2
87. (a) Find 4 cos (x) dx.
π/9

(b) A curve passes through the point (3, −2)and has a gradient of 2x 2 − 2.
Determine the equation of the curve.
88. (a) Determine the mass of the frustum of a cone made of wood of density
0.9 g/cm3 given that the diameters of the two parallel surfaces are 20 and 40
cm and the vertical height of the frustum is 16 cm.
(b) The frustum was made by removing a smaller top cone base diameter 20 cm
from the large original cone, base diameter 40 cm.
Determine the ratio by mass of the smaller cone which was removed to the
mass of the remaining frustum.
89. A glass vessel has a uniform circular bore and a hemispherical end of the same
radius. With the hemispherical end at the bottom, the container is filled to 0.7 of its
overall height. When the container is inverted the level of the liquid drops down to
0.6 of the vessel’s height and is below the hemispherical section.
The vessel contains 100 ml of water. Determine:

(i) the bore;


(ii) the overall height.

90. Determine the volume of the solid of revolution formed when the shaded area
shown is rotated about the x axis.
412 • Mathematics

15
y

10 5
y = x2 +
2

x
1  2 3 4

Note: Volume of revolution =π y2 dx.


91. The LCR circuit shown below consists of a resistor R, a capacitor C and an inductor
L connected in series together with a voltage source v(t).
Prior to closing the switch at time t = 0 both the charge on the capacitor and the
resulting current in the circuit are 0.
When Kirchoff’s laws are applied to the circuit the following second-order differen-
tial equation is obtained:
d2 q dq 1
L· 2
+R· + · q = v(t)
dt dt C
Determine the charge q(t) on the capacitor and the resultant current i(t) in the
circuit at time t given that R = 160 ohms, L = 1 H, C = 10−4 F and v(t) = 20 v.
(Current equals rate of change of charge with respect to time.)

R L
C
t=0

v(t )
i(t )

92. A pair of simultaneous equations is given by:


dx dy
2 + = 5et
dt dt
with the initial conditions x = 0 and y = 0 when t = 0.
dy dx
−3 =5
dt dt
Use Laplace Transforms to show that x = et − t − 1.
Without finding an expression for y, determine y.
Selection of Examination Standard Questions • 413

93. Show that the simultaneous differential equations


dx
+ x + 2y = et
dt
dy
− 2x − 3y = −2et
dt
give the Laplace transforms of functions x(t) and y(t) as being
s+1 −2s
x= and y=
(s − 1) 3
(s − 1)3
given that x = y = 0 at t = 0, that is, x0 = 0 and y0 = 0.
Hence, find the original functions x(t) and y(t).
94. The function f (x) is defined by:

f (x) = 10x 0<x<1


f (x) = 20 − 10x 1<x<2

Show that the Fourier Sine series for f (x) is given by:
 πx  1    
80 3πx 1 5πx
f (x) = 2 · sin − sin + sin − ···
π 2 9 2 25 2
95. Sketch the graph of the function f (x) defined by

x, 0 < x < π
f (x) = ⎣ such that f (x) = f (x + 2π)
0, π < x < 2π

for values of x between −2π and 2π.


Construct the Fourier series for this function.
π2 1 1 1
Show that = 1 + 2 + 2 + 2 Λ.
8 3 5 7

Suggested Solutions to Examination


Questions
2
1. (a) y = x 4 − + 7x − 9 = x 4 − 2x −2 + 7x − 9
x2
dy d2 y 12
= 4x 3 + 4x −3 + 7 ⇒ 2 = 12x 2 − 12x −4 = 12x 2 − 4
dx dx x
414 • Mathematics

dy 1
(b) y = ln (x) + sin (x) ⇒ = + cos (x) = x −1 + cos (x)
dx x
d2 y 1
2
= −x −2 − sin (x) = − 2 − sin (x)
dx x
 2  2  
1 1 1 1 1
2. (a) x + − x− =4 ⇒ x +x· + ·x+ 2
2
x x  x x x
1 1 1
− x −x· − ·x+ 2 =4
2
x x x
   
1 1 1 1
⇒ x + 2 + 2 − x − 2 + 2 = 4 ⇒ x2 + 2 + 2 − x2 + 2 − 2 = 4
2 2
x x x x

Therefore 4 = 4.
and so as x is not present in the final ‘solution’ it can take any value.
 
1 2 1 1
(b) x + = 9 ⇒ x2 + 2 + 2 = 9 ⇒ x2 + 2 = 7
x x x
1 1
Squaring both sides gives x + 2 + 4 = 49 ⇒ x 4 + 4 = 47
4
  x x
1 2 1 √
x+ =9⇒x+ = 9
x x
1
⇒x+ = 3 ⇒ x 2 + 1 = 3x
x
so x 2 − 3x + 1 = 0
Solving this quadratic gives x = 2. 618 or x = 0. 382.
These values were obtained using the positive square root of 9. If the negative
value is used the quadratic equation becomes x 2 + 3x + 1 = 0 giving the
solutions x = −2. 618 and x = −0. 382.
3.

25 mm
α 50 mm

2. 5
Working in cm: cos (α) = = 0. 5 ⇒ α = 60◦
5
Selection of Examination Standard Questions • 415

240 4π
Therefore θ = 360 − (2 × 60) = 240◦ ⇒ θ = × 2π = radian
360 3
sin (θ ) = −0. 866

So area of segment of angle θ = 0. 5r2 (θ − sin (θ ))


 

Area = 0. 5 × 52 − (−0. 866) = 63. 2 cm2
3

240 240
Wetted perimeter of pipe surface = × πd = × π × 10 = 20. 94 cm
360 360
63. 2
Therefore hydraulic mean depth = = 3. 017 cm or 30. 17 mm
20. 94
V2
4. Work done = p dV
V1

6.5  
120 6. 5
W= dV = 120 [ln (V)]6.5
0.5 = 120 (ln (6. 5) − ln (0. 5)) = 120 · ln
V 0. 5
0.5

= 120 · ln (13)

Therefore W = 307. 8 Nm.


5. (a) Suppose the added amount of water is x litres.
The original 1 litre of mixture has become (1 + x) litres.
The original amount of alcohol was 18% of 1 litre and this is to be 15% of the
new mixture.

0.68 + x Water
Water
1 litre 0.68

Alcohol 0.18 Alcohol


1+x
Solids 0.14 Solids
litres

15
Therefore (1 + x) = 0. 18
100
15 (1 + x) = 0. 18 × 100
15 + 15x = 18
416 • Mathematics

15x = 3
3
x=
15
So x = 0. 2 litre (or 200 ml).
(b) Percentage analysis of new mixture:
0. 68 + 0. 2
Water × 100 = 73. 3%
1 + 0. 2
Solids 100 − 15 − 73. 3 = 11. 6%
Alcohol 15%
Cx 3 dy 3Cx 2
6. (a) y = Rgx − ⇒ = Rg −
64L dx 64L
b dy 2b
(b) y = ax 3 + 2
⇒ y = ax 3 + bx −2 ⇒ = 3ax 2 − 2bx −3 = 3ax 2 − 3
x dx x
2  2    
 2.6
 k3.6 23.6 1
(c) k − 1 dk = −k = −2 − −1
3. 6 1 3. 6 3. 6
1
= (1. 368‘) − (−0. 722) = 2. 090

 2  2v 3
(d) 2v − 4 v dv = − 4v + C
3
7. 20
y

15

10

x
1 2 3

dy
y = 11 + 7x − 2x 2 ⇒ = 7 − 4x
dt
dy 7
When = 0, 7 − 4x − 0 ⇒ x = = 1. 75
dx   4 
7 7 7 2
When x = , y = 11 + 7 −2 = 17. 125
4 4 4
d2 y
From the sketch it is obvious that this is a maximum, but = −4 which implies
dx 2
a maximum as the value is negative.
Selection of Examination Standard Questions • 417

8.

45 mm

52.5 mm

40 mm

60 mm

1
Radius of cone base = × 52. 5 = 26. 25 mm
2

Slant height of cone = 452 + 26. 252 = 52. 1 mm
Curved surface area = πrL = π × 26. 25 × 52. 1 = 4296. 5 mm2

π 2 
Area of annulus between top of cylinder and base of cone = D − d2
4
π 2 
Area of annulus = 60 − 52. 52 = 662. 7 mm2
4

Curved surface area of cylinder = πDh = π × 60 × 40 = 7539. 8 mm2


Total surface area exposed = 4296. 5 + 662. 7 + 7539. 8 = 12499 mm2
So the exposed area is approximately 12500 mm2 or 125 cm2
ds dv
9. (a) s = t3 − 5. 5t2 − 4t + 68. 5 ⇒ v = = 3t2 − 11t − 4 ⇒ a = = 6t − 11
dt dt

When the velocity is 0, 3t2 − 11t − 4 = 0 ⇒ (3t + 1) (t − 4) = 0


Therefore t = 4 s since the negative value is nonsensical.
At this time s = (4)3 −5. 5 ×(4)2 −4 ×4 + 68. 5 = 64 −88 −16 + 68. 5 = 28. 5 m
and a = 6 × 4 − 11 = 13 m/s2 .
 
 √   1/2  3. x 3/2 3. x 3
(b) (i) 3 x − 3x 2 dx = 3. x − 3x 2 dx = − +C
√ 3/2 3
= 2 x3 − x3 + C
   
2  −3  2x −2 1
(ii) + 2 dx = 2x + 2 dx = + 2x + C = 2x − 2 + C
x3 −2 x
418 • Mathematics

10. A

18 m

C
24 m
B D E
24 m

(i) AB = 18, BD = 12, DC = 12


By Pythagoras’ Theorem: (AD)2 = (AB)2 − (BD)2 = 182 − 122 = 180
Therefore AD = 13. 42 m
Similarly (AC)2 = (AD)2 − (DC)2 = 180 − 122 = 36
Therefore AC = 6 m
1
(ii) Area of one sloping triangular flat surface = × BE × AD
2
1
Total surface area of roof = 4 × × 24 × 13. 42 = 644. 16 m2
2
11. (a)
 3  
2x − 3x 2 + 2x + 4 x 2 − 3x + 5
= 2x 5 − 3x 4 + 2x 3 + 4x 2 − 6x 4 + 9x 3 − 6x 2 − 12x + 10x 3 − 15x 2 + 10x + 20
= 2x 5 − 9x 4 + 21x 3 − 17x 2 − 2x + 20

(b)

 3x − 2x + 5
2

x + 1 3x + x 2 + 3x + 5
3

3x 3 + 3x 2
−2x 2 + 3x + 5
−2x 2 − 2x
5x + 5
5x + 5
0
 2 3  2
(c) 2a b × 3ab2 = 8a6 b3 × 9a2 b4 = 72a8 b7
 3
3ab2 27a3 b6 27
(d)  2 = 3 9 = 3
ab 3 a b b
Selection of Examination Standard Questions • 419

sin (θ )
12. tan (θ ) = cos (θ ) ⇒ = cos (θ ) ⇒ sin (θ ) = cos2 (θ )
cos (θ )
However, cos2 (θ ) = 1 − sin2 (θ ) so sin (θ ) = 1 − sin2 (θ ), that is, sin2 (θ ) + sin (θ ) −
1=0
Solving this quadratic equation gives sin (θ ) = 0. 6180 or −1. 6180.
The second value is an impossibility and so θ = sin−1 (0. 618) = 38◦ 10 and
141◦ 50 .
dy 1
13. (a) (i) y = 4 cos (x) + ln (x) ⇒ = −4 sin (x) + = −4 sin (x) + x −1
dx x
d2 y 1
So 2 = −4 cos (x) − x −2 = −4 cos (x) − 2
dx x
4 dy
(ii) y = 3x 3 + 2
+ 6x − 2 = 3x 3 + 4x −2 + 6x − 2 ⇒ = 9x 2 − 8x −3 + 6
x dx
d2 y 24
So 2 = 18x + 24x −4 = 18x + 4
dx x
16000
(b) T = 1700 + + 0. 6x 2 = 1700 + 16000x −2 + 0. 6x 2
x2
dT 32000
Therefore = −32000x −3 + 1. 2x = − 3 + 1. 2x = 0 for max/min
dx x
32000 32000
So 1. 2x = ⇒x =4 = 26666. 6
x3 1. 2

Therefore x = ± 4 26666. 6 = ±12. 78
d2 T 96000 96000
Now 2
= 96000x −4 + 1. 2 = 4
+ 1. 2 = + 1. 2 = 4. 8 which is
dx x 2666. 6
positive.
Therefore the turning point is a minimum.
16000
When x = 12. 78, T = 1700 + + 0. 6 × 12. 782 = 1896
12. 782

10000 y

8000

6000

4000

2000

x
–30 –20 –10 10 20 30

A plot of the function showing positive and negative symmetry.


420 • Mathematics

14. 10 y

x
–2 2 4 6 8 10

–5

–10

–15

–20

The intercept is −10 and the slope is 2 so the equation is y = 2x − 10.


150
15. Area of sector = × π × 10. 22 = 136. 2 cm2
360
150
Length of arc = × 2π × 10. 2 = 27. 7 cm
360
This is the circumference of the cone base, which has a diameter d, say.
Therefore πd = 27. 7 and so d = 8. 5 cm = 85 mm
The slant height of the cone is equal to the radius of the sector = 10. 2 cm
1
Radius of cone base = × 8. 5 = 4. 25 cm
2 √
So the perpendicular height = 10. 22 − 4. 252 = 9. 27 cm = 92. 7 mm
16.

19a + 27b = 43 (i)


7a + 4b = −2 (ii)

Multiplying (i) by 7 and (ii) by 19 gives

133a + 189b = 301 (iii)


133a + 76b = −38 (iv)

Subtracting (iv) from (iii) gives 113b = 339 so b = 3


Substituting b = 3 into (ii) gives 7a + 12 = −2 so 7a = −14 and a = −2
Therefore

10y + 12 = −2x
−6 + x = y
Selection of Examination Standard Questions • 421

Rearranging gives

2x + 10y = −12 2x + 10y = −12


or subtracting gives
x −y =6 2x − 2y = 12

12y = −24 so y = −2
So 2x − 20 = −12 giving 2x = 8 so x = 4
The solution is x = 4, y = −2
πDd π × 0. 3 × 0. 2
17. Area of elliptical section = = = 0. 015π m2
4 4
Centre of ellipse from centre of wheel = 1. 8 − 0. 15 = 1. 65 m
By Pappus’ Theorem, in one revolution of the elliptical sectional area about the
centre of the wheel,
Volume swept out = area × distance moved by the centroid
Volume of the rim = 0. 015π × 2π × 1. 65 = 0. 4885 m3
Mass = V × ρ = 0. 4885 × 7. 21 × 103 = 3522. 1 kg

x3 x2 dy 3x 2 2x 3x 2 d2 y 6x
18. y = − ⇒ = − = −x ⇒ 2 = −1
4 2 dx 4 2 4 dx 4
At a turning point, the gradient = 0; so
 
3x 2 3x
−x =0⇒x −1 =0
4 4

4
Therefore, either x = 0 or x =
3
d2 y 6 (0)
When x = 0, 2
= − 1 = −1. Therefore a maximum
dx 4
4 d2 y 6 (4/3) 8
When x = , 2
= −1= −1=1
3 dx 4 4
(4/3)3 (4/3)2 8
Therefore a minimum when y = − =−
4 2 27
 
19. (i) 2x 2 − 2 = 2 x 2 − 1 = 2 (x + 1) (x − 1)
(ii) 7x 2 + 9xy − 10y2 = (7x − 5y) (x + 2y)
 
(iii) 14x 3 y2 + 22x 2 y3 − 12xy4 = 2xy2 2x 2 + 11xy − 6y2 = 2xy2 (2x − y) (x + 6y)
(iv) 3x 3 + 8x 2 y − xy2 − 10y3 divided by (x − y) leaves 3x 2 + 11xy + 10y2
3x 2 + 11xy + 10y2 = (3x + 5y) (x + 2y) so the full factorisation is:
3x 3 + 8x 2 y − xy2 − 10y3 = (3x + 5y) (x + 2y) (x − y)
422 • Mathematics

5  5
  10x 3 x 2
20. A1 = 10x + x + 4 dx =
2
+ + 4x
3 2 1
1   
10 (5) 3
52 10 (1)3
12
= + +4×5 − + +4
3 2 3 2
   
1250 25 10 1
So A1 = + + 20 − + + 4 = 441. 3
3 2 3 2

5    5
x2 1 1 x3 x
A2 = − − dx = − ln (x) −
2 x 2 6 2 1
1
   
53 5 1 1
− ln (5) −
= − − ln (1) −
6 2 6 2
   
125 5 1 1
So A2 = − ln (5) − − − ln (1) − = 17. 06
6 2 6 2
Therefore the area between the curves is 424.27 unit2 .
π/4 π 
π/4
21. (a) Area = 2 cos (x) dx = [2 sin (x)]0 = 2 sin − 2 sin (0) = 1. 414
4
0

 2 
(b) y = 3x − 4 dx = x 3 − 4x + C passes through the point (1, −2)
Therefore −2 = 13 − 4 × 1 + C ⇒ −2 = 1 − 4 + C ⇒ C = 1
The equation is y = x 3 − 4x + 1
   2    2
5 2 5 5 2 5
22. (i) x 2 + 5x = 84 ⇒ x + − = 84 ⇒ x + = 84 +
2 2 2 2
 2  
5 361 5 19 5 19 14 24
⇒ x+ = ⇒ x+ =± ⇒x=− ± = and −
2 4 2 2 2 2 2 2

Therefore the two solutions are x = 7 and x = −12


(ii) 2x 2 − 3ax − bx = 0 ⇒ 2x 2 − x (3a + b) = 0 ⇒ x · [2x − (3a + b)] = 0
3a + b
Therefore either x = 0 or 2x − (3a + b) = 0 ⇒ x =
2
1 −1 dy
23. (i) y = x − 5x + 7 − ⇒ y = x − 5x + 7 − x ⇒
2 2 = 2x − 5 + x −2
x dx
d2 y 2
⇒ = 2 − 2x −3 = 2 − 3
dx 2 x
dy d2 y
(ii) y = 2 cos (x) − 3 sin (x) ⇒ = −2 sin (x) − 3 cos (x) ⇒ 2
dx dx
= −2 cos (x) + 3 sin (x)
Selection of Examination Standard Questions • 423

24.
Cross-section area (cm2 ) Multiplier Product for area
0 1 0
2.9 4 11.6
3.8 2 7.6
4.1 4 16.4
3.7 2 7.4
2.7 4 10.8
1.1 1 1.1
Total 54.9
4
Total length of 8 cm is divided into 6 equal sections. Therefore width h = cm
3
h 4
× 54. 9
Volume = × total = 3 = 24. 4 cm3
3 3
Mass = 24. 4 × 8. 4 = 204. 96 g
25. C = a + bN
Therefore
a + 300b = 8. 5
and subtracting gives 100b = 1 so b = 0. 01
a + 200b = 7. 5
Substituting into the first equation gives a + 3 = 8. 5 and so a = 5. 5
(a) Therefore C = 5. 5 + 0. 01N
(b) When N = 150, C = 5. 5 + 0. 01 × 150 = £7. 00
(c) Minimum cost is achieved when the machine is running at idle, that is,
producing 0 articles.
Minimum cost is therefore £5.50.
π π
26. The angles must be measured as radians. So 30◦ = rad and 60◦ = rad
6 3
π/3   π/3
cos (x) sin (x)
Area = sin (x) + dx = − cos (x) +
2 2 π/6
π/6
⎛ π  ⎞ ⎛ π ⎞
 π  sin  π  sin
⎜ 3 ⎟ ⎜ 6 ⎟
Area = ⎝− cos + ⎠ − ⎝− cos + ⎠
3 2 6 2
   
0. 866 0. 5
= −0. 5 + − −0. 866 +
2 2
Therefore the area = 0.549 unit2 .
424 • Mathematics

27. Suppose the length of the first piece of wire is x cm, then the length of the second
is (100 − x) cm.
 x 2  
100 − x 2
The area of the first square is and the area of the second is .
4 4
Total area
 x 2  100 − x 2 x2 10000 − 200x + x 2
A= + = +
4 4 16 16
10000 − 200x + 2x 2
⇒A=
16
dA −200 + 4x
= and this equals 0 at a max/min.
dx 16
−200 + 4x 200
Therefore 0 = ⇒ 0 = −200 + 4x ⇒ x = = 50 cm
16 4
d2 A 4
2
= > 0 so a minimum
dx 16
10000 − (200 × 50) + 2 × 502
When x = 50 cm, the area =
16
10000 − 10000 + 5000
=
16
5000
So minimum area = = 312. 5 cm2
16
1 2
If the calculation was done in units of a metre, the minimum area would be m
32
or 0.03125 m . 2

28. Without the groove:


πDd π × 42 × 37
Area of ellipse = = = 388. 5π cm2
4 4
Volume = 388. 5π × 4. 5 = 1748. 25π = 5492. 3 cm3
At the base of the groove:

Major axis = 42 − (2 × 1. 3) = 39. 4 cm


Minor axis = 37 − (2 × 1. 3) = 34. 4 cm
π × 39. 4 × 34. 4
Area of ellipse formed at the base of the groove = = 338. 84π cm2 .
4
The difference in the areas of the ellipses gives the ‘area’ of the groove and so the
volume of the groove is the difference in areas multiplied by the groove width.
Therefore, volume of groove = (388. 5π − 338. 84π) + 1. 3 = 49. 66π × 1. 3
= 202. 8 cm3 .
So the net volume of the grooved plate is 5492. 3 − 202. 8 = 5289. 5 cm3 .
With a density of 7.86 g/cm3 , the mass is 5289. 5 × 7. 86 = 41575 g or 41.575 kg.
Selection of Examination Standard Questions • 425

29.
3 cm
1 cm
4 cm

r
x r

Referring to the sketch, and working in cm, using Pythagoras’ Theorem:

r2 = x 2 + 42 ⇒ r2 = x 2 + 16 K K (i)

r2 = (x + 1)2 + 32 ⇒ r2 = x 2 + 2x + 10 K (ii)

Equating (i) and (ii) gives

x 2 + 16 = x 2 + 2x + 10 ⇒ 16 = 2x + 10 ⇒ 6 = 2x and so x = 3 cm

From (i): r2 = 32 + 16 ⇒ r2 = 25 ⇒ r = 25 = 5 cm or 50 mm
30.
15 m

14 m Radius = 805 m

10 m
 
15 + 10
Area of cross-section = average width × depth = × 14 = 175 m2
2
6 6
Length of arc = × Circumference = × 2π × 805 m
360 360
By Pappus’ Theorem,
Volume = Area of section × Distance moved by centroid
6
Therefore Volume = 175 × × 2π × 805 = 14752. 4 m3
360
31. a2 = b2 +c2 −2bc·cos (A) = 32 +52 −2×3×5×cos (20) = 9+25−28. 191 = 5. 809

Therefore a = 5. 809 = 2. 41 cm or 24.1 mm
a b 2. 41 3 3 × sin (20)
= ⇒ = ⇒ sin (B) = = 0. 42575
sin (A) sin (B) sin (20) sin (B) 2. 41
426 • Mathematics

Therefore B = sin−1 (0. 42575) = 25. 2◦ or 25◦ 12


 
So C = 180◦ − 20◦ + 25◦ 12 = 134◦ 48

b · c · sin (A) 3 × 5 × sin (20)


Area = = = 2. 565 cm2
2 2
Note: any combination of sides and angle could have been used for the area, for
a · b · sin (C)
example, . The reason for the combination used is that these rely
2
on values given by the question and not on calculated values (which may be
incorrect).
32. y

x
–2 –1 1 2 3

–2

The coordinates of the intersections of the graphs are: (−1. 14, 0. 3) and (2.64, 5.96).
So the solutions to the simultaneous equations are x = −1. 14, y = 0. 3 and
x = 2. 64, y = 5. 96
Note: the answers given are probably more accurate than those you would expect
to get from this generally inaccurate method of solving.
78 7
33. p = 1.2
= 78V −1.2 , p = 1.15 = 7V −1.15
v v
5  −0.2 5
V  
So A1 = 78 · V −1.2 dV = 78 · = −390 5−0.2 − 2−0.2
−0. 2 2
2
= −390 × −0. 1458 = 56. 85

5  5
−1.15 V −0.15  
A2 = 7·V dV = 7 · = −46. 6 5−0.15 − 2−0.15
−0. 15 2
2

= −46. 6 × −0. 1157 = 5. 40

So area between curves is 56. 85 − 5. 40 = 51. 45 units2 .


Selection of Examination Standard Questions • 427

ds dv
34. s = t3 − 6t2 + 9t + 3 ⇒ v = = 3t2 − 12t + 9 ⇒ a = = 6t − 12
dt dt
(i) When the body is stationary, 3t2 − 12t + 9 = 0 ⇒ t2 − 4t + 3 = 0
Factorising gives (x − 1) (x − 3) = 0 so v = 0 when t = 1 s and when
t = 3 s.
(ii) At these times the accelerations are 6 − 12 = −6 m/s2 and 18 − 12 = 6 m/s2 .

x+2 x−1 7 (x + 2) (3x − 4) − (x − 1) (x − 1) 7


35. − = ⇒ =
x − 1 3x − 4 2 (x − 1) (3x − 4) 2
So

2 [(x + 2) (3x − 4) − (x − 1) (x − 1)] = 7 (x − 1) (3x − 4)


 2     
2 3x − 4x + 6x − 8 − x 2 − 2x + 1 = 7 3x 2 − 4x − 3x + 4
   
2 3x 2 − 4x + 6x − 8 − x 2 + 2x − 1 = 7 3x 2 − 7x + 4
   
2 2x 2 + 4x − 9 = 7 3x 2 − 7x + 4
4x 2 + 8x − 18 = 21x 2 − 49x + 28
0 = 17x 2 − 57x + 46

This either does not factorise, or is too complicated to try to factorise so the
quadratic formula gives:

− (−57) ± 572 − 4 × 17 × 46
x=
2 × 17

57 ± 3249 − 3128
=
34

57 ± 121
=
34
57 ± 11
=
34
68 46
= or
34 34
x = 2 or 1. 353

804. 2 804. 2
36. Area of circle = π · r2 ⇒ π· r2 = 804. 2 ⇒ r2 = ⇒ r =
π π
= 16 cm.
The angle at the top of the triangle = 180 − (2 × 75) = 30◦ .
One of the circle theorems states that the angle at the centre of a circle is twice that
at the circumference. Therefore the angle at the centre is 60◦ .
428 • Mathematics

30°

r 30°

75°
C B

Referring to the sketch:


Angle COB = 30◦
CB = r · sin (30) = 16 × 0. 5 = 8 cm
Base of triangle = 2 × 8 = 16 cm
OB = r · cos (30) = 16 × 0. 866 = 13. 856 cm
AB = AO + OB = 16 + 13. 856 = 29. 856 cm
base × perpendicular height
Area of triangle =
2
16 × 29. 856
=
2
= 238. 85 cm2
  dy
37. (a) (i) y = Ei − Ri2 ⇒ = E − 2Ri
  di    
c   c c
(ii) y = · V 1−n − p1−n ⇒ y = V 1−n − p1−n
1−n 1−n 1−n
 
dy c dy c
= (1 − n) V 1−n−1 ⇒ = c · v −n = n
dv 1−n dv v
1  1    
12x 8 12 (1)8 12 (0)8 12
(b) (i) 12x 7 dx = = − = = 1. 5
8 0 8 8 8
0
(ii)
r   r
3   3 x3
π r2 − x 2 dx = π r 2
· x −
2π · r3 2π · r3 3 0
0
   
3 r3
= 2
π r ·r− − (0)
2π · r3 3
 
3 r3 3 π2r3
= π r 3
− = · =1
2π · r 3 3 2π · r 3 3
Selection of Examination Standard Questions • 429

38. 2 sin2 (θ ) − 2 cos2 (θ ) − 2 = 0 ⇒ sin2 (θ ) − cos2 (θ ) − 1 = 0 but cos2 (θ ) = 1 −


 
sin2 (θ ) sin2 (θ ) − 1 − sin2 (θ ) − 1 = 0 ⇒ 2 sin2 (θ ) − 2 = 0 ⇒ sin2 (θ ) = 1 ⇒
sin (θ ) = ±1
Therefore θ = 90◦ and 270◦
50 + 60 + 70
39. s = = 90 mm
2
 √
Area = 90 (90 − 50) (90 − 60) (90 − 70) = 2160000 = 1470 mm2
2 × Area of triangle
Perimeter = 180 mm so radius R =
Perimeter of triangle
2 × 1470
= = 16. 33 mm
180
dy
40. y = 4x 3 − 3x 2 − 18x ⇒ = 12x 2 − 6x − 18 = 0 at max/min
dx
Therefore 12x 2 − 6x − 18 = 0 ⇒ 2x 2 − x − 3 = 0 ⇒ (2x − 3) (x + 1) = 0
3
Therefore x = and −1
2
d2 y 3
= 24x − 6 = 30 when x = (Minimum), and − 30 when x = −1 (Maximum)
dx 2 2
3
When x = , y = −20. 25 and when x = −1, y = 11
2
π
Area of base × Perpendicular height × 102 × 10
41. Volume of cone = = 4
3 3
= 261. 8 cm3
1 πd3 1 π × 103
Volume of hemisphere = of = × = 261. 8 cm3
2 6 2 6
Therefore total volume = 261. 8 + 261. 8 = 523. 6 cm3

Curved surface area of cone = πr×Slant height = π ×5× 102 + 52 = 175. 6 cm2

Curved surface area of hemisphere = Surface area of circumscribing cylinder


= π × Diameter × Height
= π × 10 × 5 = 157. 1 cm2

Total surface area = 175.6 + 157.1 = 332.7 cm2


Volumes of similar objects vary as the cubes of their corresponding dimensions. As
the large object has dimensions twice that of the smaller, it has a volume that is
eight times that of the smaller.
Therefore volume = 523.6 × 8 = 4188.8 cm3
430 • Mathematics

Areas vary as the squares of the dimensions therefore the larger area is four times
that of the smaller.
Therefore area = 332. 7 × 4 = 1330. 8 cm2
42. (i) Method: Determine what has to added/subtracted from x 2 − 7x + 6 = 0 to
make x 2 − 8x + 7 = 0
x 2 − 7x + 6 = 0 ⇒ x 2 − 7x − x + 6 + 1 = −x + 1 ⇒ x 2 − 8x + 7 = 1 − x

The solutions to x 2 − 7x + 6 = 0 are the intersections of y = x 2 − 8x + 7 and


y =1−x
Therefore
x = 1, y = 0 and x = 6, y = −5
(ii) x 2 − 9x + 8 = 0 ⇒ x 2 − 9x + x + 8 − 1 = x − 1
The solutions to x 2 − 9x + 8 = 0 are the intersections of y = x 2 − 8x + 7 and
y =x−1
Therefore
x = 1, y = 0 and x = 8, y = 7
20 y

15

10

x
1 2 3 4 5 6 7 8 9

–5

–10
 
43. P = V a + bV 2 ⇒ P = aV + bV 3
The simultaneous equations formed from the data are:
16a + 4096b = 13408 (i)
12a + 1728b = 6024 (ii)

(i) × 3 and (ii) × 4 gives:


48a + 12288b = 40224 (iii)
48a + 6912b = 24096 (iv)

16128
(iii)–(iv) gives 5376b = 16128 so b = =3
5376
Selection of Examination Standard Questions • 431

13408 − 12288
Substituting into (i) gives 16a + 12288 = 13408 so a = = 70
16 
When the speed is 14 knots the probable power is P = 14 70 + 3 × 142 =
9212 kW.
x3 3 x3
44. (i) y = − 3 + 3x 2 − 2x ⇒ y = − 3x −3 + 3x 2 − 2x
3 x 3
dy d2 y 36
= x 2 + 9x −4 + 6x − 3 ⇒ 2 = 2x − 36x −5 + 6 = 2x − 5 + 6
dx dx x
dy d2 y
(ii) y = 3 sin (x) + 4 cos (x) ⇒ = 3 cos (x) − 4 sin (x) ⇒ 2
dx dx
= −3 sin (x) − 4 cos (x)
45.

4.27 m 2.44 m

1.22 m

Radius of the base = 1.22 m


Perpendicular height of cone = 4.27 − 1.22 = 3.05 m

Slant height = 1. 222 + 3. 052 = 3. 285 m
Curved surface area = π · r · l = π × 1. 22 × 3. 285 = 12. 6 m2
The slant height is the radius of the developed sector where the angle subtended at
Arc of sector π × 2. 44
the centre = × 360◦ = × 360◦ = 133. 7◦
Circumference of whole circle 2π × 3. 285

Curved S.A. of hemisphere = Curved S.A. of circumscribing half cylinder


πd2 π × 2. 442
= = = 9. 352 m2
2 2

Total volume = Volume of cone + Volume of hemisphere


1 1 πd3
= × area of base × height + ×
3 2 6
1 π × 2. 44 3
= × π × 1. 222 × 3. 05 +
3 12
= 4. 754 + 3. 803
= 8. 557 m3
432 • Mathematics

46.

3a + 7b + 2c = 25 (i)
2a + 6b + c = 18 (ii)
a + b + c + d = 14 (iii)
2a + d = 9 (iv)

From (iv) d = 9 − 2a
Substituting this into (iii) gives
a + b + c + 9 − 2a = 14 so −a + b + c = 5 (v)

Subtracting (v) from (ii) gives


2a + 6b + c = 18
−a + b + c = 5

3a + 5b = 13

Multiplying (v) by 2 and subtracting from (i) gives


3a + 7b + 2c = 25
−2a + 2b + 2c = 10

5a + 5b = 15

Subtracting the last two equations gives 2a = 2, so a = 1


Substituting into either of the last two equations gives b = 2
Substituting into equation (iv) gives d = 7
Substituting a and b into equation (v) gives c = 4
The solution is therefore a = 1, b = 2, c = 4 and d = 7.
47. Q C R

P A O B S

D
Selection of Examination Standard Questions • 433

AO × BO = CO × DO
30 × 30 = 4 × DO
DO = 225 mm

Curved surface area of lens = Curved surface area of slice of sphere where
diameter of sphere is 22.9 cm and thickness is 0.4 cm
= Curved surface area of slice PQRS of circumscribing
cylinder
= π × 22. 9 × 0. 4
= 28. 78 cm2

dy
48. (a) = cos (x) − 2 sin (x) ⇒ y = (cos (x) − 2 sin (x))dx = sin (x) + 2 cos (x) + C
dx
Therefore 3 = sin (0) + 2 cos (0) + C ⇒ C = 1
Therefore y = sin (x) + 2 cos (x) + 1
4 4 4  5 4
x x π   1024π
(b) Volume = π y dx = π
2
dx = π = × 45 − 0 =
16 80 0 80 80
0 0
= 12. 8π
Therefore volume is 40.212 unit3
49. Squaring both sides gives:
⎡ ⎤
⎢ w · u2 ⎥ k+a D w · u2
P2 = ⎢

⎥ ⇒g·
⎦ + =
k+a D k (2a + k) + a2 Et P2
g· +
k (2a + k) + a2 Et

k+a D w · u2 k+a w · u2 D
⇒ + = ⇒ = −
k (2a + k) + a2 Et P2 g k (2a + k) + a2 P2 g Et
w · u2 D w · u2 · Et − DP2 g
The right-hand side − = and k (2a + k)+a2 = (k + a)2
P2 g Et P2 gEt
k+a w · u2 D k+a w · u2 · Et − DP2 g
So = − ⇒ =
k (2a + k) + a2 P2 g Et (k + a)2 P2 gEt
1 w · u2 · Et − DP2 g P2 gEt
⇒ = ⇒ =k+a
k+a P2 gEt w · u2 · Et − DP2 g
P2 gEt
⇒ k= −a
w · u2
· Et − DP2 g
z xy2
50. (a) x α 2 ⇒ = Constant
y z
434 • Mathematics

Let each of the original values of x, y and z be unity (i.e. 1), then: new value of
y = 1 − 0. 2 = 0. 8.
New value of z = 1 + 1. 12 = 1. 12
1 × 1. 12
Therefore x2 = = 1. 75 and so the fractional increase in x is
0. 82
1. 75 − 1 = 0. 75
Equivalent to a 75% increase.
 time
(b) time α length ⇒  = Constant
length
t1 t2
So √ = √
L1 L2
44
Time for one beat of first pendulum = s
105
Time for one beat of second pendulum = 1 s

44 1  105 × 174. 4
√ = √ ⇒ L2 = = 31. 51 ⇒ L2 = 993 mm
105 × 174. 4 L2 44

51.

3 cos (x) + 4 sin (y) = 2 2 − 1. 5 (i)

5 cos (x) − 2 · sin (y) = −3. 5 (ii)

Multiplying (i) by 5 and (ii) by 3 gives



15 cos (x) + 20 sin (y) = 10 2 − 7. 5 (iii)

15 cos (x) − 3 2 · sin (y) = −10. 5 (iv)

Subtracting (iv) from (iii) gives

17. 1421
24. 2426 sin (y) = 17. 1421 ⇒ sin (y) = = 0. 7071
24. 2426

Therefore y = 45◦
Substituting sin(y) = 0. 7071 into (i) gives
3 cos (x) + 2. 8284 = 1. 3284
1. 3284 − 2. 8284
Therefore cos (x) = = −0. 5

3
Therefore x = 120
19
52. Volume of frustum = × volume of cone
27
Let the volume of the complete cone be 27 and let the volume of the frustum be
19. Therefore the volume of the top removed is 8.
Volumes of similar objects vary as the cubes of their dimensions.
Selection of Examination Standard Questions • 435

Therefore

Volume of complete cone (Height of complete cone)3


=
Volume of top removed (Height of top removed)3
27 H3
= 3
8 h

3 8H3
h=
27
2
h= H
3

1
Therefore height of frustum = H
3
 /6
5π     π 
5π /6 5π
53. (a) Area = 2 sin(θ ) dθ = [−2 cos(θ )]π/6 = −2 cos − −2 cos
6 6
π/6
Area = (1. 732) − (−1. 732) = 3. 464
π   π 
(b) (i) 3 cos (θ ) dθ = [3 sin (θ )]ππ/2 = (3 sin (π)) − 3 sin = 0 − 3 = −3
2
π/2

2  2  3   
  x 3 (2) (1)3
(ii) x 2 − 6x dx = − 3x 2 = − 3 (2)2 − − 3 (1)2
3 1 3 3
1    
8 1 26
= − 12 − −3 =−
3 3 3
436 • Mathematics

3  3  
  x5 7x 2 243 63
(iii) x − 7x + 8 dx =
4
− + 8x = − + 24
5 2 1 5 2
1  
1 7
− − + 8 = 41. 1 − 4. 7 = 36. 4
5 2
f1
54. Express V in terms of v, f1 and f2 in the following expression where K = :
f2

V +v 2 √ V +v √
K= ⇒ K= ⇒ K · (V − v) = V + v
V −v V −v
√ √
⇒ K ·V − K ·v =V +v
√ 
√ √ √  √  K +1
⇒ K ·V −V =v+ K ·v ⇒V K −1 =v K + 1 ⇒ V = v · √ 
K −1

√ 12
When f1 = 12 and f2 = 18, K = = ±0. 8165
18
√ 
K +1 ±0. 8165 + 1
Therefore V = v · √  = 20 ×
K −1 ±0. 8165 − 1

+0. 8165 + 1 −0. 8165 + 1


So V = 20 × = −198 or 20 × = −2. 02
+0. 8165 − 1 −0. 8165 − 1
55. x
–4 –2 2

–2

–4

–6

–8

–10 y

When the area bounded by the graph is swept through one complete revolution
about the x axis, the y ordinates of the graph become the radii at regular intervals
along the length of the solid swept out. Putting the cross-sectional areas through
Selection of Examination Standard Questions • 437

Simpson’s Rule gives:

Radii, r Cross-Section areas, πr2 Multiplier Product for volume


0 π ×0 1 π ×0
5 π × 25 4 π × 100
8 π × 64 2 π × 128
9 π × 81 4 π × 324
8 π × 64 2 π × 128
5 π × 25 4 π × 100
0 π ×0 1 π ×0
Total π × 780

Common interval between ordinates = 1


1 × π × 780
Therefore volume generated = = 816. 8 unit3
3
16000 dA
56. (i) A = x 2 + ⇒ A = x 2 + 16000x −1 ⇒ = 2x − 16000x −2
x dx
16000
At a turning point the derivative equals 0. Hence 2x − =0
x2
16000 √
Therefore 2x = 2
⇒ 2x 3 = 16000 ⇒ x 3 = 8000 ⇒ x = 3 8000 = 20
x
dA d2 A 32000
= 2x − 16000x −2 ⇒ 2 = 2 + 32000x −3 = 2 + > 0 when x = 20
dx dx x3
Therefore this is a minimum.
16000
(ii) When x = 20, A = (20)2 + = 400 + 800 = 1200 units2
20
57. Referring to triangle ABC in the Elevation diagram
AC = radius of 2 cm ball + radius of 4 cm ball = 1 + 2 = 3 cm
AB = radius of cylinder − radius of 2 cm ball = 3 − 1 = 2 cm
!  √
BC = (AC)2 − (AB)2 = 32 − 22 = 5 = 2. 236 cm

h = radius of 2 cm ball +BC+ radius of 4 cm ball = 1 + 2. 236 + 2 = 5. 236 cm


π × d2 × h π × 62 × 5. 236
Internal volume of cylinder up to height h = =
4 4
= 148 cm3
πd3
Volume of sphere =
6
π
Volume of three small balls = 3 × × 23
6
438 • Mathematics

Water level

C
h

A B

Elevation

Plan
π
Volume of large sphere = × 43
6
π   π
Total volume of balls = × 3 × 23 + 43 = × 88 = 46. 08 cm3
6 6
Volume required to be filled with water = Volume of cylinder − Volume of balls
= 148 − 46. 08
= 101. 92 cm3
= 101. 92 ml

As volumes vary according to the cubes of corresponding dimensions, halv-


101. 92
ing a dimension means one-eighth the volume. So volume required =
8
= 12. 74 ml.
58. A regular pentagon consists of five isosceles triangles.
360◦
The angle at the centre = = 72◦
5
Selection of Examination Standard Questions • 439

Each isosceles triangle can be divided into two right-angled triangles where the
72
angle at the apex is = 36◦ and the hypotenuse is 3 cm.
2
The height of one of these triangles is 3 × cos(36◦ ) = 2. 427 cm and the base is 3
3 × sin(36◦) = 1. 763 cm.
Therefore the length of the third side of the isosceles triangle is 2 × 1. 763 =
3. 526 cm.
1 1
The area of each isosceles triangle = × base × height = × 3. 526 × 2. 427 =
2 2
4. 28 cm2
The area of the pentagon is 5 × 4. 28 = 21. 4 cm2

B
3 cm a

C
A = 30°

Angle C = 2 × (90 − 36) = 108◦


Angle B = 180 − A − C = 42◦

a b 3. 526 × sin (30)


= ⇒a= = 2. 635 cm
sin (A) sin (B) sin (42)

ab sin (C) 2. 635 × 3. 526 × sin (108)


Area of triangle ABC = = = 4. 418 cm2
2 2
The remaining section of the pentagon = 21. 4 − 4. 418 = 16. 982 cm2
The ratio of the areas of the sections is 4. 418 : 16. 982
Dividing both parts by 4.418 gives the ratio as 1 : 3. 844
 
59. (a) 2 sin2 (θ ) − cos (θ ) = 1 ⇒ 2 1 − cos2 (θ ) − cos (θ ) − 1 = 0 ⇒ 2 cos2 (θ )
+ cos (θ ) − 1 = 0
⇒ (2 cos (θ ) − 1) (cos (θ ) + 1) = 0 ⇒ cos (θ ) = 0. 5 or cos (θ ) = −1
Therefore θ = 60◦ , 360◦ − 60◦ = 300◦ and 270◦
 
(b) 3 cos (2θ ) + 2 cos (θ ) = 0 ⇒ 3 2 cos2 (θ ) − 1 + 2 cos (θ ) = 0
⇒ 6 cos2 (θ ) + 2 cos (θ ) − 3 = 0
Using the quadratic formula results in cos(θ ) = 0.5598 or – 0.8932.
Therefore q = 55◦ 57 , 304◦03 , 153◦17 and 206◦43 .
440 • Mathematics

60.
V 5 10 15 20 25
R 10 25 50 85 130
V2 25 100 225 400 625

200 y

180

160

140

120

100

80

60

40

20
x
100 200 300 400 500 600 700

Taking two points on the line (100, 25) and (400, 85)
The vertical difference is 85 − 25 = 60
The horizontal difference is 400 − 100 = 300
60
The slope is therefore c = = 0. 2
300
The point (100, 25) satisfies the equation of the straight line so
25 = 100 × 0. 2 + K and so K = 25 − 20 = 5
The data therefore fits the equation R = 5 + 0. 2V 2
When V = 40 the probable resistance is 5 + 0. 2 × 402 = 325 N
k (p + V) k (p + V)
61. p = V + ⇒p−V = ⇒ (p − V) (k + p + V) = k (p + V)
k+p+V k+p+V
⇒ pk + p2 + pV − kV − pV − V 2 = kp + kV ⇒ pk + p2 − kV − V 2 = kp + kV
p2 − V 2
⇒ p2 − V 2 = kp + kV − pk + kV ⇒ p2 − V 2 = 2kV ⇒ k =
2V
9. 22 − 0. 0422
When p = 9. 2 and V = 0. 042, k = = 1007. 6
2 × 0. 042
62. The length of each non-parallel side being equal to the radius, the trapezium is
composed of three equilateral triangles.
Therefore the angle at the base is 60◦ , the length of the short side of the trapezium
is 10 cm, the length of the long side is 2 × 10 = 20 cm and the height, h, of the
trapezium is 10 · sin(60◦) = 8. 66 cm.
Selection of Examination Standard Questions • 441

1 (10 + 20) × 8. 66
Area of the trapezium = (Sum of parallel sides × Height) =
2 2
= 129. 9 cm2

5 cm

8.66 cm

5 cm

10 cm

   
π · h · R2 + Rr + r2 π · ×8. 66 × 102 + 10 × 5 + 52
Volume of frustum = =
3 3
= 1587 cm 3

63. (a)

4a + 3b + 2w = 2 (i)
2a + 8b + 3w = 11 (ii)
3a + 5b − 6w = −36 (iii)

To eliminate w: multiply (i) by 3 and (ii) by 2, then subtract


Step 1

12a + 9b + 6w = 6
4a + 16b + 6w = 22

8a − 7b = −16

Multiply (i) by 3 and add (iii)


442 • Mathematics

Step 2
12a + 9b + 6w = 6
3a + 5b − 6w = −36
15a + 14b = −30

Multiply the result of Step 1 by 2 and add the result of Step 2


Step 3
16a − 14b = −32
15a + 16b = −30
31a = −62
−62
So a = = −2
31
Substitute this value into the result of Step 2

−30 + 14b = −30 ⇒ 14b = 0 ⇒ b = 0

Substitute both values into (i)

−8 + 0 + 2w = 2 ⇒ 2w = 10 ⇒ w = 5

Therefore a = −2, b = 0 and w = 5


(b)
x 2 + y2 = 18. 5
x−y =1

The second equation gives x = 1 + y and when this is substituted into the first
equation, the result is:
(1 + y)2 + y2 = 18. 5
1 + 2y + 2y2 = 18. 5

So 2y2 + 2y − 17. 5 = 0
Solving this by formula gives y = 2. 5 or −3. 5, which results in x = 3. 5 or −2. 5.
The pairs of answers are (3. 5, 2. 5) and (−2. 5, −3. 5)
64. (a)
sin2 (θ ) cos2 (θ ) sin2 (θ ) cos2 (θ ) − sin2 (θ )
1− −
1 − tan2 (θ ) cos2 (θ ) cos2 (θ ) cos2 (θ ) cos2 (θ )
= = =
1 + tan (θ )
2
sin (θ )
2
cos (θ )
2 sin (θ )
2
cos (θ ) + sin2 (θ )
2
1+ +
cos2 (θ ) cos2 (θ ) cos2 (θ ) cos2 (θ )
cos2 (θ ) − sin2 (θ ) 1 − tan2 (θ )
= but cos 2
(θ ) + sin 2
(θ ) = 1 ⇒
cos2 (θ ) + sin2 (θ ) 1 + tan2 (θ )
= cos2 (θ ) − sin2 (θ )
Selection of Examination Standard Questions • 443

(b)
tan (θ ) cos2 (θ ) sin (θ )
sin (θ ) + sin (θ ) · cot2 (θ ) + = sin (θ ) + sin (θ ) · +
cos (θ ) sin (θ )
2 cos 2 (θ )

cos (θ )
2 sin (θ ) cos (θ )
2 sin (θ )
sin (θ ) + sin (θ ) · + = sin (θ ) + +
sin (θ )
2 cos (θ )
2 sin (θ ) cos2 (θ )
cos2 (θ ) sin (θ )
sin (θ ) + +
sin (θ ) cos2 (θ )
sin (θ ) · sin (θ ) cos2 (θ ) + cos2 (θ ) · cos2 (θ ) + sin (θ ) · sin (θ )
=
sin (θ ) cos2 (θ )
sin2 (θ ) cos2 (θ ) + cos2 (θ ) · cos2 (θ ) + sin2 (θ )
=
sin (θ ) cos2 (θ )
 
cos2 (θ ) sin2 (θ ) + cos2 (θ ) + sin2 (θ )
=
sin (θ ) cos2 (θ )
cos2 (θ ) + sin2 (θ ) 1
= =
sin (θ ) cos2 (θ ) sin (θ ) cos2 (θ )

65. 8 y

6 4y – x – 20 = 0

4 y–x+1=0

x
–2 2 4 6 8
2y + x – 7 = 0
–2

–4

The graphs intersect at the points


• A = (−2, 4. 5)
• B = (3, 2)
• C = (8, 7)
The lengths of the sides are:
√ √
• AB = 52 + 2. 52 = 31. 25 = 5. 59
√ √
• AC = 102 + 2. 52 = 106. 25 = 10. 31
√ √
• BC = 52 + 52 = 50 = 7. 07

s = half the sum of the sides = 11. 485


444 • Mathematics


Area = 11. 485 × (11. 485 − 5. 59) × (11. 485 − 10. 31) × (11. 485 − 7. 07)

= 351. 22
Area = 18. 74 unit2

66. D E

30°

y
J
H
G F

60°
A C B

The hexagon is composed of six equilateral triangles with all angles of 60◦, and
each side of length 1 unit.

OC = OA × sin (60) = 1 × 0. 866 = 0. 866


1 1
Area of triangle OAB = × AB × OC = × 1 × 0. 866 = 0. 433
2 2
Therefore the area of the hexagon = 6 × 0. 433 = 2. 598
The area to be removed is in the shape of a trapezium consisting of a central
rectangle with a triangle at each end.
Let the depth of this trapezium DG be represented as y.
 
1
Area removed = Area of rectangle + Two triangles = DE × y + 2 × × HG × y
2
= 1 × y + (y · tan (30)) × y
= y + 0. 5774y2

This is to be 20% of the area of the hexagon so:

y + 0. 5774y2 = 0. 2 × 2. 598 ⇒ y + 0. 5774y2 = 0. 5192


⇒ 0. 5774y2 + y − 0. 5192 = 0

Dividing throughout by 0.5774 gives the quadratic y2 + 1. 732y − 0. 9 = 0


Selection of Examination Standard Questions • 445

Using the quadratic formula gives:


!

−1. 732 ± (1. 732)2 − 4 × 1 × (−0. 9) −1. 732 ± 6. 6
y= =
2×1 2
−1. 732 ± 2. 569
=
2
−1. 732 + 2. 569
As the negative value is an impossibility, y = = 0. 4185
2
67. Spitting the interval x = 0 to x = 2 into 10 interval gives a strip width of 0.2.
As y = e−2x , the ordinates are e0 , e−0.4 , e−0.6 , K, e−4

Ordinates Multiplier Product for volume


1.000 1 1.000
0.670 4 2.680
0.449 2 0.898
0.301 4 1.204
0.202 2 0.404
0.135 4 0.540
0.091 2 0.182
0.061 4 0.244
0.041 2 0.082
0.027 4 0.108
0.018 1 0.018
Total 7.36

0. 2
Area = × 7. 36 = 0. 4907 unit2
3
Area 0. 4907
Mean ordinate = = = 0. 245 unit
Length of interval 2
! √
68. Perpendicular height = (Slant height)2 − (Radius of base)2 = 502 − 302 = 40 cm

π × 302 × 40
Volume of cone = = 12000π unit3
3
 
π D 3 − d3
Volume of hollow sphere = but D = d + (2 × Thickness) = d + 3,
6
therefore  
  
π (d + 3)3 − d3 π d3 + 3d2 + 27d + 27 − d3
Volume of hollow sphere = =
 2 6  6
π 9d + 27d + 27
=
6
446 • Mathematics

Equating the two volumes gives:


 
π 9d2 + 27d + 27
= 12000π ⇒ 9d2 + 27d + 27
6
= 72000 ⇒ 9d2 + 27d − 71973 = 0

Dividing by 9 gives the quadratic equation d2 + 3d − 7997 = 0


Using the quadratic
 formula √
−3 ± 32 − 4 × 1 × (−7997) −3 ± 31997 −3 ± 178. 877
d= = =
2×1 2 2
−3 + 178. 877
The negative value can be ignored so d = = 87. 94 cm, giving
2
D = 90. 93 cm
c h2 ch2
69. a = ⇒ 2as = ⇒ 2as (d − h) = ch2 ⇒ 2asd − 2ash = ch2
2s d − h d−h
⇒ 0 = ch2 + 2ash − 2asd
Using the quadratic formula gives:
!
−2as ± (2as)2 − 4 (c) (−2asd)
h=
2c
!  

−2as ± 4a2 s2 + 8acsd −2as ± 4 a2 s2 + 2acsd
h= =
2c 2c
√ ! 
−2as ± 4 × a2 s2 + 2acsd
=
2c

−as ± as (as + 2cd)
So h =
c

− (1. 5) (16) ± (1. 5) (16) ((1. 5) (16) + 2 (15) (17))
=
15
 √
−24 ± 24 (24 + 510) −24 ± 12816 −24 ± 113. 208
h= = =
15 15 15
= 5. 947 or − 9. 147

70. N C
32
T
T
W B

68 Mast
Mast
60 20°
A B W B
Elevation Elevation
Plan on AB on WB
A
Selection of Examination Standard Questions • 447


(a) Referring to the Plan view, if 68 = 602 + 322 then Pythagoras’ Theorem is
true and therefore B must be 90◦ .
 √ √
602 + 322 = 3600 + 1024 = 4624 = 68

Therefore B is a right angle.


32
(b) sin (A) = = 0. 4706 ⇒ A = 28◦ 4
68
Therefore C = 90◦ − 28◦ 4 = 61◦ 56
height of mast
(c) Referring to the Elevation on AB: tan (20◦ ) =
60
Therefore height of mast = 60 tan (20◦) = 21. 84 m
 
(d) Referring to the Plan view: WB = 60 sin (A) = 60 sin 28◦ 4 = 28. 235 m
height of mast 21. 84
Referring to the Elevation of WB: tan (W) = =
WB 28. 235
= 0. 7734
Therefore W = tan−1 (0. 7734) = 37◦ 43
 
71. x 2 + 1 = x 2 − x − 2 + x + 3
Therefore the solution to x 2 − x − 2 = 0 are the points of intersection of the curve
y = x 2 + 1 and the line y = x + 3.
10 y
9
8
7
6
5
4
3
2
1
x
–2 –1 1 2 3

The solutions are: x = −1, y = 2 and x = 2, y = 5


72. (a) Referring to the diagram,
Area of segment = area of sector − area of triangle
θ · r2
Area of sector = where θ is the centre angle in radians.
2
ab · sin C
Area of triangle = where, in this case,
2
a = r, b = r, and C is the centre angle θ
448 • Mathematics

Area of
segment

r2 · sin θ
Therefore area of triangle =
2
Area of segment = Area of sector-area of triangle
r2 · θ r2 · sin θ
= −
2 2
r2
= · [θ − sin θ ] θ in radians
2
A

15
C 30 30
D
O
α

B
(b) By crossed chords:

AO × BO = CO × DO

15 × BO = 30 × 30
30 × 30
BO =
15
BO = 60 cm

Diameter of pipe = AO + BO = 15 + 60 = 75 cm
Therefore radius of pipe = 37. 5 cm
30
sin (α) = = 0. 8 ⇒ α = 53◦ 8
37. 5
 
θ = 360 − 2 × 53◦ 8 = 253◦44
Selection of Examination Standard Questions • 449

 
sin (θ ) = sin 253◦44 = −0. 96
π
In radians, θ = 253◦44 × = 4. 4286 rad
180
1 1
Area of segment = r2 (θ − sin (θ )) = × 37. 52 × (4. 4286 − (−0. 96))
2 2
= 3789 cm2
Volume = Area × Length = 3789 × 2 × 100 = 757771 cm3 = 757. 8 litres
73. (a) WZ = x a+b × x a−b = x a+b+a−b = x 2a

W 2 Z = W × WZ = x a+b × x 2a = x 3a+b
W2 x 2(a+b)
= a−b = x 2(a+b)−(a−b) = x a+3b
Z x
Z3 x 3(a−b)
= a+b = x 3(a−b)−(a+b) = x 2a−4b
W x
(b)

4x + y = 10
2 7
+ =3
x y
From the first equation y = 10 − 4x
Substituting into the second equation gives:
2 7 2 (10 − 4x) + 7x
+ =3 ⇒ = 3 ⇒ 20 −8x + 7x = 3x (10 − 4x)
x 10 − 4x x (10 − 4x)
Therefore

20 − x = 30x − 12x 2 ⇒ 12x 2 − 31x + 20 = 0 ⇒ (4x − 5) (3x − 4) = 0


5 4
This gives two values for x: and
4 3
5 4
However, y = 10 − 4x so, when x = , y = 10 − 5 = 5 and when x = ,
4 3
16 14
y = 10 − =
3 3    
5 4 14
The pairs of solutions are: , 5 and ,
4 3 3
An alternative method to find x is to use the quadratic formula.
!

− (−31) ± (−31)2 − 4 × 12 × 20 31 ± 1 31 + 1 31 − 1
So x = = = ,
2 × 12 24 24 24
32 30 4 5
= , = ,
24 24 3 4
and continue to find the corresponding y values.
450 • Mathematics

74. In each of the following formulae, transpose and re-write in terms of k:


2ghDk2 2ghDk2  
(i) Q =   ⇒ Q2 =   ⇒ Q2 . d s2 − k2 = 2ghDk2
d s −k
2 2 d s −k
2 2

⇒ Q2 ds2 − Q2 dk2 = 2ghDk2 ⇒ Q2 ds2 = 2ghDk2 + Q2 dk2



  Q2 ds2 Q2 ds2
⇒ Q2 ds2 = k 2 2ghD + Q2 d ⇒ k2 = ⇒k=
2ghD + Q2 d 2ghD + Q2 d

d
Therefore k = Qs
2ghD + Q2 d
 
1 1 1 1 1
(ii) F = ⇒ 2πF = ⇒ 4π 2 F 2 = ⇒k=
2π Lk Lk Lk 4Lπ 2 F 2
π (k − b) π π k b
(iii) L = (k + b) + + 2c ⇒ L = · k + · b + − + 2c
2 4c 2 2 4c 4c
   
π k π b π 1 π 1
⇒ L = · k + + · b − + 2c ⇒ L − 2c − b − =k +
2 4c 2 4c 2 4c 2 4c
 
π 1
L − 2c − b −
2 4c
⇒k=   (Multiply everything by 4)
π 1
+
2 4c
 
π 1
4L − 4 × 2c − 4 × b −
2 4c
⇒k=  
4π 4
+
2 4c

b
4L − 8c − 2bπ +
Simplify k = c (Multiply everything by c)
1
2π +
c
4Lc − 8c2 − 2bcπ + b
⇒k=
2πc + 1

75. 7x 2 − 28 + 7y2 = 0 ⇒ y2 = 4 − x 2
1 1  1
  x3
Volume of revolution = π y2 dx = π 4 − x 2 dx = π 4x −
3 −1
−1
  −1 
1 (−1) 22π
Therefore, Volume = π 4− − −4 − = = 23. 04 cm3
3 3 3
Selection of Examination Standard Questions • 451

2 y

x
–1 1

–1

–2

76. Show that, if x is numerically less than 1,


    
1 1 3
− − −
1 2 2 2
√ = (1 + {−x})−1/2 ≈ 1 + (−x) + (−x)2
1−x 1! 2!
   
1 3 5
− − −
2 2 2
+ (−x)3
3!
1 1×3 2 1×3×5 3
=1+ x+ x + x
2 2×4 2×4×6
1
√ ≈ 1 + 0. 5 × 0. 1 + 0. 375 × 0. 01 + 0. 3125 × 0. 001 = 1. 0540625
0. 9

77. Due to bad weather a ship’s speed is decreased by 20% for 8 h. The ship then runs
at normal speed for 10 h. In order to catch a tide the speed is then increased for 6 h.
Over the 24-h period the fuel consumption was found to be unchanged from the
normal consumption.
Given that the fuel consumption C is directly proportional to the cube of the ship’s
speed V determine the percentage increase in the ship’s speed in the last 6 h.

C α V3 ⇒ C = k · V3
452 • Mathematics

For the first 8 h C1 = k · (0. 8V)3 per hour


For the next 10 h C2 = k · V 3 per hour
For the last 6 h the speed is increased from V to βV
For the last 6 h C3 = k · (βV)3 per hour
For the 24-h period 8C1 + 10C2 + 6C3 = 24C2

8C1 + 6C3 = 14C2


8k · (0. 8V) + 6k · (βV)3 = 14k · V 3
3

4. 096 + 6β 3 = 14
6β 3 = 9. 904

3 9. 904
β=
6
β = 1. 18182

Therefore the percentage increase in speed during the last 6 h is 18.2%.


   
7 1
√   1 x n+4 x 3n− 2
x 4n+ 2
x n+4 x 6n−7 x n+4 x 6n−7 2
78. (a) √ = =   =   = x1
x 3n x 2n−1  1 n−
1
4n−
1
x 3n x 2n−1 2 x 3n x 2 x 2

(b) 10x = 20x−4 so taking logarithms of both sides gives:


 
log (10x ) = log 20x−4 ⇒ x log (10) = (x − 4) log (20)
⇒ x = x log (20) − 4 log (20) ⇒ 4 log (20) = x log (20) − x
4 log (20)
⇒x= = 17. 29
log (20) − 1
 √  8 12 1/2
√ a8 b12 a8 b12 a b a4 b6
−1
79. (i) 64 · a b =
8 12 = √ = =
64 64 8 8
 √ 
√ √  1/3 6
5
b
(ii) 216a−12 · b0.6 = 216 · a−12 · b0.6 = 6 · a−4 · b0.2 = 4
3 3
a
(iii)

a−b+c =3 (i)
2a + 8b − 3c = 56 (ii)
7a + 3b + 3c = 69 (iii)

Adding 3 times (i) to (ii) gives 5a + 5b = 65


Adding (ii) to (iii) gives 9a + 11b = 125
9 times the first equation gives 45a + 45b = 585
5 times the second equation gives 45a + 55b = 625
Selection of Examination Standard Questions • 453

Subtracting gives 10b = 40 so b = 4


Substituting gives 45a + 180 = 585
Therefore 45a = 405 so a = 9
Substituting into (i) gives 9 − 4 + c = 3 so c = −2
Check in (ii) 18 + 32 − (−6) = 56 Correct
Solution: a = 9, b = 4 and c = −2
80. (a)
10 y

x
–5 –4 –3 –2 –1 1 2

–5

–10

(−10) − 6 16
(b) (i) First line: gradient = =− = −2. 6
2 − (−4) 6
Equation: y = −2. 6x + c passing through (−4, 6)
Therefore: 6 = −2. 6 (−4) + c ⇒ c = −4. 6
Equation: y = −2. 6x − 4. 6
4 − (−10) 14
Second line: gradient = = =2
2 − (−5) 7
Equation: y = 2x + c passing through (0, 0)
Therefore: c = 0
Equation: y = 2x
(ii) The solution to the simultaneous equations is the point of intersection
(−1, −2).
Therefore x = −1, y = −2
81.
# = 35◦
Angle DCA
# = 55◦ − 20◦ = 35◦
Angle DAC

# = 110◦
So triangle ADC is isosceles; and it follows that CD = AD = 50 m and ADC

Angle A# # = 50◦
BC = 40◦ − 20◦ = 20◦ making angle DCB
454 • Mathematics

C
40°
50 m
55°
D
A
B

20°

Using the Sine rule

50 DB 50 sin (50◦ )
= ⇒ DB = = 112 m
sin A#
BC #
sin DCB sin (20◦ )

Therefore the distance AB = 112 − 50 = 62 m


82. Let the fuel consumption be F, the calorific value be C and the speed (knot) be S.
S3 kS3
Therefore F α ⇒ F=
C C
The ship burns 60 tons/day of fuel of calorific value 40 MJ/kg when travelling at
k · (20)3 60 × 40
20 knots, so 60 = ⇒ k= = 0. 3
40 203
0. 3 × (16) 3
Therefore F = = 29. 26 tons/day
42
83. y
The intersection of the two curves occurs when x is approximately 6.25
1.2

5
y=
2x
0.8

y = 0.5 log(x)
0.4

x
0
2 3 4 5 6 7 8 9 10

84. It is first of all necessary to find the points of intersection of the two curves.
At these points x 2 + 1 = 7 − x
Therefore x 2 + x − 6 = 0 ⇒ (x + 3) (x − 2) = 0 ⇒ x = −3 and x = 2
2   2  
Shaded area = −3 (7 − x) − x 2 + 1 dx = −3 6 − x − x 2 dx
Selection of Examination Standard Questions • 455

12 y

x
–4 –3 –2 –1 1 2 3

 2    
x2 x3 4 8 9 −27
Area = 6x − − = 12 − − − −18 − −
2 3 −3 2 3 2 3
   
22 27 125
= − − =
3 2 6
CB AB 550 × sin (10◦ )
85. = ⇒ sin (C) = = 0. 6367 ⇒ C = 39. 55◦ or 140. 45◦
sin (A) sin (C) 150
Therefore angle B = 130. 45◦ or 29. 55◦
AC CB 150 × sin (130. 45◦)
= ⇒ AC = = 657. 3 mm
sin (130. 45◦) sin (10◦ ) sin (10◦ )
A C CB  150 × sin (29. 55◦)
= ⇒ A C = = 426. 0 mm
sin (29. 55◦) sin (10◦ ) sin (10◦ )
The piston has travelled from position A to position A so the distance moved is the
difference between AC and A C, that is, 657. 3 − 426. 0 = 231. 3 mm.
x3 x2 dy 3x 2
86. y = − ⇒ = −x
4 2 dx 4
3x 2
Therefore at the points where the gradient is 0, −x =0
  4
3x 4
So − 1 x = 0 ⇒ either x = 0 or x =
4 3
That is, there are two points on the curve where the gradient is 0.
 
dy 3x 2 d2 y 6x 6 4 4
= −x ⇒ 2 = −1= − 1 = 1 > 0 when x = , making this
dx 4 dx 4 4 3 3
a minimum point.
 
4 8 4 8
When x = , y = − so the minimum point is ,−
3 27 3 27
456 • Mathematics

π/2
π/2
π  π 
87. (a) 4 cos (x) dx = [4 sin (x)]π/9 = 4 sin − 4 sin = 4 − 1. 368 = 2. 632
π/9 2 9
dy 2x 3
(b) = 2x 2 − 2 ⇒ y = − 2x + C
dx 3
2 × (3)3
When x = 3, y = −2, therefore −2 = −2×3+C
3
−2 = 18 − 6 + C ⇒ C = −2 − 18 + 6 = −14
2x 3
Therefore y = − 2x − 14
3
88. (a) Top radius = 10 cm
Base radius = 20 cm
510cm
cm

16 cm

20 cm
   
π · h · R2 + Rr + r2 π · ×16 × 202 + 20 × 10 + 102
Volume of frustum = =
3 3
π 
V= × 11200 = 11728. 6 cm3
3
Therefore mass of frustum = 11728. 6 × 0. 9 = 10555. 8 g ≈ 10. 56 kg
(b) For the height of the removed cone, let the removed cone have height h.
h h + 16
By similar triangles =
10 20
Therefore 20h = 10 (h + 16) ⇒ 20h = 10h + 160 ⇒ h = 16 cm
π · r2 · h π × 102 × 16  π 
Volume of top cone = = = × 1600 cm3
3 3 3
The ratio by mass is identical to the ratio by volume as both sections are of
equal density.
Therefore ratio of mass of cone removed to mass of frustum equals
π  π 
× 1600 : × 11200 ⇒ 1600 : 11200 ⇒ 1 : 7
3 3
89. (i) For the left-hand position, the water fills a solid comprising:
hemisphere of radius r + cylinder of radius r and height (0. 7h − r)
Volume:
2
100 = × π · r3 + π · r2 · (0. 7h − r) (i)
3
Selection of Examination Standard Questions • 457

h
0.7h
0.6h
r

For the right-hand position, the water fills a solid comprising:


Cylinder of radius r and height 0. 6h
Volume:
100 = π · r2 × 0. 6h (ii)
Equating (i) and (ii) gives:
2
π · r2 × 0. 6h = × π · r3 + π · r2 · (0. 7h − r) divide throughout by π · r2
3
2
0. 6h = × r + (0. 7h − r) simplifies to give:
3
2 r r 10r
0. 6h = × r + 0. 7h − r ⇒ 0. 6h = 0. 7h − ⇒ = 0. 1h ⇒ h =
3 3 3 3
Substituting this value into (ii) gives:

2 10r 100
⇒ 100 = 2πr3 ⇒ r =
3
100 = π · r × 0. 6 × = 2. 5154 cm
3 2π
Therefore the bore of the vessel is 5.031 cm
10 × 2. 5154
(ii) Overall height h = = 8. 38 cm
3
 
5 5 2 25
90. y = x 2 + ⇒ y2 = x 2 + = x 4 + 5x 2 +
2 2 4
 3  
2 4 2 25
Volume of revolution = π y dx = π x + 5x + dx
4
0

 3   
x5 5x 3 25x 35 5 × 33 25 × 3
V=π + + =π + + −0
5 3 4 0 5 3 4
2247π
= = 353. 0 unit3
20
d2 q dq 1 d2 q dq
91. L · 2
+R· + · q = v(t) implies 2 + 160 · + 10000 · q = 20
dt dt C dt dt
Taking transforms gives
  20
s2 · q − s. q0 − q1 + 160 (s · q − q0 ) + 10000q =
s
458 • Mathematics

Since both initial conditions are 0


20 20
s2 · q + 160s · q + 10000q = giving q =  2 
s s · s + 160s + 10000

Splitting this into partial fractions gives

20 A Bs + C
 = + 2
s · s + 160s + 10000
2 s s + 160s + 10000
 
So 20 = A s2 + 160s + 10000 + (Bs + C) · s
20 1
Let s = 0 20 = 10000A ⇒A = =
10000 500
1
Coefficients of s : 0 = A + B
2 ⇒B = −
500
160
Coefficients of s: 0 = 160A + C⇒C = −160A = −
500
1 1 160
Therefore A = , B=− , C=−
500 500 500
 
1/500 1 s + 160
Substituting gives q = −
s 500 s2 + 160s + 10000
The second term requires transposition, so completing the square gives
 
1/500 1 s + 160
q= −
s 500 (s + 80)2 + 602

The (s + 80)2 term means that the First Shift Theorem is operating, so all terms
involving s must now be in terms of s + 80.
Splitting s + 160 into s + 80 + 80 gives
 
1/500 1 s + 80 + 80
q= −
s 500 (s + 80)2 + 602
 
1/500 1 s + 80 80
= − +
s 500 (s + 80)2 + 602 (s + 80)2 + 602

At this point inverse transforms can be taken


 
1 1 80
q= − e−80t · cos(60t) + e−80t · · sin(60t)
500 500 60

Therefore
 
1 e−80t 4
q= − cos(60t) + · sin(60t)
500 500 3
dq
Since current is the rate of change of charge, i =
dt
Selection of Examination Standard Questions • 459

Using the product rule to differentiate q:


e−80t 4
let u = v = cos(60t) + · sin(60t)
500 3
du 80 −80t dv
So =− e = −60 · sin(60t) + 80 cos(60t)
dt 500 dt
Therefore
 
dq e−80t 80 −80t 4
= · (−60 · sin(60t) + 80 cos(60t)) − e cos(60t) + · sin(60t)
dt 500 500 3
−80t  
dq e 320
= −60 · sin(60t) + 80 · cos(60t) − 80 · cos(60t) − · sin(60t)
dt 500 3
1 −80t
So i = ·e · sin(60t)
3
92. Taking Laplace Transforms gives
5
2 (sx − x0 ) + (sy − y0 ) =
s−1
(sy − y0 ) − 3 (sx − x0 ) = 5

Substituting the boundary values gives


5 5
2sx + sy = 2sx + sy =
s−1 s−1
and rearranging gives
5 5
sy − 3sx = −3sx + sy =
s s
5 5
Subtracting theses equations gives 5sx = −
s−1 s
1 1 s − (s − 1) 1
and dividing by 5s gives x = − = 2 = 2
s (s − 1) s2 s (s − 1) s (s − 1)
1 A B C
Using partial fractions gives = + 2+
s2 (s − 1) s s (s − 1)
So 1 = As (s − 1) + B (s − 1) + Cs2
Let s = 0: 1 = −B ⇒ B = −1
Let s = 1: 1 = C ⇒ C=1
Coefficient of s2 : 0 = A + C ⇒ A = −C ⇒ A = −1
1 1 1
x=− − 2 +
s s s−1
Hence
x = −1 − t + et or x = et − 1 − t as required
Rearranging the second equation gives
dy dx
=5+3
dt dt
460 • Mathematics

dx dy  
but = et − 1, so = 5 + 3 et − 1 = 5 + 3et − 3
dt dt
So 
dy  t 
= 3et + 2 ⇒ y = 3e + 2 dt =3et + 2t + c
dt
As y = 0 when t = 0, 0 = 3 + c ⇒ c = −3
Therefore y = 3et + 2t − 3
93.
1
s · x − x0 + x + 2y =
s−1
2
s · y − y0 − 2x − 3y = −
s−1
Substituting the initial conditions gives
1
x · (s + 1) + 2y = Λ (i)
s−1
2
−2x + y · (s − 3) = −
s−1
To eliminate x, multiply the first equation by −2 and the second by (s + 1)
2
−2 · (s + 1) x − 4y = −
s−1
2 (s + 1)
−2 (s + 1) x + y · (s − 3) (s + 1) = −
s−1
Subtract the top equation from the bottom equation to give
2 (s + 1) 2
y [(s − 3) (s + 1) + 4] = − +
s−1 s−1
Simplifying the algebra results in the following:
  2s 2s 2s
y s2 − 2s + 1 = − ⇒ y (s − 1)2 = − ⇒ y=−
s−1 s−1 (s − 1)3
Substituting this expression in (i) gives
4s 1 1 4s
x (s + 1) − = ⇒ x (s + 1) = +
(s − 1) 3 s−1 s − 1 (s − 1)3
(s − 1)2 + 4s s2 + 2s + 1 (s + 1)2
x (s + 1) = = =
(s − 1)3 (s − 1)3 (s − 1)3
(s + 1)
So x =
(s − 1)3
Resolving into partial fractions:
(s + 1) A B C
x= = + +
(s − 1)3 (s − 1) (s − 1)2 (s − 1)3
Selection of Examination Standard Questions • 461

So s + 1 = A (s − 1)2 + B(s − 1) + C
Let s = 1: 1 + 1 = C ⇒ C = 2
Coefficient of s2 : 0 = A ⇒ A = 0
Let s = 0: 1 = A (−1)2 +B (−1)+C ⇒ 1 = A−B+C ⇒ B = A+C −1 ⇒ B = 1
1 2
Therefore x = + ⇒ x = t · et + t2 · et = t · et (1 + t)
(s − 1) 2
(s − 1)3
Resolving into partial fractions
2s A B C
y=− = + +
(s − 1) 3 (s − 1) (s − 1)2
(s − 1)3
So −2s = A (s − 1)2 + B(s − 1) + C
Let s = 1: −2 = C ⇒ C = −2
Coefficient of s2 : 0 = A ⇒ A = 0
Let s = 0: 0 = A (−1)2 + B (−1) + C ⇒ 0 = A − B + C ⇒ B = A + C ⇒ B = −2
−2 2
Therefore y = − ⇒ y = −2t · et − t2 · et = −t · et (2 + t)
(s − 1) 2
(s − 1)3
94. T
4
bn = f (x) · sin (nωx) dx where T = 4
T
0
2
= f (x) · sin (nωx) dx
0
1 2
= 10x · sin (nωx) dx + (20 − 10x) · sin (nωx) dx
0 1

Consider the first integral


let u = 10x dv = sin(nωx)
cos (nωx)
du = 10 v=−

  
10x · cos (nωx) cos (nωx)
Integral equals − − − 10 dx
nω nω
 
10x · cos (nωx) 10 sin (nωx) 1
= − +
nω n2 ω2 0
 
10 cos (nω) 10 sin (nω)
= − + − (0 + 0)
nω n2 ω2
10 cos (nω) 10 sin (nω)
=− +
nω n2 ω2
2π 2π π
but ω = = =
T 4 2
462 • Mathematics

 nπ   nπ 
20 cos 40 sin
So integral equals − 2 + 2
nπ n2 π 2
Consider the second integral
let u = 20 − 10x dv = sin (nωx)
cos (nωx)
du = −10 v=−

  
(20 − 10x) . cos (nωx) cos (nωx)
Integral equals − − − (−10) dx
nω nω
 
(20 − 10x) . cos (nωx) 10 sin (nωx) 2
= − −
nω n2 ω2 1
   
10 sin (2nω) 10 cos (nω) 10 sin (nω)
= 0− − − −
n2 ω2 n2 ω2 n2 ω2
but ω = π  nπ   nπ 
40 sin(nπ) 20 cos 2 40 sin
2
so integral equals − + +
n2 π 2 nπ n2 π 2
 nπ   nπ 
20 cos 40 sin
= 2 + 2
nπ n2 π 2
 nπ   nπ   nπ   nπ 
20 cos 40 sin 20 cos 40 sin
Therefore bn = − 2 + 2 + 2 + 2
nπ n2 π 2 nπ n2 π 2
 nπ 
80 sin
= 2
n2 π 2
 nπ 
For n = 1, 5, 9, . . . , sin =1
2
 nπ 
For n = 3, 7, 11, . . . , sin = −1
2
 πx  1    
80 3πx 1 5πx
Hence f (x) = 2 sin − sin + sin − ···
π 2 9 2 25 2
95. y

x
–2π –π π 2π
Selection of Examination Standard Questions • 463

$ &  π  
1 2π 1 π 2π 1 x2 1 π2 π
a0 = f (x) dx = x dx + 0 dx = = · =
2π 0 2π 0 π 2π 2 0 2π 2 4
⎡ π ⎤
2π  2π
1 1⎣
an = f (x) · cos(nx) dx = x · cos(nx) dx + 0 dx⎦
π π
0 0 π
 π
1 x · sin(nx) cos(nx)
= + (by parts)
π n n2 0
   
1 π · sin(nπ) cos(nπ) cos(0) 1
an = + − 0 + = (cos(nπ) − 1)
π n n2 n 2 π · n2
2
When n is even, an = 0, when n is odd an = −
π · n2
2 2 2
So a1, = − , a3 = − 2 , a5 = − 2 , etc.
π 3 π 5 π
⎡ π ⎤
2π  2π
1 1⎣
bn = f (x) · sin(nx) dx = x · sin(nx) dx + 0 dx⎦
π π
0 0 π

 
1 x · cos(nx) sin(nx) π
= − + (by parts)
π n n2 0

   
1 π · cos(nπ) sin(nπ) sin(0)
bn = − + − 0+
π n n2 n2
 
1 π · cos(nπ) cos(nπ)
= − =−
π n n
1 1 1
So b1 = 1, b2 = − , b3 = , b4 = − , etc.
2 3 4
Therefore the Fourier series for f (x) is
π 2 2 2
f (x) = − cos(x) − 2 cos(3x) − 2 cos(5x)ΛΛ
4 π 3 π 5 π

1 1 1
+ sin(x) − sin(2x) + sin(3x) − sin(4x)ΛΛ
2 3 4
i.e.
 
π 2 cos(3x) cos(5x)
f (x) = − cos(x) + + ΛΛ
4 π 32 52
 
1 1
+ sin(x) − sin(2x) + sin(3x)ΛΛ
2 3
464 • Mathematics

When x = 0, f (x) = 0 by definition So, in the Fourier series:


   
π 2 cos(0) cos(0) 1 1
0= − cos(0) + + ΛΛ + sin(0) − sin(0) + sin(0)ΛΛ
4 π 32 52 2 3
 
π 2 1 1 1
So = 1 + 2 + 2 + 2 ΛΛ
4 π 3 5 7
π2 1 1 1
Therefore = 1 + 2 + 2 + 2 ΛΛ
8 3 5 7
APPENDIX 1
ARITHMETIC
Fractions
Fractions are normally given in their simplest form; in other words, the fraction has been
cancelled as far as possible. To do this look and see if there is a number that will divide
into the top (the numerator) and the bottom (the denominator).

Example
3
Consider the fraction .
12
3 1
3 divides into the numerator and denominator so cancelling by 3 gives = .
12 4
Similarly

4 1
(a) = (cancel by 4)
8 2
15 3
(b) = (cancel by 5)
55 11
27 3
(c) = (cancel by 9)
36 4

In some cases, cancelling can be done in stages – use as many steps as you need.
466 • Mathematics

Example
27
The last fraction given above was .
36
9 3
This can be cancelled by 3 giving , which can be cancelled by 3 again to give .
12 4

Adding fractions

When the denominators are all the same, you simply add the numerators.

Example
2 4 2+4 6
(a) + = =
7 7 7 7
5 9 5+9 14 7
(b) + = = =
24 24 24 24 12
(Remember to cancel if necessary.)

When the denominators are different, the fractions have to be altered so that the
denominators are the same then the top lines can be added as above.

Example
1 1
(a) +
3 4
A common denominator has to be found. The easiest one is found by multiplying the
given denominators together. In this example the common denominator is 4 × 3 = 12.
1 1×4 4 1 1×3 3
= = and = =
3 3×4 12 4 4×3 12
1 1 4 3 7
The two fractions can now be added, so: + = + =
3 4 12 12 12

Worked example
2 1
+ =?
3 5
2 2×5 10 1 1×3 3
= = = =
3 3×5 15 5 5×3 15
2 1 10 3 10 + 3 13
+ = + = =
3 5 15 15 15 15
Appendix 1 – Arithmetic • 467

Subtracting fractions

This works in exactly the same way as addition.

Example

2 1 10 3 7
(a) − = − =
3 5 15 15 15

Multiplication of fractions

It sometimes helps if the ‘x’ sign is read as the word ‘of’.

Example

1 4 1 4 2
(a) × means of =
2 9 2 9 9
Usually the calculation is done by multiplying the numerators, multiplying the denomi-
nators and then cancelling down if needed:

Example

1 3 1×3 3
(a) × = =
2 4 2×4 8
3 2 3×2 6
(b) × = =
5 5 5×5 25
7 3 7×3 21
(c) × = =
11 8 11 × 8 88

When multiplying, it is often possible to cancel before multiplying the fractions.

Example

31
A 1 1
(a) × 1 =
4 A
3 4
1
A
4 7 7
(b) × 2 =
5 A
8 10
468 • Mathematics

Dividing fractions

The way to divide fractions is to turn the second fraction upside down and then
multiply:

Example

2 1 2 7 14
(a) ÷ = × =
3 7 3 1 3
2 4 2 9 18 9
(b) ÷ = × = =
7 9 7 4 28 14
2 3 2 8 16
(c) ÷ = × =
5 8 5 3 15

Arithmetic with mixed numbers

Here, it is usually easiest to convert to improper fractions (top heavy) before the
arithmetic is done.

Example
1 3 4 18
(a) 1 ÷ 3 = ÷
3 5 3 5
4 5
= ×
3 18
20 10
= =
54 27

2 1 2 8 17 12
(b) 2 ×4 ÷2 = × ÷
3 4 5 3 4 5
8 17 5
= × ×
3 4 12
680
=
144
85
=
18
Appendix 1 – Arithmetic • 469

Exercise 1

1 2 5 1 1 3 5 2
1. + 2. − 3. 4 −2 4. ×
5 3 7 4 3 8 6 9
4 4 9 3 1 1 2 2
5. ÷ 6. ÷ 7. 1 ×3 8. 6 ÷2
9 7 14 7 3 4 5 3

Answers

13 13 47 23
1. 2. 3. =1
15 28 24 24
5 7 3 1
4. 5. 6. =1
27 9 2 2
13 1 12 2
7. =4 8. =2
3 3 5 5

Approximating and Estimating


Estimates and approximations bring two major benefits:

1. They enable you to check that the final answer is near to the figure that the estimate
led you to expect. This is particularly valuable if you are unsure in the use of the keys
on the calculator.
2. They can provide you with a ‘short cut’, by showing plainly that you do not need to
do all the calculations (e.g. when comparing figures in tables).

Example

54
(a) Calculate of 128
97

Steps
54 50
• Approximate to (move to figures that make ‘easy’ fractions)
97 100
50 1
• Simplify to (by dividing top and bottom by 50)
100 2
• Approximate 128 to 130
470 • Mathematics

1
• Answer is × 130 = 65
2
• Accurate calculator answer: 71.26 to 2 decimal places

Ratio
Ratio is a comparison of the size of one quantity compared with another quantity or
other quantities of the same kind. It is an expression of how the parts of the total
compare with each other. It is sometimes written as a fraction.

As it is a means of comparing the size of one quantity to another it is often convenient


to write the ratio in terms of unity. The ratio sign is the colon (:)

Notation

If a quantity has to be divided into three parts where the second part is twice the first,
and the third is three times the first, the quantity is said to be divided in the ratio 1 to 2
to 3. This is shortened to 1 : 2 : 3.

Example

(a) Divide 1,500 in the ratio 2 : 3 : 5


1500
There are 2 + 3 + 5 = 10 parts so 1 part equals = 150
10
If follows that 2 parts equal 300, 3 parts equal 450 and 5 parts equal 750
So 1,500 is split 300 : 450 : 750
As a check, the 3 parts total 1,500
(b) The lengths of two bars are 250 mm and 2 m respectively. In terms of the same units
the ratio of the first to the second may be written
250 : 2000
or 1 : 8
1
Alternatively, the first is of the second.
8
(c) The mass of a solid shaft 300 mm diameter is 1200 kg. Another shaft is hollow,
300 mm outside diameter and 150 mm bore (diameter of the hole is 150 mm) and is
the same length as the first shaft. If the mass of the second shaft is 900 kg, express
the ratio of the mass of the hollow shaft to the mass of the solid shaft.
Appendix 1 – Arithmetic • 471

Ratio of masses, hollow : solid


900 : 1200
that is, 3 : 4
3
or, : 1
4

Proportion

Proportion is an equation of ratios. In other words, it expresses the ratio of one pair of
quantities as being equal to another ratio.

The proportion symbol is the double colon (::)

Example

(a) The ratio 5 : 10 is the same as the ratio 20 : 40, and so this may be written as

5 : 10 :: 20 : 40
or 5 : 10 = 20 : 40 (#)
5 20
or =
10 40

Note: In (#) the two inside terms multiplied equals the two outside terms multiplied,
that is, 10 × 20 = 5 × 40.
This is useful when one of the terms is unknown.
(b) A pump takes 55 min to deliver 4400 litres of water. Under similar conditions, how
long would it take to deliver 6000 litres?
Let the time to deliver 6000 litres equal t.
Ratio of times taken = Ratio of volumes delivered

So 55 : t :: 4400 : 6000
Therefore t × 4400 = 55 × 6000
55 × 6000 330000
t= = = 75 min
4400 4400

(c) The ratio of the areas of two circles is equal to the ratio of the squares of their diam-
eters. The diameter of one circle is 20 mm and its area is 314 mm2 , another circle
has a diameter of 30 mm. What is its area?
472 • Mathematics

Let A represent the area of the second circle.

Ratio of areas :: ratio of diameters2


314 : A :: 202 : 302
314 : A :: 400 : 900
A × 400 = 314 × 900
314 × 900 314 × 9
A= =
400 4
A = 706.5 mm2

Inverse proportion

The above are examples of cases of direct proportion because, in the pump question
for instance, an increase in time of running results in an increase in the quantity of water
delivered. Similarly a decrease in running time means a decrease in water delivered. So
the quantity of water delivered varies directly as the time.

There are situations where the increase in one quantity means a decrease in another.
These are examples of indirect proportion where one quantity varies inversely as the
other. Suppose one pump could empty a tank in 20 min, then two similar pumps work-
ing together would empty the tank in half the time, that is, 10 min. Here the greater the
number of pumps, the less time taken; the time varies inversely as the number of pumps

Method of unity

It is often more convenient to use the ‘method of unity’ to proportion problems, espe-
cially when dealing with compound proportion where there are more than two pairs of
quantities.

Example

If 8 men can build 2 engines in 18 days, how long would it take 12 men, working at the
same rate, to build 5 similar engines?

Time for 8 men to build 2 engines = 18 days

Time for 1 man to build 2 engines = 18 × 8 days


Appendix 1 – Arithmetic • 473

18 × 8
Time for 1 man to build 1 engine = days
2
18 × 8
Time for 12 men to build 1 engine = days
2 × 12
18 × 8
Time for 12 men to build 5 engines = × 5 = 30 days
2 × 12

Example

A general service pump can empty a tank in 10 h and the ballast pump can empty the
same tank in 5 h. If both pumps are working together, how long will it take to empty the
tank?

Quantity pumped by a g.s. pump in 10 h = whole tank


1
Quantity pumped by g.s. pump in 1 h = of tank
10
Quantity pumped by ballast pump in 5 h = whole tank
1
Quantity pumped by ballast pump in 1 h = of tank
5
1 1
Quantity pumped by both pumped in 1 h = + of the tank
10 5
3
= of the tank
10
10
Therefore time to empty the tank = h = 3 h 20 min.
3

Variation

Variation is a further step in ratio and proportion.

As seen before, the amount of water delivered by a pump is directly proportional to the
time.

This is written with the symbol α meaning ‘is directly proportional to’.

So Volume α Time

Also, the time taken to travel a certain distance is inversely proportional to the speed
1
So time α
speed
474 • Mathematics

Example

The cost of a propeller of 2 tonnes mass is £8000 and that of one of 3 tonnes mass is
£12000.
cost 8000
First propeller, = = £4000/tonne
mass 2
cost 12000
Second propeller, = = £4000/tonne
mass 3
cost
Hence, = constant(cost per tonne)
mass
cost1 cost2
Therefore, =
mass1 mass2

Example

A train travels a certain distance in 5 h when travelling at an average speed of 60 km/h;


another train takes 4 h over the same journey travelling at an average speed of 75 kh/h.

1st train, speed × time = 5 × 60 = 300 km

2nd train speed × time = 4 × 75 = 300 km

Hence, speed × time = constant (the distance)

Therefore Speed1 × Time1 = Speed2 × Time2

To sum up, when two quantities:

• vary directly – their quotient is constant,


• vary inversely – their product is a constant.

Example

Within a certain range, the power of an engine varies directly as the mass/h of fuel
burned. An engine uses 180 kh/h of fuel when the power developed is 800 kW, what
will be the fuel consumption when the power is 900 kW?

Fuel/h α power

fuel/h
Therefore = constant
power
fuel/h1 fuel/h2
Therefore =
power1 power2
Appendix 1 – Arithmetic • 475

180 fuel2
So =
800 900
180
fuel2 = × 900 = 202. 5kg/h
800

Example

If a gas is maintained at the same temperature its pressure varies inversely as its volume.
6 m3 of gas at atmospheric pressure is compressed until its pressure is four times as
much, the temperature remaining unaltered; find the volume of the compressed gas.

1
Pressure α
Volume
Therefore Pressure × Volume = constant
Therefore p1 × V1 = p2 × V2
1 × 6 = 4 × V2
1×6
V2 =
4
V2 = 1. 5 m3

Example

The resistance of a conducting wire varies directly as its length and inversely as its cross-
sectional area. A wire 100 m long and 1 mm2 cross-sectional area has a resistance of
2 Ω. What would be the resistance of a wire of similar material 250 m long and 0.5 mm2
cross-sectional area?
1
Resistance α length also resistance α
area

Resistance × Area
Therefore = constant
Length
R1 × A1 R2 × A2
Therefore =
L1 L2
2 × 1 R2 × 0. 5
=
100 250
2 × 1 × 250
R2 =
0. 5 × 100
R2 = 10 Ω

Note: Even though the unit of area is mm2 the length does not have to be in millimetres.
The only necessity is that both areas have the same units, as do both lengths.
476 • Mathematics

Percentages
Percentage is basically a fraction whose denominator is 100.

Example
21
21% has the same value as or 0.21
100

Representation

1
1% is expressed as a fraction is . So 1% = 0.01
100
The following shows how identical values are expressed in different ways:

The ratio of 4 to 25, that is, 4 : 25


4
can be written as in fractional form
25
16
which is equal to when the denominator is 100
100
and is written 16% in percentage form.

A fraction can be converted to a percentage by multiplying by 100 and a percentage


can be converted to a fraction by dividing by 100 (and cancelling).

Example
1 1
= × 100 = 50%
2 2
1 1 1
= × 100 = 33 %
3 3 3
3 3 1
= × 100 = 37 %
8 8 2

Percentage increase or decrease

To express percentage increase or decrease use:


Total increase
% Increase = × 100
Original amount
Appendix 1 – Arithmetic • 477

Total decrease
% Decrease = × 100
Original amount

Example

A spring of length 80 mm is stretched to a length of 100 mm. What is the percentage


increase in length?

Actual increase = 20 mm

20
% Increase = × 100 = 25%
80

Another spring of length 100 mm is compressed to a length of 80 mm. What is the


percentage decrease in length?

Actual decrease = 20 mm

20
% Decrease = × 100 = 20%
100

It is worthwhile to think about these two examples carefully. It is all too easy to say if the first
is increased by 25% then the second must be decreased by the same percentage as the two
examples are the reverse of each other.

Example

The ratio of the mass of engine A to engine B is 5 : 3.

Find (i) by what percentage is A’s mass more than B’s mass, (ii) by what percentage is B’s
mass less than A’s mass.

Difference = 5 − 3 = 2

2 2
(i) % increase of A over B = × 100 = 66 %
3 3
2
So A has a mass that is 66 % greater than that of B
3
2
(ii) % decrease of B over A = × 100 = 40%
5
So B has a mass that is 40% less than that of A.
478 • Mathematics

Percentages of quantities

Example
2
(a) Calculate 24% of
5
24
(i) 24% =
100
2 24 2
(ii) 24% of = × (note that ‘of’ becomes ‘ב)
5 100 5
(iii) To multiply fractions – multiply the numerators, and multiply denominators
24 × 2 48
=
100 × 5 500
48 12
(iv) Simplify =
500 125

Constituent Parts
The parts which make up a whole unit, such as the quantities of metal mixed together
to form an alloy, may be written in the form of ratios, fractions or percentages, and it is
often useful to convert one form into another when solving problems.

Consider a brass alloy consisting of 2 parts copper to1 partzinc. So in 3 g of brass there
2
is 2 g of copper and 1 g of zinc, therefore two-thirds 66 % of the total mass is copper
  3
1
and the remaining one-third 33 % is zinc.
3
To convert a ratio of parts into fractional amounts, add the ratios to make the com-
mon denominator and so each ratio becomes a fraction with that denominator. These
fractions can then be multiplied by 100 to convert the fractions into percentages.

Example

An alloy is composed of 5 parts bismuth, 3 parts lead and 2 parts tin.

Express this ratio as fractional quantities and percentages, and find the mass of each
element required to produce 1.5 kg of alloy.

Ratio of bismuth : lead : tin = 5 : 3 : 2


Sum of ratios = 5 + 3 + 2 = 10
Appendix 1 – Arithmetic • 479

5 1
Mass of bismuth = = = 50% of total mass
10 2
3
Mass of lead = = 30% of total
10
2 1
Mass of tin = = = 20% of total mass
10 5

To produce 1.5 kg = 1500 g of alloy:


1
Mass of bismuth = × 1500 = 750 g
2
3
Mass of lead = × 1500 = 450 g
10
2
Mass of tin = × 1500 = 300 g
10
Example

In a certain two-cylinder engine, the power developed in No. 1 cylinder is 20% more
than the power developed in No. 2 cylinder. What percentage of total power is
developed in each cylinder?

Ratio of powers No. 1 : No. 2 = 120 : 100


= 1.2 : 1
Sum of powers = 1. 2 + 1 = 2.2
1. 2
% Power developed by No. 1 cylinder = × 100 = 54.55
2. 2
1
% Power developed by No. 2 cylinder = × 100 = 45.45
2. 2

Example

Of the total heat in the fuel burned in the cylinders of a diesel engine, 40% is converted
into indicated power, 30% is carried away by the cooling water, 2% is lost by radiation
and the remainder is carried away by the exhaust gases.

(i) What percentage of the total heat of the fuel does the heat in the exhaust gases
represent?
(ii) If 80% of the indicated power is usefully imparted to the crankshaft as brake power
and the remainder lost in friction, what percentage of the total heat of the fuel does
the brake power represent?
(iii) If 40% of the heat in the exhaust gases is recovered in an exhaust gas boiler and the
remainder carried away up the stack, what percentage of the total heat of the fuel
is recovered in the boiler?
480 • Mathematics

The full amount of heat in the fuel is represented by 100%.

Heat converted to indicated power = 40%


Heat carried away in cooling water = 30%
Radiation loss = 2%
Total = 72%
Remainder = 100 − 72
= 28%
So exhaust gases carry away 28% of the total heat
brake power = 80% of 40%
80
= × 40%
100
= 32%
Heat equivalent of brake power = 32%
(Friction loss represents 40 − 32 = 8% of the total heat)
Recovery in the boiler = 40% of 28%
40
= × 28%
100
= 11.2%
(Heat carried away up the stack 28 − 11. 2 = 16. 8%)

A ‘heat-balance’ diagram can now be drawn to illustrate the distribution of heat in the
fuel.

Total heat in fuel


supplied to engine
100%

IP Cooling water Exhaust gases Radiation


40% 30% 28% 2%

Recovered Lost up
BP Friction
in boiler funnel
32% 8%
11.2% 16.8%
Appendix 1 – Arithmetic • 481

Averages
There are several ways of expressing an average. The most common way is called the
‘arithmetic mean’, or just the mean. This is a single value which is used to represent
the numbers in a group, because it is a measure of where the ‘middle’ of the group is
located.

The mean is calculated by adding up all the numbers in a group, then dividing by how
many numbers there are in the group.

Example

(a) Find the average, or mean, of the following numbers: 3, 5, 3, 7, 4

3+5+3+7+4 22
Mean = = = 4. 4
5 5

(b) Find the average, or mean, of the following numbers: 2, 3, 4, 2, 1, 30

2 + 3 + 4 + 2 + 1 + 30 42
Mean = = =7
6 6

Note how the large value of 30 has made the mean higher than the rest
of the numbers in the list. So the mean is affected by very large (or small)
values.
(c) The distances covered by a ship on four successive days were 320, 300, 310 and 330
nautical miles (nm). Find the average day’s run.

320 + 300 + 310 + 330 1260


Average day’s run = = = 315 nm
4 4

(d) In a test of tensile strength between two metals, six samples of each metal are
tested. Which metal has the higher average tensile strength?
482 • Mathematics

Metal A Metal B
7 7
15 6
8 10
9 11
12 19
13 20

7 + 15 + 8 + 9 + 12 + 15 7 + 6 + 10 + 11 + 19 + 20
Mean = Mean =
6 6
64 73
= =
6 6
= 10. 6 = 12. 16

Metal B has the higher average tensile strength.

The following example makes an important point:

A ship travels a distance of 15 nm upriver, against the current, in 72 min and then makes
the return journey, with the current, in 50 min.

Find (i) the average speed up-river,

(ii) the average speed down-river,

(iii) the average speed over the whole journey.

Distance
Speed =
Time
72 6
(i) 72 min = = h
60 5
15 5 75
Speed = = 15 × = = 12. 5 knots
6/5 6 6

50 5
(ii) 50 min = = h
60 6
15 6 90
Speed = = 15 × = = 18 knots
5/6 5 5
Appendix 1 – Arithmetic • 483

122
(iii) Total journey time = 72 + 50 = 122 min = h
60
30 60 1800
Speed = = 30 × = = 14. 75 knots (accurate to 2 decimal
122/60 122 122
places)

Note: The average speed for the whole journey is NOT the average of the two parts.
Distance
This example shows that Speed = . The units of speed are usually km/hr
Time
or m/s.

Speeds of ships and aircraft are usually measured in knots where one knot is one
nautical mile per hour. One nautical mile is approximately 1852 metres or 1.852 km

Standard form

Very large and very small numbers are difficult to manipulate.

Example

The speed of light in a vacuum is about 300 000 000 m/s; the distance of the Earth from
the Sun is about 149 500 000 km; the mass of an electron is about 0.000 000 000 000
000 000 000 000 000 911 grams.

Numbers written like this are difficult both to write and to manipulate, and so they are
normally written in standard form. All modern calculators have the ability to display
numbers in this way.

Examples of numbers in standard form are 3 × 105 , 5 × 106 and 7 × 10−3 , that is, A × 10n
where A is a number written with one digit before the decimal point and n is a positive
or negative whole number.

(The number A is called the mantissa and the number n is called the exponent.)

For example,

2700 = 2. 7 × 103
270 = 2. 7 × 102
27 = 2. 7 × 101
484 • Mathematics

and

0. 27 = 2. 7 × 10−1
0. 027 = 2. 7 × 10−2
0. 0027 = 2. 7 × 10−3 ; and so forth.

To put a number in standard form: place a decimal point after the first non-zero digit and
count how many places the point has been altered.

If the number is greater than 10 the power is positive; less than 1 the power is negative.

Therefore the speed of light is about 3 × 108 m/s; the distance from the Earth to the Sun
is about 1. 495 × 108 km; the mass of an electron is 9. 11 × 10−28 g.

Multiplication and division of numbers in standard form

The powers of 10 obey the rules of indices, so they are added when numbers are
multiplied and subtracted when numbers are divided. So
   
5 × 104 × 3 × 103 = (5 × 3) × 10(4+3) = 15 × 107

However, this is not in standard form as the first number is not between 1 and 10.
   
However, 15 = 1. 5 × 101 so the answer is 1. 5 × 101 × 107 = 1. 5 × 108
   
5 × 105 ÷ 3 × 106 = (5 ÷ 3) × 10(5−6) = 1. 667 × 10−1
   
3 × 104 ÷ 5 × 106 = (3 ÷ 5) × 10(4−6) = 0. 6 × 10−2

Again, this is not in standard form.


   
However, 0. 6 = 6. 0 × 10−1 , so the answer is 0. 6 × 10−1 × 10−2 = 0. 6 × 10−3 .

Addition and subtraction of numbers in standard form

If the exponents are the same for all the numbers in the calculation there is no problem.

Example
   
5 × 102 + 4 × 102 = (5 + 4) × 102 = 9 × 102
Appendix 1 – Arithmetic • 485

(The same applies to subtraction and it may be possible that the initial solution is not in
standard form. See above notes on multiplication and division.)

If the exponents are different to begin with they have to be made equal before the
calculation can proceed.

Example
           
5 × 104 + 3 × 103 = 5 × 101 × 103 + 3 × 103 = 50 × 103 + 3 × 103
= 52 × 103 but 52 × 103 = 5. 2 × 104

Another method is to write all the standard forms as ‘ordinary numbers’, add or subtract
them and then write the answer in standard form.

Either method is acceptable; pick the one you feel is easiest.

Example
   
5 × 104 + 3 × 103 = 50000 + 3000 = 53000 = 5. 3 × 104

Worked problems

1. Express in standard form:

(a) 56.86 (b) 1365 (c) 0.00876

Solution
(a) 56.86 must be divided by 10 to leave one digit before the decimal point and it must
be multiplied by 10 to keep the same value.
56. 86
So 56. 86 = × 10 = 5. 686 × 101
10
1365
(b) 1365 = × 1000 = 1. 365 × 103
1000
1000 1
(c) 0. 00876 = 0. 00876 × = 8. 76 × 2 = 8. 76 × 10−2
1000 10
2. Express the following numbers, which are in standard form, as decimal numbers:

(a) 1. 542 × 10−2 (b) 1. 246 × 105

Solution
1. 542
(a) 1. 542 × 10−2 = = 0. 01542
100
(b) 5. 843 × 106 = 5. 843 × 1000000 = 5843000
486 • Mathematics

Exercise 2
1. A carrier wave has a wavelength of 1200 m and a frequency of 9 × 104 Hz. Calculate
the speed of the wave if speed = wavelength × frequency, where speed is in m/s,
 
wavelength is in metres and frequency is in Hertz. 1. 08 × 108 m/s
2. The force on an area of 1500 m2 of a ship’s hull due to water pressure is 2. 4 × 108 N.
Force  
Calculate the average pressure, where pressure = 1. 6 × 105 N/m2
Area
3. A laser pulse, speed 3 × 108 m/s, is fired at the Moon, 3. 9 × 108 m from the Earth.
Calculate the time taken for the pulse to reach the Moon. (1. 3 s)
4. There are about 1. 5 × 1024 molecules in 5 g of hydrogen gas. Calculate the number
 
of molecules in 6 kg of hydrogen gas. 1. 8 × 1027
5. In an experiment to estimate the diameter of an oil molecule, a drop of oil falls onto
still water where it spreads into a circle.
Volume of drop
If diameter d = , calculate d if Volume = 4 × 10−10 m3 and
Area of circle
Area = 8 × 10−4 m2 . (5 × 10−7 m)
6. A small cube has sides of length 5 mm. What is the volume of the cube in m3 ? What
is the volume (in m3 ) of one billion of these cubes? (One billion = 1 000 000 000)
(1.25×10−7 m3 , 125 m3 )

Engineering notation is similar to standard form except that the powers can only be
multiples of 3. This allows the following prefixes to be used.

When multiplying by Prefix used Abbreviation


109 Giga G
106 Mega M
103 Kilo K
10−3 Milli m
10−6 Micro μ
10−9 Nano N
10−12 Pico p
APPENDIX 2
COMPLEX NUMBERS
It is not possible to find the value of the square root of a negative number.

In order to try to find a meaning for roots of this type −1 is represented by j.

Definitions
Expressions such as 2 + j 3 are called complex numbers. The number 2 is called the real
part and j 3 is called the imaginary part.

The general expression for a complex number is x + jy, which is said to be in rectangular
form.

Complex Conjugate
If Z = x + jy, the complex conjugate Z = x − jy.

If a complex number z = x+jy is multiplied by its conjugate the result is the real number
x 2 + y2 .

Division
To divide complex numbers the numerator and denominator are multiplied by the
conjugate of the denominator.
488 • Mathematics

4+j5 (4 + j 5) (1 + j)
For example =
1−j (1 − j) (1 + j)
4+j4+j5−5 −1 + j 9
= = = −0. 5 + j 4. 5
1+1 2

Questions

1. Add the following:


(a) 3 + j 5, 7 + j 3 and 8 + j 2 (b) 2 − j 7, 3 + j 8 and −5 − j 2

2. Subtract the following:


(a) 3 + j 5 from 2 + j 8 (b) 7 − j 6 from 3 − j 9

3. Simplify
(a) (3 + j 3)(2 + j 5) (b) (2 − j 6)(3 − j 7) (c) (4 + j 5)2
1 2+j5 −2 − j 3 7+j3
(d) (e) (f ) (g)
2+j5 2−j5 5−j2 8−j3
(1 + j 2)(2 − j) 4+j2
(h) (i)
(1 + j) (2 + j)(1 − j 3)
4. Solve the following equations:
(a) x 2 + 2x + 2 = 0 (b) x 2 + 9 = 0
1
5. Find the admittance of a circuit if Y = where Z = 1. 3 + j 0. 6
Z
6. Three impedances Z1 , Z2 and Z3 are represented by the complex numbers
Z1 = 2 + j, Z2 = 1 + j and Z3 = j 2.
Find the equivalent impedance Z if:
(a) Z = Z1 + Z2 + Z3
1 1 1 1
(b) = + +
Z Z1 Z2 Z3
1
(c) Z = + Z3
1 1
+
Z1 Z2

Diagrammatic Representation
Complex numbers can be represented by lines on an Argand diagram. The real part of
the number is the horizontal ordinate, while the imaginary part is the vertical ordinate.
The complex number is drawn as a straight line from the origin to this point.
Appendix 2 – Complex Numbers • 489

The polar form of a complex number

z = x + jy
= r · cos θ + j(r · sin θ )
= r(cos θ + j sin θ )

The expression r(cos θ + j sin θ ) is known as the polar form of the complex number z.
Using conventional notation it may be shown abbreviated as r∠θ .

r is called the modulus of the complex number z and is denoted by mod z or |z|.

The angle θ is called the argument of the complex number z, and is denoted by arg z.
 
√ 4
If z = 3 + j 4, |z| = 32 + 42 = 5 and θ = tan−1 = 53. 13◦.
3
To multiply numbers in polar form, multiply the moduli and add the arguments.

To divide numbers in polar form, divide the moduli and subtract the arguments.

Questions
1. Find the moduli and arguments of the complex numbers
4 + j 3 and 4 − j 3.
2. Express each of the complex numbers in polar form:
(a) 3 − j4 (b) 3 − j3 (c) −3 + j 3
(d) −2 − j (e) j 4 (f ) −j 3. 5
3. Express the following complex numbers in rectangular form:
(a) 3∠45◦ (b) 5∠154◦ (c) 4.6∠ − 20◦
(d) 3.2∠ − 120◦
4. Simplify, answering in polar form:
(a) 8∠30◦ × 7∠40◦ (b) 2∠ − 20◦ × 5∠ − 30◦
(c) 5∠120◦ × 3∠ − 30◦ (d) 7∠ − 50◦ × 3∠ − 40◦
5. Answer in polar and rectangular forms:
8∠20◦ 10∠ − 40◦ 3∠ − 15◦ 1. 7∠35. 28◦
(a) ◦
(b) ◦
(c) (d)
3∠50 5∠20 5∠ − 6◦ 0. 6∠ − 9. 37◦
6. Three complex numbers z1 , z2 and z3 are given in polar form as 3∠35◦, 5∠28◦ and
2∠ − 50◦ respectively. Find in rectangular form:
z1 × z2
(a) z1 × z2 × z3 (b)
z3
490 • Mathematics

Powers

The product of two numbers z1 = |z1 | ∠θ1 and z2 = |z2 | ∠θ2 is given by
z = |z1 · z2 | ∠ (θ1 + θ2 ).
   
So if z1 = z2 then z2 = z2  ∠2θ ; similarly z3 = z3  ∠3θ etc.
 
In general, zn = zn  ∠nθ
 
In rectangular form this becomes zn = zn  (cos (nθ ) + j sin (nθ ))

This equation is known as De Moivre’s Theorem. The power, n, can be either positive or
negative, integer or fraction.

Example

Find the solutions, in polar degree form, to the equation z3 = 1, that is, find the 3 cube
roots of 1.

Solution

In polar form 1 = 1∠0◦ or 1 = 1∠360◦ or 1 = 1∠720◦

The angle refers to 3θ so θ = 0◦ , 120◦ and 240◦

The roots are therefore 1 = 1∠0◦ or 1 = 1∠120◦ or 1 = 1∠240◦

In rectangular form these are

1 · (cos 0 + j sin 0) = 1

1 3
1 · (cos 120 + j sin 120) = − + j ·
2 √
2
1 3
1 · (cos 240 + j sin 240) = − − j ·
2 2

The roots are equally spaced around 360◦ starting at the angle of the first solution.

Example

Find the solutions to z5 = 1 + j.

Solution
   
√ √ 45
In polar form 1 + j = 2∠45◦ so the first root is 5
2 ∠ = 1. 07∠9◦ 360◦
5
divided by 5 gives 72◦
Appendix 2 – Complex Numbers • 491

The second root is therefore 1. 07∠81◦, the third 1. 07∠153◦, the fourth 1. 07∠225◦ and
the fifth 1. 07∠297◦.

That is, 1. 06 + j 0. 17, 0. 17 + j 1. 06, −0. 96 + j 0. 49, −0. 76 − j 0. 76 and 0. 49 − j 0. 96

The exponential form of a complex number

Since z can be expressed as z = |z| (cos (θ ) + j sin (θ )) power series expansions for
cos(θ ) and sin(θ ) mean that z can also be expressed as
   
θ2 θ4 θ3 θ5
z = |z| 1 − + − ··· + j θ − + − ···
2! 4! 3! 5!
 
θ2 θ3 θ4 θ5
= |z| 1 + jθ − −j + + j ···
2! 3! 4! 5!
Since j2 = −1, j3 = −j, j4 = 1, j5 = j this expansion can be rewritten as

(j θ )2 (j θ )3 (j θ )4 (j θ )5
|z| 1 + (j θ ) + + + + ···
2! 3! 4! 5!

This is the power series expansion of ex with x replaced by j θ .

Therefore z = |z| · ej θ . This is the exponential form of a complex number.

Since z = |z| (cos (θ ) + j sin (θ )) the implication is that e j θ = cos (θ ) + j sin (θ )

This is known as Euler’s formula.

By replacing θ with different values certain identities may be discovered:

Replacing θ with −θ gives e−j θ = cos (θ ) − j sin (θ )

Letting θ = π gives the result e j π = −1, an amazing result!!

Since ea+jb = ea · ejb , ea+jb = ea (cos (b) + j sin (b))

Similarly, ea−jb = ea (cos (b) − j sin (b))

Example

Find the exponential form of 2 + j 2

Solution
 
√ √ 2 π
In polar form the modulus is 22 + 22 = 8 and the angle is tan−1 =
2 4
√ π
So the exponential form is 8ej 4
492 • Mathematics

Example

Find the rectangular form of e2−j π

Solution
e2−j π = e2 · e−j π = e2 (cos (π) − j sin (π)) = e2 (−1) = −e2

Euler’s formula gives e j θ = cos (θ ) + j sin (θ ) so e−j θ = cos (θ ) − j sin (θ )


1 jθ
Adding these equations gives cos (θ ) = e + e−j θ
2
1 jθ
Subtracting gives sin (θ ) = e − e−j θ
2j

Questions on complex arithmetic


1. Find the modulus and arguments (lengths and angles) of the following complex
numbers:
(i) 4 + j 3 (ii) −2 + j (iii) 3 − j 4 (iv) −3. 5 − j 2
(v) j 3 (vi) −j 4 (vii) −4 − j 2 (viii) −3 + j 3
2. Convert the following complex numbers into rectangular form:
(i) 3∠45◦ (ii) 5∠154◦ (iii) 4. 6∠ − 20◦
(iv) 3. 2∠ − 120◦ (v) 5. 5∠55◦
3. Multiply each pair of complex numbers, answering in polar and rectangular form:
(i) 8∠30◦ × 7∠40◦ (ii) 2∠ − 20◦ × 5∠ − 30◦
(iii) 5∠230◦ × 3∠ − 30◦ (iv) 7∠ − 50◦ × 3∠ − 40◦
4. Divide each pair of complex numbers, answering in polar and rectangular form:
8∠20◦ 10∠ − 40◦ 3∠ − 15◦ 1. 7∠35. 28◦
(i) ◦
(ii) ◦
(iii) ◦
(iv)
3∠50 5∠20 5∠ − 6 0. 6∠ − 9. 37◦
5. Three complex numbers are given in polar form where

z1 = 3∠35◦ , z2 = 5∠28◦ and z3 = 2∠ − 50◦ .

Simplify: (i) z1 × z2 × z3 , giving the answer in polar form,


z1 × z2
(ii) , giving the answer in rectangular form.
z3
6. If the complex number z = 2 − j 3 express in polar form:
1 √
(i) (ii) z2 (iii) z
z
7. Find the square root of:
(i) 8∠38◦, answering in polar form,
(ii) 2 + j 3, answering in rectangular form.
Appendix 2 – Complex Numbers • 493

1
8. The admittance Y of a circuit is given by Y = .
Z
(i) If Z = 3 + j 5 find Y in polar form,
(ii) If Z = 17. 4∠42◦ find Y in rectangular form.
9. A simple circuit, which has a resistance R in series with an inductive reactance, XL ,
has impedance Z given by the complex number

Z = R + jXL .

A simple circuit, which has a resistance R in series with a capacitive reactance, XC ,


has impedance Z given by the complex number

Z = R − jXC .

Find the resistance and inductive or capacitive reactance for each of the following
impedances:

(iii) 29. 6∠23. 37◦ (iv) 7∠ − 12
10. The potential difference across a circuit is given by the complex number V = 40 +
j 35 volts and the current is given by the complex number I = 6 + j 3 amperes.
Sketch the appropriate phasors on an Argand diagram and find:
(i) the phase difference (i.e. the angle ϕ) between the phasors for V and I,
(ii) the power, given that power = |V| × |I| × cos ϕ

Solutions to Complex Number Exercise


1. (i) 5∠36. 9◦ (ii) 2. 23∠153. 4◦ (iii) 5∠ − 53. 1◦ (iv) 4. 03∠ − 150. 2◦
(v) 3∠90◦ (vi) 4∠ − 90◦ (vii) 4. 5∠ − 153. 4◦ (viii) 4. 24∠135◦
2. (i) 2. 12 + j2. 12 (ii) −4. 49 + j 2. 19 (iii) 4. 32 − j 1. 57
(iv) −1. 6 − j 2. 77 (v) 3. 15 + j 4. 5
3. (i) 56∠70◦ (ii) 10∠ − 50◦ (iii) 15∠200◦ (iv) 21∠ − 90◦
4. (i) 2. 67∠ − 30◦ (ii) 2∠ − 60◦ (iii) 0. 6∠ − 9◦ (iv) 2. 83∠44. 65◦
5. (i) 30∠13◦ (ii) −2. 93 + j 6. 90
6. (i) 0. 277∠56. 32◦ (ii) 13∠ − 112. 63◦ (iii) 3. 6∠ − 56. 3 and 3. 6∠123. 7◦
7. (i) 2. 83∠19◦ (ii) 1. 674 + j 0. 896
8. (i) 0. 172∠ − 59. 03◦ (ii) 0. 0427 − j 0. 0385
9. (i) R = 4. 5, XL = 2. 2 (ii) R = 23, XC = 35
(iii) R = 27. 2, XL = 11. 7 (iv) R = 6. 85, XC = 1. 46
10. (i) ϕ = 14. 62◦ (ii) Power = 345 W
494 • Mathematics

Alternating Current in a Resistor


Ohm’s Law is obeyed in an a.c. circuit just as it is in a d.c. circuit. So, when a sinusoidal
voltage, V, is applied to a resistor, R, the current flowing in the resistor will be given by:
V
.I=
R
This relationship must also hold true for instantaneous values of current, i, and
voltage, v.
v Vmax · sin (ωt)
So i = and since v = Vmax · sin (ωt), i = .
R R
The current and voltage will both have a sinusoidal shape and, since they rise and fall
together, they are said to be in-phase with one another. This can be represented by the
phasor diagram shown in Figure A2.1.

I V

 Figure A2.1

This diagram shows two rotating phasors (of magnitude I and V) rotating at an angu-
lar velocity w. The applied voltage (V) is referred to as the reference phasor and this is
aligned with the horizontal axis (i.e. it has a phase angle of 0º).

Voltage waveform

Current waveform

θ i v
θ 90 270

 Figure A2.2
Appendix 2 – Complex Numbers • 495

Alternating Current in an Inductor


When a sinusoidal voltage, V, is applied to an inductor, L, the current flowing in the
V
inductor will be given by: I = where X is the reactance of the inductor.
X
π 
The current in the inductor lags behind the voltage by a phase angle of 90º radians
 π
2
Vmax · sin ωt −
and since v = Vmax · sin (ωt), i = 2
X
Once again, the current and voltage will both have a sinusoidal shape but they are 90º
apart and this relationship has been illustrated by means of the phasor diagram shown
in Figure A2.3. The applied voltage (V) is the reference phasor (its phase angle is 0º) whilst
the current flowing (I) has a lagging phase angle of 90º.

 Figure A2.3

Alternating Current in a Capacitor


When a sinusoidal voltage, V, is applied to a capacitor, C, the current flowing in the
V
capacitor will be given by: I = where X is the reactance of the capacitor.
X
π 
The current in the capacitor leads the voltage by a phase angle of 90º radians
2
 π 
Vmax · sin ωt +
and since v = Vmax · sin (ωt), i = 2
X
496 • Mathematics

Once again, the current and voltage will both have a sinusoidal shape but they are 90º
apart and this relationship has been illustrated by means of the phasor diagram shown
in Figure A2.4. The applied voltage (V) is the reference phasor (its phase angle is 0º) while
the current flowing (I) has a leading phase angle of 90º.

I
V

 Figure A2.4

Inductive and Capacitive Reactance

1
XL = 2πfL and XC =
2πfC

Complex Impedances
The j-operator and the Argand diagram provide a useful way of representing
impedances. Any complex impedance may be represented by the relationship:

Z = R ± jX where Z represents the impedance, R the resistance and X the reactance.

The ±j term indicates whether the reactance is due to inductance, in which case the j
term is positive, or whether it is due to capacitance, in which case the j term is negative.

Inductance and resistance in series

Z = R + jXL or Z = R + j ωL where ω = 2πf


Appendix 2 – Complex Numbers • 497

Capacitance and resistance in series

j
Z = R − jXC or Z = R − where ω = 2πf
ωC

Inductance, resistance and capacitance in series

Z = R + jXL − jXC = R + j (XL − XC ) or


 
j 1
Z = R + j ωL − = R + j ωL − where ω = 2πf
ωC ωC

Complex Number Applications


Example

A series connected electrical circuit has resistance of 32 ohms and inductance of 0.15 H.
It is connected to a 200 V, 50 Hz supply. Determine:

(i) the inductive reactance,


(ii) the impedance in rectangular and polar forms,
(iii) the current and the circuit phase angle,
(iv) the voltage drop across the resistor and
(v) the voltage drop across the inductor.

Solution

(i) Reactance, XL = 2πfL = 2π (50) (0. 15) = 47. 1 ohms


(ii) Impedance, Z = R + j XL
that is, Z = 32 + j 47. 1

So, |Z| = 322 + 47. 12 = 56. 94
 
47. 1
and ϕ = tan−1 = 55◦ 48
32
498 • Mathematics

Thus, in polar form, Z = 56. 94 ∠ 55◦48 ohms


V 200∠ 0◦
(iii) Current I = = = 3. 51 ∠ − 55◦ 48
Z 56. 94 ∠ 55◦48
that is, the current is 3.51 A, lagging V by 55◦ 48
(iv) The voltage drop across the 32 ohm resistor, VR = I · R

VR = 3. 51 ∠ − 55◦ 48 (32) = 112. 3 ∠ − 55◦ 48

(v) The voltage drop across the 0.15 H inductor, VL = I · XL

VL = (3. 51 ∠ − 55◦ 48 ) (47. 1 ∠ 90◦) = 165. 3 ∠ 34◦12 volts

Example

A 240 V, 50 Hz voltage is applied across a series connected circuit having a resistance


of 12 Ω, an inductance of 0.10 H and a capacitance of 120 μF. Determine the current
flowing in the circuit.

Solution
XL = 2πfL = 2π (50) (0. 10) = 31. 4 Ω
1 1
XC = = = 26. 5 Ω
2πfC 2π (50) 120 × 10−6
Z = R + j (XL − XC ) = 12 + j (31. 4 − 26. 5) = 12 + j 4. 9
  
4. 9
In polar form, |Z| = 122 + 4. 92 = 13. 0, ϕ = tan−1 = 22◦ 10
12

So Z = 13. 0 ∠ 22◦10 ohms.


V 240 ∠ 0◦
Hence, current I= = = 18. 5 ∠ − 22◦ 10
Z 13. 0 ∠ 22◦10

that is, the current is 18.5 A, lagging the voltage by 22◦ 10

Questions

1. A resistance of 45 ohms is connected in series with a capacitor of 42 μF. If the


applied voltage is 250 V, 50 Hz determine:
(i) the capacitative reactance,
(ii) the impedance,
(iii) the current, and its phase relative to the applied voltage,
(iv) the voltage across the resistance and
Appendix 2 – Complex Numbers • 499

(v) the voltage across the capacitance.


(i) [75.79 ohms] (ii) [88.14 ohms] (iii) [2.836 A, leading by 59◦ 18 ]
(iv) [127.6 V] (v) [214.9 V]
2. A resistance of 24 ohms, an inductance of 0.2 H and a capacitance of 240 μF are con-
nected in series and a voltage of 240 V, 50 Hz applied. Calculate the current flowing
in the circuit and its phase relative to the applied voltage.
[4.36 A, lagging by 64◦ 10 ]

Test Examples – Complex


1. Two complex numbers, Z1 and Z2 , are given as Z1 = 2 − j 3 and Z2 = −1 + j 5.
Find the values of the following complex numbers, giving each answer in both
rectangular and polar forms:
(i) 3 · Z1 + Z2
(ii) Z1 × Z2
Z1
(iii)
Z2
2. A 240 V, 50 Hz voltage is applied across a series connected circuit having a resistance
of 5 Ω, an inductance of 0.2 H and a capacitance of 200 μF.
1
If XL = 2π · f · L and XC = , write down an expression, in complex number
2π · f · C
form, for the total impedance in the circuit.
Hence, determine the current flowing in the circuit and its phase angle relative to
the applied voltage.
3. Find the solutions, in polar degree form, to the equation z6 = 1.

Solutions to Test Example – Complex


1. (i) 3 · Z1 + Z2 = 3(2 − j 3) + (−1 + j 5) = 6 − j 9 − 1 + j 5 = 5 − j 4
In polar form, 6.40 ∠ − 38. 7◦
(ii) Z1 × Z2 = (2 − j 3) × (−1 + j 5) = −2 + j 10 + j 3 − j2 15 = 13 + j 13
In polar form, 18.38 ∠45. 0◦
Z 2−j3 −1 − j 5 −2 − j 10 + j 3 + j2 15
(iii) 1 = × =
Z2 −1 + j 5 −1 − j 5 12 + 52
−2 − j 7 + (−1) 15 −17 − j 7 17 7
= = =− −j
26 26 26 26
500 • Mathematics

In polar form, 0.71 ∠ − 157. 6◦


1 1
2. XL = 2πf · L = 2π · 50. 0. 2 = 62. 83185307 XC = =
2πf · C 2π · 50. 200 × 10−6
= 15. 91549431
So Z = 5 + j 62. 83185307 − j 15. 91549431 = 5 + j 46. 91635876
V 240 240∠0◦
Since I = = = = 5. 087∠ − 83. 92◦
Z 5 + j 46. 9164 47. 182∠83. 92◦
Therefore the current is 5.09 amperes, lagging by 83.92◦
3. In polar form 1 = 1∠0◦ or 1 = 1∠360◦ or 1 = 1∠720◦
The angle refers to 6θ so θ = 0◦ , 60◦ , 120◦, 180◦, 240◦ and 300◦
The roots are therefore 1 = 1∠0◦ , 1 = 1∠60◦ , 1 = 1∠120◦, 1 = 1∠180◦,
1 = 1∠240◦ and 1 = 1∠300◦
In rectangular form these are

1 · (cos 0 + j sin 0) = 1

1 3
1 · (cos 60 + j sin 60) = + j ·
2 2√
1 3
1 · (cos 120 + j sin 120) = − + j ·
2 2
1 · (cos 180 + j sin 180) = −1

1 3
1 · (cos 240 + j sin 240) = − − j ·
2 2

1 3
1 · (cos 300 + j sin 300) = − j ·
2 2
APPENDIX 3
LAPLACE
TRANSFORMS
A differential equation is one that contains an unknown function, y, and its first (and
possibly higher) derivatives.
dy
An example of a first-order differential equation is 2 + 3y = e2t .
dt
d2 y dy
An example of a second-order differential equation is 3 + 4 + 3y = 0.
dt2 dt
These are often exceptionally difficult to solve so the concept of Laplace transforms is
used.

Definition
∞
The Laplace transform of the function f (t) is defined by the integral e−st · f (t)dt,
0

where s is a parameter assumed to be a real number.

If y = f (t) the notation for the Laplace transform can be: L{f (t)}, F(s) or y.

This appears to be a very complex procedure but it is possible to integrate each function
f (t). Due to the complexity of the integration, a set of tables is used to find the transform
of any given function f (t).
502 • Mathematics

Standard Laplace Transforms

Function Laplace transform Function Laplace transform


f (t) or y F(s) or y f (t) or y F(s) or y

1 n!
1 tn (n = 1, 2, 3)
s sn+1
k n!
k eat · tn
s (s − a)n+1
1 ω
eat eat · sin ωt
s−a (s − a)2 + ω2
a s−a
sin(at) eat · cos ωt
s2 + a2 (s − a)2 + ω2
s 2ωs
cos(at) t · sin ωt  2
s + a2
2
s2 + ω 2
a s2 − ω 2
sinh(at) t · cos ωt  2
s − a2
2
s2 + ω 2

s √ 1 π 
cosh(at) t
s2 − a2 2 s3
 
1 1 π
t √
s2 t s
2!
t2
s3

Examples

Find the Laplace transforms of the following:

1. L{6sin(3t) − 4cos(5t)} = 6L{sin(3t)} − 4L{cos(5t)}


  s 
3
=6 2 − 4
s + 32 s2 + 5 2
18 4s
= 2 − 2
s +9 s + 25
1 3 2 3
2. L{2t − 3} = 2L{t} − L{3} = 2 2 − = 2 −
s s s s

1 5
3. L{5e3t } = 5L{e3t } = 5 =
s−3 s−3
Appendix 3 – Laplace Transforms • 503

 s  3s
4. L{3 cos(2t)} = 3L{cos(2t)} = 3 = 2
s2 + 2 2 s +4

Questions

Use Laplace transforms to rewrite the functions of t as functions of s.

Algebraic simplification is essential.

t6
1. 1 + 3t − 2t2 + 2. 3e2t − 4e−5t
12

3. 2 sin 4t − 3 cos 2t 4. t5 − 2t4

5. 3t − 5 6. 4t2 + 2t − 1

7. 3t3 − 4t + 7 8. t6 − 3t4 + t2

9. 2e4t 10. 3e−2t

11. 5 sin 2t 12. 4 cos 3t

13. e2t · t3 14. e3t · sin 4t

15. 2e−2t · cos 2t 16. t · sin 3t

17. t · cos 4t 18. 2t3 · e−2t

19. et · sin t 20. 4e−3t · cos 2t

Solutions
1 3 4 60 3 4 8 3s
1. + 2− 3+ 7 2. − 3. −
s s s s s−2 s+5 s2 + 16 s2 + 4
120 48 3 5 8 2 1
4. − 5 5. − 6. + 2−
s6 s s 2 s s 3 s s
18 4 7 720 72 2 2
7. − 2+ 8. − 5 + 3 9.
s4 s s s7 s s s−4
3 10 4s
10. 11. 12.
s+2 s2 +4 s2 +9
504 • Mathematics

6 4 2s + 4
13. 14. 15.
(s − 2) 4 s2 − 6s + 25 s2 + 4s + 8
6s s2 − 16 12
16.  2 17.  2 18.
s2 + 9 s2 + 16 (s + 2)4

1 4s + 12
19. 20.
s2 − 2s + 2 s2 + 6s + 13

It is also necessary to find inverse transforms, that is, the reverse procedure.

The same set of tables can be used reading from right to left, or the following table may
be used.

Standard Inverse Laplace Transforms

Laplace transforms Function Laplace transforms Function


F(s) or y f (t) or y F(s) or y f (t) or y
1 n!
1 tn (n = 1, 2, 3)
s sn+1
k n!
k eat · tn
s (s − a)n+1
1 ω
eat 3at · sin ωt
s−a (s − a)2 + ω2
a s−a
sin(at) eat · cos ωt
s + a2
2 (s − a)2 + ω2
s 2ωs
cos(at)  2 t · sin ωt
s + a2
2
s2 + ω 2
a s2 − ω2
sinh(at)  2 t · cos ωt
s − a2
2
s2 + ω 2
 
s s a · eat − b · ebt
cosh(at)
s − a2
2 (s − a) (s − b) (a − b)
 at 
1 1 e − ebt
t
s2 (s − a) (s − b) (a − b)
2!
t2
s3
Appendix 3 – Laplace Transforms • 505

Examples

Find the inverse Laplace transform of

1
1.
s2 +9
3
From the table, the inverse transform of = sin(3t).
s2 + 3 2
1 1
Therefore the inverse transform of 2 = sin(3t).
s +9 3
7s
2. 2
s +4
s
The inverse transform of 2 = cos(2t).
s + 22
Therefore the required inverse transform is 7cos(2t).

The First Shift Theorem

If a function f (t) is multiplied by e−at , the effect on the Laplace transform is simply to
replace s with (s + a).
2
Thus the transform of e−3t sin(2t) is
(s + 3)2 + 4
2
The inverse transform of is t2 · e4t .
(s − 4)3

Inverse Laplace Transforms Using


Partial Fractions
Example
4s − 5
Determine L−1
s2 − s − 2

Solution 4s − 5 4s − 5 A B
≡ ≡ +
s2 − s − 2 (s − 2)(s + 1) s−2 s+1

Hence 4s − 5 = A(s + 1) + B(s − 2)


506 • Mathematics

Let x = 2 3 = 3A ⇒ A = 1
Let x = −1 −9 = −3B ⇒ B = 3

4s − 5 1 3
Hence L−1 = L−1 + L−1 = e2t + 3e−t from tables
s2 −s−2 s−2 s+1

Inverse Laplace Transforms Using


‘Completing the Square’
Example
2(s + 1)
Determine L−1
s2 + 2s + 10

Solution

The denominator does not factorise so the technique of ‘completing the square’ is used.
The first two terms, s2 + 2s, are the first two terms of the expansion of (s + 1)2. This gives
s2 + 2s + 1 so the denominator requires the addition of 9, that is, 32 , to equate to the
Laplace transform.

2(s + 1) 2(s + 1)
So L−1 = L−1 = 2e−t cos(3t) from tables.
s2 + 2s + 10 (s + 1)2 + 32

Questions
Find the inverse Laplace Transforms of the following functions of s.

(Solutions are given in brackets.)

2s2 − 9s − 35
1. 4e−t − 3e2t + e−3t
(s + 1)(s − 2)(s + 3)

2s + 3
2. 2e2t + 7te2t
(s − 2)2

5s2 − 2s − 19
3. 2e−3t + 3et − 4tet
(s + 3) (s − 1)2
Appendix 3 – Laplace Transforms • 507

3
4. e2t sin(3t)
s2 − 4s + 13

4
5. 2e2t sin(t)
2s2 − 8s + 10

s+1
6. e−t cos(3t)
s2 + 2s + 10

3 3 −3t
7. e sin(2t)
s2 + 6s + 13 2

2 (s − 3)
8. 2e3t cos(2t)
s2 − 6s + 13

3s + 2 14 4t
9. e sin(3t) + 3e4t cos (3t)
s2 − 8s + 25 3

3s2 + 16s + 15  
10. e−3t 3 − 2t − 3t2
(s + 3) 3

Questions

Find the inverse Laplace transforms of the following functions of s.

1 3
1. 2.
s2 +4 4s − 2

1 2
3. 4.
s3 s4

2s 1
5. 6.
s2 + 9 (s − 2)4

2 s+2
7. 8.
s2 − 6s + 13 s2 + 4s + 20

3s + 1 5s2 − 2s − 4
9. 10.
s2 + 2s − 3 s(s + 1)(s − 2)

4s3 − 14s2 + 3s + 13 5s2 + 7s − 1


11. 12.  
(s + 2)(s − 1)3 (s + 4) s2 + 1
508 • Mathematics

Solutions
1 3 1t 1 2
1. sin(2t) 2. e2 3. t
2 4 2
1 3 1 2t 3
4. t 5. 2 cos(3t) 6. e ·t
3 6

7. e3t · sin(2t) 8. e−2t · cos(4t) 9. 2e−3t + et

10. 2 + e−t + 2e2t 11. 3e−2t + et − 5t · et + t2 · e−t

12. 3e−4t + 2 cos(t) − sin(t)

Example of the occurrence of a practical differential equation.

Newton’s Law of Cooling


Let y be the temperature difference of the mass compared to the background temper-
ature (◦ C) and let t be the time measured in seconds. Suppose also originally the mass
is 600◦ C above the background temperature.
dy
The rate of change of temperature is given by and this is proportional to its
dt
temperature. That is, the hotter it is, the faster it cools.
dy dy
So ∝y→ = −k · y where k is a constant dependant on the rate of cooling. The
dt dt
negative sign is present to indicate cooling, as opposed to heating.
dy
Therefore, + ky = 0, y0 = 600
dt
Taking Laplace transforms of every term gives

(s · y − y0 ) + k · y = 0 → (s · y − 600) + k · y = 0 → s · y − k · y = 600
600
Factorising the left-hand side gives y (s + k) = 600 → y =
s+k
The inverse transform gives the answer as y = 600e−kt

To find the value of k another piece of information is needed, so suppose that after 30 s
the temperature has dropped to 300◦C.
Appendix 3 – Laplace Transforms • 509

ln(0. 5)
Therefore, 300 = 600e−(30k) → e−(30k) = 0. 5 → k = = 0. 02315
−30
The cooling equation is therefore y = 600e−0.02315t which looks like

600
Temperature (above background)

550
500
450 Cooling curve
400
350
300
250
200
150
100
50
0
0 20 40 60 80 100 120 140 160 180 200
Time (s)

Solving Differential Equations Using


Laplace Transforms
Definitions:
dy
(i) L = s · y − y0 where y0 is the value of y when t = 0.
dt
d2 y dy
(ii) L = s2 · y − s · yo − y1 where y1 is the value of when t = 0.
dt2 dt

Example

d2 y dy
Solve 2
+ 6 + 9y = 3e2t , given y0 = 0 and y1 = 0.
dt dt

Solution

(i) Take transforms of everything

2  3
s · y − s · y0 − y1 + 6 (s · y − y0 ) + 9y =
s−2
510 • Mathematics

(ii) Substitute in y0 and y1


  3
s2 · y − s · 0 − 0 + 6 (s · y − 0) + 9y =
s−2
(iii) Simplify
3
s2 · y + 6s · y + 9y =
s−2
f (iv) Factorise out the transform
  3
y · s2 + 6s + 9 =
s−2
(v) Factorise s if possible
3
y · (s + 3)2 =
s−2
(vi) Isolate the y term
3
y=
(s − 2)(s + 3)2
(vii) Write as partial fractions
3 A B C
= + +
(s − 2)(s + 3) 2 s − 2 s + 3 (s + 3)2
Therefore 3 = A(s + 3)2 + B(s − 2)(s + 3) + C(s − 2)

3
Let s = 2 3 = 25A A=
25
3
Let s = −3 3 = −5C C = −
5
3
s2 : 0=A+B B=−
25
3 3 3
− −
So y= 25 + 25 + 5
s − 2 s + 3 (s + 3)2

(viii) Take inverse transforms


3 2t 3 3
y= e − e−3t − t · e−3t
25 25 5
Example

Solve
dy
4 + 6y = 2e4t given y = 5 when t = 0.
dt
Appendix 3 – Laplace Transforms • 511

Solution

Taking Laplace transforms gives

dy
4L + 6L{y} = 2L e4t
dt

1
Therefore, 4(s · y − y0 ) + 6y = 2 · but y0 = 5,
s−4
2
Therefore, 4s · y − 4 × 5 + 6y = and factorising the left-hand side for y gives
s−4

2
y(4s + 6) − 20 = and dividing by 2 gives
s−4
1 1 10
y(2s + 3) − 10 = so y = +
s−4 (s − 4)(2s + 3) (2s + 3)

The first part of the right-hand side gives

1 A B
≡ + ⇒ 1 = A(2s + 3) + B(s − 4)
(s − 4)(2s + 3) s − 4 2s + 3

1
Let x = 4 so 1 = 11A ⇒A=
11
3 11 2
Let x = − so 1 = − B ⇒B=−
2 2 11
Therefore
  
1 1 2 1 1
y= − + 10
11 s−4 11 2s + 3 2s + 3
⎛ ⎞
  
1 1 108 1 1 1 54 ⎜ 1 ⎟
= + = + ⎝
11 s−4 11 2s + 3 11 s−4 11 3⎠
s+
2

1 4t 54 − 3 t
Taking inverse transforms gives y = e + e 2
11 11
Example

Solve
d2 y dy
+ 6 + 8y = 0, given y1 = 8 and y0 = 4
dt2 dt
512 • Mathematics

Solution
Taking Laplace transforms gives
d2 y dy
L + 6L + 8L {y} = 0
dt2 dt
 
This transforms to s2 · y − s · y0 − y1 + 6(s · y − y0 ) + 8y = 0
 
Therefore, s2 · y − 4s − 8 + 6(s · y − 4) + 8y = 0
 
Factorising out y gives y s2 + 6s + 8 − 4s − 8 − 24 = 0
  4s + 32 4s + 32
Therefore, y s2 + 6s + 8 = 4s + 32 ⇒ y = ⇒y=
s2 + 6s + 8 (s + 4)(s + 2)
4s + 32 A B
By partial fractions ≡ +
(s + 4)(s + 2) s+4 s+2
Therefore, 4s + 32 = A(s + 2) + B(s + 4)

Let s = −2 24 = 2B B = 12
Let s = −4 16 = −2A A = −8

8 12  
Therefore, y = − + ⇒ y = −8e−4t + 12e−2t = 4 3e−2t − 2e−4t
s+4 s+2
Example

Solve
d2 y dy
9 2
− 24 + 16y = 0 given y0 = 3 and y1 = 3
dt dt

Solution

Taking Laplace transform gives


 
9 s2 y − sy0 − y1 − 24 (sy − y0 ) + 16y = 0 and substituting y0 and y1
 
9 s2 y − 3s − 3 − 24 (sy − 3) + 16y = 0. Factorising on y gives
   
y 9s2 − 24s + 16 − 27s − 27 + 72 = 0 ⇒ y 9s2 − 24s + 16 = 27s − 45
27s − 45 27s − 45 A B
⇒y= 2 ≡ ≡ +
9s − 24s + 16 (3s − 4)2 (3s − 4) (3s − 4)2
By partial fractions 27s − 45 = A(3s − 4) + B
4
Let s = , −9 = B
3
Coefficient of s: 27 = 3A ⇒ A = 9
Appendix 3 – Laplace Transforms • 513

9 9 3 1
Therefore y = − = −
3s − 4 (3s − 4)2 4 4 2
s− s−
3 3
4 4 4
Taking inverse transforms gives y = 3e 3 t − te 3 t = (3 − t)e 3 t from tables.

Example

Solve
d2 y dy 2 13
2
− 2 + y = 3e4t given y0 = − and y1 =
dt dx 3 3

Solution
  3
Taking transforms gives s2 y − sy0 − y1 − 2(sy − y0 ) + y =
s−4
 
2 13 2 3
Substituting gives s2 y + s − − 2 sy + +y =
3 3 3 s−4
  2 13 4 3
Factorising out y gives y s2 − 2s + 1 + s − − =
3 3 3 s−4
  3 2 17
Therefore y s2 − 2s + 1 = − s+
s−4 3 3
1
3 1 3 (2s − 17)
So y (s − 1)2 = − (2s − 17) ⇒ y = − 3
s−4 3 (s − 4)(s − 1)2 (s − 1)2
1
3 − (2s − 17)(s − 4) A B C
y= 3 ≡ + +
(s − 4)(s − 1) 2 s − 4 s − 1 (s − 1)2

1
Thus [9 − (2s − 17)(s − 4)] ≡ A(s − 1)2 + B(s − 1)(s − 4) + C(s − 4)
3
1
Let s = 4 3 = 9A A=
3
Let s = 1 −12 = −3C C=4

2 2 1
Coefficient of s2 : − = A + B ⇒ B = − − = −1
3 3 3
1
1 4 1
Therefore y = 3 − + ⇒ y = e4t − 1et + 4tet
s − 4 s − 1 (s − 1) 2 3
1
The solution is y = (4t − 1)et + e4t
3
514 • Mathematics

Example
d2 y
Solve + 16y = 10 cos 4t given y0 = 3 and y1 = 4
dt2

Solution

Taking transforms and substituting gives


2  s   10s
s y − 3s − 4 + 16y = 10 · ⇒ y s2 + 16 − 3s − 4 = 2
+4 2 s2 s + 42
2 
3s + 4 10s (3s + 4) s + 42 + 40s
Therefore y = 2 + 2 =  2
s +4 2
s2 + 4 2 s2 + 4 2

(As + B) (Cs + D)
≡ + 2
s2 + 4 2 s2 + 4 2
   
Therefore (3s + 4) s2 + 42 + 10s = (As + B) s2 + 42 + (Cs + D)

Let s = 0 64 = 16B + D
s3 : 3=A
s2 : 4 = B, therefore 64 = 64 + D so D = 0
s: 58 = 16A + C so C = 10

3s + 4 10s 3s 4 10s
y= + 2 = 2 + 2 + 2
s2 + 4 2 s +4
2 2 s + 4 2 s + 4 2
s + 42
2

5
Therefore y = 3 cos 4t + sin 4t + t · sin 4t
4

Questions

Solve the following differential equations:

dy
1. − 4y = 8 given y0 = 2.
dt
dy
2. 2 + 3y = e4t given y0 = 5.
dt
dy
3. − 3y = t · e2t given y0 = 0.
dt
Appendix 3 – Laplace Transforms • 515

dy
4. 2 + 5y = e−2t given y0 = 1.
dt
dy 1
5. 4 − 3y = sin 2t given y0 = .
dt 4
d2 y
6. + 25y = 0 given y0 = −2 and y1 = −20.
dt2
d2 y dy
7. 2
+ 4 + 5y = 0 given y0 = 1 and y1 = −4.
dt dt
d2 y
8. + 4y = 12t given y0 = 0 and y1 = 7.
dt2
d2 y dy
9. + 6 + 9y = 3e2t given y0 = 0 and y1 = 0.
dt2 dt
d2 y dy
10. − 2 + y = et given y0 = −2 and y1 = −3.
dt2 dt

Answers
8 8 8 2
1. (sy − y0 ) − 4y = ⇒ y(s − 4) − 2 = ⇒ y = +
s s s(s − 4) s − 4
8 A B
Let ≡ + ⇒ 8 = A(s − 4) + Bs
s(s − 4) s s−4
Let s = 4 8 = 4B B=2
Let s = 0 8 = −4A A = −2
2 2 2 4 2
So y=− + + = −
s s−4 s−4 s−4 s
So y = 4e4t − 2
1 1
2. 2 (sy − 5) + 3y = ⇒ y(2s + 3) = + 10
s−4 s−4
1 10
⇒y= +
(s − 4)(2s + 3) 2s + 3
1 A B
Let ≡ + ⇒ 1 = A(2s + 3) + B(s − 4)
(s − 4)(2s + 3) s − 4 2s + 3
1
Let s = 4 1 = 11A A=
11
3 11 2
Let s = − 1=− B B=−
2 2 11
1 1 2 1 10 1 1 54 1
So y = · − · + = · + ·
11 (s − 4) 11 2s + 3 2s + 3 11 (s − 4) 11 3
s+
2
516 • Mathematics

1 4t 54 − 3 t
So y= e + e 2
11 11
1 1 1
3. (sy − 0) − 3y = ⇒ y(s − 3) = ⇒y=
(s − 2)2 (s − 2)2 (s − 3)(s − 2)2
1 A B C
Let ≡ + +
(s − 3)(s − 2)2 s − 3 s − 2 (s − 2)2

So 1 = A(s − 2)2 + B(s − 2)(s − 3) + C(s − 3)


Let s = 2 1 = −C C = −1
Let s = 3 1=A A=1
s2 : 0 = A+B B = −1
1 1 1
So y= − − ⇒ y = e3t − e2t − te2t
s − 3 s − 2 (s − 2)2
1 1
4. 2(sy − 1) + 5y = ⇒ y(2s + 5) = +2
s+2 s+2
1 2 1 1
⇒y= + = +
(s + 2)(2s + 5) 2s + 5 (s + 2)(2s + 5) 5
s+
2
1 A B
Let ≡ + ⇒ 1 = A(2s + 5) + B(s + 2)
(s + 2)(2s + 5) s + 2 2s + 5
Let s = −2 1=A A=1
5 1
Let s = − 1=− B B = −2
2 2
1 −2 1 1 −1 1 1
So y= + + = + + =
s + 2 2s + 5 5 s+2 5 5 s+2
s+ s+ s+
2 2 2
So y = e−2t

1 2 2
5. 4 sy − − 3y = ⇒ y(4s − 3) = 2 +1
4 +4s2 s +4
2 1 s2 + 6
⇒y=  2  + = 2 
s + 4 (4s − 3) 4s − 3 s + 4 (4s − 3)
s2 + 6 As + B C
Let   ≡ 2 +
s2+ 4 (4s − 3) s +4 4s − 3
 
So s2 + 6 = (As + B)(4s − 3) + C s2 + 4

3 105 73 105
Let s = = C C=
4 16 16 73
6
Let s = 0 6 = −3B + 4C B=−
73
8
s2 : 1 = 4A + C A=−
73
Appendix 3 – Laplace Transforms • 517

8 6 105
−s−  s  3  2 
So y = 73 73 + 73 = − 8 − +
105 1
s2 + 4 4s − 3 73 s2 + 4 73 s2 + 4 292 s − 3/4
8 3 105 3 t
So y = − cos 2t − sin 2t + e4
73 73 292
   
6. s2 y + 2s + 20 + 25y = 0 ⇒ y s2 + 25 = −(2s + 20)
   
s  5
⇒y=− 2 2 + 4 ⇒ y = − [2 cos 5t + 4 sin 5t]
s + 52 s2 + 5 2
   
7. s2 y − s + 4 + 4(sy − 1) + 5y = 0 ⇒ y s2 + 4s + 5 − s + 4 − 4 = 0

s s s+2 1
⇒y= 2 = = − 2 ·
s + 4s + 5 (s + 2)2 + 1 (s + 2)2 + 1 (s + 2)2 + 1
⇒ y = e−2t cos t − 2e−2t sin t

2  12   12 12 7
8. s y − 7 + 4y = 2 ⇒ y s2 + 4 = 2 + 7 ⇒ y = 2  2 + 
s s s s +4 s +4
2

7s2 + 12 A B Cs + D
⇒y=  ≡ + 2+ 2
s s +4
2 2 s s s +4
   
⇒ 7s2 + 12 = As s2 + 4 + B s2 + 4 + (Cs + D)s2

Let s = 0 12 = 4B B=3
s3 : 0 = A+C
s2 : 7=B+D D=4
s1 : 0 = 4AA = 0 so C = 0
 
3 4 1 2
⇒y= 2 + 2 = 3· + 2 · ⇒ y = 3t + 2 sin 2t
s s +4 s2 s2 + 2 2
3   3 3
9. s2 y + 6sy + 9y = ⇒ y s2 + 6s + 9 = ⇒y=
s−2 s−2 (s − 2)(s + 3)2
3 A B C
Let ≡ + +
(s − 2)(s + 3) 2 (s − 2) (s + 3) (s + 3)2

So 3 = A(s + 3)2 + B(s + 3)(s − 2) + C(s − 2)

3
Let s = 2 3 = 25A A=
25
3
Let s = −3 3 = −5C C=−
5
3
s2 : 0=A+B B=−
25
518 • Mathematics

  
3 1 3 1 3 1
So y = − −
25 s − 2 25 s + 3 5 (s + 3)2
3 2t 3 3
So y = e − e−3t − te−3t
25 25 5
2  1
10. s y + 2s + 3 − 2(sy + 2) + y =
s−1
  1
⇒ y s2 − 2s + 1 + 2s + 3 − 4 =
s−1
1 1
⇒ y(s − 1)2 + 2s − 1 = ⇒ y(s − 1)2 = + 1 − 2s
s−1 s−1
1 1 − 2s
⇒y= +
(s − 1) 3 (s − 1)2
1 − 2s A B
Let ≡ + ⇒ 1 − 2s = A(s − 1) + B
(s − 1) 2 s − 1 (s − 1)2
Let s = 1 −1 = B
s1 : −2 = A
1 1 2 1
⇒y= − − ⇒ y = t2 et − tet − 2et
(s − 1)3 (s − 1)2 (s − 1) 2

Questions

Practical questions (answers in brackets)

1. The oscillations of a heavily damped pendulum satisfy the differential equation

d2 y dy
2
+ 7 + 12y = 0
dt dt
where y cm is the displacement of the bob at time t seconds. The initial displacement
dy
is equal to 3 cm and the initial velocity i. e. is 6 cm/s.
dt   
Solve the equation for y. y = 3 6e−3t − 5e−4t
d2 y dy
2. The differential equation M · 2
+K· + F · y = 0 represents the motion of the
dt dt
pointer of a galvanometer about its point of equilibrium.
M is the moment of inertia of the pointer about its pivot, K is the resistance due
to friction at unit angular velocity and F is the force required to produce unit
displacement.
If M = 0. 006, K = 0.03 and F = 0. 1875 solve the equation for y in terms of t given
that y0 = 0. 2 and y1 = 0.
 
y = e−2.5t (0. 2 cos 5t + 0. 1 sin 5t)
Appendix 3 – Laplace Transforms • 519

3. The equation of motion of a body oscillating on the end of a spring is:


d2 y
+ 225y = 0
dt2
where y is the displacement in metres of the body from its equilibrium position after
time t seconds.
Find y in terms of t given that y0 = 1 and y1 = 0. [y = cos 15t]

Many situations in engineering give rise to simultaneous differential equations.

The approach for solving these systems is roughly as follows:

• Write the equations in terms of their Laplace transforms;


• Simplify the equations in x and y.
• Solve for x and y by normal algebraic techniques.
• Determine the inverse transforms of x and y.

Example

Solve the simultaneous differential equations:


dx dy
+ + 2x = 0
dt dt given that at t = 0, x = 2 and y = 0.
dx dy
+ 4 − 2x = e−2t
dt dt

By the usual notation, L{x} = x and L{y} = y.

Solution

1. Take transforms and substitute initial values.


Transforming gives

(s · x − x0 ) + (s · y − y0 ) + 2x = 0
1
(s · x − x0 ) + 4 (s · y − y0 ) − 2x =
s+2
and substituting initial values gives

(s · x − 2) + (s · y − 0) + 2x = 0
1
(s · x − 2) + 4 (s · y − 0) − 2x =
s+2
So
(s · x − 2) + s · y + 2x = 0
1
(s · x − 2) + 4s · y − 2x =
s+2
520 • Mathematics

2. Collect terms in x and y.

(s + 2) x + s · y = 2
1
(s − 2) x + 4s · y = 2 +
s+2
These are now simultaneous equations in x and y.
3. Solve the equation simultaneously.
Multiply the first equation by 4 and then subtract the equations.
So
4 (s + 2) x + 4s · y = 8 ⇒ (4s + 8) x + 4s · y = 8
1
(s − 2) x + 4s · y = 2 +
s+2
Subtracting gives
1
(3s + 10) x = 6 −
s+2
4. Rearrange to find x.
6 1
x= −
3s + 10 (s + 2) (3s + 10)
6 2
Manipulation gives = which is a standard form.
3s + 10 s + 10/3
1 1/4 −3/4
can be split into partial fractions +
(s + 2) (3s + 10) (s + 2) (3s + 10)
1/4 −1/4
which is equal to + .
(s + 2) (s + 10/3)
2 1/4 −1/4 9/4 1/4
Therefore, x = − + ⇒x= −
s + /3
10 (s + 2) (s + /3) 10 (s + /3) (s + 2)
10

5. Take inverse transforms gives x = 94 e−10t/3 − 14 e−2t


It is now necessary to find y.
Returning to the original transformed equations and multiplying the first by (s − 2)
and the second by (s + 2) gives

(s − 2) (s + 2) x + s (s − 2) y = 2 (s − 2)
1
(s − 2) (s + 2) x + 4s (s + 2) y = 2(s + 2) + · (s + 2)
s+2
2 
(s − 2) (s + 2) x + s − 2s y = 2s − 4
 
(s − 2) (s + 2) x + 4s2 + 8s y = 2s + 4 + 1

When the top equation is subtracted from the bottom equation the result is:
  9 9 3
3s2 + 10s y = 9 ⇒ y =  = =
3s2 + 10s s (3s + 10) s (s + 10/3)
Appendix 3 – Laplace Transforms • 521

Splitting this into partial fractions gives:


9/10 −9/10
y= +
s s + 10/3

Taking the inverse transform of each component gives


9 9 10t 9  10t 
y= − e− /3 = 1 − e− /3
10 10 10
9 −10t/3 1 −2t
Therefore the solutions are x = e − e .
4 4

Question

The following simultaneous differential equations are subject to the given initial condi-
tion x0 = 1 and y0 = 1:

dx dy
3 + 3 − 2x = et
dt dt
dx dy
+2 −y =1
dt dt
(3s − 4)
Show that x = .
(s − 1) (3s − 1)
Find x(t) and hence find y(t) in a manner that does not include finding y.

Answer
dx dy
3 + 3 − 2x = et (1)
dt dt
dx dy
+2 −y =1 (2)
dt dt

Take Laplace transforms


1
3 (s · x − x0 ) + 3 (s · y − y0 ) − 2x =
s−1
1
s · x − x0 + 2 (s · y − y0 ) − y =
s
Substitute initial conditions
1
3 (s · x − 1) + 3 (s · y − 1) − 2x =
s−1
1
s · x − 1 + 2 (s · y − 1) − y =
s
522 • Mathematics

So
1 6s − 5
x · (3s − 2) + y · 3s = +6=
s−1 s−1
1 3s + 1
x · s + y · (2s − 1) = + 3 =
s s
Eliminate y

(6s − 5) (2s − 1)
x · (3s − 2) (2s − 1) + y · 3s (2s − 1) =
(s − 1)
3s + 1
x · 3s2 + y · 3s (2s − 1) = · 3s = 3(3s + 1)
s
Subtracting gives
  (6s − 5) (2s − 1)
x · 6s2 − 7s + 2 − 3s2 = − 3(3s + 1)
(s − 1)
 2   
12s − 16s + 5 − 3 3s2 − 2s − 1
=
(s − 1)
  3s2 − 10s + 8 (s − 2) (3s − 4)
x · 3s2 − 7s + 2 = ⇒ x · (s − 2) (3s − 1) =
(s − 1) (s − 1)

Therefore
(3s − 4)
x=
(s − 1) (3s − 1)
Splitting this into partial fractions gives:
−1/2 9/2 −1/2 3/2
x= + = +
(s − 1) (3s − 1) (s − 1) (s − 1/3)

Taking inverse Laplace transforms gives


1 3 t
x = − et + e /3 (3)
2 2

To find y, use this function in equation (1)

From (3)
dx 1 1 t
= − et + e /3 (4)
dt 2 2

Substituting (3) and (4) into (1) gives:


   
1 1 t dy 1 3 t
3 − et + e /3 + 3 − 2 − et + e /3 = et
2 2 dt 2 2
Appendix 3 – Laplace Transforms • 523

Solving this gives:


dy 1 1 t 1 3 t
= et + e /3 ⇒ y = et + e /3 + c
dt 2 2 2 2
Using the fact that y = 1 when t = 0 gives:
1 t 3 t/3
y= e + e −1
2 2
The solution to the simultaneous equations is therefore:
1 3 t
x = − et + e /3
2 2
1 t 3 t/3
y = e + e −1
2 2

Loaded beams

If a horizontal beam of length 2c is subjected to a vertical load function, F(x), the


resulting deflection y at a point in the beam, distance x from one end, is given by
d4 y
EI = F(x) for 0 < x < 2c
dx 4

E = modulus of elasticity of material of the beam,


I = second moment of area of cross-section of the beam.

d2 y
At any point along the beam, the bending moment = EI ·
dx 2
d3 y
and the shearing force = EI ·
dx 3
Also:

• for cantilever beam


dy
◦ = 0.
at point of support y = 0,
dx
d2 y d3 y
◦ for free end of beam
2
= 0, 3 = 0.
dx dx
• for a supported beam
d2 y
◦ freely supported, at points of support y = 0, =0
dx 2
dy
◦ clamped, at points of support y = 0, = 0.
dx
524 • Mathematics

Example

A beam of length 2c is simply supported at both ends and carries a uniformly distributed
load of W n/m. Find an expression for the deflection y in terms of x, the distance from
one end.

Solution
d4 y
EI =W 0 < x < 2c
dx 4
At x = 0, y = 0, y = 0
At x = 2c, y = 0, y = 0
d4 y d4 y W
EI 4 = W so 4
=
dx dx EI
Taking Laplace transforms gives
W
s4 · y − s3 · y0 − s2 · y1 − s · y2 − y3 =
EI · s
but y0 = 0, y2 = 0, therefore
W
s4 · y − s2 · y1 − y3 =
EI · s
W
Let y1 = A and y3 = B therefore s4 · y = As2 + B +
EI · s
A B W 1 Bx 3 W x4
So y= + 4+ · giving y = Ax + + · (1)
s2 s EI s5 6 EI 24
At x = 2c, y = 0
B8c3 W 16c4 B4c3 W 2c4
Therefore 0 = 2Ac + + · so 0 = 2Ac + + · (2)
6 EI 24 3 EI 3
At x = 2c, y = 0
Bx 3 W x4 Bx 2 W x3 W x2
Now y = Ax + + · so y = A + + · and y = Bx +
6 EI 24 2 EI 6 EI 2
W 2 Wc
So 0 = 2Bc + 2c giving B=−
EI EI
Substituting the value for B in equation (2)
4Wc4 2Wc4 Wc3
0 = 2Ac − + giving A=
3EI 3EI 3EI

Wc3 x Wcx 3 Wx 4
Equation (1) becomes y= − +
3EI 6EI 24EI
Appendix 3 – Laplace Transforms • 525

Wx  3 
Therefore y= 8c − 4cx 2 + x 3 0 < x < 2c
24EI

Test Examples – LAPLACE


1. Find the Laplace transforms of the following functions of t:
(i) 3t4 + 2t · sin(2t)
(ii) e−2t · t · cos (3t)
 3
2t
(iii) + 2 cos (3t) · e3t
3
2. Find the inverse Laplace  transforms of the following functions of s:
s+6 π
(i) 2 −
s − 8s + 25 (s + 3)3
2s2 + 10s + 9
(ii)
s (s + 2)2
3. The concentration, C, of impurities in an oil purifier varies with time t and is
described by the equation:
dC
a· = b + d · m − Cm
dt
where a, b, d and m are constants.
Given that C = 5 when t = 0, also m = 4, a = 2, b = 7 and d = 1, solve the
differential equation for C in terms of t.
4. Find the Laplace transforms of the following functions of t:
(i) 4t2 + 5 cos(3t)
(ii) (3t − 7) H (t − 2)
 4
t
(iii) + 1. 5 sin (4t) · e−3t
5
5. Find the inverse Laplace transforms of the following functions of s:
3 2
(i) 2 −
s + 9 s5
5s2 + 20s + 18
(ii)
s (s + 3)2
6. Use Laplace transforms to find the particular solution of the second-order linear
ODE given below:

d2 y
+ 9y = 3 cos (3t) given that y0 = 2 and y1 = 4
dt2
526 • Mathematics

7. Given that the functions x(t) and y(t) obey the simultaneous equations:

dx
− 6x + 3y = 8et
dt
dy
− 2x − y = 4et
dt

with initial conditions x0 = −1 and y0 = 0,


2
show that the Laplace transform of y(t) is y=
(s − 1)(s − 4)
Hence find the function y(t).
Find also the function x(t) in a way that does not involve finding x.
8. Given the following differential equation with the stated initial conditions:

d2 y
+ 9y = 27t y0 = 3, y1 = 0
dt2

3s2 + 27
(i) Show that y =  
s2 + 9
s2
(ii) Hence find the function y(t)
9. Find the Laplace transform of f (t) = 2e4t + cosh(3t) + e−2t · cos(t) and simplify
where applicable.
3 2 2s − 6
10. Find L−1 4 − + 2
s 3s + 1 s + 9
4 4s + 3
11. Find L−1 +
(s + 1) 3 s2 + 6s + 13

Solutions to Test Examples – LAPLACE


1. (i) f (t) = 3t4 + 2t · sin (2t)

4! 2 × 2s 72 8s
Lf (t) = 3 × 5
+2×  2 = 5 +  2
s s2 + 2 2 s s2 + 4

(ii) f (t) = e−2t · t · cos (3t)

(s + 2)2 − 32 s2 + 4s − 5
Lf (t) =  2 = 2
(s + 2)2 + 32 (s + 4s + 13)
Appendix 3 – Laplace Transforms • 527


2t3
(iii) f (t) = + 2 cos (3t) · e3t
3
2 × 3! (s − 3) 12 2 (s − 3)
Lf (t) = +2× = + 2
3 (s − 3)4 (s − 3)2 + 32 3 (s − 3)4 s − 6s + 9 + 9
4 2s − 6
Lf (t) = + 2
(s − 3) 4 s − 6s + 18
 
s+6 π s+6 1 π
2. (i) F(s) = 2 − = − 2 ×
s − 8s + 25 (s + 3) 3
(s − 4) + 3
2 2 2 (s + 3)3

(s − 4) + 10 1 π
= −2×
(s − 4) + 3
2 2 2 (s + 3)3
 
10 √
f (t) = e4t · cos(3t) + sin (3t) − 2e−3t t.
3

2s2 + 10s + 9
(ii) F(s) =
s (s + 2)2
Using partial fractions:

2s2 + 10s + 9 A B C
= + +
s (s + 2)2 s s + 2 (s + 2)2
⇒ 2s2 + 10s + 9 = A (s + 2)2 + Bs (s + 2) + Cs
3
s = −2 ⇒ 8 − 20 + 9 = −2C ⇒ −3 = −2C ⇒ C =
2
9
s=0 ⇒ 9 = 4A ⇒A=
4
9 1
s2 : ⇒2=A+B ⇒B=2− =−
4 4
9/4 1/4 3/2
F(s) = − +
s s+2
(s + 2)2
9 1 3
f (t) = − e−2t + t · e−2t
4 4 2
dC dC
3. 2 = 11 − 4C ⇒2 + 4C = 11
dt dt
  11   11 11
2 sC − C0 + 4C = ⇒ 2 sC − 5 + 4C = ⇒ 2sC − 10 + 4C =
s s s
11 10s + 11 10s + 11
2sC + 4C = + 10 ⇒ C (2s + 4) = ⇒C=
s s s (2s + 4)
10s + 11 A B
= + ⇒ 10s + 11 = A (2s + 4) + Bs
s (2s + 4) s 2s + 4
11
s = 0 ⇒ 11 = 4A ⇒A=
4
66 84 66 18 9
s = 1 ⇒ 21 = 6A + B ⇒ B = 21 − = − = =
4 4 4 4 2
528 • Mathematics

11/4 9/2 11/4 9/4 11 9 −2t


So C= + = + ⇒C= + e
s 2s + 4 s s+2 4 4
..
.
or
11 11 10
2sC + 4C = + 10 ⇒C= +
s s (2s + 4) (2s + 4)
11 5
⇒C= +
s (2s + 4) (s + 2)
11 A B
= + ⇒ 11 = A (2s + 4) + Bs
s (2s + 4) s (2s + 4)
11
s = 0 : 11 = 4A A =
4
11 11/4 11/2 5
s = −2 : 11 = −2B B = − ⇒C= − +
2 s 2s + 4 s + 2
11/4 11/4 5
= − +
s s+2 s+2
11/4 9/4 11 9 −2t
C= + ⇒C= + e
s s+2 4 4
4. (i) f (t) = 4t2 + 5 cos (3t)

2! s 8 5s
Lf (t) = 4 × +5× 2 = 3+ 2
s 3 s +3 2 s s + 32
(ii) f (t) = (3t − 7) · H (t − 2) = [3 (t − 2) − 1] · H (t − 2)

3 1
Lf (t) = 2
− · e−2s
s s

t4
(iii) f (t) = + 1. 5 sin (4t) · e−3t
5

4! 4 24 6
Lf (t) = + 1. 5 × = +
5 (s + 3) (s + 3)
5 2
+ 42 5 (s + 3)
5 s2 + 6s + 9 + 16
24 6
Lf (t) = + 2
5 (s + 3)5 s + 6s + 25

3 2
5. (i) F(s) = −
s2 + 9 s5
2 4 1
f (t) = sin(3t) − t = sin(3t) − t4
4! 12

5s2 + 20s + 18
(ii) F(s) =
s (s + 3)2
Appendix 3 – Laplace Transforms • 529

Using partial fractions:


5s2 + 20s + 18 A B C
= + + ⇒ 5s2 + 20s + 18
s (s + 3) 2 s s + 3 (s + 3)2
= A (s + 3)2 + Bs (s + 3) + Cs
s = −3 ⇒ 45 − 60 + 18 = −3C ⇒ 3 = −3C ⇒ C = −1
s = 0 ⇒ 18 = 9A ⇒ A = 2
s2: ⇒ 5 = A + B ⇒ B = 3
2 3 1
F(s) = + −
s s + 3 (s + 3)2
f (t) = 2 + 3e−3t − t · e−3t
d2 y
6. + 9y = 3 cos (3t) given that y0 = 2 and y1 = 4
dt2

Taking transforms gives


2  3s
s · y − sy0 − y1 + 9y =
s2 + 3 2
Substituting and expanding:
2  3s   3s
s · y − 2s − 4 + 9y = ⇒ y s2 + 9 = 2 + 2s + 4
s2 + 3 2 s +9
 
Dividing by s2 + 9 :
3s 2s 4
y= 2 + +
s2 +9 s2 + 9 s2 + 9
2ωs
Now the inverse of  2 = t · sin (ωt) so the inverse of
s2 + ω2
3s 1
 2 = · t · sin (3t)
s2 +9 2
1 4
So y= · t · sin (3t) + 2 cos (3t) + · sin (3t)
2 3
7. Take transforms:
8
(s · x − x0 ) − 6x + 3y =
s−1
4
(s · y − y0 ) − 2x − y =
s−1
Substitute boundary values
8
s · x + 1 − 6x + 3y =
s−1
4
s · y − 2x − y =
s−1
530 • Mathematics

Re-arrange and factorise


8 9−s
x · (s − 6) + 3y = −1=
s−1 s−1
4
−x · 2 + y · (s − 1) =
s−1
Eliminate the y variable
18 − 2s
x · 2 (s − 6) + 6y =
s−1
4(s − 6)
−x · 2(s − 6) + y (s − 1) (s − 6) =
s−1
Add
18 − 2s 4s − 24
y [(s − 1) (s − 6) + 6] = +
s−1 s−1
2  2s − 6 2
⇒ y s − 7s + 12 = ⇒y=
s−1 (s − 1) (s − 4)
2 A B
Let = + ⇒ 2 = A (s − 4) + B (s − 1)
(s − 1) (s − 4) s−1 s−4
2 2
s = 1 gives A = − , s = 4 gives B =
3 3
2 2
Taking the inverse transforms gives y = e4t − et
3 3
From the second of the equations

dy
2x = − y − 4et
dt
 
8 4t 2 t 2 4t 2 t
= e − e − e − e − 4et
3 3 3 3
= 2e4t − 4et

Therefore x = e4t − 2et


27
8. (i) s2 y − s · y0 − y1 + 9y =
s2
  27
y s2 + 9 − 3s = 2
s
2  27 3s3 + 27
y s + 9 = 2 + 3s =
s s2
3s3 + 27
So y=  
s 2 s2 + 9
3s3 + 27 A B (Cs + D)
(ii) y =  ≡ + 2+ 2
s s +9
2 2 s s s +9
Appendix 3 – Laplace Transforms • 531

   
Thus 3s3 + 27 = As s2 + 9 + B s2 + 9 + (Cs + D) s2

s = 0 27 = 9B ⇒B=3
s3: 3 = A+C
s2: 0=B+D ⇒ D = −3
s: 0 = 9A ⇒A=0 ⇒C=3

3 3s − 3 3 3s 3
y=+ 2 = 2+ 2 − 2
s2 s +9 s s + 32 s + 32
Therefore y = 3t + 3 cos(3t) − sin(3t)
9. f (t) = 2e4t + cosh(3t) + e−2t · cos(t)
2 s (s + 2) 2 s (s + 2)
L f (t) = + + = + +
s − 4 s2 − 9 (s + 2)2 + 12 s − 4 s2 − 9 s2 + 4s + 5
10.
3 2 2s − 6 3 2/3 2s 6
− + 2 = 4− + − 2
s 4 3s + 1 s + 9 s s + 1/3 s2 + 32 s + 32
 s  
3 2/3 3
= 4− +2 2 − 2
s s + 1/3 s + 32 s2 + 3 2
t3 2
Inverse Laplace transform = − e−t/3 + 2 cos(3t) − 2 sin(3t)
2 3
11.
4 4s + 3 4 4s + 3
+ = +
(s + 1)3 s2 + 6s + 13 (s + 1)3 (s + 3)2 + 22
4 4 (s + 3) − 9
= +
(s + 1)3
(s + 3)2 + 22
4 4 (s + 3) 9
= + −
(s + 1)3
(s + 3) + 2
2 2 (s + 3)2 + 22
9 −3t
Inverse Laplace transform = 2 · t2 · e−t + 4 · e−3t · cos(2t) − · e · sin(2t)
2
APPENDIX 4
FOURIER ANALYSIS
Many phenomena that are studied in engineering and science are periodic in
nature.

For example, the current and voltage in an a.c. circuit, the displacement, velocity and
acceleration of a slider-crank mechanism of a reciprocating engine.

The best-known mathematical periodic functions are the sine and cosine functions.

Fourier analysis is a method of separating a periodic function into its constituent


components called its fundamental and harmonics.

Periodic Functions
A function f (x) is said to be periodic and have period 2π if

f (x + 2π) = f (x) for all values of x

for example,

sin(x + 2π) = sin(x)


cos(x + 2π) = cos(x)
Appendix 4 – Fourier Analysis • 533

0
f (x) = sin(x)

2π 2π

A typical square wave function has period 2π and is defined by:



⎨ +k 0 < x <π
f (x) =
⎩ −k π< x < 2π

and is discontinuous at π, 2π, 3π, etc.

f(x)

–3π –2π –π 0 π 2π 3π 4π x

–k

Trigonometric Series
If a function is repetitive with a period of 2p it is possible to represent it in the form:

f (x) = a0 + a1 · cos(x) + a2 · cos(2x) + a3 · cos(3x) + · · ·


+ b1 · sin(x) + b2 · sin(2x) + b3 · sin(3x) + · · ·

where a1 , a2 , a3 , b1 , b2 , b3 , are real numbers.

Such a series is called a trigonometric series and an and bn are called the coefficients of
the series.

If the coefficients can be determined, the series is then called the Fourier Series corre-
sponding to f (x).
534 • Mathematics



In general, f (x) = a0 + (an · cos(nx) + bn · sin(nx))
n=1

a1 · cos(x) + b1 · sin(x) is called the fundamental

an · cos(nx) + bn · sin(nx) is called the nth harmonic.

Fourier coefficients

π 2π
1 1
a0 = f (x)dx or a0 = f (x)dx
2π 2π
−π 0
π 2π
1 1
an = f (x) · cos(nx)dx or an = f (x) · cos(nx)dx n = 1, 2, 3, . . .
π π
−π 0
π 2π
1 1
bn = f (x) · sin(nx)dx or bn = f (x) · sin(nx)dx n = 1, 2, 3, . . .
π π
−π 0

Example

Find the Fourier series for the periodic function f (x) defined as:

⎨−1 when − π ≤ x < 0
f (x) =
⎩ 1 when 0 ≤ x < π

The function is periodic outside that range with period 2π, that is, f (x + 2π) = f (x)

Solution

Since f (x) has two values for x in the range −π to π, the integration is divided into two
parts – π to 0 and 0 to π.

1
a0 = f (x)dx

−π

0 π
1 1
= π −1dx + π 1dx
2 2
−π 0
1 1
= π [−x]0−π + [x]π
2 2π 0
Appendix 4 – Fourier Analysis • 535

1 1 π π
= [(−0) − (−π)] + [π − 0] = − + =0
2π 2π 2π 2π
0 π
1 1
an = (−1) cos(nx)dx + (1) cos(nx)dx
π π
−π 0
 0  π
1 1 1 1
= − sin(nx) + sin(nx)
π n −π π n 0

=0 for all n.

Therefore no cosine terms

0 π
1 1
bn = (−1) sin(nx)dx + (1) sin(nx)dx
π π
−π 0
 0  π
1 1 1 1
= cos(nx) + − cos(nx)
π n −π π n 0
   
1 1 1 1 1 1
= cos(0) − cos(−nπ) + − cos(nπ)− cos(0)
π n n π n n
1 1 1 1 1
= − cos(−nπ) − cos(nπ) +
π n n n n
1 2 2
= − cos(nπ) since cos(x) = cos(−x)
π n n

When n is odd, cos(nπ) = cos(−nπ) = −1

1 2 2 4
∴ bn = + =
π n n nπ

When n is even, cos(nπ) = cos(−nπ) = 1

1 2 2
∴ bn = − =0
π n n

4 4 4
∴ f (x) = 0 + · sin(x) + · sin(3x) + · sin(5x) + · · ·
π 3π 5π
 
4 1 1
= sin(x) + · sin(3x) + · sin(5x) + · · ·
π 3 5
536 • Mathematics

Example

Find the Fourier series for the waveform shown:

f(x)

x
–4π –2π 0 2π 4π

Solution

The waveform has period 2π and is of the form f (x) = mx.


π 1
The gradient, m, is equal to =
2π 2
1
Therefore, f (x) = x
2

2π 2π  2π


1 1 x 1 x2 1 (2π)2 π
a0 = f (x) dx = dx = = =
2π 2π 2 2π 4 0 2π 4 2
0 0
2π 2π
1 x 1
an = · cos(nx) dx = x · cos(nx) dx
π 2 2π
0 0

Integrating by parts gives

⎡ ⎤
2π
1 ⎣x 1
an = · sin(nx) − · sin(nx)⎦
2π n n
0
 2π
1 x 1
an = · sin(nx) + 2 · cos(nx)
2π n n 0
 
1 2π 1 0 1
= · sin(2nπ) + 2 · cos(2nπ) − · sin(0) + 2 · cos(0)
2π n n n n
 
1 1 1
= 0+ 2 − 0+ 2 =0
2π n n
Appendix 4 – Fourier Analysis • 537

Therefore no cosine terms.

2π 2π
1 x 1
bn = · sin(nx) dx = x · sin(nx) dx
π 2 2π
0 0

Integrating by parts

    2π
1 x 1 1 x 1
bn = − · cos(nx) + · cos(nx) dx = − · cos(nx) + 2 · sin(nx)
2π n n 2π n n 0

1
Substituting and evaluating gives bn = − · cos(2nπ).
n
1
However, for all values of n, cos(2nπ) = 1, so bn = −
n
 
π 1 1
Therefore the Fourier series for f (x) = − sin(x) + sin(2x) + sin(3x) · · · .
2 2 3
Graphs of the first n terms of the Fourier series for the last example.

1 4

π

2

2 5

3 75
538 • Mathematics

Aids to simplify Fourier analysis

It is often useful to think of a wave as being a function of time: therefore the variable x
has been replaced by the variable t. The assumption is that you have been given f (t) in
analytical form.

• Sketch the waveform over two or three cycles, showing the curve for positive and
negative values of t.
• a0 is the average value of f (t) – normally found by observation.
• Sketch the waveform again having removed a0 from the curve.
• If the curve is reflective in the vertical axis, then it is an EVEN function and therefore is
composed only of cosine terms. Therefore bn = 0.
• If the curve can be rotated 180◦ about the origin it is an ODD function and therefore
is composed only of sine terms. Therefore an = 0 and a0 = 0.
T
• If f (t) = f t + the Fourier series has only even harmonics.
2
T
• If f (t) = −f t + the Fourier series has only odd harmonics.
2
• If the curve can be rotated 180◦ about the point a0 on the vertical axis then it contains
only sine terms, but it cannot be called an ODD function.

Half Range Series


Sometimes a function of period 2π is defined over the range 0 to π, instead of the usual
−π to π, or 0 to 2π. This results in a choice of how to proceed.

For example, if f (t) = 2t between t = 0 and t = π, then, since the period is 2π, we have
no indication of how the function behaves over the other half-range.

–π 0 π
Appendix 4 – Fourier Analysis • 539

If the waveform was as shown in (a), the function would be even, and the series would
have cosine terms only (plus possibly a0 ).

(a) 2π

–π 0 π

On the other hand, if the waveform was as shown in (b), the function would be odd and
the series would have sine terms only.

(b) 2π

–π 0 π

–2π

Alternatively, if something completely different is chosen for f (t) in the ‘missing’ range,
the series will have both sine and cosine terms including a0 .

(c) 2π

–π 0 π

In each case, we are making an assumption on how the function behaves between
t = −π and t = 0, and the resulting Fourier series will apply only to f (t) between
t = 0 and t = π for which it is defined. For this reason, such series are called half-range
series.
540 • Mathematics

Example

A function f (t) is defined by f (t) = 2t 0<t<π

f (t) = f (t + 2π)

Obtain a half-range cosine series for f (t)

(a) 2π

–π 0 π

Solution

To obtain a cosine series, f (t) has to be an even function. Therefore the waveform looks
like (a) above.

By inspection, a0 = π.

1 0 1 π
Now, an = (−2t) cos (nt) dt + (2t) cos (nt) dt
π −π π 0
2 π 4 π
= (2t) cos (nt) dt (by symmetry) = t · cos (nt) dt
π 
0 π  π 0   π 
4 t · sin(nt) 1 π 4 1 1
= − sin(nt) dt = (0 − 0) − − cos(nt)
π n 0 n0 π n n 0
4
= (cos(nπ) − 1)
π · n2

cos(nπ) = 1 if n is even

cos(nπ) = −1 if n is odd
8
Therefore, an = 0 if n is even, and an = − if n is odd.
πn2
Since f(t) is an even function there are no sine terms.
 
8 1 1
Therefore, f (t) = π − cos(t) + cos(3t) + cos(5t) + · · ·
π 9 25

Example

Determine the half-range sine series to represent

f (t) = 1 + t 0<t<π
f (t) = f (t + 2π)
Appendix 4 – Fourier Analysis • 541

Solution
The ‘missing’ section of the graph is chosen to make f (t) into an odd function and
therefore only contains sine terms.

1+π

–π 0 π


2
bn = (1 + t) · sin(nt) dt
π
0
⎧ ⎫
⎨  π π ⎬
2 − cos(nt) 1
= (1 + t) · + cos(nt) dt
π⎩ n 0 n ⎭
0
   
2 1+π 1 1 sin(nt) π
= − · cos(nπ) + +
π n n n n 0
 
2 1 1+π 2  
= − · cos(nπ) = 1 − (1 + π) cos(nπ)
π n n πn

1+π

–π –1 0 π

–(1 + π)

When n is even, cos(nπ) = 1, and when n is odd, cos(nπ) = −1


2
Therefore, for even values of n, bn = −
n
4 + 2π
and for odd values of n, bn =
πn
542 • Mathematics

Therefore,
 
4 + 2π 1 1
f (t) = sin(t) + sin(3t) + sin(5t) + · · ·
π 3 5
 
1 1 1
−2 sin(2t) + sin(4t) + sin(6t) + · · ·
2 4 6

Functions with Periods Other than 2π


In practice, most functions are defined over periods other than 2π; for example, from 0
to T, from −1 to +1.

Functions with period T

T T
If y = f (x) is defined over − < x < , that is, has period T, this can be converted to an
2 2
interval of 2π by changing the units of the variable. As most functions are dealing with
time the variable is t seconds.

We have f (t) = f (t + T), every oscillation of the wave is completed in T seconds and the,
1
frequency f hertz of the periodic function is given by f = .
T
If the angular velocity, ω radians per second, is defined by ω = 2πf , then
2π 2π
ω= and T = .
T ω
The angle, x radians, at any time t seconds is therefore x = ωt and the Fourier series to
represent the function can be expressed as:


  
f (t) = a0 + an · cos(nωt) + bn · sin(nωt) (1)
n=1

which can also be written in the form:



f (t) = A0 + Bn · sin(nωt + ϕn ) n = 1, 2, 3, . . . (2)
n=1
Appendix 4 – Fourier Analysis • 543

Comparing equations 1 and 2, it can be seen that

A0 = a0 ; Bn sin (ϕn ) = an ; Bn cos (ϕn ) = bn

and from this, it follows that:


an
Bn = a2n + b2n ; ϕn = tan−1
bn

So:

B1 · sin (ωt + ϕ1 ) is the first harmonic or fundamental (lowest frequency);

B2 · sin (ωt + ϕ2 ) is the second harmonic with frequency twice that of the fundamental;

Bn · sin (ωt + ϕn ) is the nth harmonic with frequency n times that of the fundamental.

For the series to converge, the values of Bn must decrease as n increases.

Fourier Coefficients
With the new variable, the Fourier coefficients become:


  
f (t) = a0 + an · cos(nωt) + bn · sin(nωt)
n=1
2π/ω
T 
1 ω
a0 = f (t) dt = f (t) dt
T 2π
0 0
2π/ω
T 
2 ω
an = f (t) · cos(nωt) dt = f (t) · cos(nωt) dt
T π
0 0
2π/ω
T 
2 ω
bn = f (t) · sin(nωt) dt = f (t) · sin(nωt) dt
T π
0 0

There is little difference between these formulae and the earlier ones.
T T π π
The limits can be 0 to T, − to , − to , so long as they cover a complete period.
2 2 ω ω
544 • Mathematics

Example

Determine the Fourier series for a periodic function defined by:

f (t) = 2(1 + (t) −1 < t < 0


f (t) = 0 0<t<1 f (t) = f (t + 2)

Solution

The first step is to sketch the waveform which is:

y = 2(1 + t )

t
–1 0 1

∞ 
 
We have f (t) = a0 + an · cos(nωt) + bn · sin(nωt) where T = 2
n=1

1
By inspection, a0 =
2
−T/2
 1 0
2 2
an = f (t) · cos(nωt) dt = f (t) · cos(nωt) dt = 2(1 + t) · cos(nωt) dt
T 2
T/2 −1 −1

T T
Note: The limits 0 < t < T have been replaced with − <t< .
2 2
Integrating by parts gives:
⎧ ⎫
⎨ sin(nωt)
0
1
0 ⎬
an = 2 (1 + t) − sin(nωt) dt
⎩ nω −1 nω ⎭
−1
  0 
1 cos(nωt)
= 2 (0 − 0) − −
nω nω −1
2
= (1 − cos(nω))
n2 ω2
2π 2π 2π 2
Now, T = ∴ ω= = =π ∴ an = (1 − cos(nπ))
ω T 2 n2 π 2
Appendix 4 – Fourier Analysis • 545

4
Therefore, an = 0 (n even); an = (n odd)
n2 π 2
2 T/2 2 1 0
Similarly, bn = f (t) · sin(nωt) dt = f (t) · sin(nωt) dt = 2(1 + t) · sin(nωt) dt
T −T/2 2 −1 −1

Integrating by parts gives:


⎧ ⎫
⎨ − cos(nωt)
0
1
0 ⎬
bn = 2 (1 + t) + cos(nωt) dt
⎩ nω −1 nω ⎭
−1
   
1 1 sin(nωt) 0
=2 − +
nω nω nω −1
 
1 1
=2 − + 2 2 sin(nω)
nω n ω

2
As before, ω = π ∴ bn = −

 
1 4 1 1
Therefore, f (t) = + 2 cos (πt) + cos (3πt) + cos (5πt) + · · ·
2  π 9 25 
2 1 1
− sin (πt) + sin(2π(t) + sin(3π(t) + · · ·
π 2 3

Half-range series for non-2π period

The theory behind the half-range sine and cosine series still applies with the new
variable.

(a) Even function Half-range cosine series


T
y = f (t) 0<t<
2
f (t) = f (t + T) Symmetrical about the y axis
2 T/2 4 T/2
a0 = f (t) dt an = f (t) · cos(nωt) dt
T 0 T 0
(b) Odd function Half-range sine series
T
y = f (t) 0<t<
2
f (t) = f (t + T) Symmetrical about the origin
4 T/2
bn = f (t) · sin(nωt) dt
T 0
546 • Mathematics

Example

Find the half-range cosine series to represent f (t) where

f (t) = 4 − t 0<t<4
f (t) = f (t + 8)

Solution

Since a half-range cosine series is required, an even function has to be formed, that is,
symmetrical in the vertical axis.

f(t )
4
4

t t
0 4 –4 0 4

Now for a useful trick. If we lower the waveform by two units to its ‘average’ posi-
tion, balanced half above and half below the t axis, a0 = 0, and the new function is
F(t) = 2 − t.

t
–4 0 4

Since this is an even function, only the an values have to be found.

T/2 4 4
4 4 1
So, an = F(t) · cos(nωt) dt = (2 − t) · cos(nωt) dt = (2 − t) · cos(nωt) dt
T 8 2
0 0 0
⎧ ⎫
 4 4
1 ⎨ (2 − t) 1 ⎬
an = . sin (nωt) + sin(nωt) dt
2⎩ nω 0 nω ⎭
0
 4 
1 2 1 
= − · sin(4nω) − 2 2 cos(nωt)
2 nω n ω 0
Appendix 4 – Fourier Analysis • 547

 
1 2 1
= − · sin(4nω) − 2 2 (cos(4nω) − 1)
2 nω n ω

2π 2π 2π π
Now, T = ∴ω= = =
ω T 8 4
   
1 8 16 1 16
Therefore, an = − · sin(nπ) − 2 2 (cos(nπ) − 1) = − 2 2 (cos(nπ) − 1)
2 nπ n π 2 n π
8
= − 2 2 (cos(nπ) − 1)
n π
When n is even, an = 0
16
When n is odd, an =
n2 π 2
Since we originally subtracted 2 from f (t) to get F(t), we have to add 2 to return to f (t).
Therefore,
 
16 πt 1 3πt 1 5πt
f (t) = 2 + cos + cos + cos + ···
π2 4 9 4 25 4

Example

Find the half-range sine series to represent f (t) where

f (t) = 3 + t 0<t<2
f (t) = f (t + 4)

Solution

A sine series is required, so rotational symmetry about the origin is needed.

f(t)

t
0 2
548 • Mathematics

Therefore,

–2 2 a0 = 0; an = 0; T = 4

–3

–5

2 2
4
bn = f (t) · sin(nωt) dt = (3 + t) · sin(nωt) dt
T
0 0

This has to be evaluated using ‘integration by parts’, and then substituting n = 1, 2, 3, . . .


to obtain the series.

So, by parts (not shown here),

1  1 
bn = 3 − 5 cos(2nω) + 2 2 sin(2nω)
nω n ω

2π 2π 2π π
However, T = ∴ω= = =
ω T 4 2
2 4
Therefore, bn = (3 − 5 cos(nπ)) + 2 2 (sin(nπ))
nπ n π
For any value of n, sin(nπ) = 0
4
For even values of n, cos(nπ) = 1, so bn = −

16
For odd values of n, cos(nπ) = −1, so bn =

Therefore,
 
4 πt 1 4 3πt 1 5πt
f (t) = 4 sin − sin(πt) + sin − sin + ···
π 2 2 3 2 4 2
Appendix 4 – Fourier Analysis • 549

Rectified Sinusoidal Waveforms


The following integrals are useful when calculating the Fourier coefficients.

If m and n are any integers (except 0)

T T
(a) cos(nωx)dx = 0 (b) sin(nωx)dx = 0
0 0
T T T T
(c) cos2 (nωx)dx = if = 0 (d) sin2 (nωx) dx= if n = 0
0 2 0 2
T
(e) cos(mωx) · cos(nωx)dx = 0 if m = n [(c) if m = n]
0
T
(f ) sin(mωx) · sin(nωx)dx = 0 if m = n [(d) if m = n]
0
T
(g) sin(mωx) · cos(nωx)dx = 0
0

For the case where T = 2π, these integrals become:


2π 

(a) cos(nx)dx = 0 (b) sin(nx)dx = 0
0 0

2π 

(c) cos2 (nx)dx = π if n = 0 (d) sin2 (nx)dx = π if n = 0
0 0

2π 

(e) cos(mx) · cos(nx)dx = 0 if m = n (f ) sin(mx) · sin(nx)dx = 0 if m = n
0 0


(g) sin(mx) · cos(nx)dx = 0
0

Note: The integrals could equally have been between −π and π.

(These results may be stated in any solution and need not be proved.)

Example

Consider a half-rectified sinusoidal waveform of period T (Figure A4.1)

This can be described by


⎧ T
⎨ A sin (ωt) 0≤t≤ 2π
f (t) = 2 ω= ⇒ ωT = 2π
⎩ 0
T
≤t<T T
2
550 • Mathematics

A f (t )

t
T

 Figure A4.1

Solution

 T/2  T 
1
a0 = A sin (ωt) dt + 0 dt
T 0 T/2

A T/2 A ωT
=− [cos (ωt)]0 = − cos − cos(0)
Tω 2π 2
A A A
=− (cos (π) − cos(0)) = − · (−2) =
2π 2π π
 T/2  T/2
2 A
an = A sin (ωt) cos(nωt) dt = [sin (1 + n) ωt + sin (1 − n) ωt] dt
T 0 T 0

When n = 1
 T/2
A A T/2 A
a1 = sin(2ωt) dt = − [cos(2ωt)]0 = − (cos (ωT) − cos(0))
T 0 2ωT 2ωT
A
=− (cos (2π) − cos(0)) = 0
2 · 2π

For n > 1
 T/2
A 1 1
an = − cos (1 + n) ωt − cos(1 − n)ωt
T (1 + n) ω (1 − n) ω 0

A 1 ωT 1 ωT
= − cos (1 + n) − cos(1 − n)
T (1 + n) ω 2 (1 − n) ω 2

1 1
− − cos 0 − cos 0
(1 + n) ω (1 − n) ω
Appendix 4 – Fourier Analysis • 551


A 1 1
= − cos (1 + n) π − cos(1 − n)π
T (1 + n) ω (1 − n) ω

1 1
− − −
(1 + n) ω (1 − n) ω
For even values of n:

cos(1 + n)π = −1 and cos(1 − n)π = −1

and so
 
A 1 1 1 1
an even = + − − −
ωT (1 + n) (1 − n) (1 + n) (1 − n)
 
A 2 2
= +
2π (1 + n) (1 − n)
A 2(1 − n) + 2(1 + n) A 4 2A
= = ! " = ! "
2π (1 + n) (1 − n) 2π 1 − n2 π 1 − n2
2A
=− ! 2 "
π n −1
For odd values of n (other than n = 1):

cos(1 + n)π = 1 and cos(1 − n)π = 1

and so
 
A 1 1 1 1
an odd = − − − − − =0
ωT (1 + n) (1 − n) (1 + n) (1 − n)
Similarly:
 T/2  T/2
2 A
bn = A sin (ωt) sin(nωt) dt = [cos (1 − n) ωt − cos (1 + n) ωt] dt
T 0 T 0

When n = 1
  T
A T/2 A sin(2ωt) /2 A T sin (ωT)
b1 = (1 − cos(2ωt)) dt = t − = − −0
T 0 T 2ω 0 T 2 2ω
A T A
= · =
T 2 2
For n > 1
 T/2
A 1 1
bn = sin (1 − n) ωt − sin(1 + n)ωt
T (1 − n) ω (1 + n) ω 0
1 1 ωT
= sin (1 − n) π − sin(1 + n)π as =π
(1 − n) ω (1 + n) ω 2
552 • Mathematics

However, for all n > 1, sin(1 − n)π = sin(1 + n)π = 0 so bn = 0

The Fourier series for the half-rectified sinusoid is therefore


A 2A  cos(nωt) A 2π
f (t) = − + sin(ωt) where ω =
π π even n n − 1
2 2 T
A 2A 2A A
= − cos(2ωt) − cos(4ωt) − · · · + sin(ωt)
π 3π 15π 2

Example

Consider the rectified sine wave f (t) = |A sin (ωt)| , 0 ≤ t < T (Figure A4.2)

A f(t )

t
T

 Figure A4.2

Solution
T
f t+ = f (t) so there are only even harmonics present and since f (t) is an even
2
function there are only cosine terms.
2A
a0 =
π
 
4A T/2 2A T/2
an = sin (ωt) cos(nωt) dt = [sin (n + 1) ωt − sin (n − 1) ωt] dt
T 0 T 0
 T/2
2A 1 1
an = − cos (1 + n) ωt + cos(n − 1)ωt
T (n + 1) ω (n − 1) ω 0

2A 1 ωT 1 ωT
= − cos (1 + n) + cos(n − 1)
T (n + 1) ω 2 (n − 1) ω 2

1 1
− − cos 0 + cos 0
(n + 1) ω (n − 1) ω
Appendix 4 – Fourier Analysis • 553


2A 1 1
= − cos (n + 1) π + cos(n − 1)π
T (n + 1) ω (n − 1) ω

1 1
− − +
(n + 1) ω (n − 1) ω

For even values of n:

cos(1 + n)π = −1 and cos(1 − n)π = −1

and so
 
2A 1 1 1 1
an = − − − +
T (n + 1) ω (n − 1) ω (n + 1) ω (n − 1) ω
 
2A 2 2
= −
ωT (n + 1) (n − 1)
 
2A 2 (n − 1) − 2 (n + 1) A −4 4A
= = =− ! 2 "
2π (n + 1) (n − 1) π n −1
2 π n −1

The Fourier series for the rectified sine wave is therefore


2A 4A  cos (nωt) 2π
f (t) = − where ω =
π π even n n − 1
2 T

2A 4A 1 1 1
That is, f (t) = − cos(2ωt) + cos(4ωt) + cos(6ωt) + · · ·
π π 3 15 35
Example
t
Find the Fourier series for the full wave rectified sine wave i = 5 sin with
2
period 2π.

Solution

The function is even so has only cosine terms


 2π  2π
1 t 5 t
a0 = 5 sin dt = −2 cos
2π 0 2 2π 2 0
5 2π
= −2 cos − (−2 cos(0))
2π 2
5 10
= [2 + 2] ⇒ a0 =
2π π
   
1 2π t 5 2π 1 t t
an = 5 sin · cos(nt) dt = sin + nt + sin − nt dt
π 0 2 π 0 2 2 2
⎡     ⎤2π
1 1
− cos t + n cos t − n
5 ⎢⎢ 2 2 ⎥

an = ⎣ − ⎦
2π 1 1
+n −n
2 2 0
554 • Mathematics

⎡⎛    ⎞
1 1
⎜ − cos 2π +n cos 2π −n
5 ⎢
⎢⎜ 2 2 ⎟

= −
2π ⎣⎝ 1 1 ⎠
+n −n
2 2
⎛ ⎞⎤
⎜ − cos [0] cos [0] ⎟ ⎥
−⎜
⎝ 1 − ⎟⎥
⎠ ⎦
1
+n −n
2 2
When n is both odd and even
⎡⎛ ⎞ ⎛ ⎞⎤
5 ⎢⎜ 1 1 ⎟ ⎜ −1 1 ⎟⎥
an = ⎢⎜ + ⎟−⎜ − ⎟⎥
2π ⎣⎝ 1 1 ⎠ ⎝ 1 1 ⎠⎦
+n −n +n −n
2 2 2 2
⎛ ⎞ ⎛ ⎞
5 ⎜
⎜ 2 2 ⎟ ⎜
⎟= 5 ⎜ 1 1 ⎟

= + +
2π ⎝ 1 1 ⎠ π⎝ 1 1 ⎠
+n −n +n −n
2 2 2 2
Rationalising this expression gives:
20
an = − ! 2 "
π 4n − 1

10 20 
∞ cos(nt) 20 1 1 1 1
Hence i = − ! "= − cos(t) − cos(2t) − cos(3t) − · · ·
π π n=1 4n2 − 1 π 2 3 15 35
Example

Determine the Fourier series for the waveform shown in Figure A4.3.

Voltage

Time

 Figure A4.3
Appendix 4 – Fourier Analysis • 555

Solution


⎪ T

⎨ 0 0≤t<
2
v(t) = f (t + T) = f (t)

⎪ ωt T
⎩ V sin ≤t<T
2 2

T  T
V ωt 2V ωt V  π V
a0 = sin dt = − cos =− cos (π) − cos =
T 2 ωT 2 T/2 π 2 π
T/2

T T
2V ωt V 1 1
an = sin cos (nωt) dt = sin n + ωt − sin n − ωt dt
T 2 T 2 2
T/2 T/2

⎡ ⎤T
1 1
cos n + ωt cos n − ωt ⎥
V ⎢⎢ 2 2 ⎥
=− −
ωT ⎣ 1 1 ⎦
n+ n−
2 2 T/2

⎡⎛ ⎞
V ⎢⎜ cos (2n + 1) π cos (2n − 1) π ⎟
=− ⎣⎝ − ⎠
2π 1 1
n+ n−
2 2
⎛ ⎞⎤
π π
cos (2n + 1) cos (2n − 1)
⎜ 2 − 2⎟ ⎥
−⎝ ⎠⎦
1 1
n+ n−
2 2

⎡ ⎤
V ⎢ 1 1 ⎥ V 2V
an = − ⎣− + =− =− ! 2 "
2π 1 1⎦ 1 π 4n − 1
n+ n− 2π n2 −
2 2 4
Therefore

T T
2V ωt V 1 1
bn = sin sin (nωt) dt = cos n + ωt − cos n − ωt dt
T 2 T 2 2
T/2 T/2

⎡ ⎤T
1 1
sin n + ωt sin n − ωt ⎥
V ⎢⎢ 2 2 ⎥
= ⎣ − ⎦
ωT 1 1
n+ n−
2 2 T/2
556 • Mathematics

⎡⎛ ⎞
V ⎢⎜ sin (2n + 1) π sin (2n − 1) π ⎟
= ⎣⎝ − ⎠
2π 1 1
n+ n−
2 2
⎛ ⎞⎤
π π
sin (2n + 1) sin (2n − 1)
⎜ 2 − 2⎟ ⎥
−⎝ ⎠⎦
1 1
n+ n−
2 2
⎡ ⎛ ⎞⎤ ⎡ ⎤
V ⎢ ⎜ −1 1 ⎟⎥ V ⎢ ⎥
⎢ −2n ⎥ = − ! 4nV "
For odd n = ⎣(0) − ⎝ − ⎠ ⎦ =
2π 1 1 2π ⎣ 2 1 ⎦ π 4n2 − 1
n+ n− n −
2 2 4

4nV
For even n = ! "
π 4n2 − 1
Therefore
∞ ∞
V 2V  cos(nωt) 4V  (−1)n · n · sin(nωt)
v(t) = − +
π π 4n2 − 1 π 4n2 − 1
n=1 n=1

Assume V = 1 and T = 2, then, to the first 4 harmonics,


 
1 2 cos t cos 2t cos 3t cos 4t
v(t) = − + + +
π V 3 15 35 63
 
4V sin t 2 sin 2t 3 sin 3t 4 sin 4t
+ − + − +
π 3 15 35 63

Test Examples – Fourier


1. Show that the half range Fourier Cosine series up to and including the third
harmonic for the function:
f (t) = 2t, 0 ≤ t ≤ π.
f (t + 2π) = f (t)
 
8 1 1
is f (t) = π − cos(t) + cos(3t) + cos(5t) + · · ·
π 9 25
2. A periodic function is defined by:

f (t) = 2(1 + (t) −1 < t < 0


f (t) = 0 0<t<1 f (t) = f (t + 2)
Appendix 4 – Fourier Analysis • 557

Show that the Fourier series for f (t) can be written as:
 
1 4 1 1
f (t) = + 2 cos (πt) + cos (3πt) + cos (5πt) + · · ·
2 π 9 25
 
2 1 1
− sin (πt) + sin(2π(t) + sin(3π(t) + · · ·
π 2 3

3. Given the function f (t) = 1 − t over the interval 0 ≤ t ≤ 1


f (t) = f (t + 2)
Find (i) the Fourier Cosine series of f (t)
(ii) the Fourier Sine series of f (t)

Solution to Test Examples – Fourier


1. To obtain a cosine series, f (t) has to be an even function. So f (t) resembles

(a) 2π

–π 0 π

By inspection, a0 = π

0 π
1 1
Now, an = (−2t) cos (nt) dt + (2t) cos (nt) dt
π π
−π 0
π π
2 4
= (2t) cos (nt) dt (by symmetry) = t · cos (nt) dt
π π
0 0

⎧ ⎫
⎨  π π ⎬   π 
4 t · sin(nt) 1 4 1 1
an = − sin(nt) dt = (0 − 0) − − cos(nt)
π⎩ n 0 n ⎭ π n n 0
0
4
= (cos(nπ) − 1)
π · n2
cos(nπ) = 1 if n is even
cos(nπ) = −1 if n is odd
558 • Mathematics

8
Therefore, an = 0 if n is even, and an = − if n is odd.
πn2
Since f (t) is an even function
 there are no sine terms. 
8 1 1
Therefore, f (t) = π − cos(t) + cos(3t) + cos(5t) + · · ·
π 9 25
2. The first step is to sketch the waveform which is:

y = 2(1 + t)

t
–1 0 1

∞ 
 
We have f (t) = a0 + an · cos(nωt) + bn · sin(nωt) where T = 2
n=1
1
By inspection, a0 =
2
−T/2
 1 0
2 2
an = f (t) · cos(nωt) dt = f (t) · cos(nωt) dt = 2(1 + t) · cos(nωt) dt
T 2
T/2 −1 −1

Integrating by parts gives:


⎧ ⎫
⎨ 
sin(nωt) 0 1
0 ⎬
an = 2 (1 + t) − sin(nωt) dt
⎩ nω −1 nω ⎭
−1
   
1 cos(nωt) 0
= 2 (0 − 0) − −
nω nω −1
2
= (1 − cos(nω))
n2 ω2
2π 2π 2π 2
Now, T = ∴ω= = = π ∴ an = 2 2 (1 − cos(nπ))
ω T 2 n π
4
Therefore, an = 0 (n even); an = 2 2 (n odd)
n π
Similarly,

T/2 1 0
2 2
bn = f (t) · sin(nωt) dt = f (t) · sin(nωt) dt = 2(1 + t) · sin(nωt) dt
T 2
−T/2 −1 −1
Appendix 4 – Fourier Analysis • 559

Integrating by parts gives:


⎧ ⎫
⎨ 
− cos(nωt) 0 1
0 ⎬
bn = 2 (1 + t) + cos(nωt) dt
⎩ nω −1 nω ⎭
−1
  0 
1 1 sin(nωt)
=2 − +
nω nω nω −1
 
1 1
=2 − + 2 2 sin(nω)
nω n ω
2
As before, ω = π ∴ bn = −

 
1 4 1 1
Therefore, f (t) = + 2 cos (πt) + cos (3πt) + cos (5πt) + · · ·
2 π 9 25
 
2 1 1
− sin (πt) + sin(2πt) + sin(3πt) + · · ·
π 2 3

3. (i) Lower the function by 0.5 to give a new function F(t) = 0. 5 − t.


T/2 1
4 4
So, an = F(t) · cos(nωt) dt = (0. 5 − t) · cos(nωt) dt
T 2
0 0
1
= (1 − 2t) · cos(nωt) dt
0
⎧ ⎫
⎨ (1 − 2t) 1
2
1 ⎬
an = · sin (nωt) + sin(nωt) dt
⎩ nω 0 nω ⎭
0
 
1 2 1
= − · sin(nω) − 2 2 [cos(nωt)]0
nω n ω
 
1 2
= − · sin(nω) − 2 2 (cos(nω) − 1)
nω n ω
2π 2π 2π
T= ∴ ω= = =π
ω T 2
 
1 2
Therefore, an = − · sin(nπ) − 2 2 (cos(nπ) − 1)
nπ n π
 
2
= − 2 2 (cos(nπ) − 1)
n π
560 • Mathematics

When n is even, an = 0
4
When n is odd, an =
n2 π 2
Since we originally subtracted 0.5 from f (t) to get F(t), we have to add 0.5 to
return to f (t). Therefore,
 
4 1 1
f (t) = 0. 5 + 2 cos (πt) + cos (3πt) + cos (5πt) + · · ·
π 9 25
(ii) A sine series is required, so rotational symmetry about the origin is needed.
Therefore, a0 = 0; an = 0; T = 2
1 1
4
bn = f (t) · sin(nωt) dt = 2 (1 − t) · sin(nωt) dt
T
0 0

This has to be evaluated using ‘integration by parts’, and then substituting


n = 1, 2, 3, . . . to obtain the series.
So, by parts (not shown here),
2 2
bn = − 2 2 (sin(nω))
nω n ω
2π 2π
However, T = ∴ω= =π
ω 2
2 2
Therefore, bn = − 2 2 (sin(nπ))
nπ n π
2
For any value of n, sin(nπ) = 0 so bn =

Therefore,
 
2 1 1 1
f (t) = sin (πt) + sin(2πt) + sin (3πt) + sin (4πt) + · · ·
π 2 3 4
APPENDIX 5
SOLUTIONS TO
TEST EXAMPLES
Solutions to Test Examples 1
103/2 × 104 × 103/4 10(6/4+16/4+3/4) 1025/4
1. = = = 104/4 = 101 = 10
103 × 10 /4 × 102
1
10( /4+ /4+ /4)
12 1 8
1021/4
42 × 42/3 × 4−2 4(2+2/3−2) 42/3 42/3
2. = = = =1
41/2 × 41/6 4(3/6+1/6) 44/6 42/3
8−2
3. = 8(−2−(−5)) = 8(−2+5) = 83 = 512
8−5
 2/5 5/3 √
3 × 3×3 310/15 × 31/2 × 31 34/6 × 33/6 × 36/6 313/6
4. = = = = 33/6
3−4/3 × 33 3−4/√3 × 39/3 35/3 310/6
= 31/2 = 3 = 1. 732 (3 decimal place accuracy)
 2   2 √
2 4 √ 5 4 √ 25 2 √ √
5. 1 + + 200 = + √ + 100 × 2 = + + 100 × 2
3 81 3 81 9 9
27
= + 10 × 1. 414 = 3 + 14. 14 = 17. 14 (2 decimal places)
9
6. Let V1 and V2 represent the volumes of the first and second spheres respectively
and let d1 and d2 represent their diameters.

V1 (d1 )3 24. 25 (d1 )3


Given that = and that V 1 = 24. 25 cm 2
, then =
V2 (d2 )3 V2 (d2 )3
562 • Mathematics

24. 25 × (d2 )3 24. 25 × (2 × d1 )3


Therefore V2 = and if d 2 = 2 × d 1 , V 2 =
(d1 )3 (d1 )3
24. 25 × (2 × d1 )3
If V2 = then
(d1 )3
24. 25 × 23 × (d1 )3
V2 = = 24. 25 × 8 = 194 cm3
(d1 )3
1
7. In 1 h the first pump can empty of a tank.
12
1
In 1 h the second pump can empty of a tank.
4
1
In 1 h the third pump can empty of a tank.
9
1 1 1 3 9 4 16
Working together the three pumps can empty + + = + + =
12 4 9 36 36 36 36
36 9
of a tank in 1 h, so it takes = = 2. 25 h.
16 4
The time taken will be 2 h 15 min.
1
8. Strength ∝ Breadth × (Depth)2 × , that is,
Length
k × Breadth × (Depth) 2
Strength =
Length
k × 40 × (100)2
Inserting the dimensions gives Strength =
5000
400000k
∴S= = 80k
5000
For the second beam
k × Breadth × (Depth)2 k × Breadth × (80)2
Strength = =
Length 3000
As the beams have equal strength, the strength of the second beam is also 80k.
k × Breadth × 6400 80k × 3000 240000
Therefore 80k = so Breadth = =
3000 6400k 6400
= 37. 5 mm

9. Elongation = 62. 5 − 50 = 12. 5 m


Elongation 100 12. 5 100
The percentage elongation = × = × = 25%
Original length 1 50 1
Reduction in area = 80 − 48 = 32 mm2
Reduction 100 32 100
The percentage reduction = × = × = 40%
Original area 1 80 1
10. Ratio of powers No. 1 : No. 2 : No. 3 = 115% : 95% : 100% = 1.15 : 0.95 : 1.00
The total of the ratios is 3.10. Therefore
Appendix 5 – Solutions to Test Examples • 563

1. 15 100
No. 1 cylinder develops × = 37. 097% of the total power
3. 1 1
0. 95 100
No. 2 cylinder develops × = 30. 645% of the total power
3. 1 1
1. 00 100
No. 3 cylinder develops × = 32. 258% of the total power
3. 1 1
11. % Zinc content = 100 − (71 + 1 + 3) = 25%
Total mass = 500 kg. Therefore
71
Content of copper = × 500 = 355 kg
100
1
Content of tin = × 500 = 5 kg
100
3
Content of lead = × 500 = 15 kg
100
25
Content of zinc = × 500 = 125 kg
100
12. Total = 27 + 39 + 47 + 51 + 48 + 32 + 20 + 11 + 8 + 5 = 288
288
Mean = = 28. 8 mm
10
13. Cost at first port = 200 × $600 = $120000
Cost at second port = 600 × $700 = $420000
Total cost = $540000, total volume = 800 barrels
540000
Therefore average cost per barrel = = $687. 5
800
Note: It is important to realise that the overall average is the total cost divided by
the total volume and not just the average of the costs.

Solutions to Test Examples 2


1. (i) (3x + 4y − 5z) + (−2x − 5y + 4z) = (3x − 2x) + (4y − 5y) + (−5z + 4z)
=x−y−z
     
(ii) 2a2 b − ab + 3ab2 + 5ab2 − a2 b + ab = 2a2 b − a2 b + (−ab + ab)
 
+ 3ab2 + 5ab2 = a2 b + 8ab2
(iii) (5x − 4z + 3y) − (2x + 5y − 3z) = (5x − 2x) + (3y − 5y) + (−4z − (−3z))
= 3x − 2y + (−4z + 3z) = 3x − 2y − z
564 • Mathematics

(iv) (3a + 6c − 2b) − (−4c − 5b − a) = (3a − (−a)) + (−2b − (−5b))


+ (6c − (−4c)) = (3a + a) + (−2b + 5b) + (6c + 4c) = 4a + 3b + 10c

2. (i) 5x − 3z − 4x − 2y + 4y + 2z − y = 5x − 4x − 2y + 4y − y − 3z + 2z = x + y − z

(ii) 2. 5a + c − 1. 2a + 2. 5b − 3c + b = 2. 5a − 1. 2a + 2. 5b + b + c − 3c
= 1. 3a + 3. 5b − 2c

(iii) b2 − 3ab2 + 2a2 b − 4a2 − 2b2 + 5a2 − 2ab2 = 5a2 − 4a2 + b2 − 2b2 − 3ab2
−2ab2 + 2a2 b = a2 − b2 − 5ab2 + 2a2 b
a × a3 × a5 a9 3 or a × a × a = a1+3+5−(2+4) = a3
3 5
3. (i) = = a
a2 × a4 a6 a2 × a4
(ii) b5 × b−3 × b−2 = b5+(−3)+(−2) = b0 = 1

(iii) c × c1/2 × c−1/3 = c1/2 × c1/2 × c−1/3 = c1/2+1/2+1/3 = c2/3
   1/2  4 1/2  2 1/2
(iv) d 4 e 2 = d4 e 2 = d e = d2 e
 3 √  1/3
(v) f 2 × f 1/2 × f −5 = f 6 × f 1/2 × f −5 = f 6 × f 1/6 × f −5 = f 7/6
3

4. (i) (x + 2y) (2x + y) = 2x 2 + xy + 4yx + 2y2 = 2x 2 + 5xy + 2y2


(ii) (2x + y) (3x − 2y) = 6x 2 − 4xy + 3yx − 2y2 = 6x 2 − xy − 2y2
(iii) (3x − 4y) (2x − 3y) = 6x 2 − 9xy − 8yx + 12y2 = 6x 2 − 17xy + 12y2
5. (i) (a + b)2 = a2 + 2ab + b2
(ii) (a − b)2 = a2 − 2ab + b2
 
(iii) (a + b)3 = a2 + 2ab + b2 × (a + b) = a3 + a2 b + 2a2 b + 2ab2 + b2 a + b3
= a3 + 3a2 b + 3ab2 + b3
 
(iv) (a − b)3 = a2 − 2ab + b2 × (a − b) = a3 − a2 b − 2a2 b + 2ab2 + b2 a − b3
= a3 − 3a2 b + 3ab2 − b3
6. (i)

 4a + 2b

2a − 3b 8a2 − 8ab − 6b2

8a2 − 12ab
4ab − 6b2
4ab − 6b2
0
Appendix 5 – Solutions to Test Examples • 565

(ii)
 3x 2 + xy − 2y2

3x − 4y 9x 3 − 9x 2 y − 10xy2 + 8y3

9x 3 − 12x 2 y
3x 2 y − 10xy2
3x 2 y − 4xy2
− 6xy2 − 8y3
− 6xy2 − 8y3
0
(iii)
 x 2 + xy + y2

x − y x 3 − 0x 2 y − 0xy2 − y3

x3 − x2y
x 2 y − 0xy2
x 2 y − xy2
xy2 − y3
xy2 − y3
0
7. (i) (a + b) + (c − d) − (a − b) − (c + d) + (a − b) = a + b + c − d − a + b − c − d
+a − b = a + b − 2d

(ii) 2 {a − 3 (a + 2) + 4 (2a − 1) + 5} = 2 {a − 3a − 6 + 8a − 4 + 5}
= 2 {6a − 5} = 12a − 10

(iii) 2x −[2x − {2x − (2x − 2) − 2} − 2]−2 = 2x −[2x − {2x − 2x + 2 − 2} − 2]−2


= 2x − [2x − {0} − 2] − 2 = 2x − [2x − 2] − 2 = 2x − 2x + 2 − 2 = 0
 
8. (i) 3b2 − 6b + 9 = 3 b2 − 2b + 3
(ii) pv + pvx = pv (1 + x)
 
(iii) ax 3 − 2bx 2 + 3cx = x ax 2 − 2bx + 3c
 
(iv) 12a3 b3 c3 − 8a2 b2 c2 + 4abc = 4abc 3a2 b2 c2 − 2abc + 1
9. (i) D2 − d2 = (D + d) (D − d)
(ii) 1 − a2 = (1 + a) (1 − a)
566 • Mathematics

(iii) 4x 2 y2 − 9z2 = (2xy + 3z) (2xy − 3z)


    
(iv) T14 − T24 = T12 + T22 T12 − T22 = T12 + T22 (T1 + T2 ) (T1 + T2 )

10. (i) a2 + 8a + 16 = (a + 4)2


(ii) d2 − 10d + 25 = (d − 5)2
(iii) 9v 2 + 12v + 4 = (3v + 2)2
(iv) 4x 2 − 12xy + 9y2 = (2x − 3y)2

x x 2x 6x 3x 8x x
11. (i) + − = + − =
2 4 3 12 12 12 12
2a − 1 2a + 3 3a − 8 6 (2a − 1) 3 (2a + 3) 2 (3a − 8)
(ii) − − = − −
b 2b 3b 6b 6b 6b
(12a − 6) (6a + 9) (6a − 16) 12a − 6 − 6a − 9 − 6a + 16 1
= − − = =
6b 6b 6b 6b 6b
4 18 3 4 18 3
(iii) + 2 − = + −
x + 2 x − 2x − 8 x − 4 x + 2 (x + 2) (x − 4) x − 4
4 18 3 4 (x − 4) 18
= + − = +
x+2 (x (x
+ 2) − 4) x − 4 (x (x
+ 2) − 4) (x + 2) (x − 4)
3 (x + 2) 4x − 16 18 3x + 6
− = + −
(x + 2) (x − 4) (x + 2) (x − 4) (x + 2) (x − 4) (x + 2) (x − 4)
4x − 16 + 18 − 3x − 6 x−4
= =
(x + 2) (x − 4) (x + 2) (x − 4)
1
=
x+2

12. 2x 3 − x 2 y + xy2 − 3y3 = 2 × (2)3 − (2)2 (−2) + (2) (−2)2 − 3 (−2)3 = 2 × 8


− (−8) + (8) − (−24) = 16 + 8 + 8 + 24 = 56

Solutions to Test Examples 3


1.
8 + 5x − 7 = 3x + 9
5x − 3x = 9 − 8 + 7
2x = 8
x=4
Appendix 5 – Solutions to Test Examples • 567

2.
2(a + 3) + 3(2a − 4) = 4(11 − 3a)a = 2. 5
2a + 6 + 6a − 12 = 44 − 12a
2a + 6a + 12a = 44 − 6 + 12
20a = 50
3.
3[3 − {x + 2(1 − x)} − 4x] = 2[x − 3(2 + x) − 4]
3[3 − {x + 2 − 2x} − 4x] = 2[x − 6 − 3x − 4]
3[3 − {2 − x} − 4x] = 2 [−2x − 10]
3 [3 − 2 + x − 4x] = −4x − 20
9 − 6 + 3x − 12x = −4x − 20
9 − 6 + 20 = −4x − 3x + 12x
23 = 5x
x = 4. 6
4.
2x x 5 4x 1
+ − = +
3 4 6 5 3
2x x 4x 1 5
+ − = +
3 4 5 3 6
40x + 15x − 48x 2 + 5
=
60 6
7x 7
=
60 6
7 × 60
x=
6×7
x = 10
5.  
1 − 2a2 1 a 1 5
− + = −
6a 4 3 6 2a
 
2 1 − 2a − 3a + a × 4a 2a − 30
2
= multiply by 12a
12a 12a
 
2 1 − 2a2 − 3a + 4a2 = 2a − 30
2 − 4a2 − 3a + 4a2 = 2a − 30
2 − 3a = 2a − 30
2 + 30 = 2a + 3a
32 = 5a
a = 6. 4
568 • Mathematics

6.
2 3 4
+ =
x − 3 x + 2 x2 − x − 6
2 (x + 2) + 3 (x − 3) 4
= 2
(x − 3) (x + 2) x −x−6
2x + 4 + 3x − 9 4
= 2 multiply by x 2 − x − 6
x −x−6
2 x −x−6
2x + 4 + 3x − 9 = 4
5x − 5 = 4
5x = 9
x = 1. 8

7. The equation produced is based upon the fact that the distance from meeting
place to the port is the same for both ships. Therefore the equation is:

Distance travelled by fast ship = Distance travelled by slow ship

Distance = Speed × Time and let x = time taken by fast ship (hours)
Therefore time taken by slow ship = (x + 4. 5) h
So

Speed × Time of fast ship = Speed × Time of slow ship


17. 5x = 16 (x + 4. 5)
17. 5x = 16x + 72
1. 5x = 72
72
x=
1. 5
x = 48 h

The distance is therefore 17. 5 × 48 = 840 nm.


8. Let b = breadth of plate, in m.
Therefore length = 4b m and since area = length × breadth,
area = 4b × b = 1 m2

So 4b2 = 1 giving b2 = 0. 25

Breadth b = 0. 25 = 0. 5 m
Length 4b = 4 × 0. 5 = 2 m
Appendix 5 – Solutions to Test Examples • 569

9.
R2 (1 + αθ2 )
=
R1 (1 + αθ1 )
R2
· (1 + αθ1 ) = 1 + αθ2
R1
R2
· (1 + αθ1 ) − 1 = αθ2
R1
R2
· (1 + αθ1 ) − 1
R1
= θ2
α
1 R2
or θ2 = · (1 + αθ1 ) − 1
α R1
When R1 = 200, R2 = 240, θ1 = 15 and α = 0. 0042,
1 240
θ2 = · (1 + 0. 0042 × 15) − 1 = 65. 62
0. 0042 200
10.
D3
d=
3. 5 × n × R
D3
d2 =
3. 5 × n × R
d2 (3. 5 × n × R) = D3
D3
R=
d2 × 3. 5n
When d = 82. 5 mm, D = 381 mm and n = 8 bolts per coupling,
3813
R=
82. 52 × 3. 5 × 8
Therefore R = 290. 2 mm
11.
 n−1/n
T1 p1
=
T2 p2
 n−1/n
797 34. 4
=
301 1. 05
n−1/n
2. 6478 = 32. 7619 ‘

 
n−1
Therefore log (2. 6478) = log (32. 7619)
n
   
n−1 n−1
So 0. 4229 = × 1. 5154 ⇒ = 1. 7473
n n
⇒ n − 1 = 1. 7473n
570 • Mathematics

1
Therefore − 1 = 0. 747n ⇒n=− = −1. 338
0. 747

log (56. 36)


12. (a) 40.59x = 56. 36 ⇒ 0. 59x × log (4) = log (56. 36) ⇒x=
0. 59 × log (4)
= 4. 929

(b) x
1.95
= 12. 4x 0.53 ⇒ 1. 95 × log (x) = log (12. 4) + 0. 53 × log (x)
1. 95 × log (x) − 0. 53 × log (x) = log (12. 4) ⇒ 1. 42 × log (x) = log (12. 4)
log (12. 4)
log (x) = = 0. 7700 ⇒ x = 100.7700 = 5. 889
1. 42
9. 97586
(c) e5x = 46. 382.6 ⇒ 5x = 2. 6 × ln (46. 38) = 9. 97586 ⇒x=
5
= 1. 995
√  3
(d) 3 x = e2.5 ⇒ x = e2.5 = (12. 182)3 = 1808
Note: Logarithms to any base can be used to solve questions 11, (12a) and (12b) but
if the question involves the value ‘e’ then the natural logarithm (ln) should be used.

Solutions to Test Examples 4


1. Let the first number be ‘x’ and the second ‘y’. Then

2. 5x + 3. 5y = 19
−3. 5x + 2. 5y = 3

Multiplying each equation by 2 gives

5x + 7y = 38
−7x + 5y = 6

Multiplying the first equation by 7 and the second by 5 gives

35x + 49y = 266


−35x + 25y = 30

Adding gives

74y = 296
296
y= =4
74
Appendix 5 – Solutions to Test Examples • 571

Substituting this value into the original first equation gives

2. 5x + 14 = 19
2. 5x = 5
5
x=
2. 5
x=2

Checking by substitution into the original second equation gives


(−3. 5 × 2) + (2. 5 × 4) = −7 + 10 = 3 Therefore x = 2 and y = 4.

2.
2x 3y 3 x y 13
− = and − =
3 5 4 2 4 16
Multiplying the first by 60 and the second by 16 gives

40x − 36y = 45(1)


8x − 4y = 13(2)

Multiplying the second by 5 gives

40x − 36y = 45

40x − 20y = 65

Second subtract first gives

16y = 20
20 5
y= =
16 4
Substituting into (1) gives

40x − 45 = 45
40x = 90
90 9
x= =
40 4
Subtituting into (2) gives
   
9 5 9 5
8× − 4× = 18 − 5 = 13 therefore x = ,y=
4 4 4 4
572 • Mathematics

3.
a (1 + 2b) = 3 and a (1 − 3b) = 0. 5
3
1 + 2b =
a
0. 5
1 − 3b =
a
3
1= − 2b
a
0. 5
1= + 3b
a
Multiplying the first by 2 and the second by 12 gives
6
2 = − 4b
a
6
12 = + 36b
a
Subtracting gives

10 = 40b
10 1
b= =
40 4
Substituting into the second equation gives
6
12 = +9
a
6
=3
a
a=2

Checking
6 1 1
− 4 × = 3 − 1 = 2 therefore a = 2, b =
2 4 4

4. Let the age of the grandson be ‘x’ and the age of the granddaughter be ‘y’. Then

4x + 3y = 72
3x + 4y = 68

Multiplying the first equation by 4 and the second by 3 gives

16x + 12y = 288

9x + 12y = 204
Appendix 5 – Solutions to Test Examples • 573

Subtracting the second from the first gives

7x = 84
x = 12

Substituting into the first equation gives

48 + 3y = 72
3y = 24
y=8

Therefore the grandson is 12 years old and the grandaughter is 8 years old.

5. Let the numbers be ‘x’ and ‘y’.

x−y =2
x 2 − y2 = 6
x 2 − y2 = (x + y) (x − y) but x − y = 2
So (x + y) · 2 = 6 giving x + y = 3

Adding these equations gives 2x = 5, so x = 2. 5


Substituting this value into the first equation gives 2. 5 − y = 2 and so y = 0. 5
Check: 2. 52 − 0. 52 = 6. 25 − 0. 25 = 6

6.
35 = a + 30b

55 = a + 70b

Subtracting gives 20 = 40b so b = 0. 5


Substituting into the first equation gives 35 = a + 15 so a = 20
Check: 20 + 70 × 0. 5 = 20 + 35 = 55
Therefore F = 20 + 0. 5 m
When m = 60 kg, F = 20 + 0. 5 × 60 = 20 + 30 = 50N
7. Let the original speed of A be ‘x’ and the original speed of B be ‘y’.
The time for B to overtake A is 8 h, but since B leaves 1 h later than A, the time for A
to be overtaken is 9 h.
The distance from the port to the position of overtaking is the same for both ships,
and as distance = Speed × Time

x×8= y×9
9x
y= (i)
8
574 • Mathematics

If the speed had been 4 knots slower, the speed of A would have been (x − 4) knots
and the speed of B would have been (y − 4) knots.
The time for B to overtake A is 6 h and for A to overtake B is 7 h. Therefore (x − 4) ×
7 = (y − 4) × 6

7x − 28 = 6y − 24
7x − 6y = 4 (ii)

Substituting the value of y from (i) into (ii) gives:


9x
7x − 6 × =4
8
54x
7x − =4
8
56x
As 7x ≡
8
56x 54x
− =4
8 8
2x
=4
8
8×4
x=
2
x = 16
9 × 16 144
Substituting this value into (i) gives y = = = 18
8 8
Therefore the original speed of A was 16 knots and B was 18 knots.

8. Given the simultaneous equations


x 6 2x 9
+ = 4 and − =1
2−y x 2−y x
find the values of
x 1
, , x and y.
2−y x
 
2x x
Since =2
2−y 2−y
x 6
+ =4
2−y x
 
x 9
2 − =1
2−y x
x 1
To simplify the equations let be ‘a’ and let be ‘b’
2−y x
Appendix 5 – Solutions to Test Examples • 575

So
a + 6b = 4

2a − 9b = 1

Multiplying the first equation by 2 gives

2a + 12b = 8

2a − 9b = 1

Subtracting gives

21b = 7
1
b=
3
Substituting
1
a+6× =4
3
a+2=2
a=2
1 1 1
Since b = ,x= = so x = 3
x b 1/3

x 3 3
Since a = ,2 = so 2 − y =
2−y 2−y 2
3 1
This means that y = 2 − so y =
2 2
Therefore
x 1 1 1
= 2, = , x = 3 and y =
2−y x 3 2

9. Find the values of x and y in the simultaneous equations:

2x = 4y and 4x−1 = 2y+1

 y
2x = 4y ⇒ 2x = 22 ⇒ 2x = 22y (i)
 x−1
4x−1 = 2y+1 ⇒ 22 = 2y+1 ⇒ 22x−2 = 2y+1 (ii)

Equating the powers as the base numbers are the same:


From (i) x = 2y
From (ii) 2x − 2 = y + 1
Substituting gives 2 (2y) − 2 = y + 1 ⇒ 4y − 2 = y + 1 ⇒ 3y = 3
576 • Mathematics

Therefore y = 1
Substituting y = 1 into (i) gives x = 2.
So the answers are x = 2 and y = 1.

10.

1. 259x+1 × 1. 175y−1 = 2. 323


3. 162x × 1. 778y = 25. 12

Taking logarithms of both sides and using the laws of logarithms gives

(x + 1) log (1. 259) + (y − 1) log (1. 175) = log (2. 323)


x · log (3. 162) + y · log (1. 778) = log (25. 12)

Therefore

x · log (1. 259) + log (1. 259) + y · log (1. 175) − log (1. 175) = log (2. 323)
x · log (3. 162) + y · log (1. 778) = log (25. 12)

Inserting the values of the logarithms gives

0. 1000x + 0. 1000 + 0. 0700y − 0. 0700 = 0. 3660


0. 5000x + 0. 2500y = 1. 4000

So

0. 1000x + 0. 0700y = 0. 3360


0. 5000x + 0. 2500y = 1. 4000

Multiplying by 1000 gives

100x + 70y = 336 (i)


500x + 250y = 1400 (ii)

Multiplying (i) by 5 gives


500x + 350y = 1680

500x + 250y = 1400

Subtracting gives
100y = 280 so y = 2. 8
Substituting this value into (i) gives

100x + 196 = 336 ⇒ 100x = 140


Appendix 5 – Solutions to Test Examples • 577

So x = 1. 4
Check in (ii): (500 × 1. 4) + (250 × 1. 8) = 700 + 700 = 1400
d2 x2 y2
11. 2
= 2 = 2
x y D
Taking the square root of everything gives
d x y
= =
x y D

d y dD
From = ,y= (i)
x D x
d x 2
From = , x = dy (ii)
x y
Substituting the value of y from (i) into (ii) gives

dD d2 D
x2 = d · ⇒ x2 = ⇒ x 3 = d2 D
x x

3
Therefore x= d2 D
√ √
When D = 75 and d = 25, x = 252 × 75 = 3 46875 = 36. 056
3

dD 25 × 75
As y = ,y = = 52. 00
x 36. 056
12.

3a + 6b − 2c = 7. 25 (i)
2a + 3b + 4c = 26 (ii)
4a − 2b + c = 10. 25 (iii)

2 × (i) + (ii) gives

8a + 15b = 40. 5 (iv)

(i) + 2× (iii) gives

11a + 2b = 27. 75 (v)

11 × (iv) − 8 × (v) gives 149b = 223. 5


So b = 1. 5
Substituting this value into (i) and (ii) gives

3a − 2c = −1. 75 (vi)
2a + 4c = 21. 5 (vii)
578 • Mathematics

2 × (vi) + (vii) gives


8a = 18

So a = 2. 25
Substituting into (vii) gives
4c = 17

So c = 4. 25
Checking in (i): (3 × 2. 25) + (6 × 1. 5) − (2 × 4. 25) = 6. 75 + 9 − 8. 5 = 7. 25
Therefore a = 2. 25, b = 1. 5 and c = 4. 25

Solutions to Test Examples 5

1. (i) (2x +8)(3x − 5) = 0


So either 2x + 8 = 0 so 2x = −8 giving x = −4
5
or 3x − 5 = 0 so 3x = 5 giving x =
3
(ii) (0. 5x − 10)(0. 25x + 5) = 0
So either 0. 5x − 10 = 0 so 0. 5x = 10 giving x = 20
or 0. 25x + 5 = 0 so 0. 25x = −5 giving x = −20
(iii) (5x + 0. 5)(4x + 0. 8) = 0
So either 5x + 0. 5 = 0 so 5x = −0. 5 giving x = −0. 1
or 4x + 0. 8 = 0 so 4x = −0. 8 giving x = −0. 2
2. (i)
3x 2 + 2x − 33 = 0 ⇒ (3x + 11) (x − 3) = 0
11
So either 3x + 11 = 0 so 3x = −11 giving x = −
3
or x − 3 = 0 giving x = 3
(ii)
4x 2 − 17x + 4 = 0 ⇒ (4x − 1) (x − 4) = 0
1
So either 4x − 1 = 0 so 4x = 1 giving x =
4
or x − 4 = 0 giving x = 4
(iii)
12x 2 + 10x − 12 = 0 ⇒ (6x − 4) (2x + 3) = 0
Appendix 5 – Solutions to Test Examples • 579

2
So either 6x − 4 = 0 so 6x = 4 giving x =
3
3
or 2x + 3 = 0 so 2x = −3 giving x = −
2
3. (i)
   
15 1 2 1 15 1 2
x2−x− =0 ⇒ x− − − =0 ⇒ x− −4=0
 24 2 4 4 2
1 1 √ 1
x− =4 ⇒x− =± 4 ⇒x = ±2
2 2 2
3 5
Therefore x = − and
2 2
(ii)
 
2 1 1 2 1 1
+ 2x − 1 = 0 ⇒ + x − = 0 ⇒ x +
3x 2 x2 − − =0
 2 3 3 3 9 3
1 4 1 4 1 2
x+ = ⇒x+ =± ⇒x=− ±
3 9 3 9 3 3
1
Therefore x = −1 and
3  
9 2 9 2 81 2
(iii) 4x − 9x + 2 = 0
2 ⇒ x − x + = 0⇒ x −
2 − + =0
 2 4 4
 8 64 4
9 81 32 49 9 49 9 7
x− = − = ⇒x− =± ⇒x= ±
8 64 64 64 8 64 8 8
2 1
Therefore x = 2 and =
8 4
4. (i) 3x 2 − 2x + 0. 25 = 0 ⇒ a = 3, b = −2 and c = 0. 25
√ √
− (−2) ± 4 − 4 × 3 × 0. 25 2± 1 2±1
x= ⇒x= ⇒x=
2×3 6 6

3 1 1 1
Therefore x = and so x = and
6 6 2 6
(ii) 5x 2 + 4x − 5. 52 = 0 ⇒ a = 5, b = 4 and c = −5. 52

−4 ± 16 − 4 × 5 × (−5. 52) 4 ± 126. 4 4 ± 11. 24
x= ⇒x= ⇒x=
2×5 10 10

Therefore x = 1. 524 and −0. 724

(iii) 10x 2 − x − 0. 2 = 0 ⇒ a = 10, b = −1 and c = −0. 2



− (−1) ± 1 − 4 × 10 × (−0. 2) 1± 9 1±3
x= ⇒x= ⇒x=
2 × 10 20 20

4 2 1 1
Therefore x = and − so x = and − (or x = 0. 2 and − 0. 1)
20 20 5 10
580 • Mathematics


5. log (0. 5x) = 2 × log (x − 6) ⇒ log (0. 5x) = log (x − 6)2 ⇒ 0. 5x = (x − 6)2
0. 5x = x 2 − 12x + 36 ⇒ 0 = x 2 − 12. 5x + 36
This factorises as (x − 8) (x − 4. 5) = 0 giving x = 8 and 4. 5.
This may also be solved by completing the square or by formula.

6. 6b4 − 2. 46b2 + 0. 24 = 0
Let B = b2 then 6B2 − 2. 46B + 0. 24 = 0
√ √
2. 46 ± 6. 0516 − 4 × 6 × 0. 24 2. 46 ± 0. 2916 2. 46 ± 0. 54
B= = =
2×6 12 12
3 1. 92
Therefore B = = 0. 25 and = 0. 16 but B = b2
12√ √12
Therefore b = ± 0. 25 and ± 0. 16
So b = ±0. 5 and ±0. 4

7. V 2.8 − 5. 1V 1.4 + 5. 6 = 0
Let V 1.4 = v, therefore v 2 − 5. 1v + 5. 6 = 0
√ √
This equation gives v = 3. 5 and v = 1. 6 V = 1.4 v therefore V = 1.4
3. 5

= 2. 447 and V = 1.4 1. 6 = 1. 399

8.

x 2 − xy + 2y2 = 16 (i)
x + 2y = 8 (ii)

From (ii) x = 8 − 2y, and substituting into (i)

(8 − 2y)2 − (8 − 2y) y + 2y2 = 16 ⇒ 64 − 32y + 4y2 − 8y + 2y2 + 2y2 = 16


Therefore 8y2 − 40y + 48 = 0 ⇒ y2 − 5y + 6 = 0 ⇒ (y − 2) (y − 3) = 0
The solutions are y = 2 and y = 3.
When y = 2, x = 4 and when y = 3, x = 2.

9. Let the speed of the slow ship be ‘x’ knots.


330
The time to travel 330 nm h
x
The speed of the fast ship is x + 3. 5 knots.
334
The time to travel 334 nm is h but this time is 5 h faster than that of the
x + 3. 5
slow ship.
Appendix 5 – Solutions to Test Examples • 581

330 334
Therefore −5=
x x + 3. 5
330 5x 334 330 − 5x 334
⇒ − = ⇒ =
x x x + 3. 5 x x + 3. 5
⇒ (330 − 5x) (x + 3. 5) = 334x ⇒ 330x + 1155 − 5x 2 − 17. 5x = 334x
⇒ 0 = 5x 2 + 21. 5x − 1155 and dividing throughout by 5 gives
x 2 + 4. 3x − 231 = 0

Using the formula the two possible solutions are x = 13. 2 and x = −17. 5 knots.
So the speed of the slow ship is 13.2 knots and that of the fast ship 16.7 knots.

10. The area of a rectangle is 76 cm2 and the perimeter is 350 mm.
Find the length and breadth.
Let the length of the rectangle be ‘x’ and the breadth be ‘y’ cm.
Therefore x · y = 76 and 2x + 2y = 35 (note the change in unit)
2x + 2y = 35 ⇒ x + y = 17. 5 and so x = (17. 5 − y)
Substituting this value into the area equation gives (17. 5 − y) · y = 76
The resulting equation is y2 − 17. 5y + 76 = 0 giving y = 9. 5 cm and 8 cm.
When y = 9. 5 cm, x = 8 cm and when y = 8 cm, x = 17. 5 cm.
So both rectangles are the same size: 9.5 cm by 8 cm.

11. To find the roots of the equation: 2x 3 + 3x 2 − 17x − 30 = 0, use trial and error to
find the first solution.
When x = 1, y = 2 + 3 − 17 − 30 = −42
When x = 2, y = 16 + 12 − 34 − 30 = −36
When x = 3, y = 54 + 27 − 34 − 30 = 0 so (x − 3) is a factor.
Dividing 2x 3 + 3x 2 − 17x − 30 by (x − 3) gives

 2x 2 + 9x + 10

x − 3 2x 3 − 3x 2 y − 17x −30

2x 3 − 6x 2
+ 9x 2 − 17x − 30
+ 9x 2 − 27x
10x − 30
10x − 30
0
582 • Mathematics

Factorising 2x 2 + 9x + 10 = 0 gives (2x + 5) (x + 2) = 0


This gives x = −2. 5 and x = −2.
The three solutions are therefore x = 3, −2. 5 and −2.
 
16 3 32 48
12. 2x − 2− = 3, removing the brackets gives 2x − + 2 =3
x x x x
Multiplying throughout by x 2 gives 2x 3 − 32x + 48 = 3x 2
Rearranging gives 2x 3 − 3x 2 − 32x + 48 = 0
Using trial and error to find the first solution:
When x = 1, 2x 3 − 3x 2 − 32x + 48 = 15
When x = 2, 2x 3 − 3x 2 − 32x + 48 = −12
When x = 3, 2x 3 − 3x 2 − 32x + 48 = −21
When x = 4, 2x 3 − 3x 2 − 32x + 48 = 0 so (x − 4) is a factor.
Dividing 2x 3 − 3x 2 − 32x + 48 by (x − 4) gives

 2x 2 + 5x − 12

x − 4 2x 3 − 3x 2 y − 32x + 48

2x 3 − 8x 2
+ 5x 2 − 32x + 48
+ 5x 2 − 20x
− 12x + 48
− 12x + 48
0

2x 2 + 5x − 12 = 0 factorises to give (2x − 3) (x + 4) = 0


This has solutions x = 1. 5 and x = −4 so the full solution is x = 1. 5, 4 and −4.

Solutions to Test Examples 6


1. For each line find three points (two for the line and the third as a check)
x=0 x = 6 x = 12 Three points
(i) y =2+x y=2 y = 8 y = 14 (0, 2), (6, 8) and (12, 14)
(ii) y = 12 − 1. 5x y = 12 y = 3 y = −6 (0, 12), (6, 3) and (12, −6)
Appendix 5 – Solutions to Test Examples • 583

(iii) y = −1 − 0. 5x y = −1 y = −4 y = −7 (0, −1), (6, −4) and (12, −7)


(iv) y = −4 + 1. 25x y = −4 y = 3 y = 11 (0, −4), (6, 3) and (12, 11)

15
+x
y=2
25x
10 –4 + 1.
y=

0
0 2 4 6 8 y= 12
12
y = –1 –1
– 0.5x .5x
–5

–10

2. A straight line passes through the pair of points (−2, 14.5) and (8, −3).
change in y (−3) − 14. 5 −17. 5
The gradient = = = −1. 75
change in x 8 − (−2) 10
Therefore the graph has equation y = −1. 75x + c and passes through (8, −3).
So −3 = (−1. 75 × 8) + c ⇒ −3 = −14 + c ⇒ c = 11
Therefore the equation of the line is y = −1. 75x + 11
3. x −2 −1 0 1 2 3
y 10 7 4 1 −2 −5

10

0
–2 –1 0 1 2 3

–5

(−5) − 1 −6
Slope = = = −3
3−1 2
Intercept = 4
Therefore the line has
equation y = −3x + 4
584 • Mathematics

4. P 20 25 30 35 40 45 50
m 220 265 315 365 410 455 505

550

500

450

400

350

300

250

200
25 30 35 40 45 50

505 − 315 190


The slope, b = = = 9. 5
50 − 30 20
Therefore m = a + 9. 5P and as this line passes through the point (50, 505)
505 = a + 9. 5 × 50 which gives a = 505 − 475 = 30
So the equation is m = 30 + 9. 5P

5.

3x + 5y = 23
5x − 2y = 12. 5

6
5
4
3
2
1
0
2 3 4 5
–1 x = 3.5
–2 y = 2.5
–3
–4
Appendix 5 – Solutions to Test Examples • 585

6. Find, graphically, the values of p and q in the simultaneous equations

5p − 2q = 5. 6 and

2p − 3q = −4. 8

3.5

3.3
p = 2.4
q = 3.2

2.5

2
2 2.2 2.4 2.6 2.8 3

21
7. Find, graphically, the value of x in the equation, x 2 − 5x + = 0.
4
1.5

0.5

0
1 1.5 2 2.5 3 3.5 4

–0.5

–1

–1.5

Therefore the values of x are the values where the curve crosses the x axis
So the solutions are x =. 5 and x = 3. 5.
8. Draw the graph of y = 0. 5x 2 − 2x − 6 between the values of x = −4 and x = +8.
A table of values for x and y is:

x −4 −3 −2 −1 0 1 2 3 4 5 6 7 8
y 10 4.5 0 −3. 5 −6 −7. 5 −8 −7. 5 −6 −3. 5 0 4.5 10
586 • Mathematics

10
8
6
4
2
(ii) (iii) (iii) (ii)
0
–4 –2 0 2 4 6 8
–2
(iv) (v) (v) (iv)
–4
–6
–8

From the graph read the values of x in the following equations:

(i) 0. 5x 2 − 2x − 6 = 0 has solutions x = −2 and x = 6


(ii) 0. 5x 2 − 2x − 4 = 0 means that 0. 5x 2 − 2x − 6 = −2, so x = −1. 5
and x = 5. 5
(iii) 0. 5x 2 − 2x − 1 = 0 means that 0. 5x 2 − 2x − 6 = −5, so x = −0. 5
and x = 4. 5
(iv) 0. 5x 2 − 2x = 0 means that 0. 5x 2 − 2x − 6 = −6, so x = 0
and x = 4
(v) 0. 5x 2 − 2x + 1 = 0 means that 0. 5x 2 − 2x − 6 = −7, so x = 0. 6
and x = 3. 4

9.
y1 = x 2
y2 = 3. 5 + 2. 5x

16
14 y2
12
10
8
6
4
2
0
–2 –1 0 1 2 3 4
–2
Appendix 5 – Solutions to Test Examples • 587

If y1 = y2 then x 2 = 3. 5 + 2. 5x and so x 2 − 2. 5x − 3. 5 = 0.
Therefore the solutions to the equations are the x coordinates of the points of
intersection of the two graphs.
The solutions are x = −1 and x = 3. 5

10.

y1 = 0. 4x 2 − 3x + 2and
y2 = 1. 4x − 2

Calculating values for x between 0 and 12 gives

x 0 1 2 3 4 5 6 7 8 9 10 11 12
y1 2 −0.6 −2.4 −3.4 −3.6 −3 −1.6 0.6 3.6 7.4 12 17.4 23.6
y2 −2 −0.6 0.8 2.2 3.6 5 6.4 7.8 9.2 10.6 12 13.4 14.8

20

15

10

0
0 2 4 6 8 10 12

–5

The solutions are x = 1, y = −0. 6 and x = 10, y = 12

11. y2 = 16x and y = 5 + 8x − 2x 2

x 0 1 2 3 4 5 6
y1 0 ±4 ±5. 66 ±6. 93 ±8 ±8. 94 ±9. 8
y2 5 11 13 11 5 −5 −19
588 • Mathematics

15

10

0
0 1 2 3 4 5 6
–5

–10

–15

An accurate drawing gives x = 3. 65, y = 7. 6 and x = 5. 4, y = −9. 7 as the two


solutions.

12. By drawing a graph of y = e−x and y = x 2 solve the equation:

x 2 · ex = 1

Note: plot values between x = 0 and x = 1.

x 0 0.05 0.1 0.15 0.2 0.25 0.3 0.35 0.4 0.45 0.5 0.55 0.6 0.65 0.7 0.75 0.8 0.85 0.9 0.95 1
y1 1 0.95 0.9 0.86 0.82 0.78 0.74 0.7 0.67 0.64 0.61 0.58 0.55 0.52 0.5 0.47 0.45 0.43 0.41 0.39 0.37
y2 0 0 0.01 0.02 0.04 0.06 0.09 0.12 0.16 0.2 0.25 0.3 0.36 0.42 0.49 0.56 0.64 0.72 0.81 0.9 1

1
0.9
0.8
0.7
0.6
0.5
0.4
0.3
0.2
0.1
0
0 0.2 0.4 0.6 0.8 1

The solution is x = 0. 71, y = 0. 5.


Appendix 5 – Solutions to Test Examples • 589

Solutions to Test Examples 7

π 180
1. (i) To convert to radians multiply by , to convert to degrees multiply by
180 π
π
114◦36 = 114. 6◦ = 114. 6 × = 2. 000 rad
180
π
286◦30 = 286. 5◦ = 286. 5 × = 5. 000 rad
180

(ii) Diameter of 10 m means that r = 10 and s = r · θ

s = 10 ⇒ 10 = 10θ ⇒ θ = 1 radian ≡ 57. 3◦


s = 30. 4 ⇒ 30. 4 = 10θ ⇒ θ = 3. 04 radian ≡ 174. 2◦

2. v = ω · r where ω = 10. 52
r = 100 mm means that r = 0. 1 m and so v = 10. 52 × 0. 1 = 1. 052 m/s
r = 500 mm means that r = 0. 5 m and so v = 10. 52 × 0. 5 = 5. 26 m/s
D= 2m means that r = 1 m
125 × 2π
125 rev/min = = 13. 09 rad/s
60
Therefore v = 13. 09 × 1 = 13. 09 m/s

3. A

c
36 mm

B C
27 mm

By Pythagoras (AB)2 = (BC)2 + (AC)2

(AB)2 = 272 + 362


= 729 + 1296
= 2025

AB = 2025
= 45 mm
36 27 36
sin (B) = = 0. 8; cos (B) = = 0. 6; tan (B) = = 1. _3
45 45 27
590 • Mathematics


4. sin (θ ) + cos2 (θ ) = 1 ⇒ sin (θ ) =
2
1 − cos2 (θ ) = 1 − (0. 4924)2

= 0. 75754 = 0. 8704
sin (θ ) 0. 8704
tan (θ ) = = = 1. 7677
cos (θ ) 0. 4924
5.
10◦ 33 46◦55 150◦47 201◦ 21 287◦ 14
sin 0.1831 0.7304 0.4881 −0. 3641 −0. 9551
cos 0.9831 0.6831 −0. 8728 −0. 9314 0.2963
tan 0.1862 1.0692 −0. 5593 0.3909 −3. 2238

1 y

y = sin(x )

0.5 y = cos(x )

x
90 180 270 360

–0.5

–1

6. (a) sin−1 (0. 3783) = 22. 23◦ and 180 − 22. 23 = 157. 77◦
sin−1 (−0. 7005) = −44. 47◦ and 180 − (−44. 47) = 224. 47◦
Replace −44. 47 with −44. 47 + 360 = 315. 53◦
(b) cos−1 (0. 9687) = 14. 37◦ and 360 − 14. 37 = 345. 63◦
cos−1 (−0. 8769) = 151. 27◦ and 360 − 151. 27 = 208. 73◦
(c) tan−1 (0. 2010) = 11. 37◦ and 180 + 11. 37 = 191. 37◦
tan−1 (−3. 2006) = −72. 65◦ and 180 + (−72. 65) = 107. 35◦
Replace −72. 65◦ by −72. 65◦ + 360 = 287. 35◦
7. If θ = 80◦ :
sin θ = 0. 9848, cos θ = 0. 1736, sin 2θ = 0. 3420, cos 2θ = −0. 9397, sin2 θ =
0. 9698, cos2 θ = 0. 0302

8. cos2 θ − sin2 θ = 1 ⇒ cos (2θ) = 1


Therefore 2θ = 0◦ , 360◦, 720◦ so θ = 0◦ , 180◦, 360◦
Appendix 5 – Solutions to Test Examples • 591

a2 2b2 4 sin2 (θ ) 2 × 25 cos2 (θ )


9. = −4⇒ = − 4 ⇒ 2 sin2 (θ ) = 10 cos2 (θ ) − 4
2 5 2 5
 
But sin2 (θ ) = 1 − cos2 (θ ), and so 2 1 − cos2 (θ ) = 10 cos2 (θ ) − 4
1
So 2 − 2 cos2 (θ ) = 10 cos2 (θ ) − 4 ⇒ 6 = 12 cos2 (θ ) ⇒ cos2 (θ ) =
2
Therefore cos (θ ) = ±0. 7071
When cos (θ ) = +0. 7071, θ = 45◦
When cos (θ ) = −0. 7071, θ = 135◦
   
sin 2θ cos 2θ
10. v = ωr sin θ + m/s a = ω r cos θ +
2 m/s2
2n n
where ω = angular velocity of crank in rad/s, r = throw of crank ( 12 stroke) in m,
θ = angle of crank past top centre, in deg, n = ratio of connecting rod length to
crank length.
Calculate
(i) the velocity, and (ii) the acceleration of the piston of an engine of 1 m stroke, con-
necting rod length 2 m, at the instant the crank is 80◦ past top centre and running
at 150 rev/min.
150 × 2π
(i) 150 rev/min ≡ = 5π rad/s
60
r = 12 stroke = 0. 5 m
connecting rod length 2
n= = =4
crank length 0. 5
   
sin 2θ sin (160)
v = ωr sin θ + = 5π × 0. 5 sin (80) + = 8. 0704 m/s
2n 8
   
cos 2θ cos (160)
(ii) a = ω2 r cos θ + = (5π)2 × 0. 5 cos (80) +
n 4
= −7. 5595m/s2
That is, deceleration
11.
P1 v2 P2 v2
+ 1 + h1 = + 2 + h2
γ 2g γ 2g
P1 v2 v2 P2
+ 1 − 2 + h1 − h2 =
γ 2g 2g γ
P1 v2 v2
P2 = γ + 1 − 2 + h1 − h2
γ 2g 2g
  2  
P1 v1 − v22
P2 = γ + + (h1 − h2 )
γ 2g
350 8. 4
P2 = 10 + + 2 = 374. 28 to 2 decimal places
10 2 × 9. 81
592 • Mathematics

1. 8 sinh(1. 8) + sin(1. 8)
12. λ = = 1. 2236 to 5 significant figures
2 cosh(1. 8) + cos(1. 8)
   
2 2 2
13. 15 = 10 × log ⇒ log = 1. 5 ⇒ = 31. 62 ⇒ Y = 0. 06325
Y Y Y
   
2. 3 2. 3 2. 3
14. 4 = 2 × ln ⇒ ln =2⇒ = 7. 389 ⇒ PN = 0. 2707
PN PN PN
 x   x 
e + e−x e − e−x
15. A +B = 6ex − 3e−x
2 2

(A + B) · ex + (A − B) · e−x = 12ex − 6e−x


A + B = 12
A − B = −6
Add 2A = 6 ⇒ A = 3 ⇒ B = 9

16.
 1−n 
T2 v2 T2 V2
ln = ln ⇒ ln = (1 − n) · ln
T1 v1 T1 V1
ln (T2 ) − ln (T1 ) ln (T2 ) − ln (T1 )
1−n= ⇒ n=1−
ln (V2 ) − ln (V1 ) ln (V2 ) − ln (V1 )
n = 1. 1768

17. λ = 1. 4761
   
0. 415 U2 U2 U2
18. 2. 5 − 1. 5 = × ln ⇒ ln = 0. 72289 ⇒ = 2. 0604
0. 3 50 50 50
⇒ U2 = 103. 02

Solutions to Test Examples 8


1. Mast

hm
45°34′
15 m
Appendix 5 – Solutions to Test Examples • 593

45◦ 34 = 45. 56◦


h
tan (45. 56) = 1. 020 =
15
h = 1. 020 × 15 = 15. 3m

2.
14° 25′

d
Cliff,

95 m

(i) the angle of elevation of the top of the cliff from the boat is also 14◦ 25
  95 95 95
so tan 14◦ 25 = ⇒h= ◦ 
= = 369. 55 m
h tan (14 25 ) 0. 2571
(ii) Using Pythagoras’ Theorem d2 = 952 + 369. 552 = 145595.

So the distance is 145595 = 381. 57 m

3.
120 mm

38°37′
w

 
h = 120 sin 38◦ 37 = 120 × 0. 6241
= 74. 89 mm
 
w = 120 cos 38◦ 37 = 120 × 0. 7813
= 93. 76 mm
74. 89 × 93. 76
Area = = 3511 mm2
2
594 • Mathematics

4.

48°30′ 37°38′

d 10 m

h  
= tan 48◦ 30
d  
∴ h = d × tan 48◦ 30
So h = 1. 1303d
h  
= tan 37◦ 38
(d + 10)
 
∴ h = (d + 10) · tan 37◦ 38
Soh = 0. 771d + 7. 71

Equating h gives:

7. 71
1. 1303d = 0. 771d + 7. 71 ⇒ 0. 3593d = 7. 71 ⇒ d = = 21. 46 m
0. 3593
But h = 1. 1303d so h = 24. 56 m

5. D

AB = BC = AD = 60 mm

C B

The solution relies upon using Pythagoras’ Theorem twice.

(AC)2 = (BC)2 + (AB)2 = 602 + 602 = 3600 + 3600 = 7200



Therefore the diagonal of a side is length 7200 = 84. 9 mm

(DC)2 = (AC)2 + (AD)2 = 7200 + 602 = 7200 + 3600 = 10800



Therefore a cross diagonal is of length 10800 = 103. 9 mm
Appendix 5 – Solutions to Test Examples • 595

6. C

a
B
105°30′
b Load
15 m 35°30′

Angle C = 180 − (35◦ 30 + 110◦ 30 ) = 34◦


Using the Sine Rule:
 
15 b 15 × sin 105◦30
= ⇒b= = 22. 97 m
sin (39◦) sin (105◦ 30 ) sin (39◦ )
15 a 15 × sin (35◦ 30)

= ◦ 
⇒a= = 13. 84 m
sin (39 ) sin (35 30 ) sin (39◦ )
7. Top of stroke
x mm
C
B

1600 mm

35°
400 mm

x is the distance the crosshead has moved from the top of its stroke when the crank
is 35◦ past top centre.
Crank length = 1
2 stroke = 400 mm
596 • Mathematics

When crank is on top dead centre, the distance from shaft centre to crosshead
OC = 1600 + 400 = 2000 mm

When crank is 35◦ past top dead centre, distance from shaft centre to crosshead OB
is now to be calculated:

sin (B) sin (35) 400 sin (35)


= ⇒ sin (B) = = 0. 1434
400 1600 1600

Therefore B = 8. 244◦ = 8◦ 15


A = 180◦ − (35◦ + 8◦ 15 ) = 136◦ 45

 
OB 1600 1600 sin 136◦45
= ⇒ OB =
sin (136◦ 45 ) sin (35) sin (35)
= 1911. 3

Therefore x = 2000 − 1911. 3 = 88. 7 mm


8. Top of stroke
0.1 C

2
2.4

0.5

Let stroke = 1 so crank length = 0.5, connecting rod length = 2.


When crank is on top dead centre, distance from shaft centre to crosshead
= 0. 5 + 2 = 2. 5
Appendix 5 – Solutions to Test Examples • 597

When crosshead has moved down 0.1 of its stroke, the distance from shaft centre
to crosshead OB = 2. 5 − 0. 1 Therefore OB = 2. 4,

2. 42 + 0. 52 − 22
By Cosine rule, cos (O) = = 0. 8375
2 × 2. 4 × 0. 5

Therefore angle at O = 33. 12◦ = 33◦ 7


9.

θ°

x
knots
17
knots

50° 4
knots

By the Cosine rule, x 2 = 172 + 42 − 2 × 17 × 4 cos (50)


So x 2 = 217. 58 ⇒ x = 14. 75 knots

sin (θ ) sin (50) 4 sin (50)


By the Sine rule, = ⇒ sin (θ ) = = 0. 2077

4 14. 75 14. 75
Therefore, θ = 12
The direction of the ship is measured from North.
The direction is therefore 168◦ or 12◦ East of South.

10.
32 naut. miles x nm

23°
Port
2x naut. miles

Using the Cosine rule: a2 = b2 + c2 − 2 × b × c × cos(A)


322 = x 2 + (2x)2 − 2. x · (2x) · cos 23
Therefore 1024 = x 2 + 4x 2 − 3. 682x 2 ⇒ 1. 318x 2 = 1024 ⇒ x 2 = 776. 95
So x = 27. 87 nm
The ships are therefore 27.87 and 55.74 nm from port
598 • Mathematics

11. Half an hour at 18 knots is equivalent to a distance of 9 nm.


Let the distance of the first ship from port be ‘x’, then the distance of the second
ship is ‘x + 9’.

50 naut. miles x naut. miles

73°39′
Port
(x + 9) naut. miles

By the Cosine rule: a2 = b2 + c2 − 2 × b × c × cos(A)


 
502 = x 2 + (x + 9)2 − 2 · x · (x + 9) · cos 73◦ 39
 
Therefore 2500 = x 2 + x 2 + 18x + 81 − 2x 2 + 18x × 0. 2815
2500 = 2x 2 + 18x + 81 − 0. 563x 2 − 5. 067x
0 = 1. 437x 2 + 12. 933x − 2419 divide by 1. 437
0 = x 2 + 9x − 1683. 37
This quadratic equation gives the solutions x = 36. 77 and x = −45. 77
Therefore one ship is 36.77 nm and the other is 45.77 nm from port.
12. B

9.8 10.2

C A
16.4

b2 + c2 − a2
From the Cosine rule: cos A =
2bc
16. 42 + 10. 22 − 9. 82
Therefore cos A = = 0. 8278 ⇒ A = 34◦ 7
2 × 16. 4 × 10. 2
9. 82 + 10. 22 − 16. 42
cos B = = −0. 3445 ⇒ B = 110◦ 9
2 × 9. 8 × 10. 2
Therefore C = 180◦ − (34◦ 7 + 110◦ 9 ) = 180◦ − 144◦ 16 = 45◦ 44
13. B

7.5 m

A 46°51’
C
6.5 m
Appendix 5 – Solutions to Test Examples • 599

1 1  
Area = · a · b · sin C = × 6. 5 × 7. 5 × sin 46◦ 51 = 17. 78 m2
2 2

14. Area = s (s − a) (s − b) (s − c) where a, b and c are the sides of the triangle


(a + b + c)
and s =
2
7. 1 + 4. 2 + 5. 3
So s= = 8. 3
2
Area = 8. 3 × (8. 3 − 7. 1) × (8. 3 − 4. 2) × (8. 3 − 5. 3)
√ √
= 8. 3 × 1. 2 × 4. 1 × 3. 0 = 122. 508
Therefore the area = 11.07 cm2 .

15.

60°
x

1 1
Area = · a · b · sin C = · x · x · sin (60◦ ) = 57. 27 cm2 .
2 2
Therefore 0. 433x 2 = 57. 27 ⇒ x 2 = 132. 26 ⇒ x = 11. 5 cm

Solutions to Test Examples 9


1. A 100 mm D

60 mm 60 mm
140 mm

B 100 mm C

(BC)2 + (CD)2 − (BD)2 1002 + 602 − 1402


By the Cosine rule: cos (C) = =
2 × (BC) × (BD) 2 × 100 × 60
10000 + 3600 − 19600 6000
Therefore cos (C) = =− = −0. 5 ⇒ C = 120◦
12000 12000
600 • Mathematics

The two obtuse angles of 120◦ each mean the two acute angle total 360◦ − 240◦
= 120◦ so each acute angle is 60◦.
By the Cosine rule: (AC)2 = 602 + 1002 − 2 × 60 × 100 × cos (60)
Therefore (AC)2 = 3600 + 10000 − 6000 = 7600 ⇒ AC = 87. 18 mm
Perpendicular height, h = 60 × sin (60) = 51. 96 mm
Area = Base × Vertical height = 100 × 51. 96 = 5196 mm2
2. 32 mm
A
D

32 mm 32 mm
48 mm

B C
32 mm

(BC)2 + (CD)2 − (BD)2 322 + 322 − 482


By the Cosine rule: cos (C) = =
2 × (BC) × (CD) 2 × 32 × 32

1024 + 1024 − 2304 256


Therefore cos (C) = =− = −0. 125 ⇒ C = 97◦ 10
2048 2048

The two angles of 97◦ 10 each mean the two acute angle total 360◦ − 194◦ 20
= 165◦40 so each acute angle is 82◦ 50 .
 
By the Cosine rule: (AC)2 = 322 + 322 − 2 × 32 × 32 × cos 82◦ 50

Therefore (AC)2 = 1024 + 1024 − 2048 × 0. 1248 = 1792. 4 ⇒ AC = 42. 34 mm

Product of diagonals 48 × 42. 34


Area = = = 1016 mm2
2 2
3. 60 mm
D C

50 mm

A B
100 mm

100 + 60
The mean length of the trapezium = = 80 mm
2
The area = Mean length × Height = 8. 0 × 5. 0 = 40 cm2
Appendix 5 – Solutions to Test Examples • 601

4. 2m C
B
117°17′
1m

A 1.5 m

3.5 m

 
By the Cosine Rule: (BD)2 = 22 + 1. 52 − 2 × 2 × 1. 5 × cos 117◦17 = 9. 00
Therefore BD = 3 m
2 + 1. 5 + 3
For triangle BCD, s = = 3. 25
2 √
The area = 3. 25 × (3. 25 − 2) × (3. 25 − 1. 5) × (3. 25 − 3) = 1. 77734
Area BCD = 1. 333 m2
1 + 3 + 3. 5
Similarly, for triangle ABD, s = = 3. 75
2 √
The area = 3. 75 × (3. 75 − 1) × (3. 75 − 3) × (3. 75 − 3. 5) = 1. 9336
Area ABD = 1. 39 m2 and so total area of ABCD = 2. 72 m2
5.
x x

y y

80 mm

Assume the depth of the section removed is x mm, then the removed section
consists of a rectangle and two equal sized triangles.
x
Since the shape is a regular hexagon it can be seen that = tan (60◦ ) giving
y
x
y=
tan (60◦ )
1 x
The area of each triangle is therefore × x × , making the area of the
2 tan (60)
x2 x2
removed section + 80x + = 0. 57735x 2 + 80x
2 tan (60) 2 tan (60)
The hexagon is made from six equal equilateral triangles with side 80 mm.
602 • Mathematics

1 1
The area of one triangle is × base × height = × 80 × (80 sin (60))
2 2
= 2771. 28 mm2 .
The area of the hexagon is therefore 16627.7 mm2 .
The section has an area that is 10% that of the hexagon, so section area
= 1662. 8 mm2 .
Therefore 0. 57735x 2 + 80x = 1662. 8 ⇒ 0. 57735x 2 + 80x − 1662. 8 = 0.
Solving this quadratic equation gives x = 18. 35 or −156. 92.
Therefore, the removed section has a depth of 18.35 mm.

6. The octagon is made from eight equal triangles each having a ’top’ angle of
360
= 45◦
8

67.5°
15

Length

15

180 − 45
Therefore each base angle is = 67. 5◦
2

Length
= tan (67. 5) ⇒ Length = 15 tan (67. 5)
15

So Length = 36.21 mm
30 × 36. 21
The area of each triangle =
2
and so the area of the octagon is

30 × 36. 21
8× = 4346 mm2
2

502
Area of hole = π × = 1963 mm2
4
Therefore, area remaining = 4346 − 1963 = 2383 mm2 .
Appendix 5 – Solutions to Test Examples • 603

7.

120° 60 mm
30°
B A

The diagram gives the necessary angles.


So AB = 2 × 60 cos (30◦ ) = 103. 92 mm
Vertical height = 103. 92 sin(60) = 90 mm
90 × 103. 92
Area = = 4676 mm2
2
 
π D 2 − d2 π (D + d) (D − d) π (755 + 415) (755 − 415)
8. Area of collar = = =
4 4 4
π × 1170 × 340
So area of collar = = 99450π mm 2
4
(i) Effective area = 0. 7×99450π = 69615π mm2 but 1 m2 = (1000)2 mm3 = 106 mm3
69615π
Effective area = = 0. 2187 m2
106
(ii) At a pressure of 2000 kN/m2 total force = 0. 2187 × 2000 = 437. 4 kN

9.
9 cm

α 10 cm

θ = 2α

9
sin (α) = ⇒ α = 64◦ 9. 5
10
Therefore θ = 2α = 128◦ 19 and so sin (θ ) = 0. 7846
π
In radians θ = 128◦19 × = 2. 2395
180
r 2 102
Area of segment = (θ − sin (θ )) = (2. 2395 − 0. 7846) = 72. 45 cm2
2 2
604 • Mathematics

10. Surface area of a sphere = 4πr2 ⇒ Surface area of hemisphere = 2πr2 +πr2 (base)
 2
d 3πd2
Total surface area = 3πr2 = 3π = = 58. 9 cm2
2 4
235. 6
Therefore 3πd2 = 235. 6 ⇒ d2 = = 25

Therefore d = 5 cm

11. D

A O B
d
C
10 mm

By the crossed chords rule: AO × BO = CO × DO


Suppose the depth of the indentation CO = d mm
then DO = ball diameter −CO = 10 − d
Now, AO = BO = half of surface diameter = 2.5 mm

AO × BO = CO × DO ⇒ 2. 5 × 2. 5 = d × (10 − d) ⇒ 6. 25 = 10d − d2

This gives the quadratic equation d2 − 10d + 6. 25 = 0


which has solutions 9.33 and 0.67 mm.
Therefore the depth of the surface indentation is 0.67 mm.
The curved surface area of the indentation is equal to the curved surface area of a
slice of the same depth of a circumscribing cylinder with the same diameter as that
of the ball.
Area = π× ball diameter × depth of indentation = π × 10 × 0. 67 = 21. 05 mm2

12.
125 mm

Slant
height
75 mm

√ √
Slant height = 1252 + 752 = 21250 = 145. 77 mm
Appendix 5 – Solutions to Test Examples • 605

The radius of the sector is the slant height; the arc length of the sector is the base
circumference of the cone.
Base circumference = πd = 150π mm
A circle with radius equal to the slant height has circumference = 2π × 145. 77
Circumference = 291. 54π mm
150π
Therefore the base circumference is = 0. 51451 of the large circle.
291. 54π
This is equivalent to θ = 0. 51451 × 360 = 185◦ 13

13. The top radius is 90 mm and the base radius is 160 mm

90

170 L

160

70

√ √
The slant height L = 1702 + 702 = 33800 = 183. 85 mm
Curved surface area of frustum = π · L · (R + r) = π × 183. 85 × (160 + 90)
Therefore curved surface area = 144394 mm2
14. The outside radius is 320 mm and the inside radius is 220 mm.
Therefore the bar has a diameter of 320−220 = 100 mm and so its radius is 50 mm.
The centre of mass of the bar lies (220 + 50) = 270 mm from the centre of the ring.
The ring can be considered to be the solid of revolution made when a circle, radius
50 mm, is rotated about a point 270 mm distant.
By Pappus Theorem, the surface area equals the length of line multiplied by the
distance moved by the centre of mass.
Length = circumference of bar = 2π × 50 = 100π
Distance moved by c.o.m. = circumference of a circle, radius 270 mm = 2π × 270
= 540π
Therefore surface area = 100π × 540π =532959 mm2 = 5330 cm2

15. Areas of similar figures vary in proportion to the square of the ratio of their linear
dimensions.
Ratio of small side:large side = 125 : 175 = 1 : 1. 4 (dividing both sides by 125)
Ratio of small area to large area = (1)2 : (1. 4)2 = 1 : 1. 96
Therefore the large triangle has an area that is 96% larger than that of the small
triangle.
606 • Mathematics

150
16. Length = 150 m, 12 strips, therefore strip width = = 12. 5
12
Half ordinate Multiplier Product for area
0.1 1 0.1
3.0 4 12.0
5.85 2 11.7
7.2 4 28.8
8.1 2 16.2
8.4 4 33.6
8.4 2 16.8
8.25 4 33.0
8.1 2 16.2
7.5 4 30.0
6.3 2 12.6
3.75 4 15.0
0.5 1 0.5
Total 226.5
12. 5
Half area = × 226. 5 = 943. 75 m2 So waterplane area = 1887. 5 m2
3

Solutions to Test Examples 10


1. 100

12

124 150

10

14

140
Appendix 5 – Solutions to Test Examples • 607

Working in cm:
Area of top flange = 10 × 1. 2 = 12 cm2
Area of centre flange = 12. 4 × 1. 0 = 12. 4 cm2
Area of bottom flange = 14 × 1. 4 = 19. 6 cm2
Total area = 44.0 cm2
Therefore 1m of this girder has volume = 44 × 100 = 4400 cm3
With a density of 7.86 g/cm3 the section has a mass of 4400 × 7. 86 = 34584 g
The mass is therefore 34.58 kg/m run.
2. Working in m:
π 2 
Volume of shaft body = Area of end face × length = D − d2 × 6
4
π 2  π
Therefore volume = 0. 4 − 0. 22 × 6 = (0. 16 − 0. 04) × 6 = 0. 5655 m3
4 4
π
Volume of two couplings = 0. 762 − 0. 42 × 0. 075 × 2 = 0. 0492 m3
4
Total volume = 0. 5655 + 0. 0492 = 0. 6147 m3
Therefore mass = 0. 6147 × 7. 86 × 103 = 4832 kg = 4.832 tonne
3.

π · r2 · h
Volume of cone =
3
Volume of cylinder = π · r2 · h
4π · r3
Volume of sphere =
3
But h = 2r as the height of the cylinder (and cone) is equal to the diameter of the
sphere. Therefore:
π · r2 · 2r 2π · r3
Volume of cone = =
3 3
Volume of cylinder = π · r2 · 2r = 2π · r3
4π · r3
Volume of sphere =
3
Dividing the volumes of the cylinder and sphere by the volume of the cone gives
the ratios:
2π · r3 3
Cylinder: 2π · r3 ÷ = 2π · r3 × =3
3 2π · r3
4π · r 3 3
Sphere: × =2
3 2π · r3
608 • Mathematics

4. Working in m:
240 2
(i) Area of sector = × Area of full circle = × π × 0. 182 = 0. 06786 m2
360 3
Mass = 0. 06786 × 0. 4411 kg
(ii) Working in mm:
2
Circumference of base of cone = Arc length of sector = × 2π × 180
3
= 240π mm
240π
Therefore diameter of cone = = 240 mm
π
(iii) Slant height of cone = Radius of sector = 180 mm

Therefore perpendicular height = 1802 − 1202 = 134. 2 mm
1 1
(iv) Volume of cone = × area of base × perpendicular height= × π × 1202
3 3
×134. 2
So volume = 2023688 mm3 but 1 litre = 106 mm3 so volume = 2.024 litres.

1 πd3 π
5. Volume of hemisphere = × = × 63 cm3
2 6 12
1 πd2 π
Volume of cone = × ×h= × 62 × 5 cm3
3 4 12
π π π 396π
Total volume = × 63 + × 62 × 5 = × 62 × (6 + 5) = cm3
12 12 12 12
396π
Mass = × 8. 4 = 871 g
12
6. Suppose the inner diameter is d (cm), then the outer diameter is d + 2 (cm).
π × (d + 2)3 π × d3 π 3 
The volume of the shell = − = d + 6d2 + 12d + 8 − d3
π 2  6 6 6
Volume = 6d + 12d + 8 cm3
6
Mass (g) 3000
However, volume of shell = =
Specific density 11. 4
π 2  3000 3000 6
Therefore 6d + 12d + 8 = ⇒ 6d + 12d + 8 =
2 × = 502. 6
6 11. 4 11. 4 π
Therefore 6d2 + 12d − 494. 6 = 0
Solving this quadratic gives d = 8. 13 or −10. 13 cm.
The inner diameter is 8.13 cm so the outer diameter is 10.13 cm.
7. 1.2
h

x
2.55
Appendix 5 – Solutions to Test Examples • 609

1
Radius of sphere = × 5. 1 = 2. 55 cm
2
1
Radius of hole = × 2. 4 = 1. 2 cm
2 √ √
Half depth of hole = x cm. Therefore x = 2. 552 − 1. 22 = 5. 0625 = 2. 25 cm
Suppose the thickness of the spherical segments at the top and bottom of the
hole = h cm
So h = radius of sphere −x = 2. 55 − 2. 25 = 0. 3 cm.
πd3 π
(i) Volume of sphere = = × 5. 13 = 69. 456 cm3
6 6
πd2 · l π
Volume of cylindrical hole = = × 2. 42 × (2 × 2. 25) = 20. 36 cm3
4 4
π π
Volume of segment = · h2 (3d − 2h) so volume of both segments = 2 ×
6 6
× 0. 32 (3 × 5. 1 − 2 × 0. 3) = 1. 385cm3
Therefore the volume of the solid = 69. 456 − 20. 36 − 1. 385 = 47. 711 cm3
(ii) Density = 7. 86 g/cm3 and mass = volume × density = 47. 711 × 7. 86
= 375. 0 g
8.
h
D

16

60 mm

A B 6 C 8
28 mm
24 mm
64 mm

h 60 60
By similar triangles, from the diagram = ⇒ h = 16 × = 15 mm
16 64 64
So the height of the hole CD = 60 − 15 = 45 mm
1
Volume of the complete cone = × Area of base × Perpendicular height
3

1 πd2 1 π × 642
V= × × Height = × × 60 = 64340 mm3
3 4 3 4

1 π × 162
Volume of cone removed from top = × × 15 = 1005 mm3
3 4
610 • Mathematics

πh  2 
Volume of bored hole (frustum of a cone) = D + Dd + d2 ,
12
where D = base diameter, d = top diameter and h = height.
π × 45  2 
Volume of hole = 28 + 28 × 16 + 162 = 17530 mm3
12
Net volume = 64340 − 1005 − 17530 = 45805 mm3
or volume = 45. 805 cm3 and at 8.4 g/cm3 , the mass is 45. 805 × 8. 4 = 384. 8 g
1
9. Volume = × Area of base × height
3
The base is made from 6 equilateral triangles with sides 25 mm
1
The area of an equilateral triangle = × (Side)2 × sin (60◦ )
2
252
So the base area = 6 × (Side) × sin (60◦) = 6 ×
2
× 0. 866 = 1623. 8 mm2
2
1
The volume is therefore × 1623. 8 × 60 = 32476 mm3 = 32. 476 cm3
3
Volumes vary as the cube of the ratio of their linear dimensions so the ratio of top
 3  3
3 1 1
cone to whole cone is = = .
6 2 8
Therefore the whole cone has 8 times the volume of the top cone, so the frustum
7
has a volume that is the volume of the whole cone.
8
7
Therefore the volume of the frustum is × 32. 476 = 28. 42 cm3
8
10. Using cm as the basic unit, volumes of similar objects vary as the cube of their
corresponding dimensions, therefore volume varies as depth3 .
Let the volume be V when the depth is 7 cm.
V 73 73
Therefore = 3 ⇒ V = 200 × 3 = 548. 8 cm3
200 5 5
The additional volume is 548. 8 − 200 = 348. 8 cm3
11.

500 mm
h mm

250 mm

300 mm

Before inversion, let the volume of the whole cone be V and let the volume of the
top, empty cone be v.
Appendix 5 – Solutions to Test Examples • 611

As the volumes vary with the cube of their heights:


 3
V 5003 500
= = = 23 = 8
v 2503 250
V
Therefore v = , meaning that the empty cone is one-eighth the volume of
8
the whole cone and that the water is seven-eighths (0.875) the volume of the
whole cone.
V 5003
After inversion, comparing similar cones = 3 ⇒ h3 = 0. 875 × 5003
√ 0. 875V h
This makes h = 0. 875 × 5003 = 478. 2 mm
3

12. The surface area of a solid sphere is 1.5 times the surface area of a smaller sphere,
and the difference in their volumes is 10 cm3 .
Find the volume and diameter of the smaller sphere.
√ √
Ratio of surface areas = 1. 5 : 1 so the ratio of diameters is 1. 5 : 1 = 1. 225 : 1
Therefore the ratio of volumes is (1. 225)3 : 13 = 1. 8375 : 1
Suppose the volume of the smaller sphere is v cm3 then the volume of the larger
sphere is (v + 10)cm3 .
v + 10 1. 8375
Therefore = ⇒ v + 10 = 1. 8375 v ⇒ 10 = 0. 8375 v
v 1
so v = 11. 94 cm 3

πd3 11. 94 × 6
Now volume = ⇒ diameter = 3 = 2. 836 cm
6 π
13.
πd2
Diameter (cm) Area = Multiplier Product
4

39.5 1225.4 1 1225.4

47.7 1787.0 4 7148.0

50.0 1963.5 2 3927.0

47.7 1787.0 4 7148.0

39.5 1225.4 1 1225.4

Total 20673.8

58. 1
Length = 58.1 cm, so strip width = = 14. 525 cm
4
14. 525
Therefore volume = × 20673. 8 = 100095. 65 cm3
3
1 litre = 1000 cm3
Therefore volume = 100.1 litres
612 • Mathematics

14. A water trough has a regular isosceles triangular section, the angle at the bottom
being 80◦ . Calculate the volume flow of water along the trough, in m3 /h, when the
depth of the water in the trough is 180 mm and it is flowing at a velocity of 0.5 m/s.

180 mm 0.5 m/s


80°

w
Suppose the water level has width 2w, then = tan (40◦ ) ⇒ w = 0. 151 m
0. 18
1
The area of the cross-section = × base × perpenicular height = 0. 151 × 0. 18 m2
2
At a rate of 0.5 m/s, the volume per hour = 0. 151 × 0. 18 × 0. 5 × 3600 = 48. 94 m3
15. Find the height of the centre of gravity of a frustum of a cone which is 80 mm
diameter at the base, 60 mm diameter at the top and 40 mm perpendicular height.

H
60 mm

40 mm

10 cm
80 mm

Working in cm:
h h+4
By similar triangles = ⇒ 8h = 6h + 24 ⇒ 2h = 24 ⇒ h = 12 cm
6 8
Therefore the total height, H = 12 + 4 = 16 cm
Appendix 5 – Solutions to Test Examples • 613

The centre of gravity of a cone is one-quarter the height from the base.
π
The volume of the whole cone = × 42 × 16 cm
3
1
Its centre of gravity is of 16 cm, that is, 4 cm from the base.
4
π
The volume of the top cone removed = × 32 × 12 cm
3
1
Its centre of gravity is 4 + × 12 = 7 cm from the base.
4 
moment of volumes
Taking moments about the base: y = 
volumes
Moment of whole cone − Moment of top cone
y=
Volume of whole cone − Volume of top cone
π π
× 42 × 16 × 4 − × 32 × 12 × 7  π
= 3 π 3
π cancel by
× 42 × 16 − × 32 × 12 3
3 3
42 × 16 × 4 − 32 × 12 × 7
=
42 × 16 − 32 × 12
1024 − 756
=
256 − 108
268
=
148

y = 1. 811 cm

Therefore the centre of gravity is 18.11 mm above the base.

Solutions to Test Examples 11


dy
1. (a) y = x 3 + 3x 2 − 9x + 4 ⇒ = 3x 2 + 6x − 9
dx
2x 3 7 2x 3 dy 14
(b) y = − 2 +x ⇒ y = −7x −2 +x ⇒ = 2x 2 +14x −3 +1 = 2x 2 + 3 +1
3 x 3 dx x
√ dy 3 −2/5 3
(c) y = x 3 + 1 ⇒ y = x 3/5 + 1 ⇒
5
= x = √5
dx 5 5 × x2
dy
(d) y = 3 cos (x) + 2 sin (x) ⇒ = −3 sin (x) + 2 cos (x)
dx
dy
2. y = x 2 + 3x − 7 ⇒ = 2x + 3
dx
614 • Mathematics

dy dy
When x = 3, = 2 × 3 + 3 = 9 and when x = −2, = 2 × (−2) + 3 = −1
dx dx
ds dv
3. s = 20t − 5t2 + 4 ⇒ v = = 20 − 10t ⇒ a = = −10 The velocity after 2 s.
dt dt
(a) When v = 0, 20 − 5t = 0 ⇒ t = 4, and so s = 20 × 4 − 5 × (4)2 + 4
= 80 − 80 + 4 = 4 m

(b) The acceleration = −10 m/s2 .


1 dy 1
4. y = 4x + = 4x + x −1 ⇒ = 4 − x −2 = 4 − 2
x dx x
1 1 1 1
When the gradient = 0 4 − 2 = 0 ⇒ 4 = 2 ⇒ x 2 = and so x = ±
x x 4 2
dy
5. y = 3x 2 − x + 2 ⇒ = 6x − 1
dx
6x − 1 = −7 ⇒ 6x = −7 + 1 ⇒ 6x = −6 ⇒ x = −1

t2
6. θ = 2 + 16t − ⇒ ω = 16 − t
2
When t = 2, ω = 14 rad/s, and when ω = 0, 16 − t = 0 and so t = 16 s
dy 1
7. (a) y = x 3 + 3x + ln (x) ⇒ = 3x 2 + 3 + = 3x 2 + 3 + x −1
dx x
d2 y −2 1
It follows that 2 = 6x − x = 6x − 2
dx x
dy
(b) y = 3 cos (θ ) − 7 sin (θ ) + θ ⇒ = −3 sin (θ ) − 7 cos (θ )

2
d y
⇒ = −3 cos (θ ) + 7 sin (θ )
dθ 2
dy
8. y = 12x + 3x 2 − 2x 3 ⇒ = 12 + 6x − 6x 2
dx
dy
At a turning point = 0 and so 12 + 6x − 6x 2 = 0 ⇒ 2 + x − x 2 = 0
dx
This factorises as (2 − x) (1 + x) = 0 which has solutions x = 2 and x = −1
d2 y d2 y
= 6 − 12x and when x = 2, = 6 − 24 = −18.
dx 2 dx 2
This means that the turning point at x = 2 is a maximum.
Therefore the maximum value of y is y = 12 (2)+3 (2)2 −2 (2)3 = 24+12−16 = 20.
ωLx ωx 2 dM ωL
9. M = − ⇒ = − ωx
2 2 dx 2
dM ωL ωL L
When = 0, − ωx = 0 ⇒ = ωx and so x =
dx 2 2 2
d2 M
= −ω which shows that the turning point is a maximum.
dx 2
Therefore the maximum bending moment occurs at the midpoint of the beam.
Appendix 5 – Solutions to Test Examples • 615

 
1 4
10. (a) t = 2. 1 × 3
−√ ⇒ t = 2. 1 × θ 2/3 − 4 × θ −1/5
θ2 5
θ
dt
⇒ = −1. 4 × θ −5/3 + 0. 8 × θ −6/5
dθ  
dt 1 1
Therefore = −1. 4 × 3 5 + 0. 8 × 5 6
dθ θ θ
a · un − 1 dz a · n · un−1
(b) z = ⇒ =
c du c
 2  dy
(c) y = 3x · x − 4 ⇒ y = 3x 3 − 12x ⇒ = 9x 2 − 12
dx
dy
(d) y = 2 cos (θ ) + 5 ⇒ = −2 sin (θ )

x3
11. f (x) = − 2x 2 + 3x + 1 ⇒ f  (x) = x 2 − 4x + 3 = (x − 3) (x − 1) = 0 at max/min
3
Therefore the turning points occur when x = 1 and x = 3.
f  (x) = 2x − 4 = 2 when x = 3 and equals −2 when x = 1.
The minimum value is therefore when x = 3 and f (x) = 9 − 18 + 9 + 1 = 1.
dP
12. (a) P = a · V b ⇒ = a · b · V b−1
dV
dP
(b) when V = 100, = 0. 5 × 10−10 × 6 × 1005 = 3
dV
13. A line of length L is to be cut up into four parts and put together as a rectangle.
Show that the area of the rectangle will be a maximum if each of its sides is equal
to one quarter of L (i.e. a square).
L − 2x L
Let two of the sides be x, then the other two sides must be = −x
  2 2
L L·x
The area A = x · −x = −x 2
2 2
dA L
So = − 2x = 0 at max/min
dx 2

L L
− 2x = 0 ⇒ x =
2 4

d2 A
But − 2 which means that the turning point is a maximum.
dx 2
L
As x = , all four sides are the same and so the shape is a square.
4
1
14. (a) f (θ ) = cos (θ ) − ln (θ ) ⇒ f  (θ ) = − sin (θ ) − = − sin (θ ) − θ −1
θ
⇒ f  (θ ) = − cos (θ ) + θ −2
1
So f  (θ ) = − cos (θ ) + 2
θ
616 • Mathematics

(b) f (t) = a · t2 + 2 · ln (t) ⇒ f  (t) = 2at + 2t−1 ⇒ f  (t) = 2a − 2t−2


2
So f  (t) = 2a − 2
t
(c) f (x) = 5ex ⇒ f  (x) = 5ex ⇒ f  (x) = 5ex
dx dv
15. x = 0. 2t2 + 10. 4 ⇒ v = = 0. 4t ⇒ a = = 0. 4
dt dt
(a) When t = 5, v = 2 m/s and a = 0. 4 m/s2
100 − 10. 4
(b) When x = 100, 0. 2t2 + 10. 4 = 100 ⇒ t2 = = 448 ⇒ t = 21. 17s
0. 2
Therefore v = 8. 48 m/s and a = 0. 4 m/s2 .
dθ dω
16. θ = 2. 1 − 3. 3t + 4. 8t2 ⇒ ω = = −3. 3 + 9. 6t ⇒ = 9. 6
dt dt
Therefore the angular velocity = −3. 3 + 9. 6 × 1. 5 = 11. 1 rad/s and angular
acceleration = 9.6 rad/s2 .

17. f (x) = x 5 − 5x ⇒ f  (x) = 5x 4 − 5 = 0 at turning point ⇒ x 4 = 1 ⇒ x = ±1


f  (x) = 20x 3 so when x = +1 f  (x) > 0 so the turning point is a minimum and
when x = −1 f  (x) < 0 so the turning point is a maximum.
When x = 1, f (x) = 1 − 5 = −4 so the minimum is (1, −4)
When x = −1, f  (x) = −1 + 5 = 5 so the maximum is (−1, 4)
dy
18. y = a · x 2 + bx + c ⇒ = 2ax + b = 4x + 2
dx
Therefore b = 2 and 2a = 4, giving a = 2.

d2 y
= 4 so the turning point is a minimum.
dx 2
dy 2 1
At the turning point = 4x + 2 = 0 ⇒ x = − ⇒ x = −
dx 4 2
 2  
1 1 2 3
Therefore 1 = 2 − +2 − +c ⇒1= −1+c ⇒c =
2 2 4 2
3
So a = b = 2, c =
2

Solutions to Test Examples 12



  x5 x3
1. (a) x 4 − x 2 − 8x + 5 dx = − − 4x 2 + 5x + c
5 3
     
4 1 −2 1 4
(b) + dx = 4x + dx = −4x −1 + ln (x) + c = ln (x) − + c
x2 x x x
Appendix 5 – Solutions to Test Examples • 617

3
   3    
(c) 2x − 3x 2 − 1 dx = x 2 − x 3 − x 1 = 32 − 33 − 3 − 12 − 13 − 1
1
= −21 − (−1) = −20

dy  2  x3 x2
2. = x2 + x − 2 ⇒ y = x + x − 2 dx = + − 2x + c.
dx 3 2
The curve passes through the point (2, 5) therefore:
23 22 8 8 13
5= + − 2 (2) + c ⇒ 5 = + 2 − 4 + c ⇒ c = 5 − + 2 ⇒ c =
3 2 3 3 3
x3 x2 13
Therefore y = + − 2x +
3 2 3

3. (a) (3 cos (x) − 2 sin (x) + 4) dx = 3 sin (x) + 2 cos (x) + 4x + c

x2
(b) (4 cos (x) − sin (x) + x) dx = 4 sin (x) + cos (x) + +c
2
π/2 π 
π/2
(c) cos (x) dx = [sin (x)]0 = sin − sin(0) = 1 − 0 = 1
2
0
4. 3 3
  x4 4x 3 3x 2
Area = x 3 − 4x 2 + 3x dx = − +
4 3 2 1

1   
34 4 × 33 3 × 32 14 4 × 13 3 × 12
= − + − − +
4 3 2 4 3 2
       
81 27 1 4 3 9 5 8
Area = − 36 + − − + = − − = − (−2. 667)
4 2 4 3 2 4 12 3

C
5. pV = C ⇒ p = , between V1 and V2 .
V
V2  
C V2 V2
Area under the graph = dV = [C · ln(V)]V1 = C [ln (V2 ) − ln (V1 )] = C · ln
V V1
V1  
V2
but C = p · V, therefore Area = p · V · ln
V1

6. (a) v = 6t dt = 3t2 + c but v = 0 when t = 0, so 0 = 0 + c giving c = 0
Therefore v = 3t2
3
 3 3
3t2 dt
2t 2
(b) Average velocity = = = (3)3 − (2) 3 = 27 − 8 = 15 m/s
3−2 1
(c) When t = 2. 5 s, v = 3 × (2. 5)2 = 18. 75 m/s
618 • Mathematics


 
7. v = 3t + 8t + 12 ⇒ s =
2
3t2 + 8t + 12 dt = t3 + 4t2 + 12t + c.
Given that s = 10 when t = 0, 10 = 0 + 4 × 0 + 12 × 0 + c ⇒ c = 10
Therefore s = t3 + 4t2 + 12t + 10
so after 10, s = 103 + 4 × 102 + 12 × 10 + 10 = 1000 + 400 + 120 + 10
= 1530 m
 6 6 2 6  3 
x x3 6 216
8. V = π y dx = π
2
dx = π =π −0 = π
4 12 0 12 12
0 0
= 18π units3 (=56. 55)

9. The limits of the integral are the values of x where 0 = x 2 − x.


Therefore 0 = x (x − 1) giving x = 0 and x = 1.

1 1 1
 2   x5 x4 x3
V =π x2 − x dx = π x 4 − 2x 3 + x 2 dx = π − +
5 2 3 0
0 0
 
1 1 1 π
=π − + − (0) =
5 2 3 16

 
 x3 3x 2 
10. (a) (x + 1) (x + 2) dx =
x 2 + 3x + 2 dx = + + 2x + C
3 2
     
1 1 1 x −1 1
(b) − 2 dx = − x −2 dx = ln (x) − + C = ln(x) + + C
x x x −1 x
(c) 2 2
(a + 2b) (x + 1) dx = [(a + 2b) (x) + (a + 2b)] dx
1 1
 2 2
x
= (a + 2b) + (a + 2b) x
2 1
  2      
x2 22 12
= (a + 2b) +x = (a + 2b) +2 − +1
2 1 2 2
   
1 3 5 (a + 2b)
= (a + 2b) [2 + 2] − +1 = (a + 2b) 4 − =
2 2 2

(d) 3ex dx = 3ex + C

8 8
  0. 06x 3  
11. 0. 06x 2 + 10 dx = + 10x = 0. 02 × 83 + 10 × 8
3 6
6  
− 0. 02 × 6 + 10 × 6 = 25. 92
3
Appendix 5 – Solutions to Test Examples • 619

V2 V2 V2 V2


C −n C · V −n+1
12. Area = p dV = dV = C·V dV =
Vn −n + 1 V1
V1 V1 V1
   
C · V21−n C · V21−n1 C  1−n 
= − =− V2 − V11−n
−n + 1 −n + 1 n−1
 
C 1 1
=− − n−1
n − 1 V2 n−1
V1

as − (1 − n) = n − 1
      
1 1 1 θ 1 θ
13. (a) sin(θ ) − dθ = − cos(θ ) − +C =C− cos(θ ) +
3 2 3 2 3 2

(b) sin(x)dx = [− cos(x)]π0 = (− cos(π)) − (− cos(0)) = − (−1) − (−1) = 2
0
π/2 π/2
(c) x2
(5 cos(x) + 3 sin(x) − x) dx = 5 sin(x) − 3 cos(x) −
2 0
0 ⎛  π 2 ⎞
π  π   
⎜ ⎟ 02
= ⎝5 sin − 3 cos − 2 ⎠ − 5 sin(0) − 3 cos(0) −
2 2 2 2
 
π2 π2
= 5−0− − (0 − 3 − 0) = 8 − = 6. 7663 to 4 decimal places.
8 8
14.
2b Axis

2a

a  
2 x2 y2 x2
V=π y dx but 2 + 2 = 1 ⇒ y = b 1 − 2
2 2
a b a
−a
a  
x2
⇒V =π b2 1 − 2 dx
a
−a 
  a   
x3 a3 (−a)3
So V =π b x − 2
2 = π ·b 2 a − 2 − (−a) −
3a −a 3a 3a2
     
a −a 2a a
= π · b2 a − − (−a) − = π · b2 − (−a) +
3 3 3 3
   
2a 2a 4π · a · b2
= π · b2 − − =
3 3 3
620 • Mathematics

dv dv
15. = −kv ⇒ = −k dt
dt v
Assume that k is a positive value, but the acceleration must be decreasing, hence
the negative sign to indicatea negative
 rate of change.
dv
Integrating both sides gives = −kdt
v
Therefore ln (v) = −kt + C but when t = 0, the initial velocity is u and so,

ln (u) = −k. 0 + C ⇒ C = ln (u)


v
So ln (v) = −kt + ln (u) ⇒ ln (v) − ln (u) = −kt and so ln = −kt
u
16. y

b
a

x
–1 –0.5 0.5 1
 
1 b 1+a
Area = dx = [b ln (x + a)]10 = b ln (1 + a) − b ln (a) = b ln
0 x+a a
Area = 1. 386 when a = 1 and b = 2 (3 decimal places accuracy)

3
17. (a) dx = 3 ln(x) + C
x
 3
x x4
(b) dx = +C
a 4a

 3 
(c) 4z + 3z2 + 2z + 1 dz = z4 + z3 + z2 + z + C

(d) (2 cos (θ ) − 5 sin (θ )) dθ = 2 sin (θ ) + 5 cos (θ ) + C
18. A particle P starts from rest at O with a velocity of 5 m/s and moves along a straight
line OX with an acceleration of −2t2 at time t seconds after leaving O.
Describe the motion after 3 s.
dv 2t3 2×0
= −2t2 ⇒ v = − + C but, at t = 0 v = 5, so 5 = − +C ⇒C =5
dt 3 3
ds 2t 3 2t 4 t 4
Therefore v = =5− s = 5t − + C ie s = 5t − + C
dt 3 12 6
Appendix 5 – Solutions to Test Examples • 621

However, the particle starts at the origin so s = 0 when t = 0 which means that
C = 0.
t4
Therefore s = 5t −
6
After 3 s:
3 4 81
• s=5×3− = 15 − = 1. 5 m
6 6
2 × 33
• v =5− = 5 − 18 = −13 m/s
3
• a = −2 × 32 = −18 m/s2
Therefore the particle is 1.5 m along OX moving in the direction XO at 13 m/s,
decelerating at 18 m/s2 .

Solutions to Test Examples 13



1. x · ex dx

u=x dv = ex
du = 1 v = ex

Integral = x · ex − ex · 1dx = x · ex − ex + C

2. 2x · e3x dx

u = 2x dv = e3x
e3x
du = 2 v=
3
 
e3x  e3x 2x · e3x 2 · e3x 2 · e3x 1
Integral = 2x · − · 2dx = − +C = x− +C
3 3 3 9 3 3

3. x · sin(x) dx
π/2

u=x dv = sin (x)


du = 1 v = − cos (x)
 π
Integral = −x · cos(x) + cos(x)dx = −x · cos(x) + sin(x)
π/2
 π π   π 
= (−π · cos(π) + sin(π)) − − · cos + sin
2 2 2
= (π + 0) − (0 + 1) = 2. 142
622 • Mathematics


5x − 3
4. dx
(x − 3)(x + 3)
4 1
5 − x = A(x + 3) + B(x − 3) ⇒ B = − A =
3 3
  
1/3 4/3 1 4
− dx = ln (x − 3) − ln (x + 3) + c
x−3 x+3 3 3

9x 2 + 34x + 29
5. dx
(x + 1)(x + 2)(x + 3)

9x 2 + 34x + 29 = A(x + 2)(x + 3) + B(x + 1)(x + 3) + C(x + 1)(x + 2)

⇒ A = 2, B = 3 and C = 9
Therefore the integral is 2 ln (x + 1) + 3l n (x + 2) + 9 ln (x + 3) + C
7x − 2x 2 A B C
6. = + +
(2 − x)2 (1 + x) 2 − x (2 − x)2 1+x

7x − 2x 2 = A (2 − x) (1 + x) + B (1 + x) + C (2 − x)2

x = 2 ⇒ B = 2; x = −1 ⇒ C = −1; coefficients of x 2 ⇒ A = 1
  
1 2 1 2
So + − dx = − ln (2 − x) + − ln (1 + x) + C
2 − x (2 − x)2 1+x 2−x
1  
7. x. 1 − x 2 dx
0
du du
Let u = 1 − x 2 ; then = −2x ⇒ dx =
dx −2x
1 
du 1
Substituting gives x(u) /2 =− u /2 du as the x’s cancel.
1 1

−2x 2
0
1  
1 u3/2 1 1
1 1
Therefore the integral is − · = − · (1 − x 2 )3/2 = (0) − − = .
2 3/2 0 3 0 3 3
4
x
8.   dx
3
3
x2 − 7
du du
Let u = x 2 − 7; then = 2x ⇒ dx =
 dx  2x
1/3 du 1
= u /3 du as the x’s cancel.
1
Substituting gives x(u)
2x 2
1 u2/3 3 4
3  2/3 
Therefore the integral is · = · (x 2 − 1)2/3 = 15 − 82/3 = 1. 562.
2 2/3 4 3 4
Appendix 5 – Solutions to Test Examples • 623


9. sin (θ ) · cos2 (θ ) dθ
0
du du
Let u = cos (θ ); then = − sin (θ ) ⇒ dθ =
 dθ  sin (θ )

du
Substituting gives sin (θ ) (u) 2
= − u2 du as the x’s cancel.
−sin (θ )
u3  π    
Therefore the integral is − = − cos3 (θ ) 0 = − cos3 (π) − − cos3 (0)
3
= 2.
 
1
10. 5 sin(7t) cos(3t)dt = 5. (sin(7t + 3t) + sin(7t − 3t)) dt
 2
5
= (sin(10t) + sin(4t)) dt
2   
5 cos(10t) cos(4t) 5 cos(10t) 5 cos(4t)
= − − +C = − − +C
2 10 4 20 8

11. π/4 π/4


1
cos(4x) cos(2x)dx = (cos(4x + 2x) + cos(4x − 2x))dx
2
0 0
π/4   π/4
1 1 sin(6x) sin(2x)
= (cos(6x) + cos(2x))dx ⇒ I = +
2 2 6 2 0
⎡ ⎧
0   ⎫ ⎤
⎪ 3π π ⎪

⎨ sin sin ⎪
⎬  
⎢1 2 ⎥ 1 −1 1 1
=⎣⎢ + 2 ⎥
− 0⎦ = + =
2⎪⎪ 6 2 ⎪
⎪ 2 6 2 6
⎩ ⎭
3  
3   x3 1 32
12. Area to xaxis = x 2 + 1 dx = +x = (9 + 3) − +1 = unit2
1 3 1 3 3
 3
1 2 3 1  2 2 1 3  4  1 x5 2x 3
y dx = x + 1 dx = x + 2x 2 + 1 dx = + +x
2 1 2 21 2 5 3 1
   
1 243 54 1 2 1016
= + +3 − + +1 = unit
2 5 3 5 3 30
 
1016 32 127
Therefore y = ÷ = units = 3. 175.
30 3 40
Volume of revolution (by Pappus’ Theorem)
32
Area to axis = unit2
3
Distance moved by centroid = 2π × y = 19. 949
32
Therefore volume = × 19. 949 = 212. 8 unit3
3
624 • Mathematics

1
1 √  1  1/2
 x 3/2 x3
13. Area = x− x2 dx = x − x2 dx = −
3/2 3

0  0 0
1 1 1
Area = − − {0} = = 0. 333 unit2
3/2 3 3 √
Volume = (Vol. produced by rotating y = x) − (Vol. produced by rotating y = x 2 )
1 1   1  
ie Volume = π (x) dx − π x 4 dx = π x − x 4 dx
0 0 0
  1  
x2 x5 1 1
Volume = π − =π − − (0) = 0. 3π = 0. 942 units3
2 5 0 2 5
4 5. 6
14. s = 2t − 1. 4 ln(0. 5 + 4t) ⇒ v = 2 − 1. 4 × =2−
4t + 0. 5 4t + 0. 5
22. 4
So v = 2 − 5. 6 (4t + 0. 5)−1 ⇒ a = 5. 6 (4t + 0. 5)−2 × 4 ⇒ a =
(4t + 0. 5)2
5. 6 5. 6
When velocity = 0, v = 2 − =0⇒2=
4t + 0. 5 4t + 0. 5
Therefore 4t + 0. 5 = 2. 8 ⇒ t = 0. 575 sec
At this time s = 2 × 0. 575 − 1. 4 ln(4 × 0. 575 + 0. 5) = −0. 292 m
22. 4
and a= = 2. 857 m/s2
(2. 8)2

Solutions to Test Examples 14


1. (a) p (x < 256) = Φ (1. 33) = 0. 9082
(b) p (x > 257) = 1 − Φ (2) = 0. 0228 Proportion= 2. 28%
(c) p (250 < x < 255) = Φ (0. 67) − Φ (−2. 67) = 0. 7486 − 0. 0038 = 0. 7448
Expect 149
2. For the population: NP = 5000, μ = 16. 5, σ = 0. 5. For the sample N = 400
   
σ NP − N 0. 5 5000 − 400
(a) σx = √ · =√ · = 0. 02398
N NP − 1 400 5000 − 1
 
16. 495 − 16. 5 16. 506 − 16. 5
(b) p=1 − p <Z<
0. 024 0. 024
= Φ (0. 25) − Φ (−0. 21)
= 0. 5987 − (1 − 0. 5832)
= 0. 1819
 
16. 51 − 16. 5
(c) p = 1 − p Z < = 1 − Φ (0. 42) = 1 − 0. 6628 = 0. 3372
0. 024
Appendix 5 – Solutions to Test Examples • 625

3. 0.12
L = 0.6 L

Dislike S = 0.4 S 0.08


0.2
L 0.20*
L = 0.4
Underpaid

0.5
S = 0.6
S 0.30
Absence L
L = 0.9 0.27
0.3

S = 0.1
S 0.03

p (dislike ∩ L)
p(dislike|L) =
p(L)
0. 12
=
(0. 12 + 0. 20 + 0. 27)
0. 12
= = 0. 203
0. 59
4.
x y x2 y2 xy
4.8 3.5 23.04 12.25 16.8
9.3 8.2 86.49 67.24 76.26
12.8 10.1 163.84 102.01 129.28
17.7 15.6 313.29 243.36 276.12
21.6 18.4 466.56 338.56 397.44
26 20.8 676 432.64 540.8
92.2 76.6 1729.22 1196.06 1436.7

n=6
Mean x = 15. 36667
Mean y = 12. 76667
var(x) = 52. 06889
626 • Mathematics

var(y) = 36. 35556


Cov = 43. 26889
Cor. coeff = 0. 994492
Slope = 0. 830993
Int.t = −0. 00293

25 Tensile strength test


y = 0.831x – 0.0029
20
Extension (mm)

15

10

5
Tensile force (kN)
0
0 5 10 15 20 25 30

However, the nature of the experiment of measuring extension for a given load implies
that zero load gives zero extension, and therefore the regression line must pass through
the origin.

x·y 1436. 7
This leads to a line with equation y = bx where b =  2 = = 0. 830837
x 1729. 22
In this case the difference between the slope through the origin, 0.830837, and that of
the regression line, 0.830993, is miniscule. This may not always be the case.
APPENDIX 6
MATHEMATICAL
FORMULAE
Laws of indices

am  m n
am × an = am+n n
= am−n a = amn
a
m √ 1
a−n = n a0 = 1
n m
an = a
a

Quadratic formula

−b ± b2 − 4ac
If ax 2 + bx + c = 0 then x =
2a
Factor theorem

If x = a is a root of the equation f (x) = 0, then (x − a) is a factor of f (x).

Remainder theorem

If f (x) is divided by (x − p), the remainder will be f (p)

Partial fractions

Provided that the numerator f (x) is of degree less than the denominator, the following
identities are typical examples of the types of partial fractions used:
628 • Mathematics

Expression Form of partial fraction


f (x) A B C
+ +
(x + a) (x − b) (x + c) x+a x−b x+c
f (x) A B C
+ +
(x + a)3 (x + a) (x + a) 2
(x + a)3
f (x) Ax + B D
   +
ax 2 + bx + c (x + d) ax 2 + bx + c (x + d)

Definition of a logarithm

If y = ax then x = loga y

Laws of logarithms

log (A × B) = log A + log B


 
A
log = log A − log B
B
 
log An = n × log A

Series
x2 x3
ex = 1 + x + + + · · · valid for all x
2! 3!
x2 x3 x4
ln(1 + x) = x − + − + · · · valid for −1 < x ≤ 1
2 3 4
x2 x3 x4
ln(1 − x) = −x − − − + · · · valid for −1 ≤ x < 1
2 3 4
For x in radians and valid for all x:
x3 x5
sin(x) = x − + − ···
3! 5!
x2 x4
cos(x) = 1 − + − ···
2! 4!
x3 x5
sinh(x) = x + + + ···
3! 5!
x2 x4
cosh(x) = 1 + + + ···
2! 4!

Hyperbolic functions
ex − e−x ex + e−x ex − e−x
sinh(x) = cosh(x) = tanh(x) = x
2 2 e + e−x
Appendix 6 – Mathematical Formulae • 629

Binomial series
n (n − 1) 2 n (n − 1) (n − 2) 3
(1 + X)n = 1 + nX + X + X + ···
2! 3!

Basic expansions/factorisations

x 2 − y2 = (x + y) (x − y)
  
x 4 − y4 = x 2 + y2 x 2 − y2
 
x 3 + y3 = (x + y) x 2 − xy + y2
 
x 3 − y3 = (x − y) x 2 + xy + y2
(x + y)2 = x 2 + 2xy + y2
(x − y)2 = x 2 − 2xy + y2
(x + y)3 = x 3 + 3x 2 y + 3xy2 + y3
(x − y)3 = x 3 − 3x 2 y + 3xy2 − y3

Calculus formulae
Differentiation

dy
y or f (x) or f  (x)
dx

(ax + b)n a · n · (ax + b)n−1

sin(ax + b) a · cos(ax + b)

cos(ax + b) −a · sin(ax + b)

tan(ax + b) a · sec2 (ax + b)

eax+b a · ea+bx

a
ln(ax + b)
ax + b
(continued)
630 • Mathematics

dy
y or f (x) or f  (x)
dx
a
sin−1 (ax) √
1 − a2 x 2
−a
cos−1 (ax) √
1 − a2 x 2
a
tan−1 (ax)
1 + a2 x 2
a
sinh−1 (ax) √
1 + a2 x 2
a
cosh−1 (ax) √
a x2 − 1
2

a
tanh−1 (ax)
1 − a2 x 2
f  (x)
ln [f (x)]
f (x)

Product rule
dy
If y = u · v where u and v are functions of x, then = u · dv + v · du
dx
Quotient rule
u dy v · du − u · dv
If y = where u and v are functions of x, then =
v dx v2
Function of a function rule
dy dy du
If u is a function of x, then = ×
dx du dx
Integration


y ydx

(ax + b)n+1
(ax + b)n , n = −1
a · (n + 1)
1
sin(ax + b) − · cos(ax + b)
a
1
cos(ax + b) sin(ax + b)
a
1 ax+b
eax+b ·e
a
(continued)
Appendix 6 – Mathematical Formulae • 631


y ydx

1 
= x −1 ln(x)
x
1 1
· ln(ax + b)
ax + b a
1
tan(ax) · ln [sec(ax)]
a
 
1 sin(2x)
cos2 (x) x+
2 2
 
1 sin(2x)
sin2 (x) x−
2 2
1 1 x
· tan−1 ( )
a2 + x 2 a a
1 1 x
√ · sin−1 ( )
a − x2
2 a a
1 x
√ sinh−1 ( )
x + a2
2 a
1 x
√ cosh−1 ( )
x − a2
2 a
f  (x)
ln [f (x)]
f (x)

Integration by parts

If u and v are both functions of x, then

u · dv = u · v − v · du

Applications of Integration
Basic formulae

If y is a function of x, that is, y = f (x) and the range is a ≤ x ≤ b then the following
formulae apply:

Areas and volumes of revolution


b

Area A = ydx
a
632 • Mathematics

Volume 360◦ about the x axis,

b b

Vx = π y2 dx, the y axis, Vy = 2π x · y dx


a a

Centroids of plane areas

The centroid of an area is the point at which the total area is considered to be situated
for calculation purposes. (If the area was a thin lamina the centroid would indicate the
position of the centre of mass.)

If x and y represent the coordinates of the centroid of area A, then:

b 1 b 2
x · y dx y dx
a 2a
x= and y=
b b
y dx ydx
a a

Second moment of area

A measure of the resistance to the bending of a structural member. The further the
majority of the mass of a body is from its centroid the more resistive it is to bending. If Ix
and Iy represent second moments of area about the x axis and y axis respectively, then:

b b
1
Ix = 3
y dx and Iy = x 2 · y dx
3
a a

Second moment of mass (moment of inertia)

If the mass per unit volume of the volume of revolution generated by the rotation of
area A about the x axis is m, then the moment of inertia of the solid about the x axis is:

b
1
IOx = y4 dx
2
a

Theorem of Pappus

Volume of revolution obtained by rotating area A about the x axis = 2πyA

Volume of revolution obtained by rotating area A about the y axis = 2πxA


Appendix 6 – Mathematical Formulae • 633

Mean and r.m.s. values


b b
1 1
Mean value = y dx r. m. s. value = y2 dx
b−a b−a
a a

Laplace Transforms

y y

1
1
s
k
k
s
1
eat
s−a
a
1 − e−at
s (s − a)
a
sin(at)
s + a2
2

s
cos(at)
s + a2
2

a
sinh(at)
s − a2
2

s
cosh(at)
s − a2
2

1
t
s2
n!
tn n+1
s
n!
eat · tn
(s − a)n+1
ω
eat · sin ωt
(s − a)2 + ω2
s−a
eat · cos ωt
(s − a)2 + ω2
 1 1
f (t) dt · F(s) + · y0
s s
634 • Mathematics

dy d2 y
The transform of is s · y − y0 , and the transform of 2 is s2 · y − s · y0 − y1 where y
dt dt
dy
is the Laplace transform of y, y0 is the value of y when t = 0 and y1 is the value of
dt
when t = 0.

The First Shift Theorem

If a function, f (t), is multiplied by e−at , the effect on the Laplace transform is simply to
replace s with (s + a).

The Laplace transform of a function, f (t), with period ω is given by

ω
1
L{f (t)} = e−st · f (t) dt
1 − e−ωs
0

Fourier coefficients

Functions with period 2π

1 π
a0 = f (x) dx
2π −π
1 π
an = f (x) · cos(nx) dx
π −π
1 π
bn = f (x) · sin(nx) dx
π −π


Functions with period T : ω =
T

1 T
a0 = f (x) dx
T 0
2 T
an = f (x) · cos(nωx) dx
T 0
2 T
bn = f (x) · sin(nωx) dx
T 0
Appendix 6 – Mathematical Formulae • 635

Half-range series
Series with period 2π

(a) Fourier cosine series

1 π
a0 = f (t) dt
π 0
2 π
an = f (t) · cos(nt) dx
π 0

(b) Fourier sine series

2 π
bn = f (t) · sin(nx) dt
π 0

Series with period T

(a) Fourier cosine series

1 T
a0 = f (t) dx
T 0
T/2
4
an = f (t) · cos(nωt) dt
T
0

(b) Fourier sine series


T/2
4
bn = f (t) · sin(nωt) dt
T
0

Theorem of Pythagoras

a2 = b 2 + c 2

a
b

c
636 • Mathematics

Trigonometry
Identities
1 1
sec(x) = cosec(x) =
cos(x) sin(x)
1 sin(x)
cot(x) = tan(x) =
tan(x) cos(x)
cos2 (x) + sin2 (x) = 1
cosh2 (x) − sinh2 (x) = 1
1 + tan2 (x) = sec2 (x)
cot2 (x) + 1 = cosec2 (x)
sin(−x) = − sin(x) cos(−x) = cos(x)
tan(−x) = − tan(x) sinh(−x) = sinh((x)
cosh(−x) = cosh(x) tanh(−x) = − tanh(x)

Triangle formulae

c b

B C
a

Sine rule
a b c
= =
sin(A) sin(B) sin(C)

Cosine rule
a2 = b2 + c2 − 2bc · cos(A)

Area of any triangle

(i) 1/2 × base × vertical height


Appendix 6 – Mathematical Formulae • 637

a · b · sin (C) a · c · sin (B) b · c · sin (C)


(ii) , or
2 2 2
a+b+c
(iii) s (s − a) (s − b) (s − c) where s =
2
Compound angle formulae

sin (A ± B) = sin(A) cos(B) ± cos(A) sin(B)

cos (A ± B) = cos(A) cos(B) ± sin(A) sin(B)


tan(A) ± tan(B)
tan (A ± B) =
1 ± tan(A) tan(B)

Double angle formulae

sin(2A) = 2 sin(A) cos(A)


cos(2A) = cos2 (A) − sin2 (A)
= 2 cos2 (A) − 1 = 1 − 2 sin2 (A)
2 tan(A)
tan(2A) =
1 − tan2 (A)

Products into sums and differences


1
sin(A) cos(B) = [sin(A + B) + sin(A − B)]
2
1
cos(A) sin(B) = [sin(A + B) − sin(A − B)]
2
1
cos(A) cos(B) = [cos(A + B) + cos(A − B)]
2
1
sin(A) sin(B) = − [cos(A + B) − cos(A − B)]
2

Sums and differences into products


x + y  
x−y
sin x + sin y = 2 sin cos
2 2
x + y x − y
sin x − sin y = 2 cos sin
2 2
x + y  
x−y
cos x + cos y = 2 cos cos
2 2
x + y x − y
cos x − cos y = −2 sin sin
2 2
638 • Mathematics

For a general sinusoidal function

y = A sin(ωt ± α), then


A = amplitude,
ω = angular velocity = 2πf rad/s

= periodic time, T seconds
ω
ω
= frequency, f hertz

α = angle of lead or lag


If R sin(ωt + α) 
= a· sin(ωt) + b · cos(ωt) then a = R cos(α), b = R sin(α), R = a2 + b2 ,
−1 b
and α = tan
a

Cartesian and polar co-ordinates

If co-ordinate (x, y) = r∠θ then


y
r= x 2 + y2 and θ = tan−1
x

If co-ordinate r∠θ = (x, y) then

x = r cos θ and y = r sin θ

Areas/Volumes

The circle

Area = πr2 Circumference = 2πr = πd

π radians = 180◦
s
r
θ
r
Appendix 6 – Mathematical Formulae • 639

Sector of circle: (θ in radians)

s = rθ
 
1 2 α◦
shaded area = r θ = × πr 2
2 360◦
Segment of circle
r2
Area = (θ − sin θ )
2

Ellipse
 
D+d
Circumference = π
2
πDd
Area = πRr =
4

Annulus
  π 2 
Area = π R2 − r2 = D − d2
4
Parallelogram
Area = Base × Perp. height

Rhombus
Product of diagonals
Area =
2

Trapezium
Sum of parallel sides
Area = × height
2

Triangle

Base × Perp. Height a · b · sin C


Area = OR
2 2
a+b+c
OR s (s − a) (s − b) (s − c) where s =
2

Equilateral triangle

3
Area = · (side)2
4
640 • Mathematics

Curved surface areas


L = slant height

Cylinder
Area = π · d · h

Sphere
Area = πd2 OR 4πr2

Cone
Area = π · r · L

Frustum of cone
Area = πL (R + r)

Volumes

Prism
Volume = Area of end face × Length

Cone/pyramid
Base area × Perp. height
Volume =
3
Sphere
d3 4
Volume = π · = πr3
6 3
Spherical segment
π 2
Volume = h (3d − 2h)
6
Frustum of cone
πh  2 
Volume = D + Dd + d2
12
πh  2 
OR Volume = R + Rr + r
3

Centroids and centres of gravity

Triangle
Perp. height
x= above base
3
Appendix 6 – Mathematical Formulae • 641

Pyramid
Perp. height
x= above base
4
Semi-circle
4r
x= from diameter

Hemisphere
3r
x= from diameter
8

Statistics

Mean and standard deviation

If x is the variable with frequency f then


 
f ·x f · x2
the mean x =  and the standard deviation σ =  − ( x )2
f f

Normal distribution
X −μ
The standardised normal variable, Z, is given by Z = , where μ is the mean and σ
σ
is the standard deviation defined by the question.

Product Moment Correlation Coefficient


  n 

n 
n 
n· (xi · yi ) − xi · yi
i=1 i=1 i=1
r=   2    2 

n 
n 
n 
n
n· (xi ) −
2
xi n· (yi ) −
2
yi
i=1 i=1 i=1 i=1

Regression line

y = bx + a where
 n  n 
n · ni=1 (xi · yi ) − i=1 xi · i=1 yi
b=  n 2 and a = y − b·x
n · ni=1 (xi )2 − i=1 xi

Spearman’s rank correlation coefficient, rs



6 d2
rs = 1 −  
n n2 − 1

where d is the difference between the rankings and n is the number of data pairs.
642 • Mathematics

Normal Probabilities for Positive Values


of z
z p(Z < z) z p(Z < z) z p(Z < z) z p(Z < z) z p(Z < z) z p(Z < z) z p(Z < z) z p(Z < z)

0.00 0.5000 0.50 0.6915 1.00 0.8413 1.50 0.9332 2.00 0.9772 2.50 0.9938 3.00 0.9987 3.50 0.9998
0.01 0.5040 0.51 0.6950 1.01 0.8438 1.51 0.9345 2.01 0.9778 2.51 0.9940 3.01 0.9987 3.51 0.9998
0.02 0.5080 0.52 0.6985 1.02 0.8461 1.52 0.9357 2.02 0.9783 2.52 0.9941 3.02 0.9987 3.52 0.9998
0.03 0.5120 0.53 0.7019 1.03 0.8485 1.53 0.9370 2.03 0.9788 2.53 0.9943 3.03 0.9988 3.53 0.9998
0.04 0.5160 0.54 0.7054 1.04 0.8508 1.54 0.9382 2.04 0.9793 2.54 0.9945 3.04 0.9988 3.54 0.9998
0.05 0.5199 0.55 0.7088 1.05 0.8531 1.55 0.9394 2.05 0.9798 2.55 0.9946 3.05 0.9989 3.55 0.9998
0.06 0.5239 0.56 0.7123 1.06 0.8554 1.56 0.9406 2.06 0.9803 2.56 0.9948 3.06 0.9989 3.56 0.9998
0.07 0.5279 0.57 0.7157 1.07 0.8577 1.57 0.9418 2.07 0.9808 2.57 0.9949 3.07 0.9989 3.57 0.9998
0.08 0.5319 0.58 0.7190 1.08 0.8599 1.58 0.9429 2.08 0.9812 2.58 0.9951 3.08 0.9990 3.58 0.9998
0.09 0.5359 0.59 0.7224 1.09 0.8621 1.59 0.9441 2.09 0.9817 2.59 0.9952 3.09 0.9990 3.59 0.9998
0.10 0.5398 0.60 0.7257 1.10 0.8643 1.60 0.9452 2.10 0.9821 2.60 0.9953 3.10 0.9990 3.60 0.9998
0.11 0.5438 0.61 0.7291 1.11 0.8665 1.61 0.9463 2.11 0.9826 2.61 0.9955 3.11 0.9991 3.61 0.9998
0.12 0.5478 0.62 0.7324 1.12 0.8686 1.62 0.9474 2.12 0.9830 2.62 0.9956 3.12 0.9991 3.62 0.9999
0.13 0.5517 0.63 0.7357 1.13 0.8708 1.63 0.9484 2.13 0.9834 2.63 0.9957 3.13 0.9991 3.63 0.9999
0.14 0.5557 0.64 0.7389 1.14 0.8729 1.64 0.9495 2.14 0.9838 2.64 0.9959 3.14 0.9992 3.64 0.9999
0.15 0.5596 0.65 0.7422 1.15 0.8749 1.65 0.9505 2.15 0.9842 2.65 0.9960 3.15 0.9992 3.65 0.9999
0.16 0.5636 0.66 0.7454 1.16 0.8770 1.66 0.9515 2.16 0.9846 2.66 0.9961 3.16 0.9992 3.66 0.9999
0.17 0.5675 0.67 0.7486 1.17 0.8790 1.67 0.9525 2.17 0.9850 2.67 0.9962 3.17 0.9992 3.67 0.9999
0.18 0.5714 0.68 0.7517 1.18 0.8810 1.68 0.9535 2.18 0.9854 2.68 0.9963 3.18 0.9993 3.68 0.9999
0.19 0.5753 0.69 0.7549 1.19 0.8830 1.69 0.9545 2.19 0.9857 2.69 0.9964 3.19 0.9993 3.69 0.9999
0.20 0.5793 0.70 0.7580 1.20 0.8849 1.70 0.9554 2.20 0.9861 2.70 0.9965 3.20 0.9993 3.70 0.9999
0.21 0.5832 0.71 0.7611 1.21 0.8869 1.71 0.9564 2.21 0.9864 2.71 0.9966 3.21 0.9993 3.71 0.9999
0.22 0.5871 0.72 0.7642 1.22 0.8888 1.72 0.9573 2.22 0.9868 2.72 0.9967 3.22 0.9994 3.72 0.9999
0.23 0.5910 0.73 0.7673 1.23 0.8907 1.73 0.9582 2.23 0.9871 2.73 0.9968 3.23 0.9994 3.73 0.9999
0.24 0.5948 0.74 0.7704 1.24 0.8925 1.74 0.9591 2.24 0.9875 2.74 0.9969 3.24 0.9994 3.74 0.9999
0.25 0.5987 0.75 0.7734 1.25 0.8944 1.75 0.9599 2.25 0.9878 2.75 0.9970 3.25 0.9994 3.75 0.9999
0.26 0.6026 0.76 0.7764 1.26 0.8962 1.76 0.9608 2.26 0.9881 2.76 0.9971 3.26 0.9994 3.76 0.9999
0.27 0.6064 0.77 0.7794 1.27 0.8980 1.77 0.9616 2.27 0.9884 2.77 0.9972 3.27 0.9995 3.77 0.9999
0.28 0.6103 0.78 0.7823 1.28 0.8997 1.78 0.9625 2.28 0.9887 2.78 0.9973 3.28 0.9995 3.78 0.9999
0.29 0.6141 0.79 0.7852 1.29 0.9015 1.79 0.9633 2.29 0.9890 2.79 0.9974 3.29 0.9995 3.79 0.9999
0.30 0.6179 0.80 0.7881 1.30 0.9032 1.80 0.9641 2.30 0.9893 2.80 0.9974 3.30 0.9995 3.80 0.9999
0.31 0.6217 0.81 0.7910 1.31 0.9049 1.81 0.9649 2.31 0.9896 2.81 0.9975 3.31 0.9995 3.81 0.9999
0.32 0.6255 0.82 0.7939 1.32 0.9066 1.82 0.9656 2.32 0.9898 2.82 0.9976 3.32 0.9995 3.82 0.9999
0.33 0.6293 0.83 0.7967 1.33 0.9082 1.83 0.9664 2.33 0.9901 2.83 0.9977 3.33 0.9996 3.83 0.9999
Appendix 6 – Mathematical Formulae • 643

z p(Z < z) z p(Z < z) z p(Z < z) z p(Z < z) z p(Z < z) z p(Z < z) z p(Z < z) z p(Z < z)
0.34 0.6331 0.84 0.7995 1.34 0.9099 1.84 0.9671 2.34 0.9904 2.84 0.9977 3.34 0.9996 3.84 0.9999
0.35 0.6368 0.85 0.8023 1.35 0.9115 1.85 0.9678 2.35 0.9906 2.85 0.9978 3.35 0.9996 3.85 0.9999
0.36 0.6406 0.86 0.8051 1.36 0.9131 1.86 0.9686 2.36 0.9909 2.86 0.9979 3.36 0.9996 3.86 0.9999
0.37 0.6443 0.87 0.8078 1.37 0.9147 1.87 0.9693 2.37 0.9911 2.87 0.9979 3.37 0.9996 3.87 0.9999
0.38 0.6480 0.88 0.8106 1.38 0.9162 1.88 0.9699 2.38 0.9913 2.88 0.9980 3.38 0.9996 3.88 0.9999
0.39 0.6517 0.89 0.8133 1.39 0.9177 1.89 0.9706 2.39 0.9916 2.89 0.9981 3.39 0.9997 3.89 0.9999
0.40 0.6554 0.90 0.8159 1.40 0.9192 1.90 0.9713 2.40 0.9918 2.90 0.9981 3.40 0.9997 3.90 1.0000
0.41 0.6591 0.91 0.8186 1.41 0.9207 1.91 0.9719 2.41 0.9920 2.91 0.9982 3.41 0.9997 3.91 1.0000
0.42 0.6628 0.92 0.8212 1.42 0.9222 1.92 0.9726 2.42 0.9922 2.92 0.9982 3.42 0.9997 3.92 1.0000
0.43 0.6664 0.93 0.8238 1.43 0.9236 1.93 0.9732 2.43 0.9925 2.93 0.9983 3.43 0.9997 3.93 1.0000
0.44 0.6700 0.94 0.8264 1.44 0.9251 1.94 0.9738 2.44 0.9927 2.94 0.9984 3.44 0.9997 3.94 1.0000
0.45 0.6736 0.95 0.8289 1.45 0.9265 1.95 0.9744 2.45 0.9929 2.95 0.9984 3.45 0.9997 3.95 1.0000
0.46 0.6772 0.96 0.8315 1.46 0.9279 1.96 0.9750 2.46 0.9931 2.96 0.9985 3.46 0.9997 3.96 1.0000
0.47 0.6808 0.97 0.8340 1.47 0.9292 1.97 0.9756 2.47 0.9932 2.97 0.9985 3.47 0.9997 3.97 1.0000
0.48 0.6844 0.98 0.8365 1.48 0.9306 1.98 0.9761 2.48 0.9934 2.98 0.9986 3.48 0.9997 3.98 1.0000
0.49 0.6879 0.99 0.8389 1.49 0.9319 1.99 0.9767 2.49 0.9936 2.99 0.9986 3.49 0.9998 3.99 1.0000
INDEX
addition of sine and cosine waves 173 complementary angles 149
algebra 9 complex numbers 487
angles, constant of integration 291
acute 142 constituent parts 478
complementary 149 coordinates,
obtuse 142 polar 162
over 90◦ 142 rectangular/Cartesian 161
properties of angles and lines 150 cosecant 147
reflex 142 cosine 146
right 141 cosine rule 198
supplementary 150 cotangent 147
area, cubic equations 112
annulus 220 curve sketching 138
circle 218
circular ring (torus) surface 227 degree, minute, second 142
cone surface 225 density 235
cylinder surface 223 differential calculus 267
ellipse 223 differentiation,
frustum surface 226 chain rule (function of a function) 318
parallelogram 214 distance, velocity, acceleration 278
polygon 216 first principles 271
rhombus 215 inverse trigonometric functions 319
sector of circle 221 of ln (x) and ex 286, 287
segment of circle 221 of powers 275
sphere surface 224 of sin(x), cos(x) 285
trapezium 215 product rule 314
triangle 206, 214 quotient rule 316
units 213 rates of change 342
areas, double angles 210
between curves 337
by integration 297 equation,
averages 356, 481 logarithmic 71
of a straight line 122
BIDMAS/BODMAS 27 quadratic 96
binomial probability distribution 380 simple 33
brackets 12 simultaneous linear 75
equilateral triangle 151
centre of gravity (mass) 244 evaluation 27
centroid 244, 334 examination,
circular measure 162 questions 399
common logarithms 5 solutions 413
compass bearings 158 expansion of terms 20
Index • 645

factor theorem 113 laws of 6


factorization 12, 24 Naperian (natural) 5
first (and second) moments 255 longitude 183
flow of liquid 252
forcer 244 mass 224
formulae, maxima and minima 279
transposition 51 measurement,
Fourier analysis 582 of areas 213
fractions 465 of volumes 234
frustum, measurement of angles 142
surface area 226 meridians 183
volume 239 method of unity 472
functional notation 111, 287 moments 255, 334
multiplication,
geometrical facts 150, 190 algebraic 12
gradient 271 of powers 1
graphs 119
graphs, Naperian logarithms 5
sine, cosine, tangent 163 nautical mile 186
Guldinus, theorem of 225, 242 negative indices 2
non right angled triangles 198
hyperbolic functions,
equations 181 obtuse angle 142
identities 180, 183
sinh(x), cosh(x), tanh(x) 179 Pappus theorem 225, 242
parallelogram 214
indices 1 partial fractions 66
integral calculus 291 percentage 476
integration, polynomial division 21, 110
1
of and ln (x) 296 powers 1
x
definite integrals 297 prisms 235
distance and velocity 309 probability 382
of powers 292 addition law 384
of sin(x), cos(x) 296 binomial distribution 388
inverse proportion 472 conditional 387
isosceles triangle 151 multiplication law 385
normal distribution (Gaussian) 390
latitude 183 tree 385
laws, determination of 135 problems,
LCM (lowest common multiple) 15, 37 involving quadratic equations 79
lift of valve 253 involving simple equations 43, 45
linear graphs (straight line graphs) 120 proportion,
logarithmic equations 71 direct 471
logarithms, inverse 472
change of base rule 6 pyramids 237
common 5 Pythagoras’ theorem 151, 175
646 • Index

quadratic equations 96 mode 360


quadratics, correlation 372
completing the square 99 data– diagrammatic 351
graphical solution 128 data– tabular 349
solution by factorization 96 dispersion,
solution by formula 102 range 366
standard deviation 367
radian 142 ranked correlation 378
ratios 470 regression (linear) 372
ratios, supplementary angles 150
hyperbolic 179 surds 4
trigonometric 146
reflex angle 142 tangent 146
relative density 235 transposition of formulae 51
rhombus 215 trapezium 215
right angle 141 triangle,
right angled triangles 146 area 206, 214
roots 96 centroid 246
r.p.m. (revolutions per minute) 143 equilateral 151, 207
isosceles 151, 205
secant 147
scalene 151
sector (circle) 221
solution of 195
segment (circle) 222
trigonometric,
similar figures 228, 248
identities 179
simple equations 33
ratios 146, 175
simplification of algebraic expressions 10
Simpson’s first rule 229, 249 unity,
simultaneous equations, method of 472
linear,
including logarithms 88 variation 473
other methods of solution 89 volume,
solution by elimination 75 cone 237
linear/quadratic, frustum 239
algebraic solution 108 prisms 235
graphical solution 127, 133 pyramid 237
sine 146 sphere 241
sine rule 198 units 234
sketching of curves 135 volumes,
specific gravity 235 by integration (solid of revolution)
sphere, 306, 333
surface area 224 Simpson’s rule 249
volume 241
statistics 348 weight 244
average,
mean 356 zero gradient 270
median 358 zero power 2

You might also like